2020 PREP Self-Assessment
 2016309916

Table of contents :
Question 1
Question 2
Question 3
Question 4
Question 5
Question 6
Question 7
Question 8
Question 9
Question 10
Question 11
Question 12
Question 13
Question 14
Question 15
Question 16
Question 17
Question 18
Question 19
Question 20
Question 21
Question 22
Question 23
Question 24
Question 25
Question 26
Question 27
Question 28
Question 29
Question 30
Question 31
Question 32
Question 33
Question 34
Question 35
Question 36
Question 37
Question 38
Question 39
Question 40
Question 41
Question 42
Question 43
Question 44
Question 45
Question 46
Question 47
Question 48
Question 49
Question 50
Question 51
Question 52
Question 53
Question 54
Question 55
Question 56
Question 57
Question 58
Question 59
Question 60
Question 61
Question 62
Question 63
Question 64
Question 65
Question 66
Question 67
Question 68
Question 69
Question 70
Question 71
Question 72
Question 73
Question 74
Question 75
Question 76
Question 77
Question 78
Question 79
Question 80
Question 81
Question 82
Question 83
Question 84
Question 85
Question 86
Question 87
Question 88
Question 89
Question 90
Question 91
Question 92
Question 93
Question 94
Question 95
Question 96
Question 97
Question 98
Question 99
Question 100
Question 101
Question 102
Question 103
Question 104
Question 105
Question 106
Question 107
Question 108
Question 109
Question 110
Question 111
Question 112
Question 113
Question 114
Question 115
Question 116
Question 117
Question 118
Question 119
Question 120
Question 121
Question 122
Question 123
Question 124
Question 125
Question 126
Question 127
Question 128
Question 129
Question 130
Question 131
Question 132
Question 133
Question 134
Question 135
Question 136
Question 137
Question 138
Question 139
Question 140
Question 141
Question 142
Question 143
Question 144
Question 145
Question 146
Question 147
Question 148
Question 149
Question 150
Question 151
Question 152
Question 153
Question 154
Question 155
Question 156
Question 157
Question 158
Question 159
Question 160
Question 161
Question 162
Question 163
Question 164
Question 165
Question 166
Question 167
Question 168
Question 169
Question 170
Question 171
Question 172
Question 173
Question 174
Question 175
Question 176
Question 177
Question 178
Question 179
Question 180
Question 181
Question 182
Question 183
Question 184
Question 185
Question 186
Question 187
Question 188
Question 189
Question 190
Question 191
Question 192
Question 193
Question 194
Question 195
Question 196
Question 197
Question 198
Question 199
Question 200
Question 201
Question 202
Question 203
Question 204
Question 205
Question 206
Question 207
Question 208
Question 209
Question 210
Question 211
Question 212
Question 213
Question 214
Question 215
Question 216
Question 217
Question 218
Question 219
Question 220
Question 221
Question 222
Question 223
Question 224
Question 225
Question 226
Question 227
Question 228
Question 229
Question 230
Question 231
Question 232
Question 233
Question 234
Question 235
Question 236
Question 237
Question 238
Question 239
Question 240
Question 241
Question 242
Question 243
Question 244
Question 245

Citation preview

PREP ® Self-Assessment

American academy of pediatrics

PREPSA 2020

1

PREP ® Self-Assessment

PREPSA 2020

Question 1 A 10-year-old boy is seen for evaluation of left heel pain. He was previously healthy until the onset of pain several months ago, shortly after the beginning of his soccer season. The pain initially occurred during and after running, but it is now present throughout the day and is associated with a limp. The boy and his mother report no traumatic injury, and there has been no associated swelling, bruising, or other musculoskeletal concerns. The pain improved significantly during a 1-week break from soccer, but it recurred within several days of return to practice. He has focal tenderness over the medial and lateral aspects of his left heel and prefers to walk on his toes. The remainder of his physical examination findings are unremarkable. Of the following, the BEST next step in his care is A. non–weight bearing for 2 weeks B. radiographs of left foot C. reassurance D. trial of heel cups or cushions

American academy of pediatrics

2

PREP ® Self-Assessment

PREPSA 2020

Correct Answer: D The boy in this vignette has Sever apophysitis. Initial treatment should focus on protecting the injured calcaneal growth plate with a trial of heel cushions. Application of ice for 10 to 15 minutes several times per day and use of over-the-counter analgesics as needed for several days can also be helpful. Sever apophysitis is a stress reaction through the cartilaginous calcaneal apophysis. It can occur in any child who is skeletally immature but is most common in pre-adolescent boys participating in running sports. The calcaneal apophysis is subject to multiple stressors in young athletes, as shown in Item C1A. These stressors include competing shear stresses of: • • •

The Achilles tendon superiorly The plantar fascia inferiorly Direct ground impact force from running, jumping, or other high-impact activity

Any combination of these stressors can contribute to Sever disease, and they can be easily assessed during a clinic visit, as described below. For impact assessment, activity and footwear histories are key. Recent increase in high-impact sports or training are often irritators to the calcaneal apophysis, especially when performed in American academy of pediatrics 3

PREP ® Self-Assessment

PREPSA 2020

cleats or other poorly cushioned footwear. Shoes in these young athletes should provide adequate cushioning through the heel. A well-structured running shoe is often a better choice than shoes that are lighter weight or shoes that are designed for basketball or other court sports. If the patient is training in cleats or shoes with poor cushioning, then an added heel cushion will take stress off the apophysis. There are a variety of heel cushions on the market, and there can be some trial and error in determining what fits best in the shoe and feels best on the heel. Most cases of Sever disease will respond well to the added cushioning and a brief period of restrictedimpact activity until symptoms improve. During Achilles assessment, growth patterns should be queried as traction from the Achilles tendon often increases during periods of rapid linear growth. The patient should be able to achieve at least 10° of foot dorsiflexion, and it is not uncommon for males who are growing rapidly to lack this degree of dorsiflexion. Caregivers can be counseled that maintenance of adequate flexibility through the Achilles decreases this stress across the apophysis. Traction from the plantar fascia is higher in a foot that excessively pronates with activity. Office assessment for this is easily performed by having the patient squat or jump in the office (Item C1B). The foot should remain in neutral alignment with activity, and the knees should track over the middle of the foot. The video demonstrates increased pronation with the feet "rolling in" and the knees falling into valgus alignment (ie, "knock-kneed") as the patient is preparing to push-off and with landing. This is more evident on the left than the right. In patients who demonstrate this movement pattern, footwear with appropriate amounts of support through the medial arch are often of benefit. Unfortunately, many youth sports use cleats that lack support, and a trial of supportive over-the-counter orthotics may be warranted. In addition, exercises that strengthen the intrinsic muscles of the foot, as well as the abductors and external rotators of the hip, are often helpful, and physical therapy can assist with this strengthening. Non–weight bearing is not the best initial treatment of Sever disease in the vast majority of cases. On occasion, a patient who continues to limp and experience pain in the heel despite the above recommendations may benefit from a trial in a walking boot or crutches. These should only be used until symptoms abate, which is typically much less than 2 weeks. Sever disease is a clinical diagnosis, and radiographs are generally not indicated in the initial presentation. Appearance of the normal calcaneal apophysis is highly variable (Item C1A), often with multiple ossification centers. This irregular appearance on radiographs is not diagnostic. Sever disease is a self-limited condition that resolves with skeletal maturity. However, it can be recurrent during growth and is painful enough to interfere with enjoyment and pursuit of healthy levels of physical activity. Simple reassurance is not sufficient. Families should be counseled on the above measures for symptomatic treatment and relief.

American academy of pediatrics

4

PREP ® Self-Assessment

PREPSA 2020

PREP Pearls • Sever disease often results from participation in sports with cleats or poorly cushioned footwear. • Assuring adequate cushioning in shoes worn for sports and daily activity can markedly reduce the symptoms of Sever disease. ABP Content Specifications(s) • Recognize the clinical findings and etiologic characteristics of Sever disease, and manage appropriately Suggested Readings • Herman MJ, Martinek M. The limping child. Pediatr Rev. 2015;36(5):184-195. doi:10.1542/pir.36-5-184. • James AM, Williams CM, Haines TP. Effectiveness of interventions in reducing pain and maintaining physical activity in children and adolescents with calcaneal apophysitis (Sever’s disease): a systematic review. J Foot Ankle Res.2013;6(1):16. doi:10.1186/17571146-6-16. • Kannikeswaran N, Suresh S. Sports musculoskeletal injuries. In: McInerny TK, Adam HM, Campbell DE, DeWitt TG, Foy JM, Kamat DM, eds. American Academy of Pediatrics Textbook of Pediatric Care. 2nd ed. Elk Grove Village, IL: American Academy of Pediatrics; 2017:2675-2686. Pediatric Care Online. • Pacicca DM. Osteochondroses. In: McInerny TK, Adam HM, Campbell DE, DeWitt TG, Foy JM, Kamat DM, eds. American Academy of Pediatrics Textbook of Pediatric Care. 2nd ed. Elk Grove Village, IL: American Academy of Pediatrics; 2017:24412446. Pediatric Care Online.

American academy of pediatrics

5

PREP ® Self-Assessment

PREPSA 2020

Question 2 A 13-year-old adolescent boy is seen for ongoing difficulty swallowing. He has no trouble swallowing liquids, but he has stopped eating chicken and steak because he feels like they get stuck in his chest. His parents report that it takes him a long time to eat compared with his siblings, and he prefers to have food cut up into small bites. He has a medical history of asthma and seasonal allergies. He was diagnosed with gastroesophageal reflux disease and required a proton pump inhibitor until 12 months of age. He currently takes albuterol as needed for asthma and antihistamines as needed for allergy symptoms. He appears well and is in no acute distress. He has normal height, weight, and body mass index for age. His physical examination findings are within normal limits. Of the following, the test that is MOST likely to establish the diagnosis is A. B. C. D.

abdominal ultrasonography gastric scintigraphy magnetic resonance cholangiopancreatography upper gastrointestinal series

American academy of pediatrics

6

PREP ® Self-Assessment

PREPSA 2020

Correct Answer: D The adolescent boy in this vignette has dysphagia with solid foods, resulting from an esophageal stricture related to eosinophilic esophagitis. The test most likely to diagnose the esophageal stricture is an upper gastrointestinal series, a fluoroscopic test used to diagnose structural anomalies of the esophagus, stomach, and proximal small bowel. This presentation (dysphagia with solids) in an older child, particularly with a history of atopy, is common for eosinophilic esophagitis. A previous history of gastroesophageal reflux disease (GERD) as an infant does not increase his risk for eosinophilic esophagitis. Esophageal causes of dysphagia can include structural anomalies of the esophagus, including esophageal stricture (congenital or acquired), esophageal web, achalasia, esophageal duplication cyst, and extrinsic compression of the esophagus. Inflammatory conditions, including eosinophilic esophagitis, GERD, and pill esophagitis, can lead to poor esophageal motility and resultant dysphagia. Acute onset of esophageal dysphagia should prompt consideration of a foreign body, caustic ingestion, or infection (including herpes simplex virus and candidal esophagitis). Gastroesophageal reflux (GER) is a physiologic condition describing the passage of gastric contents into the esophagus. Most common in young infants, GER occurs because of transient relaxation of the lower esophageal sphincter, which occurs most often after eating larger meals. When GER causes troublesome symptoms or complications (either to the child or perceived by the caregiver), it is termed GERD. Diagnosis of GERD is made clinically, however further testing may be of some benefit if concerning signs or symptoms are present. An upper gastrointestinal series may be helpful to diagnose structural anomalies, for instance in a child with dysphagia or forceful vomiting, but it is not a diagnostic test for GERD. Impedance/pH study of the esophagus can evaluate for acid and nonacid reflux; however, this study requires specific training and may not be easily available at all pediatric hospitals. Endoscopy is useful to assess esophageal mucosa and can identify disorders that may mimic GERD (eg, eosinophilic esophagitis), but it is not necessary to diagnose GERD in children. Abdominal ultrasonography and gastric scintigraphy (otherwise known as a gastric emptying scan) are not indicated for the evaluation of dysphagia in infants or children. These tests may be helpful to evaluate for causes of abdominal pain and/or vomiting, but they are not used to evaluate disorders of the esophagus. Magnetic resonance cholangiopancreatography is useful to diagnose biliary tract disease but cannot evaluate for causes of dysphagia. PREP Pearls • Upper gastrointestinal series is an important tool to diagnose structural anomalies of the upper intestine, however it is not indicated to diagnose gastroesophageal reflux disease. • Esophageal dysphagia may be caused by structural anomalies, including esophageal stricture, or inflammatory conditions, including gastroesophageal reflux disease, eosinophilic esophagitis, and infectious esophagitis.

American academy of pediatrics

7

PREP ® Self-Assessment

PREPSA 2020

ABP Content Specifications(s) • Recognize the structural anomalies that interfere with normal esophageal function • Plan the appropriate evaluation of gastroesophageal reflux, and manage appropriately Suggested Readings • Adamiak T, Plati KF. Pediatric esophageal disorders: diagnosis and treatment of reflux and eosinophilic esophagitis. Pediatr Rev. 2018;39(8):392-402. doi:10.1542/pir.20170266. • Lightdale JR. Gastroesophageal reflux disease. In: McInerny TK, Adam HM, Campbell DE, DeWitt TG, Foy JM, Kamat DM, eds. American Academy of Pediatrics Textbook of Pediatric Care. 2nd ed. Elk Grove Village, IL: American Academy of Pediatrics; 2017:2063-2076. Pediatric Care Online. • Rosen R, Vandenplas Y, Singendonk M, et al. Pediatric gastroesophageal reflux clinical practice guidelines: joint recommendations of the North American Society for Pediatric Gastroenterology, Hepatology, and Nutrition and the European Society for Pediatric Gastroenterology, Hepatology, and Nutrition. J Pediatr Gastroenterol Nutr. 2018;66(3):516-554. doi:10.1097/MPG.0000000000001889.

American academy of pediatrics

8

PREP ® Self-Assessment

PREPSA 2020

Question 3 On initial physical examination, a 4,500-g neonate is noted to have an umbilical hernia, macroglossia, unusual ear creases, hemihypertrophy, and visceromegaly. Soon after delivery, he develops persistent hypoglycemia warranting treatment with intravenous dextrose. Of the following, this neonate is at INCREASED risk for A. B. C. D.

astrocytoma bone marrow failure leukemia Wilms tumor

American academy of pediatrics

9

PREP ® Self-Assessment

PREPSA 2020

Correct Answer: D This neonate's clinical presentation is consistent with a diagnosis of Beckwith-Wiedemann syndrome (BWS), placing him at increased risk for Wilms tumor. Beckwith-Wiedemann syndrome is an overgrowth disorder manifested by macrosomia, macroglossia, neonatal hypoglycemia, ear creases/pits, hemihypertrophy, and visceromegaly. Patients can also have embryonal tumors (Wilms tumor, hepatoblastoma, neuroblastoma), umbilical hernia/omphalocele, nephrocalcinosis, medullary sponge kidney disease, cardiomegaly, and nephromegaly. The macrosomia, macroglossia, and hypoglycemia are typically noted in the neonatal period. Hemihyperplasia can be noted in segmental regions of the body or specific organs. Beckwith-Wiedemann syndrome can present with variable expression, with some affected individuals having multiple clinical features while others have only 1 to 2 phenotypic findings. Development and cognitive outcomes are typically normal. Because of the risk for embryonal tumors, routine surveillance is recommended. The highest risk is in young children through age 8 years, therefore abdominal ultrasonography is recommended every 3 months until age 8 years. Serum α-fetoprotein concentration should be monitored every 2 to 3 months until age 4 years for early detection of hepatoblastoma. The estimated tumor risk for children with BWS is 7.5%. Between age 8 years and mid-adolescence, annual renal ultrasonography is recommended to assess for renal disease; measurement of urinary calciumcreatinine ratio should also be considered. Most individuals have BWS in association with an imprinting disorder, such as a methylation abnormality or paternal uniparental disomy of chromosome 11p15. Thus, most cases are not inherited. The differential diagnosis for a child with hemihyperplasia should include BWS, Proteus syndrome (PTEN hamartoma tumor syndrome), Klippel-Trenaunay-Weber syndrome, isolated hemihyperplasia, and neurofibromatosis type 1. A subset of individuals with isolated hemihyperplasia could actually have BWS but with few clinical findings. These children have a tumor risk of approximately 5.9%, and should undergo tumor surveillance akin to that recommended for children with BWS. Other disorders that can present with macrosomia in the neonatal period include SimpsonGolabi-Behmel syndrome, Costello syndrome, and Sotos syndrome. These disorders typically also have developmental delay along with the macrosomia, which is not common with BWS. Infants of mothers with gestational diabetes can also have macrosomia and neonatal hypoglycemia. Astrocytoma can be seen in association with neurofibromatosis type 1, tuberous sclerosis complex, and melanoma-astrocytoma syndrome. Bone marrow failure can be seen in association with Fanconi anemia, Pearson syndrome, Diamond Blackfan syndrome, dyskeratosis congenita, Shwachman Diamond syndrome, and amegakaryocytic anemia. Leukemia risk is increased with Down syndrome and Li Fraumeni syndrome.

American academy of pediatrics

10

PREP ® Self-Assessment

PREPSA 2020

PREP Pearls • Beckwith-Wiedemann syndrome is an overgrowth disorder manifested by macrosomia, macroglossia, neonatal hypoglycemia, ear creases/pits, hemihypertrophy, and visceromegaly. • The estimated tumor risk for children with Beckwith-Wiedemann syndrome is 7.5%. • For children with BWS, the risk for embryonal tumors is highest in young children through age 8 years. Surveillance is recommended, including abdominal ultrasonography every 3 months until age 8 years. Measurement of serum α-fetoprotein concentration is recommended every 2 to 3 months in the first 4 years of age for early detection of hepatoblastoma. ABP Content Specifications(s) • Recognize the clinical features and risks associated with overgrowth syndrome Suggested Readings • Clericuzio CL, Martin RA. Diagnostic criteria and tumor screening for individuals with isolated hemihyperplasia. Genet Med. 2009;11(3):220-222. doi:10.1097/GIM.0b013e31819436cf. • Ripperger T, Bielack SS, Borkhardt A, et al. Childhood cancer predisposition syndromes—A concise review and recommendations by the Cancer Predisposition Working Group of the Society for Pediatric Oncology and Hematology. Am J Med Genet A. 2017;173(4):1017-1037. doi:10.1002/ajmg.a.38142. • Shuman C, Beckwith JB, Weksberg R. Beckwith-Wiedemann syndrome. GeneReviews. https://www.ncbi.nlm.nih.gov/books/NBK1394/.

American academy of pediatrics

11

PREP ® Self-Assessment

PREPSA 2020

Question 4 A 4-year-old girl with a history of agenesis of the corpus callosum and epilepsy is brought to her primary care clinician by her mother to discuss seizure management. The girl has been taking antiseizure medications since age 6 months when she had her first clinical seizure. The girl's neurologist feels that her current medical management is optimized, but she continues to have several seizures each week. Her mother is interested in acupuncture as an adjunctive treatment and seeks advice. Her neurologist has stated that there is no conclusive evidence that acupuncture can reduce seizure frequency. The primary care clinician is not concerned that acupuncture would be dangerous for this patient. Of the following, the ethical principle MOST applicable to this situation is A. assent B. autonomy C. beneficence D. justice

American academy of pediatrics

12

PREP ® Self-Assessment

PREPSA 2020

Correct Answer: B Complementary and alternative medicine (CAM) treatments are becoming increasingly popular in the United States. A 2015 study reported that approximately 10% of children use CAM; several studies have found that this rate is more than 50% in children with chronic or lifethreatening medical conditions. The efficacy and safety of many CAM treatments is not well studied, particularly in children, and many CAM treatment modalities have unclear or implausible mechanisms of action. Clinicians may be asked to make treatment recommendations under conditions of uncertainty, which can lead to ethical dilemmas. These dilemmas typically involve questions about who the appropriate decision-makers are, what ethical principles should contribute to decision-making, and what obligations the clinician must uphold. The ethical principles of autonomy, parental authority, beneficence, nonmaleficence, and justice can help to develop an ethical framework within which to consider these questions. Autonomy is the right of competent persons to make decisions about their own medical care that are consistent with their individual values. Children are generally not considered to be "competent" to make medical decisions, though this becomes increasingly complex with adolescents. Accordingly, parents are typically delegated to the role of decision-makers for their children. When appropriate, based on their ability to understand the situation, children should assent to treatment. Some ethicists consider the principles of autonomy and parental authority, or the right of parents to make decisions on behalf of their child, to be distinct. Parental authority has limits, including situations in which the parents' decision is clearly not in accordance with their child’s best interest. Beneficence is the duty of the clinician to act in the best interest of the patient, and nonmaleficence is the responsibility to not harm patients. Justice is the need for fairness, or equal distribution of benefit and harm, across individuals and groups. The girl in the vignette has a complex chronic illness with daily seizures despite optimized medication management. Her mother is interested in pursuing CAM as an adjunctive treatment, not as a replacement for evidence-based treatment. Although there is no evidence that the CAM therapy can reduce her seizure frequency, the clinician is not concerned that the treatment would be dangerous. Given these factors, the ethical principle that is most applicable in guiding the clinician’s advice to the family is autonomy (parental authority would also be an appropriate answer if it were listed). A 4-year-old child with this complex neurologic condition could not be expected to provide assent. Beneficence does not apply because the clinician does not believe that treatment has proven benefit or likely risk for the patient. Principles of justice do not apply to this situation. The American Academy of Pediatrics Committee on Children With Disabilities has published a framework for clinicians facing questions about CAM, which recommends that the clinician: 1. Seek information and be prepared to share it with families. 2. Evaluate the scientific merits of specific therapeutic approaches. 3. Identify risks or potential harmful effects. 4. Provide families with information on a range of treatment options (avoid therapeutic nihilism). 5. Educate families to evaluate information about all treatment approaches. American academy of pediatrics

13

PREP ® Self-Assessment

PREPSA 2020

6. Avoid dismissal of CAM in ways that communicate a lack of sensitivity or concern for the family’s perspective. 7. Recognize when they feel threatened and guard against becoming defensive. 8. Offer to assist in monitoring and evaluating the response if CAM treatment is endorsed. 9. Actively listen to families and their children with chronic illness. PREP Pearls • Many children use complementary and alternative medicine; more than 50% of children with chronic or life-threatening illness have reported complementary and alternative medicine use. • Patient autonomy and parental authority (the right of parents to make decisions on behalf of their child) are important factors to consider when discussing complementary and alternative medicine with families. • Parental authority has limits, including situations in which a parent’s decision is clearly not in accordance with their child’s best interest. ABP Content Specifications(s) • Recognize and apply ethical principles regarding the use of complementary and alternative medicine Suggested Readings • American Academy of Pediatrics Committee on Children with Disabilities. Counseling families who choose complementary and alternative medicine for their child with chronic illness or disability. Pediatrics. 2001;107(3):598601. http://pediatrics.aappublications.org/content/107/3/598. • Black LI, Clarke TC, Barnes PM, Stussman BJ, Nahin RL. Use of complementary health approaches among children aged 4-17 years in the United States: National Health Interview Survey, 2007-2012. Natl Health Stat Report. 2015;78:1-19. PMCID:PMC4562218.. • Cummings CL, Mercurio MR. Ethics for the pediatrician: autonomy, beneficence, and rights. Pediatr Rev. 2010;31(6):252-255. doi:10.1542/pir.31-6-252. • Friedman Ross L, Lantos JD. Ethical and legal issues for the primary care physician. In: McInerny TK, Adam HM, Campbell DE, DeWitt TG, Foy JM, Kamat DM, eds. American Academy of Pediatrics Textbook of Pediatric Care. 2nd ed. Elk Grove Village, IL: American Academy of Pediatrics; 2017:61-72. Pediatric Care Online. • Mears BJ. Ethics for the pediatrician: the ethics of complementary and alternative medicine. Pediatr Rev. 2010;31(7):e49-e51. doi:10.1542/pir.31-7-e49.

American academy of pediatrics

14

PREP ® Self-Assessment

PREPSA 2020

Question 5 A 14-year-old girl is being evaluated for fatigue and left knee pain. Over the past several weeks she has been feeling fatigued. She is unable to keep up with peers in physical education classes and has recently starting taking midday naps. The girl’s left knee has been tender, red, and swollen for several days. She denies any trauma. She reports that a similar event occurred with the right knee a couple of weeks ago that resolved on its own. The girl has been otherwise well, aside from an upper respiratory tract infection that occurred while out of town on vacation about 1 month ago. On physical examination, her vital signs are normal. She has a 2/6 blowing holosystolic murmur at the apex that was not present on previous examinations. Her left knee is erythematous and tender to palpation. An antistreptolysin O titer result is positive. Of the following, in addition to a cardiology referral, the MOST appropriate next step in management for this girl is A. B. C. D.

aspirin and penicillin orally aspirin orally and intravenous immunoglobulin penicillin and glucocorticoids orally penicillin orally and intravenous immunoglobulin

American academy of pediatrics

15

PREP ® Self-Assessment

PREPSA 2020

Correct Answer A The girl in the vignette has evidence of preceding group A streptococcal infection as well as carditis and polyarthritis. These findings meet the Jones criteria for acute rheumatic fever (ARF). The management of ARF includes an anti-inflammatory medication (aspirin), eradication of the streptococcal infection (penicillin), and treatment of carditis or heart failure. There is no role for intravenous immunoglobulin in ARF. In cases of severe rheumatic carditis, the addition of glucocorticoids is believed to have therapeutic benefit, but contemporary studies are lacking, and it is not the current standard of care. The diagnosis of ARF is based on the Jones criteria which were recently revised (Item C5), given the development and advancement of echocardiography. The diagnosis is made when there is evidence of preceding streptococcal infection and either 2 major manifestations or 1 major manifestation as well as 2 minor manifestations are present.

The management of ARF is based on 3 principles. First, the inflammatory state should be managed with anti-inflammatory medication. The typical drug used is aspirin, though naproxen use has also been reported. Second, carditis or heart failure symptoms need to be managed. Drugs used in other clinical heart failure contexts, such as diuretics, are used in ARF as well. Valvular disease may warrant cardiac surgery; this is avoided during the acute phase because it is associated with worse outcomes. Third, the group A streptococcal infection needs to be American academy of pediatrics

16

PREP ® Self-Assessment

PREPSA 2020

eradicated and recurrence prevented. Penicillin, given orally or intramuscularly, is the drug of choice. PREP Pearls • The initial diagnosis of acute rheumatic fever, based on the revised Jones criteria, is made when there is evidence of preceding streptococcal infection and either 2 major manifestations or 1 major manifestation as well as 2 minor manifestations are present. • The management of acute rheumatic fever includes anti-inflammatory medication (aspirin), eradication of the streptococcal infection (penicillin), and treatment of carditis or heart failure. •

ABP Content Specifications(s) • Plan the appropriate initial management of rheumatic fever • Plan appropriate diagnostic evaluation of rheumatic fever Suggested Readings • Gersony WM, Starc TJ. Rheumatic fever. In: McInerny TK, Adam HM, Campbell DE, DeWitt TG, Foy JM, Kamat DM, eds. American Academy of Pediatrics Textbook of Pediatric Care. 2nd ed. Elk Grove Village, IL: American Academy of Pediatrics; 2017:2572-2578. Pediatric Care Online. • Gewitz MH, Baltimore RS, Tani LY, et al. Revision of the Jones criteria for the diagnosis of acute rheumatic fever in the era of Doppler echocardiography A scientific statement form the American Heart Association. Circulation. 2015;131(20):1806-1818. doi:10.1161/CIR.0000000000000205. • Mirkinson L. The diagnosis of rheumatic fever. Pediatr Rev. 1998;19(9):310-311. doi:10.1542/pir.19-9-310. • Zuhle LJ, Beaton A, Engel ME, et al. Group A Streptococcus, acute rheumatic fever and rheumatic heart disease: epidemiology and clinical considerations. Curr Treat Options Cardiol Med. 2017;19(2):15. doi:10.1007/s11936-017-0513-y.

American academy of pediatrics

17

PREP ® Self-Assessment

PREPSA 2020

Question 6 The parents of a 4-year-old girl call their pediatrician's office for advice. The girl’s father has been diagnosed with an aggressive cancer and he is now being cared for by hospice; his physicians do not expect him to live longer than a few months. The girl's parents are unsure how to discuss her father's illness with her, and ask how a preschool-aged child might respond to the death of a family member. Of the following, a COMMON response in this age group is A. B. C. D.

focus on unfairness magical thinking preoccupation with fact projection to the future

American academy of pediatrics

18

PREP ® Self-Assessment

PREPSA 2020

Correct Answer: B Children of various ages understand death in different ways, though developmental understanding is individual and may be fluid in times of stress. It is important to consider these common developmental patterns when talking with patients and families about terminal illness or death. The preschool-aged child in the vignette would be most likely to express magical thinking when considering her father’s terminal illness and anticipated death. A focus on unfairness, preoccupation with fact, or projection to the future would not be developmentally expected responses at this age. Although infants and toddlers (4 cm) may be associated with underlying skull defects.

Item C74C: Aplasia cutis congenita appears as an atrophic scar. Reprinted with permission from Mancini AJ, Krowchuk DP, eds. Pediatric Dermatology: A Quick Reference Guide. 3rd ed. Elk Grove Village, IL: American Academy of Pediatrics; 2016:595.

Epidermal nevi are benign hamartomas that are present at birth or develop during infancy. They are often brown and have a rough surface, features that may make them difficult to differentiate from a nevus sebaceus (Item C74D). Although they become elevated over time, there is no risk of malignant transformation. Juvenile xanthogranulomas are collections of xanthomatous cells that may be present at birth or appear during infancy. Typically, a juvenile xanthogranuloma appears as a solitary orange or yellow-brown papule or nodule; multiple lesions may occur (Item C74E). They are often located on the head and neck but may occur at any location. Juvenile xanthogranulomas usually resolve spontaneously over a period of several years.

American academy of pediatrics

262

PREP ® Self-Assessment

PREPSA 2020

Item C74D: Epidermal nevi are brown verrucous papules and plaques that may be linear. Courtesy of D. Krowchuk

Item C74E: Juvenile xanthogranuloma: an orange dome-shaped papule. Reprinted with permission from Mancini AJ, Krowchuk DP, eds. Pediatric Dermatology: A Quick Reference Guide. 3rd ed. Elk Grove Village, IL: American Academy of Pediatrics; 2016:472.

PREP Pearls • A nevus sebaceus is a congenital hamartoma that usually is located on the scalp. It appears as a tan, yellow-brown, orange, or pink hairless plaque with a velvety texture. • After puberty, there is a small risk of an adnexal tumor or basal cell carcinoma developing within a nevus sebaceus. MOCA-Peds Objective • Evaluate and manage common neonatal skin disorders. ABP Content Specifications(s) • Recognize the clinical findings associated with sebaceous nevus American academy of pediatrics

263

PREP ® Self-Assessment

PREPSA 2020

Suggested Readings • Idriss MH, Elston DH. Secondary neoplasms associated with nevus sebaceus of Jadassohn: a study of 707 cases. J Am Acad Dermatol. 2014;70(2):332-337. doi:10.1016/j.jaad.2013.10.004. • Section on Dermatology, American Academy of Pediatrics. Nevus sebaceous (of Jadassohn). Mancini AJ, Krowchuk DP, eds. Pediatric Dermatology: A Quick Reference Guide. 3rd ed. Elk Grove Village, IL: American Academy of Pediatrics; 2016:593-595.

American academy of pediatrics

264

PREP ® Self-Assessment

PREPSA 2020

Question 75 A 16-year-old boy is brought to the emergency department by his parents after they had trouble awakening him this morning. They report that he has become increasingly fatigued over the past month and that he has been getting up multiple times at night to use the bathroom. His medical history is significant for autism spectrum disorder, and he was started on risperidone 3 months ago for aggressive behavior. The boy is somnolent but answers questions appropriately. His temperature is 37°C, blood pressure is 90/50 mm Hg, heart rate is 120 beats/min, respiratory rate is 18 breaths/min, and oxygen saturation is 97% in room air. His weight is 110 kg. He has acanthosis nigricans over the nape of his neck, dry mucous membranes, and a capillary refill time of 4 seconds. The remainder of his physical examination findings are unremarkable. Laboratory evaluation shows the following. Laboratory Test Plasma glucose Sodium Bicarbonate Blood urea nitrogen Creatinine Serum osmolality Venous pH Serum ketones Urine ketones

Result 1,060 mg/dL (58.27 mmol/L) 154 mg/dL (154 mmol/L) 22 mEq/L (22 mmol/L) 60 mg/dL (21 mmol/L) 1.8 mg/dL (159 μmol/L) 388 mOsm/kg 7.34 Negative Negative

Of the following, the BEST next management step for this boy is to administer intravenous A. half-normal saline with potassium at twice-maintenance rate B. insulin at 0.1 unit/kg bolus C. insulin at 0.1 units/kg per hour D. normal saline, 20 mL/kg bolus; repeat as needed to restore perfusion

American academy of pediatrics

265

PREP ® Self-Assessment

PREPSA 2020

Correct Answer: D The boy in the vignette has hyperglycemic hyperosmolar state (HHS), most likely associated with type 2 diabetes mellitus (DM) and risperidone therapy. He has severe hyperglycemia, hyperosmolality, dehydration, and mental status change without ketosis or significant acidosis. The best next management step for this boy is to administer a 20-mL/kg bolus of intravenous normal saline. The bolus should be repeated as needed to restore perfusion. Because dehydration and electrolyte loss are often profound in HHS, fluid therapy is the first step in treatment. In contrast to diabetic ketoacidosis (DKA), large volumes of fluid are usually indicated. Potassium should be replaced aggressively. Phosphate and magnesium levels should be monitored closely and deficits replaced as indicated. Half-normal saline with potassium (40 mEq/L) at twice-maintenance rate may be an appropriate fluid regimen after peripheral perfusion is restored, but would not be the best next management step. Insulin is indicated for the treatment of HHS, but at lower doses and later in the course of treatment compared with the treatment of DKA. Guidelines recommend starting a continuous infusion of insulin at 0.025 to 0.05 units/kg per hour after glucose levels stop declining by at least 50 mg/dL per hour with fluid administration alone. Insulin boluses are not recommended. Hyperglycemic hyperosmolar state is defined as: • Significant hyperglycemia: Plasma glucose concentration >600 mg/dL (33.3 mmol/L) • Lack of acidosis: Arterial pH >7.30 or venous pH >7.25, serum bicarbonate >15 mmol/L • Absent to small ketonemia, small ketonuria • Hyperosmolarity: Serum osmolality >320 mOsm/kg • Mental status changes: Obtundation, combativeness, or seizures Hyperglycemic hyperosmolar state is associated with type 2 DM as well as treatment with antipsychotic medications (eg, risperidone). The boy in the vignette has obesity and acanthosis nigricans. Both are risk factors for type 2 DM. Although rare, HHS is associated with significant risk of mortality related to the severity of dehydration. PREP Pearls • Hyperglycemic hyperosmolar state is associated with severe hyperglycemia, hyperosmolality, dehydration, and mental status change without significant ketosis or acidosis. • In contrast to diabetic ketoacidosis, large volumes of fluid are usually indicated for the treatment of hyperglycemic hyperosmolar state. • Hyperglycemic hyperosmolar state is associated with type 2 diabetes and antipsychotic medications (eg, risperidone). ABP Content Specifications(s) • Plan appropriate fluid therapy for a patient with hyperosmolar non-ketotic coma

American academy of pediatrics

266

PREP ® Self-Assessment

PREPSA 2020

Suggested Readings • Fayfman M, Pasquel FJ, Umpierrez GE. Management of hyperglycemic crises: diabetic ketoacidosis and hyperglycemic hyperosmolar state. Med Clin North Am. 2017;101(3):587-606. doi:10.1016/j.mcna.2016.12.011. • Maletkovic J, Drexler A. Diabetic ketoacidosis and hyperglycemic hyperosmolar state. Endocrinol Metab Clin North Am. 2013;42(4):677-695. doi:10.1016/j.ecl.2013.07.001. • Wolfsdorf JI, Glaser N, Agus M, et al. ISPAD Clinical practice consensus guidelines 2018: diabetic ketoacidosis and the hyperglycemic hyperosmolar state. Pediatr Diabetes. 2018;19(suppl 27):155-177. doi:10.1111/pedi.12701. • Zeitler P, Haqq A, Rosenbloom A, Glaser N; Drugs and Therapeutics Committee of the Lawson Wilkins Pediatric Endocrine Society. Hyperglycemic hyperosmolar syndrome in children: pathophysiological considerations and suggested guidelines for treatment. J Pediatr. 2011;158(1):9-14. doi:10.1016/j.jpeds.2010.09.048.

American academy of pediatrics

267

PREP ® Self-Assessment

PREPSA 2020

Question 76 A 15-year-old boy is admitted to the hospital with cough, intermittent fevers, fatigue, and an abnormal chest radiograph that has not improved after treatment with oral cephalexin for 2 weeks. He complains of a sore throat, but denies joint pain or gastrointestinal symptoms. He has had transient dark urine, but no dysuria or flank pain. He has severe nodular acne for which he has been receiving doxycycline since 2 weeks before the onset of this illness. On physical examination, the boy appears moderately ill but is able to answer questions. His oxygen saturation is 85% in room air, which rises to 95% with high-flow nasal cannula at 40 L/min and FiO2 0.5. He has diffusely decreased breath sounds, more on the right than the left, with no crackles or wheezes. Cardiovascular examination findings are normal. There is no hepatosplenomegaly and no skin rashes other than nodular facial acne. A chest radiograph is obtained (Item Q76A and Item Q76B). Shortly after admission, he has an episode of bright red hemoptysis.

American academy of pediatrics

268

PREP ® Self-Assessment

PREPSA 2020

Item Q76A: Posteroanterior chest radiograph for the boy in the vignette. Courtesy of M. Guill

Item Q76B: Lateral chest radiograph for the boy in the vignette. Courtesy of M. Guill

American academy of pediatrics

269

PREP ® Self-Assessment

PREPSA 2020

Laboratory evaluation demonstrates the following. Laboratory Test Result Hemoglobin 7.5 g/dL (75 g/L) Hematocrit 22.3% Platelets 759,000/μL (759 × 109/L) Creatinine 1.09 mg/dL (96 μmol/L) Of the following, the MOST appropriate next step in evaluation for this boy’s hemoptysis is A. 24-hour urine collection for protein and creatinine analysis B. quantitative serum immunoglobulin levels C. renal ultrasonography D. serum antineutrophil cytoplasmic antibodies

American academy of pediatrics

270

PREP ® Self-Assessment

PREPSA 2020

Correct Answer: D The combination of hemoptysis with multifocal densities on chest radiography, urinary findings (dark urine and borderline elevated creatinine), anemia, and thrombocytosis suggests a systemic vasculitis such as granulomatosis with polyangiitis (previously called Wegener syndrome). Measurement of serum antineutrophil cytoplasmic antibodies (ANCA) would be the appropriate diagnostic test. The differential diagnosis for this boy would also include systemic lupus erythematosus, Goodpasture syndrome, and Henoch-Schonlein purpura. Infection and bronchiectasis are the most likely causes of hemoptysis in children and adults, but are not usually associated with findings of renal involvement, as seen in this child. Further evaluation of this boy’s renal function will be necessary, but neither measurement of 24hour urine protein and creatinine levels nor renal ultrasonography will lead to a specific diagnosis and would not help to explain his hemoptysis. He is not presenting with recurrent respiratory infections or radiographic signs of bronchiectasis, therefore, serum immunoglobulins, although potentially elevated, will not assist with the diagnosis. His findings suggest a multiorgan process, for which assessment of systemic inflammation is appropriate. Other studies that would be helpful in establishing a systemic inflammatory process and evaluating for other causes of hemoptysis include erythrocyte sedimentation rate, C-reactive protein, antinuclear antibody, serum myeloperoxidase, and antiproteinase-3 antibody levels. PREP Pearls • Infection with or without bronchiectasis is the most common cause of hemoptysis in children. • When hemoptysis is associated with multisystemic findings, a diffuse inflammatory process should be the primary consideration. ABP Content Specifications(s) • Plan the appropriate clinical and diagnostic evaluation of hemoptysis • Plan the appropriate management of hemoptysis in patients of various ages Suggested Readings • de Silva C, Mukherjee A, Jat KR, Lodha R, Kabra SK. Pulmonary hemorrhage in children: etiology, clinical profile and outcome. Indian J Pediatr. 2019;86(1):7-11. doi:10.1007/s12098-018-2725-x. • Greco A, Marinelli C, Fusconi M, et al. Clinic manifestations in granulomatosis with polyangiitis. Int J Immunopathol Pharmacol. 2016;29(2):151-159. doi:10.1177/0394632015617063. • Lutalo PM, D’Cruz DP. Diagnosis and classification of granulomatosis with polyangiitis (aka Wegener’s granulomatosis). J Autoimmun. 2014;2:48-49, 94-98. doi:10.1016/j.jaut.2014.01.028. • Shnayder R, Needleman JP. Hemoptysis. Pediatr Rev. 2018;39(6):319-321. doi:10.1542/pir.2017-0157. • Simon DR, Aronoff SC, DelVecchio MT. Etiologies of hemoptysis in children: a systematic review of 171 patients. Pediatr Pulmonol. 2017;52(2):255-259. doi:10.1002/ppul.23497. American academy of pediatrics

271

PREP ® Self-Assessment

PREPSA 2020

Question 77 A 12-year-old girl and her 15-year-old brother are seen for evaluation after several days of fever, headache, malaise, and bilateral swelling of the cheek and jaw area. The siblings are unvaccinated. The girl now has neck stiffness and demonstrates meningeal signs without other neurologic findings. Her brother reports testicular pain. He has unilateral testicular swelling and tenderness. Of the following, the BEST management for both siblings’ current concerns is A. antibiotics B. antiviral therapy C. immune globulin D. supportive care

American academy of pediatrics

272

PREP ® Self-Assessment

PREPSA 2020

Correct Answer: D The siblings in this vignette have clinical features consistent with mumps infection and its associated complications. Therefore, the best management for both siblings is supportive care. Antibiotics are not indicated because there is no evidence of a bacterial infection. Antiviral therapy specific for mumps is not available. Immune globulin therapy is not effective in the management of mumps. Mumps rubulavirus is an RNA virus that exclusively infects humans and is transmitted by respiratory tract secretions. The incubation period is typically 16 to 18 days but may be 12 to 25 days. Most patients experience a prodrome of fever, headache, and malaise lasting 1 to 2 days before parotitis occurs. Parotitis usually worsens over 3 days and resolves after 7 to 10 days. Bilateral parotitis occurs in 70% of patients. Parotitis may be accompanied by ipsilateral otalgia. Although parotitis is the most classic symptom of mumps infection, one-third of infected patients may not experience parotitis or may only have symptoms of a nonspecific upper respiratory tract infection. Patients infected with mumps are contagious for up to a week prior to the onset of parotitis and should be isolated for 5 days after the onset of parotitis, although viral shedding may be detected up to 2 weeks after the onset. Complications from mumps infection can affect multiple organ systems, are more likely to occur in infected adults than in children, and are usually short lived and self-resolving. Orchitis mostly affects postpubertal males aged 15 to 29 years, may be unilateral or bilateral, and may last for several weeks. Sterility is a rare complication. Viral meningitis affects less than 10% of individuals infected with mumps and usually manifests 5 days after the onset of parotitis and resolves after 3 to 10 days. Less common complications include glomerulonephritis, pancreatitis, thyroiditis, and hearing loss. Birth defects resulting from maternal infection with mumps have not been reported. Mumps is a vaccine-preventable disease with a 2-dose regimen that is 88% effective. In the United States, outbreaks often occur in vaccinated populations that reside in close quarters. In the event of an outbreak of mumps infection, local public health officials should be notified, although it is not mandatory. Vaccination after exposure to mumps virus is not effective in preventing or ameliorating mumps infection but should still be administered to protect against future exposures. After exposure, unvaccinated children 12 months of age and older should be given 1 dose, and children who have already received 1 dose of the vaccine at least 28 days prior should receive a second dose. High-risk individuals who have already received the recommended 2 doses of mumps vaccine should receive a third dose. Infants 6 to 11 months of age may also be vaccinated, although this dose would not count towards the routine vaccination schedule. Immune globulin is not effective for postexposure prophylaxis. The differential diagnosis of mumps infection includes other viruses that can cause parotitis, including Epstein-Barr virus, influenza A virus, cytomegalovirus, parainfluenza virus, and HIV. Bacterial infection from Staphylococcus aureus may also cause parotitis, but it is usually unilateral. Sjögren syndrome, lupus, and obstruction of the Stensen duct are noninfectious etiologies of parotitis.

American academy of pediatrics

273

PREP ® Self-Assessment

PREPSA 2020

PREP Pearls • Supportive care is the best management strategy for children infected with mumps, including children experiencing complications from mumps infection. • Complications from mumps infection, such as orchitis and meningitis, rarely result in long-term or permanent morbidity. • Response to outbreaks should be managed in conjunction with local public health officials and include vaccination but not administration of immune globulin. ABP Content Specifications(s) • Recognize the clinical features and complications associated with mumps • Plan the appropriate management of a patient with mumps Suggested Readings • American Academy of Pediatrics. Mumps. In: Kimberlin DW, Brady MT, Jackson MA, Long SS, eds. Red Book: 2018 Report of the Committee on Infectious Diseases. 31st ed. Itasca, IL: American Academy of Pediatrics; 2018:567573. https://redbook.solutions.aap.org/chapter.aspx?sectionId=189640136&bookId=2205 &resultClick=1. • Centers for Disease Control and Prevention. Mumps. In: Hamborsky J, Kroger A, Wolfe S, eds. Epidemiology and Prevention of Vaccine-Preventable Diseases. 13th ed. Washington DC: Public Health Foundation; 2015:247-260. • Mason WH, Gans HA. Mumps. In: Kliegman RM, St Geme JW III, Blum NJ, Shah SS, Tasker RC, Wilson KM, eds. Nelson Textbook of Pediatrics. 21st ed. Philadelphia, PA: Elsevier; 2020:1680-1683. • Murray DL, Vodzak J, Cheng Immergluck L. Contagious exanthematous diseases. In: McInerny TK, Adam HM, Campbell DE, DeWitt TG, Foy JM, Kamat DM, eds. American Academy of Pediatrics Textbook of Pediatric Care. 2nd ed. Elk Grove Village, IL: American Academy of Pediatrics; 2017:1920-1927. Pediatric Care Online.

American academy of pediatrics

274

PREP ® Self-Assessment

PREPSA 2020

Question 78 A 4-year-old developmentally normal girl is brought to the emergency department because of an abnormal gait. Her mother reports that she woke that morning with the inability to walk without holding on to something. When sitting, she cannot hold herself up without bobbing her head. Her mother also reports that her speech seems slower than usual and that she has had no fever, headache, sensory changes, vomiting, rash, or weakness. The mother recalls that her daughter had a mild viral upper respiratory infection 1 week ago, from which she has completely recovered. The patient is alert and cooperative. The general physical examination findings and vital signs are normal. Neurological examination reveals scanning speech. Cranial nerve examination findings are normal, with no papilledema and mild end-gaze nystagmus bilaterally. Her strength is normal, with normal deep tendon reflexes and toes downgoing to plantar stimulation. Sensory examination findings are normal. When encouraged to sit up, she exhibits truncal instability with titubation. She has mild dysmetria when reaching for toys bilaterally. She walks holding her mother’s hand with a wide-based unsteady gait. Magnetic resonance imaging of the brain with and without contrast has normal findings. Lumbar puncture is performed with normal opening pressure. Cerebrospinal fluid analysis reveals a lymphocytic pleocytosis with normal protein and glucose levels. Cultures of the cerebrospinal fluid are negative. Of the following, the MOST likely diagnosis for this child is A. accidental drug ingestion B. acute cerebellar ataxia C. acute disseminated encephalomyelitis D. postinfectious polyradiculopathy

American academy of pediatrics

275

PREP ® Self-Assessment

PREPSA 2020

Correct Answer: B The clinical presentation and diagnostic evaluation for the girl in this vignette are most consistent with acute cerebellar ataxia. Her examination findings are notable for features consistent with ataxia: slow scanning speech, intention tremor, titubation (head bobbing) while sitting, and a wide-based, unbalanced ataxic gait. In the acute setting, a cerebellar lesion must be excluded because tumors, demyelination, stroke, infection, and hemorrhage can present with identical signs and symptoms, necessitating a prompt targeted diagnostic evaluation to exclude these causes. Normal magnetic resonance imaging findings with mild inflammatory changes in the cerebrospinal fluid analysis exclude a majority of these diagnostic considerations and in the setting of the described clinical presentation are supportive of a postinfectious acute cerebellar ataxia. Acute cerebellar ataxia is a postinfectious condition commonly affecting young children between the ages of 2 and 5 years. It is relatively uncommon in older children and adolescents. Classically, acute cerebellar ataxia was associated with varicella infection but now a wide range of viral and bacterial causes have been associated with this condition, including Epstein-Barr virus, influenza, echoviruses, mumps, coxsackie B, and Mycoplasma. An immune-inflammatory process leading to acute demyelination from antibodies cross-reacting to cerebellar epitopes is proposed as the likely pathology. In selected cases, these antibodies may be isolated, and reports have included identification of antineuronal antibodies, anticentriolar antibodies, and glutamic acid decarboxylase antibodies. Most children will present with a viral prodrome in the weeks preceding the development of unsteady gait, truncal instability, nystagmus, and ataxia, which typically evolves over hours to days. The ataxia is predominately truncal rather than extremity ataxia, although often a subtle intention tremor is found on finger-to-nose testing. The diagnostic evaluation of acute cerebellar ataxia is targeted toward exclusion of other conditions causing acute ataxia and is shaped by the clinical history and examination findings. In children presenting with acute onset of ataxia with papilledema, headache, and vomiting, an obstructive process or mass lesion leading to increased intracranial pressure (ICP) must be excluded, necessitating urgent imaging. In patients with no viral prodrome but with accompanying mental status changes, accidental drug ingestion should be considered and, if accompanied by fever, encephalitis should be considered. The Miller Fisher variant of GuillainBarré syndrome, which includes ataxia, ophthalmoplegia, and areflexia, also often follows a viral prodrome and requires a high index of clinical suspicion. Inflammatory processes such as acutedisseminated encephalomyelitis typically present with encephalopathy and, depending on the extent of the inflammatory changes, additional upper motor neuron signs and symptoms including seizures. Stroke remains uncommon in children and should be considered if presentation is asymmetric and accompanied by other brainstem findings. Finally, presence of opsoclonus or myoclonus in association with ataxia, or failure to improve over time, should raise concern for paraneoplastic opsoclonus-myoclonus-ataxia syndrome, warranting further targeted diagnostic studies for neuroblastoma. In acute cerebellar ataxia, magnetic resonance imaging of the brain has normal findings. Lumbar puncture, if performed, can demonstrate a mild pleocytosis with a lymphocytic predominance but American academy of pediatrics

276

PREP ® Self-Assessment

PREPSA 2020

can also have normal findings. Treatment is supportive, aimed at safe ambulation, hydration, and, in severe cases, medical and surgical management of increased ICP in the setting of significant cerebellar swelling. Prognosis is favorable, with over 90% of children having full recovery within a few weeks to months after initial onset. A closely related condition, acute cerebellitis, shares many clinical features with acute postinfectious cerebellar ataxia but with a more severe clinical course. The key distinguishing characteristic at presentation is encephalopathy, which is present in acute cerebellitis but absent in acute cerebellar ataxia. Children with acute cerebellitis are at risk for potentially significant cerebellar edema and often have clinical signs and symptoms of increased ICP including nuchal rigidity, vomiting, papilledema, and severe headache. Caution should be exercised in performing lumbar puncture if cerebellar edema is present on examination. Close neurological monitoring for decline is warranted to allow prompt recognition and treatment of progressive cerebellar swelling. PREP Pearls • Acute cerebellar ataxia is a self-limited, postinfectious condition in young children that presents with acute onset of ataxia, nystagmus, and unsteady gait. • Diagnostic evaluation of acute cerebellar ataxia is aimed at exclusion of alternate diagnoses and can include toxicology screens, neuroimaging, and cerebrospinal fluid analysis. • Treatment of acute cerebellar ataxia is supportive with a favorable prognosis of recovery over weeks to months in the majority of children. ABP Content Specifications(s) • Plan the appropriate evaluation of ataxia • Recognize the presentation of acute ataxia Suggested Readings • Desai J, Mitchell WG. Acute cerebellar ataxia, acute cerebellitis, and opsoclonusmyoclonus syndrome. J Child Neurol. 2012;27(11):1482-1488. doi:10.1177/0883073812450318. • Fogel B. Childhood cerebellar ataxia. J Child Neurol. 2012;27(9):1138-1145. doi:10.1177/0883073812448231. • Overby P. Ataxia. In: McInerny TK, Adam HM, Campbell DE, DeWitt TG, Foy JM, Kamat DM, eds. American Academy of Pediatrics Textbook of Pediatric Care. 2nd ed. Elk Grove Village, IL: American Academy of Pediatrics; 2017:1217-1221. Pediatric Care Online. • Sivaswamy L. Approach to acute ataxia in childhood: diagnosis and evaluation. Pediatric Annals. 2014;43(4):153-159. doi:10.3928/00904481-20140325-13.

American academy of pediatrics

277

PREP ® Self-Assessment

PREPSA 2020

Question 79 A 16-year-old adolescent boy is brought to the emergency department by his sister after she found him acting strange when she came home from work that evening. She reports that he has no significant past medical or psychiatric history and no history of school or social problems and that he was in his normal state of health that morning. He is currently agitated and talking to himself, although he intermittently answers questions and agrees to be examined. He is wellkempt and oriented to name and place, although not to time. When asked if anything bothers him, he reports nausea and chest pain. He has a temperature of 37.9°C, heart rate of 148 beats/min, respiratory rate of 26 breaths/min, blood pressure of 168/100 mm Hg, and oxygen saturation of 100% on room air. His pupils are mydriatic but reactive, his oropharynx is clear, and his neck is supple. Auscultation reveals tachycardia without abnormal heart sounds and clear lungs. His abdomen is soft with hyperactive bowel sounds. He is diaphoretic and slightly tremulous with a capillary refill time of 2 seconds. He has a nonfocal neurological examination. Of the following, the evaluation that is MOST likely to lead to a diagnosis for this patient is A. blood culture B. comprehensive psychiatric assessment C. head computed tomography D. urine drug screen

American academy of pediatrics

278

PREP ® Self-Assessment

PREPSA 2020

Correct Answer: D The adolescent in this vignette presents after a cocaine ingestion with signs of a classic sympathomimetic toxidrome: agitation, hallucinations, tachycardia, tachypnea, hypertension, mydriasis, diaphoresis, and tremulousness. Severe adverse outcomes from cocaine ingestion include cardiac dysrhythmias, myocardial ischemia, seizures, coma, stroke, and rhabdomyolysis. Behavioral consequences from cocaine use include agitation, hallucinations, aggression, euphoria, and feelings of increased confidence. Ultimately, a combination of these behavioral changes can result in increased risk-seeking behaviors, both during acute intoxication and during attempts to obtain cocaine for future use. Long-term behavioral changes include irritability and mood disturbances even while not acutely intoxicated. Derived from the leaves of the plant Erythroxylon coca, cocaine has been used for medicinal purposes and abused for recreational use for over 150 years. Cocaine is one of the most abused illicit drugs and its use has permeated pop culture for decades. Common street names for cocaine include coke, blow, powder, snow white, flake, and yeyo. Cocaine can be snorted, ingested orally, rubbed on the oral mucosa, injected intravenously, and even inhaled after the powder has been crystalized in a process commonly known as freebasing. Each of these methods entails their own set of risks. Inhaling cocaine causes a more immediate high because it can cross the blood-brain barrier. It also entails a higher risk of pulmonary hemorrhage, pneumothorax, and other pulmonary complications. Oral ingestion of cocaine entails higher risks of gastrointestinal bleeding and hemorrhage. Cocaine can be used in conjunction with heroin (speedballing) or can be used to lace marijuana (chronic). As with other intoxications, patients, especially adolescents, may not be forthcoming about their drug use. Presentation with a classic sympathomimetic toxidrome is suggestive of cocaine intoxication although not confirmatory. Other drugs that may present with a sympathomimetic toxidrome include amphetamines, pseudoephedrine, theophylline, and even caffeine. A urine toxicology analysis can determine if cocaine was ingested by detecting the metabolite benzoylecgonine. Urine toxicology assays will remain positive for 2 to 4 days after ingestion of the drug. Certain illicit drugs, such as synthetic cannabinoids and synthetic opiates (eg, fentanyl) will not show up on a urine drug screen. Treatment of acute cocaine toxicity is mainly supportive. As with all emergencies, evaluation of an intoxicated patient begins with an assessment of the airway, breathing, and circulation (ABCs). Benzodiazepines are the treatment of choice for cocaine-induced agitation and seizures and will also facilitate lowering of the body temperature. Hyperthermic patients should be aggressively cooled as a majority of immediate complications are related to hyperthermia. Treatment of a cocaine-induced hypertensive emergency necessitates avoidance of β-blockers because resultant unopposed α-stimulation can further worsen the hypertension. Instead, benzodiazepines should be the first line of treatment with addition of vasodilators, such as nitroglycerin or nitroprusside, if needed. Phentolamine, an α-adrenergic blocker, may be useful as well. Reports of chest pain warrant electrocardiography and chest radiography. A basic laboratory evaluation is performed to evaluate possible electrolyte disturbances and the patient's coagulation status. A urinalysis positive for blood but without red blood cells on the microscopic American academy of pediatrics

279

PREP ® Self-Assessment

PREPSA 2020

analysis and an elevated creatinine kinase level are suggestive of rhabdomyolysis and would indicate the need for aggressive intravenous fluid administration. Blood cultures would be helpful in diagnosing sepsis and suspected infection. A patient with sepsis is usually tachycardic and hypotensive, not hypertensive. A comprehensive psychiatric evaluation is indicated if no organic reason for the patient’s symptoms was found after a complete evaluation. Computed tomography of the head would be helpful in discerning if there is a space-occupying lesion or intracranial hemorrhage. Presence of a headache, unilateral mydriasis (a “blown pupil”), the Cushing triad (hypertension, bradycardia, irregular respirations), or a focal neurological examination would increase the likelihood of identifying a diagnosis with computed tomography of the head. PREP Pearls • Cocaine intoxications present with a sympathomimetic toxidrome. • Supportive care and benzodiazepines are the mainstays of treatment for an acute cocaine intoxication. • Cocaine use can cause both short-term and long-term behavioral changes. •

ABP Content Specifications(s) • Recognize the clinical findings associated with acute cocaine intoxication, and manage appropriately • Recognize the major behavioral consequences of cocaine use/abuse • Identify the major physiologic consequences associated with opioid use/abuse, including those associated with the various means of administration Suggested Readings • Fine JS. Poisoning. In: McInerny TK, Adam HM, Campbell DE, DeWitt TG, Foy JM, Kamat DM, eds. American Academy of Pediatrics Textbook of Pediatric Care. 2nd ed. Elk Grove Village, IL: American Academy of Pediatrics; 2017:29242950. https://pediatriccare.solutions.aap.org/chapter.aspx?sectionId=125490789&bookId =1626&resultClick=1. • Rizkalla C, Sue YJ. Cocaine. Pediatr Rev. 2006;27(11):436-438. doi:10.1542/pir.27-11436. • Smollin CG, Hoffman RS. Cocaine. In: Nelson LS, Howland MA, Lewin NA, Smith SW, Goldfrank LR, Hoffman RS, eds. Goldfrank’s Toxicologic Emergencies. 11th ed. New York, NY: McGraw Hill; 2019:1124-1134.

American academy of pediatrics

280

PREP ® Self-Assessment

PREPSA 2020

Question 80 A 4-year-old boy with fever is brought to the emergency department. His symptoms began 3 days ago, on the last day of a family trip to Pakistan. He has had daily fever to 39.4°C, anorexia, and abdominal tenderness. He stayed with relatives in an urban area during the trip. He exclusively drank bottled water and mostly dined at home, although he had a few meals at restaurants. He has a temperature of 39°C, blood pressure of 98/65 mm Hg, heart rate of 120 beats/min, respiratory rate of 24 breaths/min, and oxygen saturation of 99% on room air. He appears ill and has mild diffuse abdominal tenderness with deep palpation but no rebound or guarding. Laboratory data are shown: Laboratory Test White blood cell count Hemoglobin Platelet count Aspartate aminotransferase Alanine aminotransferase

Result 3,500/µL (3.5 × 109/L) 9.8 g/dL (98 g/L) 115 × 103/µL (115 × 109/L) 117 U/L 135 U/L

Gram stain of the blood culture is shown (Item Q80).

Item Q80: Gram stain from the blood culture for the boy described in the vignette. Courtesy of R. Yee and reprinted with permission from Kimberlin DW, et al, eds. Red Book Online. Itasca, IL: American Academy of Pediatrics; 2018.

American academy of pediatrics

281

PREP ® Self-Assessment

PREPSA 2020

Of the following, the MOST likely source of this infection is A. B. C. D.

humans poultry reptiles rodents

American academy of pediatrics

282

PREP ® Self-Assessment

PREPSA 2020

Correct Answer: A For the boy in the vignette the most likely source of infection is humans. The boy has gramnegative bacteremia after travel abroad, which is most concerning for typhoid fever. Typhoid fever is endemic in many resource-limited countries. Most cases in the United States occur after travel to Pakistan, India, Mexico, and Bangladesh. Humans are the source of enteric fever infections, including infections caused by Salmonella enterica serovars Typhi, Paratyphi A, and Paratyphi B. Nontyphoidal Salmonella infections typically manifest as gastroenteritis. Diarrhea is typically nonbloody. Bacteremia can occur and lead to focal infections such as osteomyelitis and meningitis. Invasive disease caused by nontyphoidal Salmonella species is typically observed in infants younger than 3 months or susceptible hosts, such as individuals with sickle cell disease or other immunocompromising conditions. Infants younger than 3 months with suspected or known Salmonella gastroenteritis should have a blood culture performed to evaluate for bacteremia. Young children can have prolonged excretion of nontyphoidal organisms after infection. Typhoidal Salmonella infections are characterized by fever, constitutional symptoms, and abdominal pain. Patients can have diarrhea or constipation. Physical examination findings can include hepatosplenomegaly, jaundice, and rose spots, a macular exanthem. Laboratory abnormalities can include leukopenia, anemia, thrombocytopenia, elevated transaminase levels, and hyperbilirubinemia. Treatment of nontyphoidal gastroenteritis is generally not recommended because it does not shorten the duration of illness and can prolong the duration of shedding. However, it is recommended that individuals at risk of invasive infections (as previously described) be treated. Initial treatment of invasive nontyphoidal Salmonella infections is ceftriaxone. Antibiotic susceptibilities can be used to change to narrow-spectrum antibiotics. Uncomplicated bacteremia can be treated for a total of 7 to 10 days with transition to oral therapy after negative blood cultures and exclusion of focal infection. The duration of therapy for meningitis is typically 4 weeks and for osteomyelitis is 4 to 6 weeks. Typhoidal Salmonella organisms can have variable antibiotic susceptibility patterns depending on the country or region where they were acquired. Ceftriaxone is usually the first-line parenteral antibiotic. Duration of therapy depends on the site of infection and clinical response. Protection against Salmonella enterica serovar Typhi is provided by 2 available vaccines and should be considered prior to international travel. General recommendations can be found at the Centers for Disease Control and Prevention website (wwwnc.cdc.gov/travel) or in consultation with a travel clinic. There are approximately 1 million foodborne nontyphoidal Salmonella infections annually in the United States. Animals are the main reservoir of nontyphoidal Salmonella; poultry, reptiles, and rodents can be colonized. Although many foods have been associated with outbreaks of Salmonella infection in the United States, chicken and eggs are most commonly implicated. Implicated foods not of animal origin have been contaminated by an animal product or human carrier. American academy of pediatrics

283

PREP ® Self-Assessment

PREPSA 2020

PREP Pearls • Typhoid fever must be considered in the differential diagnosis of a febrile illness in any traveler returning from a resource-limited country. • Nontyphoidal Salmonella infections typically manifest as gastroenteritis. Typhoidal Salmonella infections are characterized by fever, constitutional symptoms, and abdominal pain. • Ceftriaxone is usually the first-line parenteral antibiotic used in the treatment of invasive Salmonella infections. MOCA-Peds Objective • Provide appropriate counseling regarding prevention of infectious diseases prior to international travel. ABP Content Specifications(s) • Understand the epidemiology of typhoidal and nontyphoidal Salmonella species • Plan appropriate management for a patient with typhoidal or nontyphoidal Salmonella infection • Recognize the clinical features associated with typhoidal and nontyphoidal Salmonella infection Suggested Readings • American Academy of Pediatrics. Salmonella Infections. In: Kimberlin DW, Brady MT, Jackson MA, Long SS, eds. Red Book: 2018 Report of the Committee on Infectious Diseases. 31st ed. Itasca, IL: American Academy of Pediatrics; 2018:711718. https://redbook.solutions.aap.org/chapter.aspx?sectionId=189640178&bookId=2205 &resultClick=24. • Centers for Disease Control and Prevention. Travelers’ Health. wwwnc.cdc.gov/travel. • Christenson J. Salmonella infections. Pediatr Rev. 2013;34(9):375-383. doi:10.1542/pir.34-9-375.

American academy of pediatrics

284

PREP ® Self-Assessment

PREPSA 2020

Question 81 A 14-year-old adolescent boy is seen for a health supervision visit. He received a kidney transplant from his father 1 year ago for end-stage renal disease resulting from focal segmental glomerulosclerosis. His medications include tacrolimus, mycophenolate mofetil, prednisone, and amlodipine. He reports good adherence to the medication regimen. His weight is 72 kg (95th percentile), and his height is 154 cm (10th percentile). His vitals signs are normal, and the remainder of his physical examination findings are unremarkable. The family inquires about the long-term medical management for their child's care.

Of the following, the family is MOST likely to be informed that A. B. C. D.

immunosuppressive medication will be weaned and discontinued lifestyle changes do not affect the long-term kidney graft survival recurrence of primary renal disease needs to be monitored risk of rejection is similar to a deceased-donor transplant

American academy of pediatrics

285

PREP ® Self-Assessment

PREPSA 2020

Correct Answer: C Kidney transplantation is the optimal treatment for children with end-stage renal disease. The survival rates are better for children who receive a kidney transplant as compared to children on long-term dialysis. Kidney transplantation also leads to improved linear growth in children. After kidney transplant, results of school function tests improve and children experience an overall better quality of life. The common diagnoses in children undergoing a kidney transplant are congenital disorders of the kidney (renal dysplasia or hypoplasia), obstructive uropathy, focal segmental glomerulosclerosis, reflux nephropathy, and other disorders (chronic glomerulonephritis, polycystic kidney disease). Focal segmental glomerulosclerosis, membranoproliferative glomerulonephritis, and atypical hemolytic uremic syndrome can have recurrence in the transplanted kidney and affect long-term graft survival. In children, almost half of all kidney transplants are from living donors. Living-donor kidneys are most commonly transplanted from a parent, resulting in better HLA matching. The cold ischemia time in living-donor transplantation is shorter as compared to deceased-donor transplantation, leading to decreased ischemia-reperfusion injury. The rejection rates of livingdonor kidney transplantation are lower as compared to deceased-donor transplant. As a result of these differences, survival rates of living-donor kidney transplantation grafts are higher as compared to deceased-donor graft survival rates. Children with kidney transplantation receive a combination of immunosuppressive medications to minimize the toxicity of any single drug and at the same time provide optimal immunosuppression. Most drug regimens consist of a calcineurin inhibitor (tacrolimus), an antiproliferative agent (mycophenolate mofetil), and corticosteroids (prednisone). Immunosuppressive medications are continued as long as the kidney graft is functioning, although the dosage and number of medications required to prevent graft rejection varies from patient to patient. Medication adherence should be monitored frequently for all children, especially adolescents. Immunosuppression increases the risk of infection and posttransplant lymphoproliferative disorders. To prevent infections, antifungal prophylaxis (nystatin, fluconazole), antiviral prophylaxis (ganciclovir, valganciclovir), and antibacterial prophylaxis (trimethoprimsulfamethoxazole) are usually continued for 6 months after kidney transplantation. Hypertension is a common concern in the posttransplant period and is related to medications (corticosteroids, calcineurin inhibitor) or excessive weight gain. In addition to antihypertensive medication, lifestyle changes are recommended to decrease cardiovascular risk factors and improve long-term kidney graft survival. The adolescent boy in this vignette has a diagnosis of focal segmental glomerulosclerosis and needs monitoring for recurrence in the transplanted kidney. His current body mass index is high (30 kg/m2), and lifestyle modification would help improve the long-term kidney graft survival. His risk of graft rejection is lower because he received a living-donor transplant. However, he needs to continue immunosuppressive therapy and demonstrate good adherence to medications to minimize the risk of acute rejection. American academy of pediatrics

286

PREP ® Self-Assessment

PREPSA 2020

PREP Pearls • Children with chronic renal failure have increased linear growth and better quality of life after kidney transplant. • Long-term kidney graft survival depends on primary renal disease, type of transplant, acute rejection episodes, cardiovascular risk factors, and medication adherence. ABP Content Specifications(s) • Understand the prognosis for a patient who has undergone renal transplantation Suggested Readings • Hooper DK, Varnell CD Jr. Renal transplantation. In: Kliegman RM, St Geme JW III, Blum NJ, Shah SS, Tasker RC, Wilson KM, eds. Nelson Textbook of Pediatrics. 21st ed. Philadelphia, PA: Elsevier; 2020:2779-2785. • Van Arendonk KJ, Boyarsky BJ, Orandi BJ, et al. National trends over 25 years in pediatric kidney transplant outcomes. Pediatrics.2014;133(4):594-601. doi:10.1542/peds.2013-2775.

American academy of pediatrics

287

PREP ® Self-Assessment

PREPSA 2020

Question 82 A 12-year-old boy is brought to the emergency department for evaluation of 6 hours of worsening abdominal pain that began after eating a breakfast of bacon and eggs. The pain is described as sharp and is present in the epigastrium and right upper quadrant. About 4 hours ago, he had nonbilious nonbloody emesis. There has been no fever, diarrhea, or hematochezia. He is otherwise healthy. He has no surgical history and reports no medication use (including over-thecounter drugs and dietary supplements). He has a weight of 80 kg (> 99th percentile for age), height of 150 cm (50th percentile for age), and body mass index greater than the 99th percentile for age. He is afebrile and has a heart rate of 120 beats/min, respiratory rate of 16 breaths/min, and blood pressure of 122/70 mm Hg. He is uncomfortable. His sclera are icteric, his abdomen is mildly distended, there is voluntary guarding, and his right upper quadrant is tender to palpation. Bowel sounds are not appreciated. Laboratory data are shown: Laboratory Test White blood cell count Hemoglobin Platelet count Sodium Potassium Chloride Carbon dioxide Alkaline phosphatase Bilirubin Alanine aminotransferase Aspartate aminotransferase Lipase

Result 12,500/µL (12.5 × 109/L) 13.6 g/dL (136 g/L) 238 × 103/µL (238 × 109/L) 140 mEq/L (140 mmol/L) 3.7 mEq/L (3.7 mmol/L) 100 mEq/L (100 mmol/L) 26 mEq/L (26 mmol/L) 600 U/L 5.3 mg/dL (90.1 µmol/L) 230 U/L 216 U/L > 10,000 U/L

Of the following, the test MOST likely to establish the diagnosis in this boy is A. abdominal radiography B. abdominal ultrasonography C. upper endoscopy D. upper gastrointestinal series

American academy of pediatrics

288

PREP ® Self-Assessment

PREPSA 2020

Correct Answer: B The boy in this vignette has acute abdominal pain due to choledocholithiasis (common bile duct stone) and resulting pancreatitis. Signs and symptoms of choledocholithiasis can include jaundice, right upper quadrant pain and tenderness, and fever. Ultrasonography is the initial test of choice to visualize gallstones and to evaluate for biliary tract dilation. Pancreatitis can occur as a result of an obstructing common bile duct stone. Signs and symptoms of pancreatitis include epigastric abdominal pain and tenderness and vomiting in the context of hyperlipasemia. The differential diagnosis of acute abdominal pain in children is broad and ranges from benign functional abdominal pain to surgical emergencies. Initial evaluation of acute abdominal pain should start with a thorough history that includes: • Location and duration of the pain • Quality of the pain • Triggering factors (ie, movement) • Other signs and symptoms including stool pattern and appearance of emesis (if present) The location of the pain (Item C82A), the color of the emesis (Item C82B), and the appearance of stool (Item C82C) can narrow the differential diagnosis.

Item C82B: Differential Diagnosis Based on the Color of Emesis. Reprinted with permission from Ross A, LeLeiko NS. Acute abdominal pain. Pediatr Rev. 2010;31(4):137.

American academy of pediatrics

289

PREP ® Self-Assessment

American academy of pediatrics

PREPSA 2020

290

PREP ® Self-Assessment

PREPSA 2020

Item C82A: Differential Diagnosis Based on Location of Abdominal Pain. Reprinted with permission from Ross A, LeLeiko NS. Acute abdominal pain. Pediatr Rev. 2010;31(4):136.

Item C82C: Differential Diagnosis Based on Stool Appearance. Reprinted with permission from Ross A, LeLeiko NS. Acute abdominal pain. Pediatr Rev. 2010;31(4):137. The physical examination should include assessment of the child’s overall appearance (ie, toxic or nontoxic). The presence of distention, tenderness, guarding, or rebound tenderness should prompt consideration for a surgical emergency. Laboratory evaluation may include complete blood cell count and differential, complete metabolic panel, ɣ-glutamyl transferase, erythrocyte sedimentation rate, C reactive protein, amylase, and lipase. Urine testing may include a urine pregnancy test (in adolescent girls) and urinalysis. Radiographic evaluation should be considered and tailored to the data accumulated from the history, physical examination, and laboratory evaluation. American academy of pediatrics

291

PREP ® Self-Assessment

PREPSA 2020

Causes of abdominal surgical emergencies in infants include necrotizing enterocolitis (most commonly seen in preterm infants), pyloric stenosis, malrotation with volvulus, intussusception, and incarcerated hernias. For children between 1 and 5 years of age, causes of abdominal surgical emergencies include: appendicitis (commonly perforated in this age group), intussusception, and torsed Meckel diverticulum. Appendicitis and ovarian or testicular torsion are causes of abdominal surgical emergencies more commonly seen in older children. Small bowel obstruction from other causes (eg, previous abdominal surgery or abdominal tumors) can occur at any age. The boy in this vignette has jaundice and epigastric and right upper quadrant pain, and his laboratory data is concerning for pancreatitis and biliary tract obstruction. Abdominal radiography would be a good first test if the history was concerning for a small bowel obstruction but will not generally be helpful in identifying causes of cholestasis or pancreatitis. Upper endoscopy would be useful if there is concern for peptic ulcer disease, but it is not helpful to understand causes of pancreatitis. Finally, an upper gastrointestinal series is not diagnostic for causes of pancreatitis or biliary tract disease. PREP Pearls • The evaluation of acute abdominal pain should include a thorough history and physical examination, focusing on location and quality of pain, triggering symptoms, appearance of emesis (if present), and stool pattern. • Causes of surgical abdominal emergencies in infants include necrotizing enterocolitis, pyloric stenosis, intussusception, malrotation with midgut volvulus, and incarcerated hernias. ABP Content Specifications(s) • Plan the appropriate evaluation of acute abdominal pain • Recognize the significance of dyspepsia in a child with recurrent abdominal pain • Formulate an age-appropriate differential diagnosis of acute abdominal pain Suggested Readings • Loizides AM, Atienza Orellana K, Thompson JF. Abdominal pain. In: McInerny TK, Adam HM, Campbell DE, DeWitt TG, Foy JM, Kamat DM, eds. American Academy of Pediatrics Textbook of Pediatric Care. 2nd ed. Elk Grove Village, IL: American Academy of Pediatrics; 2017:1181-1188. Pediatric Care Online. • McCollough M, Sharieff GQ. Abdominal surgical emergencies in infants and young children. Emerg Med Clin N Am. 2003;21:909-935. doi:10.1016/S0733-8627(03)000907. • Ross A, LeLeiko NS. Acute abdominal pain. Pediatr Rev. 2010;31(4):135-144. doi:10.1542/pir.31-4-135.

American academy of pediatrics

292

PREP ® Self-Assessment

PREPSA 2020

Question 83 A 13-year-old adolescent girl with Down syndrome is seen for a health supervision visit. She wants to join her sister on the middle school soccer team. Her mother has discussed this with the team's coach, who has agreed to allow her to participate in team practices in a modified fashion. The family has completed the required preparticipation history form, including the supplemental history form for special needs athletes (Item Q83). The results of early childhood cardiac screening were negative. There have been no new cardiac, respiratory, or neurologic symptoms. Neurologic and cardiac examination findings are unremarkable. Her corrected vision is 20/30 in her left eye and 20/20 in her right eye.

American academy of pediatrics

293

PREP ® Self-Assessment

American academy of pediatrics

PREPSA 2020

294

PREP ® Self-Assessment

PREPSA 2020

Of the following, the BEST test, if any, required for this patient's soccer clearance is A. additional visual evaluation B. cervical spine radiographs C. no further evaluation D. an updated echocardiogram

American academy of pediatrics

295

PREP ® Self-Assessment

PREPSA 2020

Correct Answer: C The adolescent girl in this vignette with Down syndrome (DS) can be cleared for sports participation without further evaluation. Preparticipation examinations should be based upon the history and physical examination templates that can be found at: https://www.aap.org/enus/advocacy-and-policy/aap-health-initiatives/Pages/PPE.aspx. These were developed by a collaboration of 6 medical societies, including the American Academy of Pediatrics (AAP), and reflect current standard of care for preparticipation examinations. These forms include a supplemental history form for special needs athletes as described in the vignette. Use of this supplemental form allows for identification of issues often unique to this population that may impact their ability to safely participate in sport. Historically, there has been significant debate about the need for cervical spine radiographs in asymptomatic patients with DS to assess for possible atlantoaxial instability (AAI). Although AAI has been reported in up to 27% of individuals with DS, only 1% to 2% experience symptoms. Best evidence indicates that identification of AAI on cervical spine radiographs in asymptomatic DS patients does not predict future injury in those who participate in unrestricted sports activity. Screening for AAI in DS patients should be accomplished clinically with annual neurologic examinations to identify possible myelopathy. Findings suggestive of symptomatic AAI are listed in Item C83A. Since many of these findings reflect changes from baseline, they are often best recognized by a provider with a longitudinal relationship with the patient. Any findings on history and physical examination suggestive of AAI should prompt cervical spine imaging and urgent neurosurgical evaluation. However, current expert opinion does not support cervical radiographic evaluation for AAI screening in DS patients without pertinent signs or symptoms.

American academy of pediatrics

296

PREP ® Self-Assessment

PREPSA 2020

The AAP 2011 clinical report on health supervision of children with DS advises close supervision and caution with trampoline use and with participation in sports with the potential for contact or collision (eg, football, soccer, gymnastics), but the AAP does not recommend screening cervical spine radiographs in these athletes if they are asymptomatic and have normal neurologic examination findings. These medical recommendations are different than current requirements mandated for participation in Special Olympics. Cervical spine radiographs (including flexion and extension views, which are not included when ordering standard cervical American academy of pediatrics

297

PREP ® Self-Assessment

PREPSA 2020

spine radiographs) are needed for athletes with DS who wish to participate in Special Olympic sports that “may result in extension, radical flexion or direct impact of the neck or upper spine.” A list of these Special Olympics sports is found in Item C83B.

Children with DS have a 50% risk of congenital cardiac defects, but those with negative newborn cardiovascular assessments do not appear to be at increased risk for acquired defects through childhood and adolescence. Additional preparticipation cardiac screening is not indicated if the child has no cardiac-related symptoms. For those athletes with congenital cardiac defects, further guidance on sports participation and clearance can be found in the AAP clinical report, “Medical Conditions Affecting Sports Participation.” Visual acuity should be measured in each eye for all athletes during the PPE. If corrected visual acuity in either eye is worse than 20/40, providers should consider referral for possible refraction assessment. If 20/40 reflects best corrected vision in either eye, then additional eye protection is warranted, and participation in contact sports that prohibit eye protection should be restricted. Eyeguards should be certified by the American Society for Testing and Materials (often noted on American academy of pediatrics 298

PREP ® Self-Assessment

PREPSA 2020

product packaging as ASTM). These recommendations are no different between athletes with and without DS.

PREP Pearls • Cervical spine radiographs in Down syndrome are warranted only when symptoms or physical examination findings suggest myelopathy. • Individuals with Down syndrome accrue multiple benefits from athletic participation. • Athletes with corrected vision worse than 20/40 need additional visual evaluation before clearance for contact/collision sports. ABP Content Specifications(s) • Understand the guidelines for sports participation for patients who have Down syndrome Suggested Readings • Bernhardt DT, Roberts WO; American Academy of Pediatrics. PPE: Preparticipation Physical Evaluation. 5th ed. Elk Grove Village, IL: American Academy of Pediatrics; 2019. https://www.aap.org/en-us/advocacy-and-policy/aap-healthinitiatives/Pages/PPE.aspx. • Bull M; Committee on Genetics. Health supervision for children with Down syndrome. Pediatrics. 2011;128(2):393-406. doi:10.1542/peds.2011-1605. • Mysliwiec A, Posluszny A, Saulicz E, et al. Atlanto-axial instability in people with Down’s (sic) syndrome and its impact on the ability to perform sports activities – a review. J Hum Kinet. 2015;48:17-24. doi:10.1515/hukin-2015-0087. • Rice SG. Medical conditions affecting sports participation. Pediatrics.2008;121(4):841848. doi:10.1542/peds.2008-0080. • Weijerman ME, de Winter JP. Clinical practice: the care of children with Down syndrome. Eur J Pediatr. 2010;169(12):1445-1452. doi:10.1007/s00431-010-1253-0.

American academy of pediatrics

299

PREP ® Self-Assessment

PREPSA 2020

Question 84 A pediatric consultation is requested for a 5-year-old child 3 days after undergoing a neurosurgical procedure that included craniotomy and tumor removal. The child is no longer intubated or under sedation, but has a percutaneous drain in place at the surgical site. She is tolerating clear liquids by mouth. The child has been reporting pain scores between 3 and 5 on the faces pain scale. Of the following, the MOST appropriate medication to manage this girl’s pain is A. B. C. D.

intravenous methadone intravenous morphine oral codeine oral tramadol

American academy of pediatrics

300

PREP ® Self-Assessment

PREPSA 2020

Correct Answer: B The girl in the vignette has moderate acute, postprocedural pain; a short-acting opiate like morphine would be the most appropriate medication for her. Short-acting opiates are generally preferred for opiate-naive children with acute pain, whereas long-acting opiates like methadone are beneficial in situations of chronic, severe pain. Codeine and tramadol are not recommended for the treatment of pain in children because of variable drug metabolism that can lead to either overdose or undertreatment of pain. In children, pain can be assessed by either self-report or with the use of behavioral observation scales in nonverbal children. There are several validated scales that use facial expressions, consolability, interactivity, and motor and verbal responses to score pain. Children should receive regular pain assessments during the course of treatment. Pain management strategies should include pharmacologic and nonpharmacologic interventions, including cognitive and behavioral strategies (eg, distraction, hypnosis, biofeedback) and complementary treatments (eg, massage, acupuncture) when appropriate. Side effects of pharmacologic treatment (eg, nausea, pruritus) should be recognized and treated. In 2012, the World Health Organization published the Guidelines on the Pharmacologic Treatment of Persisting Pain in Children with Medical Illnesses, which outlines a "2 step approach": acetaminophen and nonsteroidal anti-inflammatory medications for mild pain, with the addition of an opiate for moderate to severe pain. Chronic pain has various definitions; many define it as pain that extends beyond the expected period of healing. Chronic pain includes ongoing or episodic conditions (eg, sickle cell disease, cancer) as well as primary pain disorders (previously called "functional pain syndromes"). In children, the most common manifestations of primary pain disorders are headaches, abdominal pain, and musculoskeletal and joint pain. Primary pain disorders are the result of physical, cultural, developmental, and psychosocial factors and therefore require multidisciplinary treatment approaches. These treatments may include physical therapy, psychological treatment, parent coaching, and/or complementary and alternative treatments like yoga, hypnosis, acupuncture, or biofeedback. Medications are not typically the first-line treatment for children with primary pain disorders. PREP Pearls • Acetaminophen and nonsteroidal anti-inflammatory medications should be used for the treatment of mild acute pain in children, with the addition of an opiate for moderate to severe acute pain. • Codeine and tramadol are not recommended for the treatment of pain in children because of variable metabolism that can lead to either overdoses or undertreatment of pain. • Primary pain disorders (previously called functional pain syndromes) are the result of physical, cultural, developmental, and psychosocial factors, and therefore require multidisciplinary treatment approaches. Medications are not typically first-line treatment for children with primary pain disorders.

American academy of pediatrics

301

PREP ® Self-Assessment

PREPSA 2020

ABP Content Specifications(s) • Plan the appropriate management of pain in patients of various ages • Recognize the clinical features commonly associated with chronic pain syndromes • Recognize the behavioral and psychosocial effects of chronic pain syndromes • Understand the effects of a patient’s developmental stage on tolerating and dealing with pain Suggested Readings • American Academy of Pediatrics; Committee on Psychosocial Aspects of Child and Family Health; Task Force on Pain in Infants, Children, and Adolescents. The assessment and management of acute pain in infants, children, and adolescents. Pediatrics. 2001;108(3):793-797. doi:10.1542/peds.108.3.793. • Friedrichsdorf SJ, Giordano J, Desai Dakoji K, Warmuth A, Daughtry C, Schulz CA. Chronic pain in children and adolescents: diagnosis and treatment of primary pain disorders in head, abdomen, muscles and joints. Children (Basel). 2016;3(4):e42. doi:10.3390/children3040042. • Kost-Byerly S, Ward C. Managing chronic pain in children. In: McInerny TK, Adam HM, Campbell DE, DeWitt TG, Foy JM, Kamat DM, eds. American Academy of Pediatrics Textbook of Pediatric Care. 2nd ed. Elk Grove Village, IL: American Academy of Pediatrics; 2017:390-403. Pediatric Care Online. • World Health Organization. WHO Guidelines on the Pharmacological Treatment of Persisting Pain in Children with Medical Illnesses. 2012.

American academy of pediatrics

302

PREP ® Self-Assessment

PREPSA 2020

Question 85 A 6-month-old infant is brought to the emergency department after being found submerged in the bathtub. She was sitting in a bathtub seat when her father went to the next room to get a towel. When he returned, the infant was under the water and not responsive. She had a pulse but no spontaneous breathing. She was given mouth-to-mouth ventilation by the father with return of spontaneous breathing after a few rescue breaths. Of the following, the mechanism that is MOST likely to lead to morbidity and mortality in this patient is A. aspiration of water B. cerebral hypoxia C. injury during resuscitation D. injury from falling

American academy of pediatrics

303

PREP ® Self-Assessment

PREPSA 2020

Correct Answer: B Cerebral hypoxia is the leading cause of morbidity and mortality during drowning. Injury likely begins 3 to 5 minutes after submersion, is multifactorial, and is not reversible. The fall for the infant in this vignette would have been only a short distance and unlikely to cause significant damage. Aspiration of water during drowning is possible but it is usually only small amounts and patients can be supported with positive pressure ventilation if needed. Injury during resuscitation can occur but is not a major contributor to morbidity and mortality. On average, 3 children die each day in the United States from drowning. Children 1 to 4 years of age are at highest risk, and drowning is the second leading cause of death in this age group. In older children it remains the second leading cause of unintentional deaths, with toddlers and adolescent males being at highest risk. Anticipatory guidance including pool safety, boat safety, and bathtub safety are important. If a family has a pool at home, adding a fence that surrounds 4 sides with a self-closing, selflatching gate can prevent 50% of drownings. Parents should be advised to have all infants and toddlers or anyone with poor swimming skills wear a lifejacket. Adults supervising children in the pool should always be within arm’s length, providing “touch supervision” of young children, and avoid any distracting activities such as reading, games, phones, and alcohol. The American Academy of Pediatrics no longer discourages swimming lessons for children aged 1 to 4 years after further studies support that lessons at a young age may decrease the risk of drowning and certainly do not increase the risk. When boating or around natural bodies of water, such as lakes, rivers, and the oceans, young children and any inexperienced swimmers should wear life jackets even when not in the water. Adults supervising around these environments should also always use “touch supervision” and avoid distractions and alcohol. Infants and young children should have direct adult supervision when around water of any kind. Caregivers should be reminded to not leave the room, even for a moment, and to not leave children under the supervision of another child. As seen for the infant in this vignette, bath seats can be dangerous as infants can fall out or slip through them. Other drowning hazards in the house requiring caution include any large containers holding liquids, which should be emptied as soon as no longer in use. PREP Pearls • Cerebral hypoxia is the leading cause of morbidity and mortality during drowning. • Children 1 to 4 years of age are at highest risk and should have direct adult supervision when around water of any kind. • Anticipatory guidance including pool safety, boat safety, and bathtub safety is important. ABP Content Specifications(s) • Understand the epidemiology associated with drowning deaths • Counsel parents regarding safety measures for a home pool • Counsel parents and children regarding safe boat use (eg, flotation devices, supervision) American academy of pediatrics

304

PREP ® Self-Assessment

PREPSA 2020

Suggested Readings • American Academy of Pediatrics Committee on Injury, Violence, and Poison Prevention. Prevention of drowning. Pediatrics. 2010;126(1):178-185. doi:10.1542/peds.2010-1264. • Centers for Disease Control and Prevention. Drowning prevention. https://www.cdc.gov/safechild/drowning/index.html. • Frankel LR. Drowning and near drowning (submersion injuries). In: McInerny TK, Adam HM, Campbell DE, DeWitt TG, Foy JM, Kamat DM, eds. American Academy of Pediatrics Textbook of Pediatric Care. 2nd ed. Elk Grove Village, IL: American Academy of Pediatrics; 2017:2826-2831. Pediatric Care Online. • Thomas AA, Caglar D. Drowning and submersion injury. In: Kliegman RM, St Geme JW III, Blum NJ, Shah SS, Tasker RC, Wilson KM, eds. Nelson Textbook of Pediatrics. 21st ed. Philadelphia, PA: Elsevier; 2020:607-614.

American academy of pediatrics

305

PREP ® Self-Assessment

PREPSA 2020

Question 86 During morning report, a third-year pediatric resident discussed the case of a 16-year-old girl who was admitted to the hospital 2 days ago for abdominal pain. The patient is sexually active and her mother is not aware. The patient’s nucleic acid amplification test is positive for Chlamydia trachomatis, and her presentation is consistent with pelvic inflammatory disease. Because of resident work hour limits, this patient had 3 different residents in charge of her care over a 48-hour period. Brief handoffs were performed without using the system that was recently adopted by the hospital to help reduce errors in patient care. A key piece of information not transmitted during handoff was that the mother was not aware of the girl’s sexual activity and positive finding of Chlamydia infection. This information does not have to be disclosed to the mother, because adolescents have a legal right to testing and treatment of sexually transmitted infections without parental consent. The mother was informed by a resident that the girl tested positive for C trachomatis and pelvic inflammatory disease. The mother became irate that she was not informed that her daughter was sexually active at the time of admission, and the patient was extremely upset that her confidence had been broken. A patient advocate was contacted to assist with calming the mother. The discussion in morning report focused on what adolescents can legally consent to without parental knowledge, and that omission of important information during sign-out can lead to unfavorable situations. Of the following, the BEST next step in the management of this situation is to A. file a written complaint against the 3 residents involved in the patient's care B. initiate a root cause analysis of this sentinel event C. recommend that residents send a text message with important facts to the resident taking over patient care D. review the hospital’s handoff process, highlighting the importance of both verbal and written communication

American academy of pediatrics

306

PREP ® Self-Assessment

PREPSA 2020

Correct Answer: D The pediatric residents described in this vignette did not protect the confidentiality of the patient and upset her mother, which could have been avoided with good written and verbal communication, and by following the hospital’s guidelines for appropriate handoffs between providers. Pediatric providers are advocates for their patients; this includes enhancing patient safety through mindful prevention of misidentification of patients, medication errors, and misdiagnosis. All health care systems have increased efforts to improve patient safety. Medical errors affect onethird of all hospitalized children. Children are more at risk for safety-related problems than are adults; many medications have weight-based dosing; children cannot always communicate when something is wrong; and there are legal issues regarding confidentiality of care of minors. Efforts to improve patient safety are mandated by state and federal agencies such as Medicaid and Medicare; certifying organizations such as The Joint Commission and the American Board of Pediatrics; and professional societies such as the American Academy of Pediatrics. Patient safety requires that all team members, the patient, and family members are informed, attentive, and communicate openly with one another. Use of a standardized handoff process has been shown to reduce medical errors. Although there are numerous handoff methods, the effectiveness of the I-PASS (© 2014 I-PASS Study Group/Boston Children’s Hospital) process is the most evidence based. The I-PASS method was created to standardize verbal handoffs; studies at several pediatric institutions have shown that it reduces communication errors and preventable adverse events. I-PASS stands for:Illness severity; Patient summary; Action list; Situation awareness and contingency planning; and Synthesis by receiver. Having a structured sign-out process that involves direct communication between providers and having the receiver of the handoff, the person who is assuming care, summarize the important patient issues highlighted by the resident transferring the care of the patient, has been an excellent tool in the effort to promote patient safety. Filing a written complaint against the 3 residents does not highlight what was done incorrectly or provide constructive feedback to the involved residents, thus it is not likely to prevent recurrence in the future. The Joint Commission defines a sentinel event in relation to patient safety as an error in patient care that leads to death, permanent harm, or harm that requires life-saving measures. The event described in the vignette would not meet the criteria for a sentinel event. Performing a handoff by text message is inappropriate for several reasons. It does not allow for clarification requests by the receiver, the text may be missed or accidentally deleted, and it can potentially lead to a breach of patient confidentiality. The best option in this case would be to use this incident as a teaching opportunity and review the hospital's handoff procedures, and highlight or demonstrate how verbal and written communication is effective in improving patient safety and reducing errors. PREP Pearls • A structured verbal handoff process, such as the I-PASS method, is a useful tool in reducing errors arising from poor communication. • Patient safety requires that all team members, the patient, and family members are informed, attentive, and communicate openly with one another. American academy of pediatrics

307

PREP ® Self-Assessment

PREPSA 2020

MOCA-Peds Objective • Recognize and apply ethical principles involving confidentiality. ABP Content Specifications(s) • Understand the importance of creating and maintaining a learning environment (eg, morning report, meetings with partners) in improving patient safety Suggested Readings • I-PASS website. http://www.ipasshandoffstudy.com/home. © 2014 I-PASS Study Group/Boston Children’s Hospital. • Lautz AJ, Martin KC, Nishisaki A, et al. Focused training for the handover of critical patient information during simulated pediatric emergencies. Hosp Pediatr. 2018;8(4):227-231. doi:10.1542/hpeds.2017-0173. • Mueller BU, Neuspiel DR, Fisher ERS; Council on Quality Improvement and Patient Safety. Principles of pediatric patient safety: reducing harm to due to medical care. Pediatrics. 2019;143(2):e20183649. doi:10.1542/peds.2018-3649. • Neuspiel DR. Medical errors, adverse events, and patient safety. In: McInerny TK, Adam HM, Campbell DE, DeWitt TG, Foy JM, Kamat DM, eds. American Academy of Pediatrics Textbook of Pediatric Care. 2nd ed. Elk Grove Village, IL: American Academy of Pediatrics; 2017:2287-2295. Pediatric Care Online. • Starmer AJ, Spector ND, Srivastava R, Allen AD, Landrigan CP, Sectish TC. I-PASS, a mnemonic to standardize verbal handoffs. Pediatrics. 2012;129(2):201-204. doi:10.1542/peds.2011-2966.

American academy of pediatrics

308

PREP ® Self-Assessment

PREPSA 2020

Question 87 A 16-year-old boy is having a follow-up evaluation for stage 1 hypertension. His medical history is significant for asthma that is only symptomatic with upper respiratory infections; he is otherwise healthy. There is no evidence of end-organ damage from his hypertension. At previous visits, the boy and his parents were educated about and instructed to implement lifestyle modifications. At this visit, his blood pressure remains elevated and the clinician decides to initiate medication. Of the following, the BEST medication to initiate for this boy’s condition is A. atenolol B. clonidine C. hydralazine D. lisinopril

American academy of pediatrics

309

PREP ® Self-Assessment

PREPSA 2020

Correct Answer: D The boy in the vignette meets criteria for the initiation of medication for his hypertension. Firstline agents for hypertension, according to the 2017 guidelines, include angiotensin-converting enzyme (ACE) inhibitors, angiotensin II receptor blockers, a long-acting calcium channel blocker, or a thiazide diuretic (see Flynn et al). Of the responses, lisinopril, an ACE inhibitor, is the only medication that falls under these classes of drugs. Atenolol is a β-blocker that may prove to be problematic in a patient with a diagnosis of asthma. Clonidine is a centrally acting αagonist that decreases blood pressure. Hydralazine is a direct-acting smooth muscle relaxant. Typically, the treatment of hypertension begins with a nonpharmacologic approach, including diet and lifestyle modifications. Pediatric data mirroring those in adults demonstrate that diet and exercise have a favorable effect on blood pressure. The Dietary Approaches to Stop Hypertension (DASH) diet, in particular, is recommended, which is high in fruits, vegetables, low-fat milk products, whole grains, fish, poultry, nuts, and lean red meats; sugar and sweets are limited. Moderate exercise, at least 3 days a week, has been shown to lower blood pressure as well.

PREP Pearls • First-line medication therapy for hypertension includes angiotensin-converting enzyme (ACE) inhibitors, angiotensin II receptor blockers (ARBs), long-acting calcium channel blockers, and thiazide diuretics. • Typically, the treatment of hypertension begins with a nonpharmacologic approach including diet and lifestyle modifications (eg, exercise at least 3 days per week). • The Dietary Approaches to Stop Hypertension (DASH) diet is recommended; it is high in fruits, vegetables, low-fat milk products, whole grains, fish, poultry, nuts, and lean red meats; sugar and sweets are limited. American academy of pediatrics

310

PREP ® Self-Assessment

PREPSA 2020

ABP Content Specifications(s) • Understand the mechanism of action of the different classes of antihypertensive drugs Suggested Readings • American Academy of Pediatrics. Point-of-care quick reference: hypertension. Pediatric Care Online. • Brady T. Hypertension. Pediatr Rev. 2012;33(12):541-552. doi:10.1542/pir.33-12-541. • Daniels SR. Consultation with the specialist: the diagnosis of hypertension in children: an update. Pediatr Rev. 1997;18(4):131-135. doi:10.1542/pir.18-4-131. • Flynn JT, Kaelber DC, Baker-Smith CM, et al; Subcommittee on Screening and Management of High Blood Pressure in Children. Clinical practice guideline for screening and management of high blood pressure in children and adolescents. Pediatrics. 2017;140(3):e20171904. doi:10.1542/peds.2017-1904.

American academy of pediatrics

311

PREP ® Self-Assessment

PREPSA 2020

Question 88 A 16-year-old boy is brought to the emergency department in August with a 3-day history of fever, myalgia, headache, emesis, and confusion. There are no sick contacts or pets at home. One week before presentation, he had traveled to New York to visit family. During a hiking trip, he experienced several mosquito bites, but no known tick bites. His immunizations are up-to-date. On physical examination, the boy has a temperature of 39.4°C, heart rate of 120 beats/min, and respiratory rate of 16 breaths/min. His blood pressure is normal. His head, ears, eyes, nose, and throat appear normal. There is no rash. Neurologic examination is remarkable for altered mental status and mild stiffness on neck flexion. The rest of his physical examination findings are unremarkable. Laboratory findings are as follows. Laboratory Test Result Cerebrospinal fluid White blood cell count 630/µL (0.63 × 109/L), 60% lymphocytes Glucose 48 mg/dL (2.7 mmol/L) Protein 65 mg/dL (0.65 g/L) Gram stain Negative for bacteria Blood glucose 90 mg/dL (5 mmol/L) Of the following, the test that is MOST likely to establish the boy’s diagnosis is A. bacterial antigen test (cerebrospinal fluid [CSF]) B. culture (CSF, blood) C. polymerase chain reaction assay (CSF, blood) D. serology (CSF, blood)

American academy of pediatrics

312

PREP ® Self-Assessment

PREPSA 2020

Correct Answer: D The clinical presentation and cerebrospinal fluid (CSF) findings of the adolescent described in the vignette are highly suggestive of aseptic meningitis or meningoencephalitis. Given his recent mosquito bites while hiking in New York, the most likely etiology of his illness is West Nile virus (WNV). The diagnosis is generally made by detection of WNV IgM antibody in the CSF or blood. The differential diagnosis of aseptic meningitis or meningoencephalitis in children must include nonpolio enteroviruses, herpes simplex virus, and arboviruses (eg, La Crosse virus, West Nile virus) in addition to a host of other pathogens. The CSF profile of the patient described in the vignette is not consistent with a diagnosis of acute bacterial meningitis. West Nile virus is a common cause of arboviral neuroinvasive disease in the United States, with a reported median annual incidence of 0.07 cases per 100,000 children. Following initial detection in New York City in 1999, cases of West Nile neuroinvasive disease (WNND) have been reported in most US states, with the highest numbers seen in California, North Dakota, South Dakota, Oklahoma, Colorado, and Nebraska. The incubation period for WNV illness is 2 to 6 days. Most individuals with WNV infection are asymptomatic. Symptomatic illness is characterized by fever, headache, eye pain, gastrointestinal symptoms, and fatigue. The duration of acute illness is 3 to 5 days. During convalescence, individuals may develop a rash, which starts on the trunk and may extend to the extremities and face. Neuroinvasive disease occurs in less than 1% of infected individuals. West Nile virus can cause a wide spectrum of neuroinvasive clinical syndromes including aseptic meningitis, encephalitis, acute flaccid myelitis (causing a polio-like illness) and rarely, Guillain-Barré syndrome or movement disorder. Other unusual manifestations of WNV disease include hepatitis, cardiac dysrhythmias, myocarditis, rhabdomyolysis, and optic neuritis. Individuals older than age 70 years and individuals with comorbid conditions (eg, diabetes, hypertension, chronic renal disease, cancer) are at highest risk for severe WNND. The clinical features of WNV meningitis are similar to those of aseptic meningitis caused by other viral pathogens. Symptoms and signs may include abrupt onset of fever, headache, photophobia, altered mental status, and seizures; physical examination findings may include neck stiffness, cranial nerve palsies, and focal neurologic deficits. Cerebrospinal fluid analysis shows pleocytosis with a predominance of lymphocytes; a neutrophilic pleocytosis may be present early in the illness. Magnetic resonance imaging of the brain is often normal but may reveal signal abnormalities in the brainstem, basal ganglia, thalamus, or spinal cord. Weakness, malaise, and fatigue may last for several weeks to months. Complete recovery is expected for patients with WNV meningitis; however, WNV encephalitis and myelitis may result in adverse neurologic outcomes. The case-fatality rate of WNND is approximately 10%. The diagnosis of recent WNV infection is generally made by detection of WNV IgM antibody in the CSF or blood. West Nile virus IgM antibody may be detected in the serum for more than 1 year after infection; however, detection of WNV IgM in the CSF is indicative of a recent central nervous system infection. West Nile virus IgM antibodies are detectable in the CSF by 3 to 5 days of onset of illness. Detection of WNV by viral culture is not recommended given the American academy of pediatrics

313

PREP ® Self-Assessment

PREPSA 2020

difficulty in isolating the virus. Molecular detection of WNV with polymerase chain reaction is less sensitive in immunocompetent hosts compared with immunocompromised patients. Supportive care is the mainstay of management for WNND. No specific antiviral therapy is available. PREP Pearls • West Nile virus is a common cause of arboviral neuroinvasive disease in the United States. • West Nile virus can cause a wide spectrum of neuroinvasive clinical syndromes including aseptic meningitis, encephalitis, and acute flaccid myelitis. • The diagnosis of recent West Nile virus infection is generally made by detection of West Nile virus IgM antibody in the cerebrospinal fluid or blood. ABP Content Specifications(s) • Recognize the clinical features associated with arbovirus infection (eg, West Nile, dengue fever) • Understand the epidemiology of arbovirus, including West Nile virus Suggested Readings • American Academy of Pediatrics. West Nile virus. In: Kimberlin DW, Brady MT, Jackson MA, Long SS, eds. Red Book: 2018 Report of the Committee on Infectious Diseases. 31st ed. Itasca, IL: American Academy of Pediatrics; 2018:888891. https://redbook.solutions.aap.org/chapter.aspx?sectionId=189640220&bookId=2205 &resultClick=24. • Gaensbauer JT, Lindsey NP, Messacar K, Staples JE, Fischer M. Neuroinvasive arboviral disease in the United States: 2003-2012. Pediatrics. 2014;134(3):e642-e650. doi:10.1542/peds.2014-0498. • Tunkel A, Glaser C, Block K, et al. The management of encephalitis: clinical practice guidelines by the Infectious Diseases Society of America. Clin Infect Dis. 2008;47(3):302-327. doi:10.1086/589747.

American academy of pediatrics

314

PREP ® Self-Assessment

PREPSA 2020

Question 89 A 5-year-old previously healthy boy is being evaluated for rapidly progressive pain, redness, and swelling of his left lower leg. A few days earlier, he was bitten by a mosquito. On physical examination, the boy appears quiet and tired. He has a temperature of 39.1°C, pulse of 130 beats/min, respiratory rate of 30 breaths/min, and blood pressure of 80/47 mm Hg. On his left lower leg, there is a 5 × 5–cm area of induration underlying a 0.5 × 0.5–cm papule with central punctum at the site of the insect bite. There is a 6 × 6–cm area of surrounding erythema and duskiness. The area is exquisitely tender to palpation. A complete blood cell count shows a peripheral white blood cell count of 28 × 103/μL (28 × 109/L) with a differential of 85% neutrophils, 5% band forms, 8% lymphocytes, and 2% monocytes; C-reactive protein is significantly elevated. Parenteral antibiotic therapy is started. Of the following, the MOST appropriate next step in management is A. incision and drainage at the bedside B. incision and drainage in the operating room C. magnetic resonance imaging D. ultrasonography

American academy of pediatrics

315

PREP ® Self-Assessment

PREPSA 2020

Correct Answer: B The boy in the vignette has acute necrotizing fasciitis (ANF) from invasive group A βhemolytic Streptococcus (GAS), otherwise known as Streptococcus pyogenes, an aerobic grampositive β-hemolytic organism. Acute necrotizing fasciitis requires urgent treatment. Management of necrotizing fasciitis requires a combination of parenteral antibiotic therapy and aggressive, extensive surgical debridement in the operating room to halt the progression of tissue necrosis. Treatment should not be delayed for further diagnostic studies if clinical suspicion for ANF is high. Imaging such as computed tomography, magnetic resonance imaging, or ultrasonography may be helpful but cannot rule out ANF; often only localized tissue swelling is visualized rather than the classic finding of gas. Any delay in treatment of ANF significantly increases the risk of a poor outcome. Group A Streptococcus is responsible for a wide variety of illnesses, including noninvasive and invasive infections as well as nonsuppurative complications (Item C89).

American academy of pediatrics

316

PREP ® Self-Assessment

American academy of pediatrics

PREPSA 2020

317

PREP ® Self-Assessment

American academy of pediatrics

PREPSA 2020

318

PREP ® Self-Assessment

American academy of pediatrics

PREPSA 2020

319

PREP ® Self-Assessment

American academy of pediatrics

PREPSA 2020

320

PREP ® Self-Assessment

PREPSA 2020

Rapid antigen detection test is recommended if GAS pharyngitis is suspected in children older than 3 years. Throat culture should be performed in those who test negative. Diagnosis and treatment are less important in children younger than 3 years because they are unlikely to manifest acute rheumatic fever. In children, studies have demonstrated the unreliability of diagnosis based on clinical criteria alone. Those with viral symptoms (coryza, conjunctivitis, cough, etc) should not be tested because the likelihood of GAS infection is extremely low. If invasive GAS infection is suspected, cultures of blood and of local sites are indicated. PREP Pearls • Management of necrotizing fasciitis requires a combination of parenteral antibiotic therapy and aggressive, extensive surgical debridement in the operating room. • Group A Streptococcus is responsible for a wide variety of illnesses, including noninvasive and invasive infections as well as nonsuppurative complications. • Rapid antigen detection test is recommended if group A Streptococcus pharyngitis is suspected in children older than 3 years; diagnosis and treatment are less important in children younger than 3 years because they are unlikely to manifest acute rheumatic fever. American academy of pediatrics

321

PREP ® Self-Assessment

PREPSA 2020

ABP Content Specifications(s) • Recognize the clinical features associated with Streptococcus pyogenes infection • Recognize the clinical features associated with Streptococcus pyogenes infection • Understand the epidemiology of Streptococcus pyogenes • Plan the appropriate diagnostic evaluation of suspected Streptococcus pyogenes infection • Plan the appropriate diagnostic evaluation of suspected Streptococcus pyogenes infection Suggested Readings • American Academy of Pediatrics. Group A Streptococcus. In: Kimberlin DW, Brady MT, Jackson MA, Long SS, eds. Red Book: 2018 Report of the Committee on Infectious Diseases. 31st ed. Itasca, IL: American Academy of Pediatrics; 2018:748762. https://redbook.solutions.aap.org/chapter.aspx?sectionId=189640187&bookId=2205 &resultClick=1. • Bisno AL, Stevens DL. Streptococcal infections of skin and soft tissues. N Engl J Med. 1996;334(4):240-246. doi:10.1056/NEJM199601253340407. • Bochner RE, Gangar M, Belamarich PF. A clinical approach to tonsillitis, tonsillar hypertrophy, and peritonsillar and retropharyngeal abscesses. Pediatr Rev. 2017;38(2):81-92. doi:10.1542/pir.2016-0072. • Dietrich ML, Steele RW. Group A Streptococcus. Pediatr Rev. 2018;39(8):379-391. doi:10.1542/pir.2017-0207.

American academy of pediatrics

322

PREP ® Self-Assessment

PREPSA 2020

Question 90 A 3-year-old previously healthy, fully immunized boy is brought to the emergency department after several hours of progressive lethargy. Earlier in the day, he was in his usual state of health, without fever, decreased activity, respiratory symptoms, pain, or decreased appetite. He has not had any sick contacts or history of trauma. He has a temperature of 37°C, heart rate of 80 beats/min, respiratory rate of 12 breaths/min, blood pressure of 75/40 mm Hg, and oxygen saturation of 92% on room air. He appears lethargic. When stimulated, he opens his eyes, mumbles a few words, and moves the examiner’s hands away. Pupils are pinpoint. Muscle tone is decreased, and deep tendon reflexes are 2+ throughout. Cough and gag reflexes are present. Mucous membranes are moist. He is breathing slowly but comfortably. His lungs are clear to auscultation bilaterally. His heart has a regular rate and rhythm. His abdomen is soft, nontender, and nondistended. His extremities are warm with a capillary refill time of 2 seconds. Pulses are strong throughout. Capillary blood gas data are shown: Laboratory Test pH PCO2 PO2 HCO3 Base excess

Result 7.20 55 mm Hg 50 mm Hg 24 mEq/L 2 mEq/L

Of the following, the MOST appropriate next step in management is A. endotracheal intubation B. intravenous naloxone C. noninvasive positive-pressure ventilation D. oxygen, 100% nonrebreather facemask

American academy of pediatrics

323

PREP ® Self-Assessment

PREPSA 2020

Correct Answer: B The previously healthy boy in this vignette has a sudden onset of lethargy, pinpoint pupils, and hypopnea. Evaluation shows respiratory acidosis and hypoxia. These findings suggest an opioid toxidrome. The most appropriate next step in management is intravenous naloxone. The 2 main functions of the respiratory system are providing oxygen and removing carbon dioxide from the blood. Accordingly, the 2 major types of respiratory failure are hypoxic and hypercarbic. Hypoxic respiratory failure is characterized by insufficient oxygenation of pulmonary arterial blood. Oxygenation occurs when oxygen diffuses from open, oxygencontaining alveoli to adjacent pulmonary capillaries. Pulmonary capillary blood that perfuses alveoli that are fluid filled or collapsed returns to the heart deoxygenated, contributing to hypoxia, a process called intrapulmonary shunting. Other causes of hypoxia include barriers to diffusion such as in pulmonary fibrosis and pulmonary edema, and decreased pulmonary blood flow such as that caused by pulmonary embolism, right-sided heart failure, right-to-left intracardiac shunting, and pulmonary hypertension. While oxygenation is dependent on ventilation as well as other factors, removal of carbon dioxide is solely proportional to ventilation. Hypercarbic respiratory failure is caused by disruption of ventilation from respiratory pump failure or airway obstruction anywhere in the tracheobronchial tree and the nasopharyngeal airway. Respiratory pump failure can be caused by central nervous system (CNS) depression as well as acute, chronic, or anatomic lesions affecting the function of the diaphragm or accessory respiratory muscles. As described for the boy in this vignette, hypoxic and hypercarbic respiratory failure can coexist. Inadequate ventilation can cause decreased alveolar PO2 by several different mechanisms. Decreased air intake leads to less oxygen in the alveoli available for gas exchange. Inadequate alveolar inflation can also cause de-recruitment of alveoli, especially during exhalation, and intrapulmonary shunting. Increased PCO2 from inadequate ventilation can decrease the PO2 in accordance with the ideal gas principle. Obtaining pH, PaCO2, PaO2, serum bicarbonate levels, and base deficit/excess from an arterial blood gas sample is helpful in investigating the severity and causes of respiratory failure, as well as acid-base disturbances. Since CO2 combines with H2O to produce carbonic acid (H2CO3), elevations in pCO2 decrease the pH. Chemoreceptors in the CNS respiratory center are exquisitely sensitive to the cerebrospinal fluid pH, as well as to PaO2. With acute elevations in pCO2 significant acidemia occurs due to the limited buffering by plasma proteins. Acutely, the serum bicarbonate increases only 1 mEq/L but the pH decreases by approximately 0.08 for every 10 mm Hg increase in PaCO2. In contrast, in chronic respiratory failure the pH remains closer to normal due to renal compensation, which takes several days to occur. With renal compensation the pH decreases by 0.03 and the serum bicarbonate level increases by approximately 3.5 mEq/L for every 10 mm Hg increase in PaCO2. Since obtaining an arterial blood gas sample may be more difficult in children, a capillary or venous blood gas sample will provide reliable information about pH, pCO2, base deficit/excess, and bicarbonate levels. Based on the sudden onset of lethargy and hypoventilation and the clinical signs of pinpoint pupils, hypotension, and respiratory acidosis and hypoxia, the boy in this vignette likely has an opioid ingestion. Naloxone is a competitive opioid receptor antagonist that can be used to reverse respiratory depression and altered mental status caused by opioid overdose. It is most commonly American academy of pediatrics

324

PREP ® Self-Assessment

PREPSA 2020

given intravenously, although it can also be given intramuscularly, subcutaneously, or intranasally. A clinical effect from naloxone usually occurs within 1 minute. If no effect is seen within several minutes, doses can be repeated. The serum half-life is approximately 30 minutes, so repeated doses and/or continuous infusions are often required for a sustained effect. For the boy in the vignette, the mechanism of respiratory failure is from hypoventilation leading to hypercarbia and hypoxia. Respiratory failure can be managed by endotracheal intubation and mechanical ventilation, but this approach is too invasive to use prior to attempting treatment with naloxone. Intubation and mechanical ventilation may not be necessary because the boy has an intact cough and gag, and thus may be able to protect his airway. Furthermore, bag-valve mask ventilation and intubation could be risky if he has a full stomach. Noninvasive positive-pressure ventilation can be used to recruit alveoli in hypoxic respiratory failure and in some cases of upper airway obstruction, but it is less effective in CNS depression. In fact, it can lead to aspiration of stomach contents if the airway is not adequately protected. Oxygen can be harmful for the boy in this vignette because it can blunt the component of respiratory drive from hypoxia, leading to worsened hypoventilation. PREP Pearls • Naloxone is a useful reversal agent for opioid overdose. • The pH decreases significantly in acute respiratory acidosis because renal compensation for respiratory acidosis requires several days. • Oxygen can be a toxic therapy in patients with respiratory failure from central nervous system depression; instead, treatment of the underlying condition should be sought. ABP Content Specifications(s) • Understand the pulmonary mechanism for regulating acid-base physiology • Recognize the clinical and laboratory manifestations associated with respiratory failure of various etiologies Suggested Readings • Abdo WF, Heunks LM. Oxygen-induced hypercapnia in COPD: myths and facts. Crit Care. 2012;16(5):323. doi:10.1186/cc11475. • Brinkman JE, Sharma S. Physiology, respiratory drive. StatPearls [Internet]. Treasure Island, FL: StatPearls Publishing; 2018. • Hsu BS, Lakhani SA, Wilhelm M. Acid-base disorders. Pediatr Rev. 2016;37(9):361369. doi:10.1542/pir.2015-0093.. • Vo P, Kharasch VP. Respiratory failure. Pediatr Rev. 2014;35(11):476-484. doi:10.1542/pir.35-11-476.

American academy of pediatrics

325

PREP ® Self-Assessment

PREPSA 2020

Question 91 A 9-year-old boy is having a follow-up evaluation for attention-deficit/hyperactivity disorder. He and his parents have been working with a behavior therapist on strategies to manage his hyperactivity, impulsivity, and inattention. A Section 504 plan has been established; the boy is receiving classroom accommodations including preferential seating, modified assignments, extended time for tests, and a home-school communication system for behavioral monitoring. There has been some improvement in the boy’s function both at home and school; however, he continues to struggle academically and socially. His parents would like to discuss treatment with a prescription medication. They are concerned about medications that might increase their son’s blood pressure, because there is a family history of hypertension. Of the following, medications COMMONLY associated with this adverse effect include A. methylphenidate and amphetamine B. methylphenidate, amphetamine, and atomoxetine C. methylphenidate, amphetamine, and clonidine D. methylphenidate, amphetamine, and guanfacine

American academy of pediatrics

326

PREP ® Self-Assessment

PREPSA 2020

Correct Answer: B It is important to monitor for side effects (Item C91) when using medications to treat attentiondeficit/hyperactivity disorder (ADHD), a chronic neurodevelopmental disorder with functional impairment in multiple environments because of inattention, hyperactivity, and/or impulsivity significantly beyond that expected for developmental age. Medications approved by the United States Food and Drug Administration for the treatment of ADHD include stimulant medications (methylphenidate, amphetamine preparations), a selective norepinephrine-reuptake inhibitor (atomoxetine), and long-acting α-agonists (clonidine, guanfacine). Stimulant medications and atomoxetine can increase blood pressure whereas alpha-agonists can decrease blood pressure.

Item C91: Potential Adverse Effects of Medications Used to Treat AttentionDeficit/Hyperactivity Disorder. Courtesy of S. Guralnick Attention-deficit/hyperactivity disorder is a common chronic condition of childhood, with a prevalence of 5% to 10%. Prevalence is higher in boys than in girls (2-6:1). Girls are more likely to have inattention and be diagnosed at a later age; boys typically are seen earlier with disruptive, hyperactive, and impulsive behaviors. Comorbid conditions are seen in most children with ADHD, up to 75% have additional diagnoses such as anxiety, depression, speech/language disorder, autism spectrum disorder, learning disability, oppositional defiant disorder, and tic disorders. American academy of pediatrics

327

PREP ® Self-Assessment

PREPSA 2020

Attention-deficit/hyperactivity disorder is more commonly diagnosed in school-aged children than in preschool-aged children or adolescents, 2 groups that present diagnostic challenges. The degree of inattention, hyperactivity, and impulsive behaviors must be beyond that expected for their age, which may be difficult to assess in young children. A key to diagnosis in this age group is the resulting functional impairment. There may be significant concerns for safety for the child or others (eg, injuries resulting in emergency department visits) or the child may have been dismissed from multiple preschools. The “on the go” and “driven by a motor” behavior of the young child with hyperactive/impulsive-type ADHD may be replaced by an internal restlessness in the older child or adolescent, who may fidget, make impulsive decisions, or display high-risk behaviors. Adolescents with symptoms of inattention require evaluation for other conditions, particularly anxiety, depression, and/or substance use. These adolescents may have 1 or more of these diagnoses in addition to or instead of a diagnosis of ADHD. It is also important to recognize that children with higher levels of intelligence may be identified later because they may not have significant impairment in functioning until academic demands and organizational needs increase. Attention-deficit/hyperactivity disorder diagnosed in childhood is likely to persist into adulthood. It is associated with adverse long-term outcomes such as decreased academic attainment, and increased legal problems, mental health conditions, and substance use disorders. Thus, early identification, implementation of an effective treatment plan, and management as a chronic condition are essential to the successful support of children and adolescents with ADHD. PREP Pearls • Stimulant medications and atomoxetine can increase blood pressure; α-agonists (clonidine, guanfacine) can decrease blood pressure. • Comorbid conditions are seen in most children with attention-deficit/hyperactivity disorder; up to 75% have additional diagnoses such as anxiety, depression, speech/language disorder, autism spectrum disorder, learning disability, oppositional defiant disorder, and tic disorders. • Adolescents with symptoms of inattention may require evaluation for other conditions, particularly anxiety, depression, and/or substance use. These adolescents may have 1 or more of these diagnoses in addition to or instead of a diagnosis of attentiondeficit/hyperactivity disorder. MOCA-Peds Objective • Plan the management of ADHD. ABP Content Specifications(s) • Understand the common side effects of medications used to treat attention deficit hyperactivity disorder • Recognize the age-related clinical findings associated with ADHD and its subtypes • Understand the gender- and age-related differences in the prevalence of ADHD • Understand the expected long-term outcome of ADHD diagnosed during childhood • Recognize co-morbidities commonly associated with ADHD in patients of various ages

American academy of pediatrics

328

PREP ® Self-Assessment

PREPSA 2020

Suggested Readings • Feldman HM, Reiff MI. Attention deficit-hyperactivity disorder in children and adolescents. N Engl J Med. 2014;370(9):838-846. doi:10.1056/NEJMcp1307215. • Floet AM, Scheinet C, Grossman L. Attention-deficit/hyperactivity disorder. Pediatr Rev. 2010;31(2):56-69. doi:10.1542/pir.31-2-56. • Subcommittee on Attention-Deficit/Hyperactivity Disorder, Steering Committee on Quality Improvement and Management. ADHD: clinical practice guideline for the diagnosis, evaluation, and treatment of attention-deficit/hyperactivity disorder in children and adolescents. Pediatrics. 2011;128(5):1007-1022. doi:10.1542/peds.2011-2654. • Wissow LS. Inattention and impulsivity. In: McInerny TK, Adam HM, Campbell DE, Foy JM, Kamat DM, eds. Textbook of Pediatric Care. 2nd ed. Itasca, IL: American Academy of Pediatrics; 2016:1462-1466. Pediatric Care Online.

American academy of pediatrics

329

PREP ® Self-Assessment

PREPSA 2020

Question 92 A 1-year-old girl is being evaluated for unusual facies, growth deficiency, significant developmental delays, hypotonia, hearing loss, and a previously diagnosed generalized tonicclonic seizure disorder. Her facies are remarkable for microcephaly, "Greek warrior" helmet nose, high anterior hairline with prominent glabella, hypertelorism, epicanthus, high-arched eyebrows, downturned mouth, small jaw, and small ears with bilateral ear pits (Item Q92). Prenatal history is remarkable for intrauterine growth restriction. Evaluation for other anomalies includes echocardiography, which shows an atrial septal defect, and brain magnetic resonance imaging, which shows thinning of the corpus callosum. A diagnosis is made using a high-resolution karyotype.

Item Q92: Girl described in the vignette. Reprinted with permission from http://medgen.genetics.utah.edu/. Of the following, the MOST likely diagnosis for this girl is A. 4p− deletion B. 5p− deletion C. trisomy 13 D. trisomy 18

American academy of pediatrics

330

PREP ® Self-Assessment

PREPSA 2020

Correct Answer: A The girl in the vignette has 4p− deletion, also known as Wolf-Hirschhorn syndrome. WolfHirschhorn syndrome commonly presents with • unusual facies • prenatal and postnatal growth deficiency • significant developmental delays • hypotonia • hearing loss • antibody deficiencies (69%) • cardiac defects (50%) • urinary tract anomalies (30%) • central nervous system malformations (80%) • epilepsy or electroencephalographic abnormalities Typical craniofacial dysmorphology includes microcephaly, “Greek warrior” helmet nose, high anterior hairline with prominent glabella, hypertelorism, epicanthus, high-arched eyebrows, downturned mouth, small jaw, and small ears with bilateral ear pits.The most common congenital heart defect is an atrial septal defect followed by pulmonic stenosis, ventricular septal defect, tetralogy of Fallot, and patent ductus arteriosus. The most common central nervous system anomaly is thinning of the corpus callosum followed by ventricular enlargement, cortical atrophy, and cerebellar hypoplasia. A few affected children have hematopoietic dysfunction or hepatic adenomas. Wolf-Hirschhorn syndrome can be diagnosed based on clinical presentation, along with a highresolution karyotype, fluorescent in situ hybridization (FISH) for 4p−, or a chromosomal microarray. If highly suspected, a high-resolution karyotype could be used to make the diagnosis. Fifty percent to 60% of patients with Wolf-Hirschhorn syndrome have a de novo or sporadic deletion of 4p16; 40% to 45% have an unbalanced translocation that contains a deletion of 4p and a partial trisomy of a different chromosome. Unbalanced translocations can occur de novo or be inherited from an unaffected parent with a balanced chromosomal arrangement. Notably, the American College of Medical Genetics and the American Academy of Pediatrics recommend that a chromosomal microarray be performed on all children with developmental delay, multiple congenital anomalies, or intellectual disability of unknown etiology. This would not only pick up a gross chromosomal deletion as in the child in the vignette, but also microdeletions or microduplications that could be easily missed on a high-resolution karyotype. A microarray would also help clarify specific breakpoints and genes involved in the deletion. Genotype and phenotype correlate with the size of the actual deletion. If the specific diagnosis is clinically suspected, then FISH analysis or karyotype results may be more quickly obtained and less expensive than microarray testing. 5p− deletion, also known as Cri du Chat syndrome, is caused by a deletion of the short arm of chromosome 5. It is characterized by a high-pitched or cat-like cry (>95%), small head, epicanthal folds, micrognathia, broad nasal bridge, hypertelorism, downward-slanting palpebral fissures, and moderate-to-severe intellectual disability. Infants with trisomy 13 have severe intellectual disability and multiple congenital anomalies, which include skin defects of the posterior scalp, polydactyly, microcephaly, wide fontanelles, American academy of pediatrics

331

PREP ® Self-Assessment

PREPSA 2020

genitourinary abnormalities, defects of the eye, cleft lip or palate, holoprosencephaly, and cardiac defects. More than 80% of these infants die in the first month after birth; 5% to 10% will survive for 1 year or more. Infants with trisomy 18 are small for gestational age and have clenched hands, overlapping fingers, increased muscle tone, micrognathia, low-set abnormal ears, congenital heart defects, single umbilical artery, short palpebral fissures, nail hypoplasia, rocker bottom feet, and short sternum. Most have severe intellectual disability. Affected children have a limited capacity for survival, with most dying in the first month after birth.

PREP Pearls • Children with developmental delay should undergo chromosomal microarray testing, which will identify gross chromosomal deletions and duplications as well as microdeletions and microduplications that could be easily missed on a high-resolution karyotype. • Wolf-Hirschhorn syndrome (4p− deletion) commonly presents with unusual facies, prenatal and postnatal growth deficiency, significant developmental delays, hypotonia, hearing loss, antibody deficiencies, cardiac defects, urinary tract anomalies, central nervous system malformations, and epilepsy or electroencephalographic abnormalities. • Typical craniofacial dysmorphology seen in 4p− deletion includes microcephaly, “Greek warrior” helmet nose, high anterior hairline with prominent glabella, hypertelorism, epicanthus, high-arched eyebrows, downturned mouth, small jaw, and small ears with bilateral ear pits. ABP Content Specifications(s) • Plan appropriate parental evaluation when an infant is born with a structural chromosomal abnormality • Understand the significance of a gross chromosomal deletion Suggested Readings • Battaglia A, Carey JC, South ST. Wolf-Hirschhorn syndrome. GeneReviews. https://www.ncbi.nlm.nih.gov/books/NBK1183/#whs.Manage ment. • Battaglia A, Filippi T, Carey JC. Update on the clinical features and natural history of Wolf-Hirschhorn (4p-) syndrome: experience with 87 patients and recommendations for routine health supervision. Am J Med Genet Part C Semin Med Genet. 2008;148C:246251. doi:10.1002/ajmg.c.30187. • Battaglia A. Deletion 4p - Wolf-Hirschhorn syndrome. In: Cassidy SB, Allanson JE, eds. Management of Genetic Syndromes. Hoboken, NJ: Wiley-Liss & Sons; 2010:249261. • Moeschler JB, Shevell M, Committee on Genetics. Comprehensive evaluation of the child with intellectual disability or global developmental delays. Pediatrics. 2014;134(3):e903-918. doi:10.1542/peds.2014-1839.

American academy of pediatrics

332

PREP ® Self-Assessment

PREPSA 2020

Question 93 An infant born at 32 weeks of gestation with a birthweight of 1,400 g is evaluated for discharge from the neonatal intensive care unit. His mother had chronic hypertension. He is now 4 weeks old and weighs 2,100 g. On physical examination, there is no murmur and slightly increased lower extremity muscle tone. He remains comfortable in room air and is feeding expressed breast milk with 2 feedings per day of formula. Of the following, the screening test MOST likely to be recommended before discharge is A. echocardiography B. ophthalmologic examination C. pneumogram D. serum hemoglobin

American academy of pediatrics

333

PREP ® Self-Assessment

PREPSA 2020

Correct Answer: B For the infant in the vignette, screening for retinopathy of prematurity (ROP) with indirect ophthalmologic examination should be completed before he is discharged from the hospital. Retinopathy of prematurity only occurs in infants born prematurely, affecting approximately 20% of infants with a birthweight of less than 1,500 g. Worldwide, the incidence of ROP varies. In middle-income countries, premature infants have higher rates of ROP, possibly because of differences in respiratory management including oxygen titration. Retinal development begins at 16 weeks’ gestation. Blood vessels grow from the optic nerve toward the periphery of the retina. With premature birth, growth of these blood vessels stops briefly. When growth resumes, vessels are exposed to completely different stimuli such as higher concentrations of oxygen and inflammatory mediators. Retinopathy of prematurity may be caused in part by an imbalance between vascular endothelial growth factor and insulin-like growth factor 1. Retinopathy of prematurity describes abnormal growth of these retinal blood vessels. In severe ROP, blood vessels can grow into the vitreous, causing retinal detachment and blindness. Severe cases of ROP may require laser photocoagulation therapy of the retina to stop the growth of abnormal vessels. Prematurity is the most important risk factor for ROP, followed by exposure to oxygen. Bacteremia, necrotizing enterocolitis, and bronchopulmonary dysplasia also contribute to the pathogenesis of ROP. All neonates and infants with a gestational age less than or equal to 30 weeks or birthweight less than 1,500 g should be evaluated for ROP. Screening should begin at 31 weeks’ postconceptional age or 4 weeks after birth, whichever comes first. Item C93 summarizes the timing of the first eye examination based on the gestational age at birth. An ophthalmologist with experience in the evaluation of premature neonates should perform screenings using an indirect binocular ophthalmoscope. Infants with ROP are also at risk for amblyopia, strabismus, cataracts, and refractive errors.

American academy of pediatrics

334

PREP ® Self-Assessment

American academy of pediatrics

PREPSA 2020

335

PREP ® Self-Assessment

PREPSA 2020

Echocardiography is not indicated before discharge for the infant in the vignette. This would be indicated for an infant with severe bronchopulmonary dysplasia who continues to receive positive pressure ventilation at 36 weeks’ postconceptional age to screen for pulmonary hypertension. A pneumogram, used to evaluate periods of central versus obstructive apnea before discharge from the hospital, is not indicated in this infant who does not have episodes of apnea or bradycardia. He has no symptoms suggestive of anemia, such as tachycardia, and therefore does not need a screening hemoglobin level. PREP Pearls • Infants born at 30 weeks’ or lower gestational age or with a birthweight less than 1,500 g should be screened for retinopathy of prematurity. • Screening for retinopathy of prematurity should occur at 31 weeks’ postconceptional age or 4 weeks after birth, whichever comes first. • Infants with retinopathy of prematurity are also at risk for amblyopia, strabismus, cataracts, and refractive errors. ABP Content Specifications(s) • Plan the appropriate screening and clinical evaluation of retinopathy of prematurity

Suggested Readings • Campbell DE. Continuing care of the infant after transfer from neonatal intensive care. In: McInerney TK, Adam HM, Campbell DE, DeWitt TG, Foy JM, Kamat DM, eds. American Academy of Pediatrics Textbook of Pediatric Care. 2nd ed. Itasca, IL: American Academy of Pediatrics; 2018:1018-1050. Pediatric Care Online. • Fierson WM; American Academy of Pediatrics Section on Ophthalmology; American Academy of Ophthalmology; American Association for Pediatric Ophthalmology and Strabismus; and American Association of Certified Orthoptists. Screening examination of premature infants for retinopathy of prematurity. Pediatrics. 2018;142(6):e20183061. doi:10.1542/peds.2018-3061. • Olitsky SE, Marsh JD. Disorders of the retina and vitreous.In: Kliegman RM, St Geme JW III, Blum NJ, Shah SS, Tasker RC, Wilson KM, eds. Nelson Textbook of Pediatrics. 21st ed. Philadelphia, PA: Elsevier; 2020:3377-3385.

American academy of pediatrics

336

PREP ® Self-Assessment

PREPSA 2020

Question 94 A 14-year-old girl is being evaluated for severe abdominal pain. For each of the past 3 months, she has missed 1 to 2 days of school at the start of her menses because of cramping, nausea, and vomiting. She only experiences these symptoms during menstruation. Her mother has been treating the pain with 400 mg of ibuprofen orally every 8 hours, with minimal relief. The girl had menarche at age 12 years. Her periods occur regularly every 28 days, and last for 5 days. She had never experienced menstrual cramping before the previous 3 months. Her mother has a history of fibroids, and her maternal aunt has endometriosis. The girl has a blood pressure of 108/70 mm Hg, heart rate of 80 beats/min, respiratory rate of 18 breaths/min, and temperature of 37°C. There is mild diffuse tenderness of her lower abdomen. There is no guarding, rebound tenderness, palpable masses, or costovertebral angle tenderness. The remainder of the examination findings are unremarkable. Of the following, the girl’s MOST likely diagnosis is A. endometriosis B. irritable bowel syndrome C. Mittelschmerz D. primary dysmenorrhea

American academy of pediatrics

337

PREP ® Self-Assessment

PREPSA 2020

Correct Answer: D The most likely diagnosis for the girl in the vignette is primary dysmenorrhea. Primary dysmenorrhea is the most common gynecologic condition in adolescent girls, and one of the most common reasons for adolescent girls to miss school. Defined as painful menses without pelvic pathology, it is usually associated with crampy, lower abdominal pain that may present a few days before the onset of bleeding and can last for a few days or throughout the entire period. Symptoms tend to be most severe during the first few days of bleeding. Menstrual cramps tend to begin 1 to 3 years after menarche, when a girl has ovulatory cycles, and can be associated with nausea, vomiting, diarrhea, lower back cramping, headache, and mood lability. The pathophysiology of primary dysmenorrhea is thought to be related to increased levels of uterine leukotrienes and cyclooxygenase which are converted to prostacyclins and prostaglandins. Prostaglandin F2α mediates smooth muscle contraction and stimulates pain sensation, and prostaglandin E2 causes vasodilation in the endometrium and platelet disaggregation. Girls with more symptomatic menstrual cramps have been found to have increased levels of these inflammatory mediators in their menstrual fluid. The first-line treatment for primary dysmenorrhea is a nonsteroidal anti-inflammatory agent (NSAID), which will inhibit prostaglandin production. These NSAIDs are most helpful if taken at the onset of cramping and/or 1 to 2 days before the start of the period. Use of an NSAID may be necessary for the first few days of bleeding. If NSAIDs are not effective, the next option would be a trial of a hormonal contraceptive to suppress ovulation and decrease the thickness of the endometrium, which also decreases production of prostaglandins and leukotrienes, or both hormonal and NSAID treatment. The hormonal method of choice should be based on the adolescent’s personal preference; this can include combination oral contraceptive pills, a transdermal contraceptive patch, vaginal ring, depot medroxyprogesterone injection, progestin implant, or a progestin-containing intrauterine device. If an adolescent does not have relief with a combination of an NSAID and hormonal contraceptive, an evaluation for secondary dysmenorrhea is warranted. Secondary dysmenorrhea is defined as painful menses resulting from pelvic pathology; the pain may be intermittent or constant, and may occur at times other than during menses. The most common cause of secondary dysmenorrhea is endometriosis. The differential diagnosis of secondary dysmenorrhea also includes pelvic inflammatory disease, ovarian cysts, uterine polyps, pelvic adhesions, leiomyomata (fibroids), and obstructive congenital anomalies of the reproductive tract. The girl in the vignette has signs and symptoms consistent with primary dysmenorrhea. Her symptoms occurred 2 years after menarche, when she is likely having more ovulatory cycles, and only occurs during her menstrual cycle. This makes endometriosis less likely, because she is not experiencing chronic pain. Her abdominal pain is not associated with improvement after defecation or changes in the frequency of passing stools, placing irritable bowel syndrome lower on the differential diagnosis for this girl’s abdominal pain. Mittelschmerz is often described as a dull ache at the time of ovulation, usually occurring 1 to 2 weeks before the onset of menses, and can last for a few minutes to several hours.

American academy of pediatrics

338

PREP ® Self-Assessment

PREPSA 2020

PREP Pearls • Primary dysmenorrhea is the most common gynecologic problem in adolescent girls. • Primary dysmenorrhea is the most common reason why adolescent girls are absent from school. • First-line therapy for primary dysmenorrhea is a nonsteroidal anti-inflammatory drug; if ineffective, a trial of a hormonal contraceptive method, or a combination of both are recommended. ABP Content Specifications(s) • Understand the pathophysiology of primary dysmenorrhea • Plan the appropriate management of primary dysmenorrhea Suggested Readings • Committee opinion no. 760: dysmenorrhea and endometriosis in the adolescent. Obstet Gynecol. 2018;132(6):e249-e258. doi:10.1097/AOG.0000000000002978. • Dinerman LM. Dysmenorrhea. In: McInerny TK, Adam HM, Campbell DE, DeWitt TG, Foy JM, Kamat DM, eds. American Academy of Pediatrics Textbook of Pediatric Care. 2nd ed. Elk Grove Village, IL: American Academy of Pediatrics; 2017:12921295. Pediatric Care Online. • Laufer MR. Gynecologic pain: dysmenorrhea, acute and chronic pelvic pain, endometriosis, and premenstrual syndrome. In: Emans SJ, Laufer MR, eds. Emans, Laufer, Goldstein’s Pediatric and Adolescent Gynecology. 6th ed. Philadelphia, PA: Lippincott Williams and Wilkins; 2012:238-241. • Nickles MC, Alderman E. Noncontraceptive use of contraceptive agents. Pediatr Rev. 2014;35(6):229-242. doi:10.1542/pir.35-6-229.

American academy of pediatrics

339

PREP ® Self-Assessment

PREPSA 2020

Question 95 An 8-month-old female infant is brought to the office for decreased activity level, poor feeding, and only 2 wet diapers over the past 24 hours. She has not had fever, respiratory symptoms, vomiting, or diarrhea. Her medical history is significant for infantile spasms, for which she completed a 2-month course of prednisolone (7 mg/kg per day) the previous week. Her current medications include ranitidine and a multivitamin. The girl’s temperature is 36.8°C, heart rate is 141 beats/min, blood pressure is 94/68 mm Hg, respiratory rate is 28 breaths/min, and room air oxygen saturation is 99%. Her weight is at the 70th percentile and length is at the 10th percentile. She is sleepy, but arousable. She has a round face and excess adiposity. The remainder of her physical examination findings are normal. Laboratory evaluation shows the following. Laboratory test Result White blood cell count 10.8 × 103/μL (10.8 × 109/L) Hemoglobin 12.9 g/dL (129 g/L) Platelets 332 × 103/μL (332 × 109/L) Sodium 135 mg/dL (135 mmol/L) Potassium 4.8 mg/dL (4.8 mmol/L) Chloride 99 mg/dL (99 mmol/L) Bicarbonate 16 mg/dL (16 mmol/L) Blood urea nitrogen 3.6 mg/dL (1.3 mmol/L) Creatinine 0.2 mg/dL (18 μmol/L) Glucose 100 mg/dL (5.5 mmol/L) Of the following, the MOST likely cause of this infant’s findings is A. adrenal insufficiency B. epilepsy C. inappropriate antidiuretic hormone secretion D. urinary tract infection

American academy of pediatrics

340

PREP ® Self-Assessment

PREPSA 2020

Correct Answer: A The infant in the vignette has iatrogenic adrenal insufficiency after discontinuation of a prolonged course of glucocorticoid therapy. The hypothalamic-pituitary-adrenal (HPA) axis remains suppressed for a while after exogenous steroids are discontinued. Her relatively high weight percentile, low length percentile, round face, and excess adiposity are signs of excess glucocorticoid exposure. Her symptoms of decreased activity and poor feeding, although nonspecific, as well as her hyponatremia, are consistent with adrenal insufficiency. Inappropriate antidiuretic hormone secretion is also associated with hyponatremia, but this infant does not have risk factors for this condition. Epilepsy and urinary tract infection may be considered in the differential diagnosis for an infant with these presenting symptoms, but there are no other clinical features supportive of these diagnoses. Common symptoms and signs of adrenal insufficiency include • abdominal pain • fatigue • hypoglycemia • hyponatremia, • metabolic acidosis • nausea • vomiting • weight loss Primary adrenal insufficiency, a problem with the adrenal gland itself, may present with saltcraving and dehydration. In primary adrenal insufficiency, hyponatremia is caused by mineralocorticoid deficiency, whereas in secondary adrenal insufficiency (adrenocorticotropic hormone [ACTH] deficiency) it is caused by relative water retention. Hyperkalemia is seen in primary adrenal insufficiency because of mineralocorticoid deficiency, but is not seen in secondary adrenal insufficiency because the renin-angiotensin-aldosterone system remains intact. Cortisol is low in both types. Levels of ACTH are high in primary adrenal insufficiency and low or inappropriately normal in secondary adrenal insufficiency. Item C95 outlines clinical features distinguishing primary and secondary adrenal insufficiency.

American academy of pediatrics

341

PREP ® Self-Assessment

PREPSA 2020

The HPA axis can be suppressed with prolonged administration of glucocorticoid by any route, including inhaled. Care should be taken to not abruptly withdraw steroid therapy after a prolonged course, especially oral therapy, given the potentially life-threatening consequences of an adrenal crisis. Suppression of the HPA axis after withdrawal of exogenous steroids is particularly dangerous during an acute illness, at which time stress-dose steroids would be required. When a prolonged steroid course is to be discontinued, the dose can be quickly weaned to a physiologic replacement dose. At that point, weaning to discontinuation should advance much more slowly and may be guided by morning cortisol levels. Stress-dose steroids should be provided during acute illnesses and other episodes of significant stress until full endogenous adrenal function returns.

American academy of pediatrics

342

PREP ® Self-Assessment

PREPSA 2020

PREP Pearls • Care should be taken to not abruptly withdraw steroid therapy after a prolonged course, given the potentially life-threatening consequences of adrenal insufficiency. • The hypothalamic-pituitary-adrenal axis can be suppressed with prolonged administration of glucocorticoids by any route, including inhaled. • Common symptoms and signs of adrenal insufficiency include fatigue, nausea, vomiting, abdominal pain, weight loss, hyponatremia, metabolic acidosis, and hypoglycemia. ABP Content Specifications(s) • Recognize the clinical features associated with adrenal insufficiency after exogenous corticosteroid therapy has been discontinued, and the complications associated with sudden withdrawal Suggested Readings • Auron M, Raissouni N. Adrenal insufficiency. Pediatr Rev. 2015;36(3):92-102. doi:10.1542/pir.36-3-92. • Kapadia CR, Nebesio TD, Myers SE, et al; Drugs and Therapeutics Committee of the Pediatric Endocrine Society. Endocrine effects of inhaled corticosteroids in children. JAMA Pediatr. 2016;170(2):163-70. doi:10.1001/jamapediatrics.2015.3526. • Speiser PW. Adrenal dysfunction. In: McInerny TK, Adam HM, Campbell DE, DeWitt TG, Foy JM, Kamat DM, eds. American Academy of Pediatrics Textbook of Pediatric Care. 2nd ed. Elk Grove Village, IL: American Academy of Pediatrics; 2017:16921701. Pediatric Care Online.

American academy of pediatrics

343

PREP ® Self-Assessment

PREPSA 2020

Question 96 A black male infant is born at term after a pregnancy with no complications. On routine predischarge screening at 36 hours of age, he is noted to have room air oxygen saturation of 98% on pulse oximetry in his right upper extremity and 93% in his right lower extremity. He is vigorous and nursing well. The remainder of his physical examination findings are normal. Of the following, the MOST appropriate next management step for this neonate is to A. discharge him with instructions to return in 2 days for follow-up B. observe him for an additional 24 hours before discharge C. obtain arterial blood gas D. repeat pulse oximetry in 1 hour

American academy of pediatrics

344

PREP ® Self-Assessment

PREPSA 2020

Correct Answer: D The neonate in the vignette has a discrepancy in oxygen saturation between the upper and lower extremities as measured with pulse oximetry. The immediate concern would be for differential perfusion between the upper and lower extremities. Based on the 2018 American Academy of Pediatrics guidelines for use of pulse oximetry to screen for critical congenital heart disease (Item C96), the most appropriate next step is to repeat pulse oximetry in the right extremities in 1 and 2 hours before investigating any further.

Item C96: Algorithm for screening newborns for critical congenital heart disease with pulse oximetry. Abbreviations: F, either foot; RH, right hand. Reprinted with permission from Kemper AR, Mahle WT, Martin GR, et al. Strategies for implementing screening for critical congenital heart disease. Pediatrics. 2011;128(5):e1262.

It would not be appropriate to discharge this neonate with instructions to follow up in 2 days or to observe him for 24 hours before discharge if there is a question of the integrity of the infant’s cardiovascular system. An arterial blood gas would confirm arterial oxygen saturation in the American academy of pediatrics 345

PREP ® Self-Assessment

PREPSA 2020

affected extremities, but is more invasive than needed in this situation. Repeating pulse oximetry in 1 and 2 hours is less invasive and the interval does not jeopardize the health of the newborn. Pulse oximetry is used in many settings as an estimation of arterial oxygen saturation. However, there are factors that may limit its accuracy, including motion artifact, poor perfusion at the site of measurement, skin pigmentation, presence of abnormal hemoglobins, and probe positioning. Recently, pulse oximetry measured on the lower extremity before initial hospital discharge (>24 hours after birth) has been recommended and adopted by many states as a routine newborn screening procedure to assess for critical congenital heart disease. Using a cutoff of less than 95% as abnormal, there is a 76.5% sensitivity and 99.9% specificity for detection of critical congenital heart disease. For infants with saturation between 90% and 94% and a differential of at least 3% between the upper and lower extremities, repeating measurement in 1 and 2 hours is recommended. Failure to identify the infant with critical congenital heart disease in the newborn period may result in up to 30% mortality. Critical congenital heart disease is defined as anatomic lesions that require surgery or catheter-based interventions in the first year after birth for survival. In addition to coarctation of the aorta, detected by a discrepancy between values from the right upper and lower extremities, use of pulse oximetry as a screen may detect both cyanotic and noncyanotic congenital heart diseases. Other noncardiac conditions that may be detected with pulse oximetry as part of newborn screening include septicemia, persistent pulmonary hypertension, respiratory distress syndrome, meconium aspiration, hemoglobinopathies, pneumonia, pneumothorax, and hypothermia. PREP Pearls • Pulse oximetry can be used as an estimation of arterial oxygen saturation, but it is subject to limitations; results may need corroboration by other means. • Lower extremity pulse oximetry is an appropriate screening tool for critical congenital heart disease in the neonate more than 24 hours after birth. • When using pulse oximetry to screen for critical congenital heart disease, if oximetry shows oxygen saturation of 90% to 30). The infant in the vignette has a normal DHT level and normal testosterone-to-DHT ratio. Defects that cause low testosterone levels, such as a testosterone biosynthetic defect or some rare types of congenital adrenal hyperplasia (CAH), can cause undervirilization. The infant in the vignette has a relatively high testosterone level, making these diagnoses unlikely. Excess androgen exposure in utero causes virilization of female external genitalia, resulting in 46,XX DSD. The most common cause of 46,XX DSD is CAH due to 21-hydroxylase deficiency. The defect in adrenal glucocorticoid and mineralocorticoid production results in elevated adrenocorticotropic hormone levels that stimulate the adrenal gland to make excess androgen. Congenital adrenal hyperplasia should be suspected in an infant with virilized, atypical genitalia and no palpable gonads. A male (46,XY) infant with 21-hydroxylase deficiency will have normal male genitalia. Newborn screening is critically important for identifying male infants with CAH before a salt-wasting crisis occurs.

American academy of pediatrics

427

PREP ® Self-Assessment

PREPSA 2020

PREP Pearls • In an infant with atypical genitalia, palpable gonads are almost always testes, making a 46,XY karyotype likely. • The most common 46,XX disorder of sex development is congenital adrenal hyperplasia due to 21-hydroxylase deficiency. • Congenital adrenal hyperplasia should be suspected in an infant with virilized, atypical genitalia and no palpable gonads. MOCA-Peds Objective • Recognize and manage ambiguous genitalia in a neonate. ABP Content Specifications(s) • Identify factors associated with virilization in female infants • Recognize the clinical features associated with androgen insensitivity syndrome • Understand the normal process of sex differentiation of a fetus Suggested Readings • American Academy of Pediatrics. Point-of-care quick references: disorders of sex development. Pediatric Care Online. • Antal Z, Zhou P. Congenital adrenal hyperplasia: diagnosis, evaluation, and management. Pediatr Rev. 2009;30(7):e49-e57. doi:10.1542/pir.30-7-e49. • Indyk JA. Disorders/differences of sex development (DSDs) for primary care: the approach to the infant with ambiguous genitalia. Transl Pediatr. 2017;6(4):323334. http://tp.amegroups.com/article/view/16975/17489. • Loomba-Albrecht LA, Styne DM. Disorders of sex development. In: McInerny TK, Adam HM, Campbell DE, DeWitt TG, Foy JM, Kamat DM, eds. American Academy of Pediatrics Textbook of Pediatric Care. 2nd ed. Elk Grove Village, IL: American Academy of Pediatrics; 2017:1968-1976. Pediatric Care Online. • Wherrett DK. Approach to the infant with a suspected disorder of sex development. Pediatr Clin North Am. 2015;62(4):983-999. doi:10.1016/j.pcl.2015.04.011.

American academy of pediatrics

428

PREP ® Self-Assessment

PREPSA 2020

Question 119 Towards the end of a busy emergency department shift, an attending physician is simultaneously caring for multiple patients in the same area. A 2-year-old girl with acute lymphoblastic leukemia, fever, and neutropenia and an 18-year-old young woman with dehydration and presumed vasovagal syncope are in adjacent beds. Orders are placed in the electronic medical record for both patients. A laboratory evaluation (including a blood culture), intravenous cefepime, and an admission order are placed for the 2-year-old patient. An intravenous bolus of normal saline, a capillary blood glucose level, and electrocardiography are ordered for the 18year-old patient. Soon after the orders are placed, the nurse reports that she unintentionally administered the cefepime to the 18-year-old patient instead of the 2-year-old patient. A review of the electronic medical record confirms that cefepime was ordered for the correct patient. The amount of cefepime that the 18-year-old patient received was less than 11 mg/kg. Cefepime is ordered again for the 2-year-old patient and promptly administered. An hour later, the 18-year-old patient is re-examined. She is no longer dizzy and wants to go home. Her vital signs have remained stable, she has tolerated hydration by mouth, her blood glucose level and electrocardiogram are normal, and she is ambulating around the emergency department without issue. There are no apparent adverse effects from the cefepime. Her discharge papers are prepared, and the disclosure of the medical error, which resulted in the administration of a subtherapeutic dose with no adverse effects, is contemplated. Of the following, the BEST decision in regards to reporting this error is A. do not report the error to the hospital’s adverse event database or disclose the error to the patient B. report the error to the hospital’s adverse event database and disclose the error to the patient C. report the error to the hospital’s adverse event database but do not disclose the error to the patient D. report the error to the patient but not to the hospital’s adverse event database

American academy of pediatrics

429

PREP ® Self-Assessment

PREPSA 2020

Correct Answer: B The administration of cefepime to the 18-year-old patient in this vignette was a medical error. The appropriate course of action is to report the error to the hospital adverse event database (or committee) and disclose the error to the patient, even though there is unlikely to be any adverse events stemming from the error. Any medical error that occurs or almost occurs (a “near-miss”) must be reported to the appropriate authority because these events are often indicative of a systems issue and less frequently the result of an individual error, and they must be investigated to identify the cause of the error. The creation of a central repository for medical errors allows for evaluation of error patterns and recurrences of errors and is the first step in ensuring that safeguards are ultimately put in place to prevent future medical errors. Daily medical practice is rife with medical errors. In To Err is Human, the seminal report released by the Institute of Medicine in 1999, it was estimated that up to 100,000 deaths occur each year due to medical errors. Children are at higher risk for medication errors than adults because of the need to calculate medication doses by weight or surface area, as compared to standard dosing often used for adults. Many different factors contribute to medical errors, ranging from provider fatigue and distractions to medication names that sound alike, patients with similar names, and overreliance on electronic medical records and computerized order entry. Any unanticipated event in the health care setting that puts a patient at risk of or causes serious injury or death is known as a sentinel event. Sentinel events should lead to a root-cause analysis in which every aspect leading up to the event, whether direct or indirect, is investigated in an attempt to identify the causes. Strategies can be employed by the individual and the institution to minimize the chance of a medical error. These strategies include writing orders in a quiet area, minimizing distractions, avoiding unsafe abbreviations (eg, QD for daily, QID for 4 times a day), ensuring accurate weights (in kilograms, not pounds), using leading zeros when writing numbers, and avoiding trailing zeros. Computerized order entry should be looked upon as a physician guide and not a substitute for physicians doing their own math and checking doses. Providers may be intimidated to report medical errors. This intimidation can be the result of a fear of being labeled a whistleblower and potentially suffering repercussions from colleagues or a fear of admitting fault and exposing oneself to potential litigation. Organizations should adopt practices that encourage voluntary reporting, such as instituting anonymous methods of reporting medical errors and ensuring no retaliation or repercussions for reporting medical errors, near-misses, or unsafe conditions in good faith. PREP Pearls • Medical errors are typically caused by underlying systems issues and are often not the fault of the individual. • A sentinel event is an unanticipated event that leads to serious injury or death (or risk of serious injury or death). • A near-miss is a potential adverse event that does not actually occur, either due to intentional intervention or luck.

American academy of pediatrics

430

PREP ® Self-Assessment

PREPSA 2020

ABP Content Specifications(s) • Apply voluntary systems for reporting of adverse medical events • Identify barriers to reporting adverse events • Recognize what interventions can reduce error in situations (eg, stress, fatigue, distraction) at high risk for medical error • Understand and apply methodologies to prevent medication errors • Apply effective strategies to improve reporting of adverse events Suggested Readings • Kohn LT, Corrigan J, Donaldson MS, eds. To Err is Human: Building a Safer Health System. Washington, DC: National Academy Press; 2000. http://www.nationalacademies.org/hmd/~/media/Files/Report%20Files/1999/ToErr-is-Human/To%20Err%20is%20Human%201999%20%20report%20brief.pdf. • Leonard MS. Patient safety and quality improvement: medical errors and adverse events. Pediatr Rev. 2010;31(4):151-158. doi:10.1542/pir.31-4-151. • Neuspiel DR. Medical errors, adverse events, and patient safety. In: McInerny TK, Adam HM, Campbell DE, DeWitt TG, Foy JM, Kamat DM, eds. American Academy of Pediatrics Textbook of Pediatric Care. 2nd ed. Elk Grove Village, IL: American Academy of Pediatrics; 2017:2287-2295. Pediatric Care Online.

American academy of pediatrics

431

PREP ® Self-Assessment

PREPSA 2020

Question 120 A 9-month-old infant is being evaluated for a rash. He was febrile earlier in the week with temperatures as high as 39.4°C, but has been afebrile for the past 24 hours. This morning, his mother noted a red rash on his neck and trunk; the rash has spread and is now present on his arms and legs as well. He is fussy and not eating normally but is drinking and has appropriate urine output. He has an erythematous, blanching, maculopapular rash confluent on his neck, chest, and upper back, but involving his whole trunk and proximal extremities. His conjunctiva and oropharynx appear normal. Of the following, the virus MOST likely causing this infant’s symptoms is A. Epstein-Barr virus B. human herpesvirus type 6 C. measles D. rubella

American academy of pediatrics

432

PREP ® Self-Assessment

PREPSA 2020

Correct Answer: B The infant in the vignette had a high fever followed by a blanching maculopapular rash that began after the fever abated. These symptoms are not consistent with Epstein-Barr virus, measles, or rubella infection, but are characteristic of roseola infantum or “sixth disease,” caused by human herpesvirus type 6 (HHV-6) infection. Primary HHV-6 infection in an immunocompetent child can cause a nonspecific acute febrile illness with or without a rash or, in approximately 25% of infected children, can cause the full clinical syndrome of roseola infantum. Roseola is characterized by 3 to 5 days of high fever (typically >39.5°C) which resolves suddenly, at which time a rash appears. The blanching, maculopapular rash can last for hours to days, typically starting on the neck and trunk before spreading to the face and limbs (Item C120). Approximately 10% to 15% of children with primary HHV-6 infection have an associated febrile seizure. Nearly all children acquire HHV-6 infection before age 2 years.

Item C120: Roseola rash. Copyright Stan Block, MD and reprinted with permission from Kimberlin DW et al, eds. Red Book Online. Itasca, IL: American Academy of Pediatrics. Like other viruses in the herpesvirus family, HHV-6 remains latent in the body after primary infection and may reactivate, particularly in immunocompromised patients. Reactivation of HHV-6 is a common cause of encephalitis in patients after hematopoietic stem cell or solid organ transplantation. Diagnosis of HHV-6 infection is primarily clinical, though laboratory testing (quantitative polymerase chain reaction and/or serologic testing) may be beneficial for immunocompromised children with symptoms concerning for HHV-6 reactivation. Treatment is typically supportive, with careful attention to hydration status. Antiviral medications may be used in consultation with an infectious disease expert in immunocompromised or severely ill children. American academy of pediatrics

433

PREP ® Self-Assessment

PREPSA 2020

PREP Pearls • Roseola infantum (“sixth disease”) is caused by human herpesvirus type 6 infection. • Typical symptoms of roseola include a high fever that lasts between 3 and 5 days before abruptly resolving, followed by the development of a blanching maculopapular rash that starts on the neck and trunk and spreads to the face and extremities. ABP Content Specifications(s) • Recognize the clinical features associated with human herpesvirus type 6 infection Suggested Readings • Agut H, Bonnafous P, Gautheret-Dejean A. Laboratory and clinical aspects of human herpesvirus 6 infections. Clin Microbiol Rev. 2015;28(2):313-325. doi:10.1128/CMR.00122-14. • American Academy of Pediatrics. Human herpesvirus 6 (including roseola) and 7. In: Kimberlin DW, Brady MT, Jackson MA, Long SS, eds. Red Book: 2018 Report of the Committee on Infectious Diseases. 31st ed. Itasca, IL: American Academy of Pediatrics; 2018:454457. https://redbook.solutions.aap.org/chapter.aspx?sectionId=189640111&bookId=2205 &resultClick=1. • Hall CB, Long CE, Schnabel KC, et al. Human herpesvirus-6 infection in children: a prospective study of complications and reactivation. N Engl J Med. 1994;331(7):432-438. doi:10.1056/NEJM199408183310703. • Watson JR, Mejias A. Human herpesvirus-6 and human herpesvirus-7 infections. In: McInerny TK, Adam HM, Campbell DE, DeWitt TG, Foy JM, Kamat DM, eds. American Academy of Pediatrics Textbook of Pediatric Care. 2nd ed. Elk Grove Village, IL: American Academy of Pediatrics; 2017:2149-2153. Pediatric Care Online.

American academy of pediatrics

434

PREP ® Self-Assessment

PREPSA 2020

Question 121 A 6-month-old male infant is brought to the emergency department by his mother after being found in his crib blue, gasping, and unarousable. He is unresponsive, not breathing, and pulseless. Chest compressions and rescue breathing are initiated. Electrocardiography reveals pulseless electrical activity. After 2 minutes of attempts, intravenous access is unsuccessful. Of the following, the BEST option to obtain access to administer epinephrine is to A. B. C. D.

continue attempts to place a peripheral intravenous catheter perform venous cutdown place a central venous catheter place an intraosseous needle

American academy of pediatrics

435

PREP ® Self-Assessment

PREPSA 2020

Correct Answer: D The infant in this vignette is in cardiac arrest. It is necessary to obtain vascular access to administer epinephrine. Since attempts to place an intravenous (IV) catheter have been unsuccessful after 2 minutes, the best next step is to place an intraosseous (IO) needle. The Pediatric Cardiac Arrest Algorithm of the 2015 American Heart Association (AHA) Pediatric Advanced Life Support (PALS) Guidelines recommends starting cardiopulmonary resuscitation (CPR) immediately, giving oxygen via bag-valve mask ventilation, and attaching a monitor/defibrillator (see de Caen AR et al, doi:10.1542/peds.2015-3373F). If asystole/pulseless electrical activity is detected on the monitor (as described in the vignette), a dose of epinephrine should be administered after 2 minutes of CPR. During these initial 2 minutes of CPR, IV access should be attempted, and an IO needle should be prepared in case the IV attempt is unsuccessful. In children with shock as well as cardiac arrest, AHA PALS guidelines recommend establishing IV or IO access early during resuscitation. Medications and fluids administered intraosseously enter the bloodstream as quickly as when administered intravenously. In a large meta-analysis, the IO route achieved more frequent successful access and improved time to achieving access as compared to peripheral and central venous techniques in patients requiring emergent vascular access in the emergency department. Contraindications to IO placement include orthopedic hardware, fracture, infection at the site of insertion, and bony pathology, such as osteogenesis imperfecta. The promptness of achieving access in shock or cardiac arrest is more important than the site. The most common site for IO needle placement in children is the intramedullary space of the proximal tibia. The patient should be placed in the supine position with a towel roll under the knee. The skin overlying the flat area of the bone medial to the tibial tuberosity should be prepped with betadine or chlorhexidine. The needle should be positioned at a 90° angle to the skin. If using the manual insertion technique, downward pressure with a back-and-forth twisting motion should be applied to puncture the skin and bone. Resistance will drop when the needle enters the marrow cavity. The stylet is removed and 5 mL of fluid instilled while evaluating surrounding tissue for evidence of extravasation. If the needle is correctly placed, it can safely be used for rapid administration of fluid, vasoactive medications and infusions, and antibiotics. Other possible sites for IO placement include the proximal humerus, distal tibia, and iliac crest. A commercially available drill for IO placement was approved by the US Food and Drug Administration in 2004 and is now used extensively. The technique for insertion of an IO needle can be found at https://www.nejm.org/doi/full/10.1056/NEJMvcm0900916. A central venous catheter or a venous cutdown could be an effective and secure method for resuscitation in shock or cardiac arrest; however, these methods are too time consuming and more dependent on the skills of the operator. Repeated attempts at placing an IV catheter could lead to unnecessary delays in obtaining vascular access.

American academy of pediatrics

436

PREP ® Self-Assessment

PREPSA 2020

PREP Pearls • If attempts at placing an intravenous catheter in the first 2 minutes are unsuccessful in a patient who is in shock or cardiac arrest, an intraosseous needle should be placed. • The most common site for intraosseous needle placement in children is the intramedullary space of the proximal tibia medial to the tibial tuberosity. ABP Content Specifications(s) • Plan the appropriate use of intraosseous therapy Suggested Readings • Cornell T, Schmidt JP, Custer JR. Outpatient procedures. In: McInerny TK, Adam HM, Campbell DE, DeWitt TG, Foy JM, Kamat DM, eds. American Academy of Pediatrics Textbook of Pediatric Care. 2nd ed. Elk Grove Village, IL: American Academy of Pediatrics; 2017:3027-3047. Pediatric Care Online. • de Caen AR, Berg MD, Chameides L, et al. Part 12: Pediatric Advanced Life Support: 2015 American Heart Association Guidelines Update for Cardiopulmonary Resuscitation and Emergency Cardiovascular Care (reprint). Pediatrics.2015;136(suppl 2):S176-S195. doi:10.1542/peds.2015-3373F. • Nagler J, Krauss B. Videos in clinical medicine. Intraosseous catheter placement in children. N Engl J Med. 2011;364(8):e14. doi:10.1056/NEJMvcm0900916. • Pifko EL, Price A, Busch C, et al. Observational review of paediatric intraosseous needle placement in the paediatric emergency department. J Paediatr Child Health. 2018;54(5):546-550. doi:10.1111/jpc.13773. • Voigt J, Waltzman M, Lottenberg L. Intraosseous vascular access for in-hospital emergency use: a systematic clinical review of the literature and analysis. Pediatr Emerg Care. 2012;28(2):185-199. doi:10.1097/PEC.0b013e3182449edc.

American academy of pediatrics

437

PREP ® Self-Assessment

PREPSA 2020

Question 122 A 3-month-old female infant is seen for concerns of poor growth and progressive abdominal distention. She is breastfed exclusively every 3 hours, although she has been less interested in feeding for the last 2 weeks. She has occasional nonbilious, nonbloody emesis with feeds. Her stools are yellow, greasy, and malodorous, occurring multiple times daily. She has not had any fevers. She was delivered at home at 38 weeks’ gestation (birth weight, 3.5 kg), she passed meconium within 24 hours, and she initially breastfed well. Her parents declined newborn screening. She has a temperature of 37.2°C, weight of 4.5 kg (3rd percentile for age), length of 55 cm (3rd percentile for age), and weight-for-length at the 50th percentile for age. She appears thin. Her abdomen is distended with a fluid wave. Her liver edge is firm and palpable 4 cm below the right costal margin. Her spleen is not palpable. She has severe diaper dermatitis with ulceration. She has 1+ pitting edema in bilateral lower extremities. Laboratory data are shown: Laboratory Test Sodium Potassium Chloride Carbon dioxide Blood urea nitrogen Creatinine Glucose Bilirubin, total Alanine aminotransferase Aspartate aminotransferase Albumin White blood cell count Neutrophils Lymphocytes Monocytes Hemoglobin Hematocrit Platelet count Prothrombin time International normalized ratio (INR) Cholesterol Triglycerides

Result 138 mEq/L (138 mmol/L) 4.6 mEq/L (4.6 mmol/L) 107 mEq/L (107 mmol/L) 21 mEq/L (21 mmol/L) 4 mg/dL (1.4 mmol/L) 0.3 mg/dL (26.5 µmol/L) 85 mg/dL (4.7 mmol/L) < 0.5 mg/dL (8.6 µmol/L) 62 U/L 106 U/L 1.7 mg/dL 13,300/µL (13.3 × 109/L) 37% 61% 2% 11.8 g/dL (118 g/L) 35.4% 420 × 103/µL (420 × 109/L) 16.9 s 1.4 71 mg/dL (1.8 mmol/L) 126 mg/dL (1.4 mmol/L)

Abdominal ultrasonography reveals an enlarged and heterogeneous-appearing liver without focal mass.

American academy of pediatrics

438

PREP ® Self-Assessment

PREPSA 2020

Of the following, the test MOST likely to establish the diagnosis is A. B. C. D.

bone marrow aspiration serum ceruloplasmin sweat chloride test urine cytomegalovirus polymerase chain reaction

American academy of pediatrics

439

PREP ® Self-Assessment

PREPSA 2020

Correct Answer: C The infant in this vignette has steatorrhea (suggested by frequent greasy, malodorous stools in the context of severe diaper dermatitis), malnutrition (failure to thrive, hypoalbuminemia with ascites), and hepatomegaly consistent with a diagnosis of cystic fibrosis–associated liver disease. The best test to establish the diagnosis of cystic fibrosis is a sweat chloride test. Hepatomegaly is detected through physical examination by percussion of the liver span and palpation of the liver edge. Normal liver span varies by age. In the neonate, normal liver span is between 4.5 and 5 cm. In older children, normal liver span is between 6 and 8 cm. In infants and young children, palpation of the liver edge below the right costal margin can be normal. In neonates, a liver edge greater than 3.5 cm below the right costal margin is considered abnormal. In children, a liver edge greater than 2 cm below the costal margin is considered abnormal. Palpation of the liver and spleen should begin in the lower abdomen/pelvis because massive hepatomegaly and/or splenomegaly can be missed with palpation only in the upper abdomen. In cases of significant lung hyperinflation or air or fluid accumulation in the thorax, the liver can be pushed inferiorly. Thus, it is ideal to both percuss the entire liver span and palpate the liver edge when assessing liver size. The texture of the liver edge is important to describe. A soft liver edge is normal, however a firm or hard liver edge is suggestive of liver disease. The presence of splenomegaly in addition to hepatomegaly on examination suggests portal hypertension, metabolic storage disease (eg, glycogen storage disease, Niemann-Pick disease, Gaucher disease), infiltrative diseases (metastatic disease, leukemia or lymphoma, hemophagocytic lymphohistiocytosis), or congestive heart failure. Infectious causes and primary hepatic malignancy (hepatoblastoma in infants and children and hepatocellular carcinoma in older children and adolescents) should always be in the differential diagnosis of hepatomegaly. Portal hypertension may present with hepatosplenomegaly or isolated splenomegaly with a small liver with a hard liver edge (caused by cirrhosis). In addition to splenomegaly, fatigue, jaundice, palmar erythema, spider angiomata, and ascites/abdominal distention may be present with portal hypertension. These possible findings are important to assess on physical examination because if they are present with acute liver injury/acute liver failure, chronic liver disease should be suspected. Laboratory features of portal hypertension include elevated transaminase levels and thrombocytopenia or pancytopenia. Initial evaluation of an infant or child with hepatomegaly should include: complete metabolic panel including liver function tests, complete blood cell count, coagulation panel, evaluation for infectious causes, and abdominal ultrasonography with Doppler flow. Further testing should be tailored to the results of the history, physical examination, and laboratory/radiographic evaluation. In this vignette, bone marrow aspiration is not indicated because there are no other physical examination findings or laboratory results indicating an oncologic etiology (ie, no splenomegaly and a normal complete blood cell count). Serum ceruloplasmin would be the test of choice for Wilson disease; however, this disease does not present in infants or young children. Cytomegalovirus (CMV) polymerase chain reaction would test for CMV infection. American academy of pediatrics

440

PREP ® Self-Assessment

PREPSA 2020

However, CMV infection would not result in the severe chronic steatorrhea and failure to thrive described in this vignette. PREP Pearls • Hepatomegaly is determined by percussion of the liver span and palpation of the liver edge. • Liver size varies by age, and displacement of the liver can occur with abnormalities in the diaphragm or thoracic organs. • Physical examination findings of splenomegaly, palmar erythema, spider angiomata, or ascites should raise concern for portal hypertension. ABP Content Specifications(s) • Identify the physical and laboratory features associated with portal hypertension • Understand the significance of hepatomegaly with or without splenomegaly in children of various ages, and evaluate appropriately Suggested Readings • Chapin CA, Bass LM. Cirrhosis and portal hypertension in the pediatric population. Clin Liver Dis. 2018;22(4):735-752. doi:10.1016/j.cld.2018.06.007. • Clemente MG, Schwarz K. Hepatitis: general principles. Pediatr Rev. 2011;32(8):333340. doi:10.1542/pir.32-8-333. • Ozuah PO, Reznik M. Hepatomegaly. In: McInerny TK, Adam HM, Campbell DE, DeWitt TG, Foy JM, Kamat DM, eds. American Academy of Pediatrics Textbook of Pediatric Care. 2nd ed. Elk Grove Village, IL: American Academy of Pediatrics; 2017:1427-1429. Pediatric Care Online. • Wolf AD, Levine JE. Hepatomegaly in neonates and children. Pediatr Rev. 2000;21(9):303-310. doi:10.1542/pir.21-9-303.

American academy of pediatrics

441

PREP ® Self-Assessment

PREPSA 2020

Question 123 A 6-year-old boy is seen for evaluation after several weeks of a limp and increasing pain in the right hip and knee. The pain is worsening, especially with activity. He reports no acute injury, but he did start playing soccer one month ago. He has tried to continue with soccer, but he is no longer able to run without a significant limp. He is otherwise healthy and reports no other medical issues. He is uncomfortable during the physical examination. His vital signs are normal for age. His body mass index is tracking along the 35th percentile. He has an antalgic gait and prefers to hold his right leg in external rotation. Knee examination findings are unremarkable, without evidence of swelling or pain with active and passive range of motion. He has pain and restriction with active and passive flexion of the right hip and marked weakness when testing hip flexion strength. His back is normal on inspection and appears to have full range of motion. He is intact to light touch and deep tendon reflexes in the right distal extremity. Radiographs are shown in Item Q123.

Item Q123: Anteroposterior radiograph of the pelvis of the patient described in the vignette. Reprinted with permission from Bunnell WP. Pediatr Rev. 1986;7(10):301.

Of the following, the BEST next step in management is to A. order a complete blood cell count, C-reactive protein level, and erythrocyte sedimentation rate B. place him on crutches and instruct him on non–weight bearing C. refer him for physical therapy D. start nonsteroidal anti-inflammatory medication

American academy of pediatrics

442

PREP ® Self-Assessment

PREPSA 2020

Correct Answer: B The boy in this vignette has Legg-Calvé-Perthes disease (LCPD). Once LCPD is identified, steps should be taken to prevent collapse of the femoral head. This patient should be provided crutches and instructed on the importance of non–weight bearing while awaiting orthopedic consultation. Legg-Calvé-Perthes disease is the idiopathic interruption of blood supply to the femoral head epiphysis, with subsequent development of avascular necrosis. The blood supply is generally restored over 2 to 4 years, and the epiphysis undergoes serial fragmentation, collapse, and subsequent regeneration. Symptom onset is typically between 2 and 12 years of age but most commonly occurs in boys aged 6 to 8 years. Patients who recover from LCPD with a normally shaped femoral head have a favorable natural history with maintenance of full hip function. However, patients who experience collapse or deformity of the femoral head may have ongoing pain and loss of function and will progress to early osteoarthritic change. Favorable outcomes are more likely in children younger than 7 years and with early recognition and treatment. Given the critical role of early detection in the long-term prognosis for LCPD, the primary goal for pediatricians is to identify LCPD as early as possible. Providers should consider the potential for interarticular hip pathology in any patient presenting with a limp and painful, restricted hip range of motion. There should be a low threshold for obtaining radiographs in these patients, which should include an anteroposterior view of the pelvis and a “frog leg” lateral view of the hip. It is important to remember that hip pathology may present as knee pain, and the hip should always be examined in patients presenting with knee pain. As soon as LCPD is identified, patients should be made non–weight bearing and instructed on activity restriction to reduce strain across the femoral head. Prompt orthopedic consultation should also be requested. Over-thecounter analgesics or anti-inflammatory medications may assist with comfort but are not a key component to the treatment plan. Progressive extrusion of the femoral head out from under the roof of the acetabulum appears to be a dominant factor leading to collapse of the epiphysis. Current treatment focuses on maintaining containment of the femoral head within the acetabulum. Children who are older than 8 years or who show radiographic signs of femoral head extrusion have improved outcomes with surgical intervention to reposition the femoral head within the acetabulum. Children younger than 6 years tend to have favorable outcomes with either conservative or surgical management, but they need to be monitored closely. Physical therapy may be beneficial in restoring strength and function as patients recover from LCPD, but it is not indicated early in the disease process. Patients with clinical examination findings suggestive of hip pathology but who have normal radiographs should undergo additional testing to determine the potential for an inflammatory or infectious process in the hip. This testing should include a complete blood cell count, erythrocyte sedimentation rate, and C-reactive protein level. In children who are febrile or have signs of systemic illness, a blood culture should be obtained. Lyme testing should be considered in areas where Lyme disease is endemic.

American academy of pediatrics

443

PREP ® Self-Assessment

PREPSA 2020

PREP Pearls • Pain with hip flexion and rotation suggests a possible interarticular process in the hip and should prompt a low threshold for imaging. • Patients with Legg-Calvé-Perthes disease should be made non–weight bearing while awaiting orthopedic evaluation. ABP Content Specifications(s) • Formulate a differential diagnosis of avascular necrosis (Legg-Calvé-Perthes disease) in a patient with a limp • Understand the natural history of avascular necrosis (Legg-Calvé -Perthes disease) Suggested Readings • Herman MJ, Martinek M. The limping child. Pediatr Rev.2015;36(5):184-197. doi:10.1542/pir.36-5-184. • Nguyen NT, Klein G, Dogbey G, et al. Operative versus nonoperative treatments for Legg-Calvé-Perthes disease: a meta-analysis. J Pediatr Ortho. 2012;32(7):697-705. doi:10.1097/BPO.0b013e318269c55d. • Pacicca DM. Osteochondroses. In: McInerny TK, Adam HM, Campbell DE, DeWitt TG, Foy JM, Kamat DM, eds. American Academy of Pediatrics Textbook of Pediatric Care. 2nd ed. Elk Grove Village, IL: American Academy of Pediatrics; 2017:24412446. Pediatric Care Online. • Shah H. Perthes disease: evaluation and management. Orthop Clin North Am. 2014;45(1):87-97. doi:10.1016/j.ocl.2013.08.005.

American academy of pediatrics

444

PREP ® Self-Assessment

PREPSA 2020

Question 124 A 16-year-old boy with cystic fibrosis and end-stage lung disease is at the office for a scheduled follow-up appointment. He and his family have decided that they are not interested in pursuing lung transplantation and his pulmonary team has determined that he is receiving maximum medical therapy. An advanced directive is discussed with the boy and his parents, who ask what purpose an advanced directive serves. Of the following, the BEST response is that it provides A. a plan to dictate end-of-life care B. a do-not-intubate order C. a do-not-resuscitate order D. home hospice support

American academy of pediatrics

445

PREP ® Self-Assessment

PREPSA 2020

Correct Answer: A An advanced directive is a written document that describes the wishes of a child or the family (as appropriate) regarding the important elements of advanced care planning including, but not limited to, end-of-life care. Given that, the correct response for the child in the vignette is a "plan to dictate end of life care"; the other response choices may be included as a part of that plan. The field of pediatric palliative care is evolving and has undergone significant growth in the last decades. The focus of palliative care is on the physical, social, psychological, and spiritual needs of the child and his/her family, both before and after death. Palliative care can be appropriately offered from the time of a life-threatening diagnosis through death, disease "control," or even cure. Palliative care is often provided by a multidisciplinary team. Common elements of expert care provided by the palliative care team include treatment (pharmacologic and nonpharmacologic), identification of fears, coping techniques, bereavement support, spiritual assessment, help with identifying decision makers, and aid in the discussion of illness trajectory and goals of care. Important barriers to the provision of palliative care include the fact that advances in the care of previously terminal illnesses or complex conditions may result in false hope for some families; prognostication remains complicated and medical providers are sometimes wrong; and legal policies differ from state to state, which can add complexity to situations that include minors. PREP Pearls • Advanced directives are written documents that describe the wishes of a child or the family regarding the important elements of advanced care planning, including, but not limited to end-of-life care. • The focus of palliative care is on the physical, social, psychological, and spiritual needs of the child and his/her family, before and after death. It can be appropriately offered at the time of diagnosis of a life-threatening condition through death, disease “control,” or even cure. ABP Content Specifications(s) • Understand the use of advance directives in pediatrics • Recognize and apply ethical principles involved in the patient-parent-pediatrician relationship regarding advance care planning Suggested Readings • Himelstein BP, Hilden JM, Boldt AM, Weissman D. Pediatric palliative care. N Engl J Med. 2004;350(17):1752-1762. doi:10.1056/NEJMra030334. • Liberman DB, Phung PK, Nager AL. Pediatric advance directives: parents’ knowledge, experience and preferences. Pediatrics. 2014;134(2):e436-e443. doi:10.1542/peds.20133124. • Zinner SE. The use of pediatric advance directives: a tool for palliative care physicians. Am J Hospice Palliative Med. 2009;25(6):427-430. doi:10.1177/1049909108322294.

American academy of pediatrics

446

PREP ® Self-Assessment

PREPSA 2020

Question 125 A 6-month-old infant is seen for a health supervision visit in the spring. She is developing normally, and her mother has no concerns. During anticipatory guidance, outdoor safety is reviewed. The mother asks about how to best protect her infant from insects. Of the following, the BEST option to address the mother’s concern is A. B. C. D.

an aerosolized spray with 30% DEET a backyard bug zapper a citronella oil and lemon eucalyptus wristband a combination product with 30 SPF sunscreen and 30% DEET

American academy of pediatrics

447

PREP ® Self-Assessment

PREPSA 2020

Correct Answer: A Questions regarding tick and mosquito repellents, including what methods are safe and most effective, are common in pediatric practice. Repellents with DEET (N,N-diethyl-3methylbenzamide) are commonly used and are the most effective in protecting infants and children from ticks and mosquitos. The duration of protection depends on the concentration of DEET. Products containing 10% DEET typically last about 2 hours, and products with 24% DEET typically last around 5 hours. There is no known benefit in duration of protection with concentrations of DEET greater than 30% to 50%. Of the response choices, an aerosol of 30% DEET is the best option to protect a 6-month-old infant from ticks and mosquitos. The American Academy of Pediatrics states that it is safe to use DEET-containing products on children as young as 2 months of age. The product should be applied to clothing and only exposed skin. Children should be bathed at the end of the day to remove repellant from the skin, and their clothes should be washed before wearing again. Other safety precautions include avoiding direct contact to the face or any wounds, encouraging the use of products to be supervised by an adult, and only reapplying if needed. A backyard bug zapper would not be recommended because it can actually attract more bugs to the area. Wristbands soaked in repellents such as citronella or eucalyptus are likely safe but do not offer adequate protection. There are topical repellents made from eucalyptus or oil of lemon that appear to repel mosquitos for about 1 to 2 hours. Testing for the ability to repel ticks is still needed, and allergic reactions to the essential oils can occur. Combination products with sunscreen and DEET are not recommended because sunscreen needs to be reapplied every 1 to 2 hours but DEET-containing products should be reapplied infrequently. The DEET can also make the sunscreen less effective. PREP Pearls • Products containing DEET are safe in children 2 months of age and older. • Higher concentrations of DEET provide longer protection. • Concentrations of DEET greater than 30% provide little added benefit for protection from ticks and mosquitos. ABP Content Specifications(s) • Identify the measures to prevent tick- and mosquito-borne infections • Advise parents regarding the appropriate use of topical insect repellants in children

American academy of pediatrics

448

PREP ® Self-Assessment

PREPSA 2020

Suggested Readings • American Academy of Pediatrics. Choosing an insect repellent for your child. HealthyChildren.org. https://www.healthychildren.org/English/safety-prevention/atplay/Pages/Insect-Repellents.aspx. • American Academy of Pediatrics. Patient education handouts: parent’s guide to insect repellents, A. Pediatric Patient Education. • Mutebi J-P, Hawley WA, Brogdon WG. Protection against mosquitoes, ticks, and arthropods. In: Brunette GW, ed. CDC Health Information for International Travel 2018 (Yellow Book 2018). New York, NY: Oxford University Press; 2017:8187. https://wwwnc.cdc.gov/travel/yellowbook/2018/the-pre-travelconsultation/protection-against-mosquitoes-ticks-other-arthropods.

American academy of pediatrics

449

PREP ® Self-Assessment

PREPSA 2020

Question 126 A 6-year-old boy is seen for a health supervision visit. He was born prematurely at 30 weeks of gestation. His weight is at the 50th percentile, and his height is at the 75th percentile. He has a heart rate of 82 beats/min, respiratory rate of 16 breaths/min, and blood pressure of 122/82 mm Hg in the right arm that was measured by using an oscillometric device. A repeat blood pressure measured by the auscultatory method is 118/80 mm Hg. The normative blood pressure tables are shown in Item Q126.

Item Q126: Normative Blood Pressure Tables for Boys.

American academy of pediatrics

450

PREP ® Self-Assessment

PREPSA 2020

Reprinted with permission from Flynn JT, Kaelber DC, Baker-Smith CM, et al. Clinical practice guideline for screening and management of high blood pressure in children and adolescents. Pediatrics. 2017;140(3):9.

Of the following, the BEST next step in evaluation of this child is to A. B. C. D.

obtain electrocardiography obtain renal ultrasonography refer to a cardiologist repeat blood pressure in 2 weeks

American academy of pediatrics

451

PREP ® Self-Assessment

PREPSA 2020

Correct Answer: D The child in this vignette has stage 1 hypertension as defined by average systolic and/or diastolic blood pressure (BP) greater than or equal to 95th percentile for age, sex, and height. Pediatric hypertension guidelines were updated in 2017 and underwent significant changes as compared to the "Fourth Report on the Diagnosis, Evaluation and Treatment of High Blood Pressure in Children and Adolescents." One of the key changes is that the current guidelines use normative BP tables based on normal weight children and do not include values for children with overweight and obesity. In the current guideline, the term prehypertension is replaced with elevated blood pressure. The updated definition and stages of hypertension are: normal BP (< 90th percentile); elevated BP (≥ 90th percentile to < 95th percentile); stage 1 hypertension (≥ 95th percentile to 99% • Trisomy 18: 97% • Trisomy 13: 91% • Sex chromosomal aneuploidy: 91% • Microdeletion/microduplication: Uncertain Cell-free DNA originates from the placenta, mostly from the cytotrophoblast; the test can be performed after 9 weeks’ gestation. The chromosomal makeup of the placenta is typically identical to that of the fetus; however, sometimes a mutation is confined to the placenta and the fetus has a normal karyotype. Placental-fetal discordance can occur in 1% to 2% of cases and is known as confined placental mosaicism. A fetus with confined placental mosaicism is at risk for intrauterine growth restriction. Other reasons for placental-fetal discordance in cfDNA include a vanishing twin, chromosomal anomalies in the mother, or chromosomal anomalies resulting from a maternal malignancy. Therefore, an abnormal cfDNA result always warrants confirmation with a diagnostic test such as a fetal karyotype or prenatal microarray, especially before considering termination of a pregnancy. Chorionic villus sampling can be performed at 10 to 13 weeks’ gestation, and amniocentesis is ideally performed after 15 weeks’ gestation. In this specific case, a diagnostic fetal karyotype through amniocentesis would be recommended. In addition, this mother’s maternal quadruple screen was abnormal and also indicative of a trisomy 18 diagnosis. This screening test is performed between 15 and 22 weeks of pregnancy; it measures 4 substances in the maternal blood: α-fetoprotein, human chorionic gonadotropin, estriol, and inhibin-A. A mathematical calculation is used to find a numeric risk for certain chromosomal abnormalities or defects in the fetus. This is done by comparing known normative levels for the quadruple screen markers for that specific week of gestation with levels seen in the current pregnancy, along with considerations for maternal age, weight, race, and diabetic American academy of pediatrics

614

PREP ® Self-Assessment

PREPSA 2020

status. Item C172 describes typical patterns seen on quadruple screen testing with certain aneuploidies or an open neural tube defect:

The prenatal detection rate for trisomy 21, trisomy 18, and neural tube defects is approximately 80% with maternal quadruple screen alone, with a 5% false-positive rate. The rate of detection is even higher when results are combined with first-trimester serum screening, cell-free fetal DNA, and targeted ultrasonography. Health care providers also use prenatal ultrasonography to monitor an infant’s growth and evaluate for nuchal thickening or other birth defects. If anomalies are noted, a maternal-fetal medicine referral may be initiated to perform targeted high-risk ultrasonography which has better resolution of fetal anatomy and can detect up to 50% of major structural abnormalities. In clinical scenarios in which multiple anomalies are identified, a fetal karyotype will be ordered; if this is unremarkable, a prenatal chromosomal microarray would be indicated. This diagnostic approach can result in significantly increased identification of pathogenic gene variants, but is accompanied by increased detection of variants of unknown clinical significance, which can complicate genetic counseling and prognostic discussions.

American academy of pediatrics

615

PREP ® Self-Assessment

PREPSA 2020

The recent explosion of genomics in the world of maternal-fetal medicine has vastly improved our ability to diagnose trisomies, other chromosomal aberrations, and various genetic syndromes before birth. It is now possible to order prenatal customized gene panels for situations such as skeletal dysplasias and even proceed to prenatal exome sequencing. Exome sequencing in adult and pediatric patients with developmental delay, dysmorphic features, and anomalies has a diagnostic yield of 30%. Prenatal exome sequencing has a distinct role in its ability to identify a molecular diagnosis in a fetus with a normal karyotype, chromosomal microarray, and targeted molecular sequencing panel that has failed to elucidate a specific diagnosis. The challenges resulting from this advancement in technology include counseling dilemmas because of uncertain results or clinical significance, high cost, and the lack of genetic providers to provide adequate counseling and informed consent. PREP Pearls • The maternal quadruple screen is a serum screening test performed between 15 and 22 weeks of pregnancy that measures 4 substances in the blood (α-fetoprotein, human chorionic gonadotropin, estriol, and inhibin-A) to ascertain risk for certain trisomies or open neural tube defects based on result patterns. • Maternal noninvasive cell-free DNA analysis is not a diagnostic test, but rather a screen for aneuploidies that should be followed up by a confirmatory diagnostic test such as an amniocentesis or chorionic villus sampling with fetal karyotype or prenatal chromosomal microarray. • Maternal cell-free fetal DNA analysis does not detect microdeletions, microduplications, unbalanced translocations, or single gene mutations. ABP Content Specifications(s) • Understand the usefulness of maternal blood screening in prenatal diagnosis • Understand the role of fetal ultrasonography in prenatal diagnosis • Understand which genetic disorders can be diagnosed prenatally • Understand the role and limitations of prenatal genetic testing Suggested Readings • Chen CP, Chien SC. Prenatal sonographic features of trisomy 13. J Med Ultrasound. 2007;15(1):58-66. doi:10.1016/S0929-6441(08)60024-7. • Chen CP, Chien SC. Prenatal sonographic features of trisomy 18. J Med Ultrasound. 2007;15(1):67-76. doi:10.1016/S0929-6441(08)60025-9. • Committee on Genetics and the Society for Maternal-Fetal Medicine. Committee opinion no. 682: microarrays and next-generation sequencing technology: the use of advanced genetic diagnostic tools in obstetrics and gynecology. Obstet Gynecol. 2016;128(6):e262e268. doi:10.1097/AOG.0000000000001817. • Gregg AR, Gross SJ, Best RG, et al. ACMG statement on noninvasive prenatal screening for fetal aneuploidy. Genet Med. 2013;15(5):395-398. doi:10.1038/gim.2013.29. • Normand EA, Braxton A, Nassef S, et al. Clinical exome sequencing for fetuses with ultrasound abnormalities and a suspected Mendelian disorder. Genome Med. 2018;10(1):74. doi:10.1186/s13073-018-0582-x.

American academy of pediatrics

616

PREP ® Self-Assessment

PREPSA 2020

Question 173 A 16-year-old adolescent boy is seen for concerns of abdominal pain and loose stools. His symptoms began several years ago but have worsened over the last year. He now has pain several times a week, accompanied by loose bowel movements. Defecation seems to improve his abdominal pain. He reports fecal urgency and nocturnal stooling, although he has not seen blood in his stool. He has missed 22 days of school in the last year because of these symptoms. He has a history of exercise-induced asthma. His father has psoriasis. He does not take any prescribed or over-the-counter medications. His weight is greater than the 99th percentile for age, his height is at the 62nd percentile for age, and his body mass index is greater than the 99th percentile for age. He is in no apparent distress. His abdominal and rectal examination findings are normal. His stool is negative for occult blood. Laboratory and endoscopic evaluation (upper intestinal endoscopy and colonoscopy) are normal. Irritable bowel syndrome is suspected. Of the following, the component of his history that BEST meets the diagnostic criteria for the suspected diagnosis is A. family history of psoriasis B. improvement in abdominal pain with defecation C. more than 20 missed days of school in 1 year D. nocturnal stooling

American academy of pediatrics

617

PREP ® Self-Assessment

PREPSA 2020

Correct Answer: B The adolescent boy in this vignette has suspected irritable bowel syndrome (IBS), a common pediatric functional gastrointestinal disorder defined as abdominal pain associated with a change in defecation. He describes abdominal pain associated with loose stools with improvement in the pain after defecation. This finding, in the absence of an organic cause, is consistent with IBS. Like other functional gastrointestinal disorders, IBS is a clinical diagnosis that may be established following a thorough history and physical examination. However, symptoms of organic gastrointestinal disease, including inflammatory bowel disease, celiac disease, food allergies, and infection, may be similar to those of IBS. If red flags for organic gastrointestinal diseases are present, including family history of autoimmune disease, weight loss/failure to thrive, poor linear growth, hematochezia or hematemesis, nocturnal stooling, or tenesmus, further testing to evaluate these conditions should occur. The diagnostic criteria (Rome IV) developed by the Rome Foundation for pediatric IBS include (Hyams JS, Di Lorenzo C, Saps M, Shulman RJ, Staiano A, van Tilburg M): • “Abdominal pain more than 4 times monthly associated with defecation, change in frequency of stool, and/or change in stool appearance • Continued abdominal pain despite treatment of constipation (when constipation is present) • After appropriate evaluation, the symptoms cannot be fully explained by another medical condition.” Factors that may exacerbate IBS include increased visceral hypersensitivity, psychosocial events/coping style, and a genetic predisposition. Visceral hypersensitivity (increased sensitivity to pain in the abdominal cavity) may be increased due to intestinal dysmotility, intestinal inflammation, change in intestinal flora, or abdominal distention. Treatment with probiotics may be helpful to alter the intestinal flora. Dietary changes, including trial of soluble fiber and limiting dairy and higher-fat foods, may improve symptoms in some patients. A low-FODMAP (fermentable oligosaccharides, disaccharides, monosaccharides, and polyols) diet may improve symptoms in children, however it is fairly restrictive (limits lactose, high-fructose corn syrup, legumes, and several fruits/vegetables) and nutritional counseling is recommended. Peppermint oil may be helpful in reducing pain, and strategies directed at improving coping skills (such as cognitive behavioral therapy) may be of significant benefit for children with IBS. The adolescent in this vignette is suspected to have IBS. Findings of a family history of psoriasis and nocturnal stooling may be suggestive of an organic gastrointestinal disease. The number of days missed in a school year is not a diagnostic criteria for IBS, although increased school absences are frequently seen in children with functional gastrointestinal disorders.

American academy of pediatrics

618

PREP ® Self-Assessment

PREPSA 2020

PREP Pearls • Irritable bowel syndrome is a functional gastrointestinal disorder associated with abdominal pain and changes in defecation. • The cause of irritable bowel syndrome is multifactorial and includes increased visceral hypersensitivity, psychosocial factors (including coping style), and genetic predisposition. • Treatment of irritable bowel syndrome may include probiotics, dietary changes, and cognitive behavioral therapy. ABP Content Specifications(s) • Recognize the clinical features associated with irritable bowel syndrome, and manage appropriately Suggested Readings • Hyams JS, Di Lorenzo C, Saps M, Shulman RJ, Staiano A, van Tilburg M. Childhood functional gastrointestinal disorders: child/adolescent. Gastroenterology. 2016;150(6):1456-1468. doi:10.1053/j.gastro.2016.02.015. • Koppen IJ, Nurko S, Saps M, Di Lorenzo C, Benninga MA. The pediatric Rome IV criteria: what’s new? Expert Rev Gastroenterol Hepatol. 2017;11(3):193-201. doi:10.1080/17474124.2017.1282820. • Leiby A, Vazirani M. Complementary, integrative, and holistic medicine: integrative approaches to pediatric irritable bowel syndrome. Pediatr Rev. 2016;37(4):e10-e15. doi:10.1542/pir.2015-0036.

American academy of pediatrics

619

PREP ® Self-Assessment

PREPSA 2020

Question 174 A 9-year-old boy is seen for evaluation of mouth lesions. The lesions were noted 2 days ago after he developed an unusual sensation in his mouth. He received a course of amoxicillin 4 months ago for streptococcal pharyngitis and has a history of mild intermittent asthma treated with an inhaled controller medication taken twice daily. He has a temperature of 38°C, blood pressure of 100/57 mm Hg, heart rate of 80 beats/min, and respiratory rate of 18 breaths/min. Examination of the oropharynx is shown in Item Q174.

Item Q174: Oropharynx of the patient described in the vignette. Courtesy of E. Ledbetter and reprinted with permission from Kimberlin DW, et al, eds. Red Book Online. Itasca, IL: American Academy of Pediatrics; 2018.

In addition to prescribing nystatin, the BEST next step in management is A. culture for susceptibilities B. evaluation of cellular immunity C. KOH preparation on scraping D. water rinses twice daily

American academy of pediatrics

620

PREP ® Self-Assessment

PREPSA 2020

Correct Answer: D For the boy in this vignette, the best next step in his management is water rinses twice daily. Inhaled steroids are a risk factor for oral candidiasis, and this risk can be mitigated by rinsing with water after steroid inhalation. Other risk factors for Candida infections include primary or acquired immunodeficiencies, neutropenia or neutrophil functional defects, extreme prematurity, and indwelling catheters. Medications that increase the risk for candidiasis include broad-spectrum antibiotics, systemic corticosteroids, and cytotoxic chemotherapy. Candida infections have a wide array of manifestations. Oropharyngeal candidiasis (thrush) manifests as white plaques. Symptoms of thrush can include an abnormal sensation in the mouth, loss of taste, and odynophagia. Esophagitis is considered a severe manifestation that typically does not occur in immunocompetent hosts. Candida infections can affect the skin and nails. Dermatitis tends to occur in intertriginous areas. Vulvovaginitis is typically characterized by a curd-like discharge, itching, and vaginal erythema. Bloodstream infections can occur in individuals with indwelling central venous catheters, urinary tract infections can occur in individuals with indwelling urinary catheters, and peritonitis can occur in individuals with peritoneal dialysis catheters. Invasive infections can involve nearly all organs and include meningitis, endocarditis, ophthalmitis, and disease of the liver, spleen, and kidneys. Thrush is usually treated with topical antifungals including nystatin, clotrimazole, or miconazole. Vulvovaginal candidiasis usually responds better to clotrimazole or miconazole than nystatin. Oral fluconazole can be considered if first-line treatments for mucocutaneous infections fail. Other antifungals with activity against Candida species include other azoles (itraconazole, voriconazole, posaconazole), the echinocandins (micafungin, caspofungin), and amphotericin. These latter antifungals are typically only used for fluconazole-refractory or invasive infections. If an infection is associated with an indwelling catheter, prompt removal of the catheter is recommended. Culture for susceptibilities is recommended for invasive infections but is generally not performed for mild mucocutaneous infections. Some Candida species such as Candida glabrata and Candida krusei can have decreased susceptibility to azole drugs, and Candida lusitaniae can have amphotericin resistance. While defects of cellular immunity can be associated with Candida infections, the boy in this vignette has inhaled corticosteroid use as a predisposing factor. Evaluation of immunity may be considered in older children without predisposing factors or with severe, recurrent, or refractory candidiasis. The diagnosis of thrush can usually be made through physical examination and does not require scraping and KOH (potassium hydroxide) preparation.

American academy of pediatrics

621

PREP ® Self-Assessment

PREPSA 2020

PREP Pearls • Risk factors for Candida infections include immunodeficiencies, neutropenia or neutrophil defects, extreme prematurity, indwelling catheters, and medications such as broad-spectrum antibiotics, systemic corticosteroids, and cytotoxic chemotherapy. • There is a wide spectrum of Candida infections ranging from mucocutaneous disease to invasive infections that can involve nearly all organs including the central nervous system, eyes, heart, kidneys, liver, and spleen. • Thrush is usually treated with topical antifungals including nystatin, clotrimazole, or miconazole. Oral fluconazole can be considered if first-line treatments for mucocutaneous infections fail. MOCA-Peds Objective • Recognize and manage fungal skin infections. ABP Content Specifications(s) • Identify the risk factors for candidiasis in patients of various ages • Recognize the clinical features associated with Candida infection • Plan appropriate management for a patient with Candida infection Suggested Readings • American Academy of Pediatrics. Candidiasis. In: Kimberlin DW, Brady MT, Jackson MA, Long SS, eds. Red Book: 2018 Report of the Committee on Infectious Diseases. 31st ed. Itasca, IL: American Academy of Pediatrics; 2018:263269. https://redbook.solutions.aap.org/chapter.aspx?sectionId=189640060&bookId=2205 &resultClick=1#192297268. • Pappas P, Kauffman C, Andes D, et al. Clinical practice guideline for the management of candidiasis: 2016 update by the Infectious Diseases Society of America. Clin Infect Dis. 2016;62(4):e1-e50. doi:10.1093/cid/civ933.

American academy of pediatrics

622

PREP ® Self-Assessment

PREPSA 2020

Question 175 A 6-month-old male infant is brought to the emergency department after 3 days of vomiting and diarrhea. During the illness, his parents gave him diluted powdered formula and small amounts of water. He is lethargic and has a weak cry. He has a heart rate of 136 beats/min, respiratory rate of 22 breaths/min, and blood pressure of 88/52 mm Hg. His mucous membranes are dry, and his capillary refill time is more than 2 seconds. Laboratory data are shown: Laboratory Test Sodium Potassium Chloride Bicarbonate Blood urea nitrogen Creatinine Glucose Serum osmolality

Result 119 mEq/L (119 mmol/L) 3.4 mEq/L (3.4 mmol/L) 90 mEq/L (90 mmol/L) 18 mEq/L (18 mmol/L) 28 mg/dL (10.0 mmol/L) 0.5 mg/dL (44 µmol/L) 90 mg/dL (5.0 mmol/L) 258 mOsm/kg (258 mmol/kg)

Of the following, the MOST accurate statement regarding this infant’s condition is A. fluid has shifted from the extracellular to the intracellular compartment B. the intracellular sodium has increased because of fluid shifts C. rapid correction will lead to cerebral edema caused by osmolar shifts D. rapid onset may lead to subarachnoid hemorrhage

American academy of pediatrics

623

PREP ® Self-Assessment

PREPSA 2020

Correct Answer: A The infant in this vignette has hyponatremic dehydration because of an excessive loss of sodium from gastroenteritis and attempted rehydration with diluted formula. The result is low serum osmolality, which in turn leads to fluids shifting from the extracellular fluid (ECF) compartment to the intracellular fluid (ICF) compartment. Total body water is distributed in the ECF and ICF compartments. One-third of the total body water is present in ECF and two-thirds is present in the ICF. The ECF is further divided into interstitial fluid or extravascular compartment (75%) and the blood volume or intravascular compartment (25%). The solute composition of the ECF and ICF varies. Sodium is the major cation and chloride is the major anion in ECF, whereas potassium is the major cation and phosphorus is the major anion in ICF. Hydrostatic pressure, oncotic pressure provided by albumin, and normal capillary permeability are responsible for movement of fluids between the ECF and ICF and help to maintain a state of equilibrium. The intravascular compartment of ECF is decreased in diarrhea, vomiting, and hypoalbuminemia and is increased in conditions like congestive heart failure. The extravascular (interstitial) compartment of ECF is increased in nephrotic syndrome, liver failure, and heart failure. The ICF is decreased in prolonged diarrhea and dehydration, increased insensible losses, and diabetes insipidus. The ICF is increased in conditions like syndrome of inappropriate antidiuretic hormone. Osmolality is the concentration of all the solutes in a given weight of water. Sodium along with glucose and urea are major osmoles in the blood. The serum osmolality is calculated by using the formula: 2 × sodium (mmol/L) + blood urea nitrogen (mg/dL) + glucose (mg/dL) 2.8 18 These solutes create an osmotic pressure gradient and are responsible for shift of fluid between the ECF and ICF from the compartment with lower osmolality to the compartment with higher osmolality. Sodium, as evident from the formula, has the maximal osmolar contribution and is important to maintain the ECF volume. Low serum sodium (hyponatremia) mostly equates to hypo-osmolality and indicates relative excess of water as compared to sodium in the ECF. To balance the osmoles, water moves from ECF to ICF in hyponatremic conditions. Hyponatremia results from net depletion of sodium that is in excess of body water depletion (sodium deficit), dilution of sodium by increase in the body water (dilutional hyponatremia), or a combination of both. Children with diarrhea, vomiting, and burns have sodium losses through the gastrointestinal tract or skin and present with hyponatremia when the fluid replacement occurs with excess free water, as described for the infant in this vignette. The ECF sodium is decreased in this infant with hyponatremic dehydration, and the ICF sodium is also decreased with fluids shifting from the ECF to ICF. Hyponatremia can present with headache, altered sensorium, or seizures as a result of cerebral edema from the osmotic shift of water into the brain cells. Neurological symptoms occur when hyponatremia develops acutely and the brain is not able to counteract the osmotic shifts. Treatment of hyponatremia should be gradual, and the target rise in sodium should not be more American academy of pediatrics

624

PREP ® Self-Assessment

PREPSA 2020

than 0.5 mmol/L/h or 10 to 15 mmol/L in 24 hours. A rapid correction of hyponatremia leads to osmotic shift of water from the brain cells into the ECF causing central pontine myelinolysis. Rapid onset of hypernatremia, rather than hyponatremia, may lead to subarachnoid hemorrhage. PREP Pearls • Hyponatremia or hypo-osmolality leads to fluid shifts from the extracellular fluid compartment to the intracellular fluid compartment. • Acute development of hyponatremia can cause cerebral edema because of movement of fluid into the brain cells. • Rapid correction of hyponatremia may increase the risk of central pontine myelinolysis because of the movement of fluid from brain cells to the extracellular fluid compartment. MOCA-Peds Objective • Evaluate and manage a patient with hyponatremic dehydration. ABP Content Specifications(s) • Understand how equilibrium is maintained between extracellular and intracellular fluid • Recognize the causes of abnormal increases or decreases in intracellular/extracellular fluid volumes • Estimate plasma osmolality by assessing appropriate serum concentrations Suggested Readings • Jain A. Body fluid composition. Pediatr Rev. 2015;36(4):141-152. doi:10.1542/pir.36-4141. • Mahajan P, Felt JR. Fluids, electrolytes, and acid-base composition. In: McInerny TK, Adam HM, Campbell DE, DeWitt TG, Foy JM, Kamat DM, eds. American Academy of Pediatrics Textbook of Pediatric Care. 2nd ed. Elk Grove Village, IL: American Academy of Pediatrics; 2017:419-433. Pediatric Care Online. • Powers KS. Dehydration: isonatremic, hyponatremic, and hypernatremic recognition and management. Pediatr Rev. 2015;36(7):274-285. doi:10.1542/pir.36-7-274.

American academy of pediatrics

625

PREP ® Self-Assessment

PREPSA 2020

Question 176 A middle school sports coach is concerned about the possibility of dehydration during afternoon practice. He seeks advice in outlining a strategy to help athletes stay well hydrated during these 60-minute practice sessions. Of the following, the BEST advice for this coach is A. carbohydrate-containing fluids (sports drinks) are the beverages of choice for these young athletes B. dehydration in young athletes is unlikely in practices that are only 60 minutes in duration C. ensure that young athletes are adequately hydrated prior to the beginning of practice D. sodium-containing food or fluids should be encouraged in these young athletes to help maintain euhydration during exercise

American academy of pediatrics

626

PREP ® Self-Assessment

PREPSA 2020

Correct Answer: C Adequate hydration is important to assure athlete safety and optimal performance. Athletes who practice daily are at risk for cumulative fluid deficits. The coach described in this vignette should counsel his young athletes that they should recoup fluid losses after each practice and should assure that they start any training session well hydrated. Young athletes should drink enough fluids throughout the day to keep their urine pale yellow (like lemonade), and 1 to 2 hours before training, they should prehydrate with water or a nutritive beverage of choice (eg, milk, “nut milks,” or vegetable juices). This timing allows for gastric emptying and absorption before exertion begins. Although fluid recommendations should accommodate individual variability in fluid loss and dietary intake, good starting points for prehydration are: 90 to 180 mL (3-6 oz) in athletes who weigh less than 40 kg, and 180 to 360 mL (6-12 oz) in athletes who weigh more than 40 kg. Fluid losses during exercise are mostly from sweat and evaporative cooling, and sweat rates are highly variable. In adolescents, fluid losses typically range from 500 to 1,300 mL/h, but losses of over 3.5 L/h have been recorded. Environmental conditions and exercise intensity are dominant factors determining fluid loss during physical activity, but there is also marked interindividual variability. Adolescent athletes who are older, highly fit, or male generally have higher sweat rates than athletes who are younger, less fit, or female. Thirst is often recognized when dehydration approaches 3% to 5%, but athletic performance (particularly in endurance activity) falls off at approximately 2% dehydration. Hydration strategies should aim to keep fluid losses to less than 2% body weight. Athletes can easily determine individual rates of fluid loss by changes in pre- and post-exercise weight. Post-exercise weight should be obtained after the athlete has dried off and removed any wet clothing. Each pound of lost weight signifies an uncompensated loss of 16 oz of fluid. The athlete should replenish this volume before the next bout of exercise, and hydration strategies during subsequent workout sessions should be adapted to prevent this excess fluid loss. Sample calculation: Pre-exercise weight: 100 lbs “Allowed” loss of 2%: 98 lbs Post-exercise weight: 95 lbs Excess weight loss during exercise: 3 lbs = 48 oz of excess fluid loss The goal for postexercise rehydration is restoration of pre-exercise weight. For many physically active children and adolescents, water is the fluid of first choice, particularly for training sessions lasting less than an hour or of relatively low intensity. Athletes training for longer durations may benefit from ingesting added carbohydrate during their workout to help fuel working muscles. Although gastric tolerance for ingestion during intense exercise is highly variable, performance in adolescents may be enhanced by carbohydrate ingestion of up to 60 g/h. This approximates the amount of carbohydrates found in 2 medium bananas, a large bagel, or 6 fig-filled cookies. Sports drinks often contain carbohydrate concentrations of about 6% to 8% and may be a more convenient way to provide added carbohydrates. Fluids with this carbohydrate concentration appear to empty from the stomach American academy of pediatrics

627

PREP ® Self-Assessment

PREPSA 2020

more quickly than plain water, and may reduce stomach discomfort in some athletes. This concentration can also be obtained by diluting apple or other nonacidic fruit juices by half with water. Most young athletes do not require added sodium during sports activity. Athletes who participate in efforts lasting longer than several hours or who are participating multiple times per day (ie, tournaments or twice daily practices) in warm weather should assure that they are replenishing sodium after activity. Sodium content in adolescent sweat is typically 40 to 70 mmol/L, but some athletes appear to have high salt concentrations in their sweat and are considered “salty sweaters.” These athletes are often identified by salt crusting on skin and clothes after activity, and they may benefit from increasing sodium intake during and around the time of activity. Many sports drinks contain 10 to 20 mmol/L of sodium, which stimulates further drinking, but this amount of sodium is not sufficient to significantly replace sweat-related sodium losses. A clear distinction should be made between sports drinks and energy drinks. Energy drinks contain added caffeine or other herbal stimulants (eg, guarana) and are not recommended for children and adolescents. Sports drinks containing added sodium, carbohydrate, and flavoring may be a convenient way to combine fluid and carbohydrate ingestion during exercise but offer no benefit over water or other nutritive beverages before or after activity. PREP Pearls • Water is the fluid of choice for hydration during physical activity. • Fluid loss with exercise is highly variable, and therefore hydration strategies need to be individualized. • Many children and adolescents are hypohydrated even before onset of sports activity. Young athletes should recoup fluid losses after each practice and should start any training session well hydrated. ABP Content Specifications(s) • Plan optimal age-appropriate replacement for fluid losses associated with athletic activity Suggested Readings • Bergeron MF. Reducing sports heat illness risk. Pediatr Rev. 2013;34(6):270-279. doi:10.1542/pir.34-6-270. • Buoite Stella A, Francescato MP, Sims ST, Morrison SA. Fluid intake behavior in athletes during typical training bouts. J Sports Med Phys Fitness. 2017;57(11):15041512. doi:10.23736/S0022-4707.16.06722-0. • Committee on Nutrition and the Council on Sports Medicine and Fitness. Sports drinks and energy drinks for children and adolescents: are they appropriate? Pediatrics. 2011;127(6):1182-1189. doi:10.1542/peds.2011-0965. • LaBotz M. Sports nutrition. In: Harris SS, Anderson SJ, eds. Care of the Young Athlete. 2nd ed. Elk Grove Village, IL: American Academy of Pediatrics; 2010:71-80. • LaBotz M. Sports nutrition. In: Kleinman RE, Greer FR, eds. Pediatric Nutrition. 7th ed. Elk Grove Village, IL: American Academy of Pediatrics; 2014:265-298.

American academy of pediatrics

628

PREP ® Self-Assessment

PREPSA 2020

Question 177 A 5-year-old child is seen for a routine health supervision visit. He recently moved from another state and has a history of severe persistent asthma with multiple hospital admissions, eczema, and allergic rhinitis. He was also diagnosed with egg allergy at the age of 12 months when he developed hives after eating scrambled eggs. Since that time, his previous physician has recommended against ever receiving the influenza vaccine. The child’s mother is therefore hesitant to consent to the vaccine, but is also interested in helping to improve her child’s asthma. Of the following, the BEST approach to influenza vaccination for this child is to A. administer immune globulin instead of the influenza vaccine B. divide the intramuscular influenza vaccine into smaller doses C. first perform a skin test with the intramuscular influenza vaccine itself D. reassure the mother and administer the intramuscular influenza vaccine

American academy of pediatrics

629

PREP ® Self-Assessment

PREPSA 2020

Correct Answer: D The best approach for the child in this vignette is to reassure the mother and administer the intramuscular influenza vaccine. The amount of egg protein in the influenza vaccine is very small, and the benefits of preventing influenza disease with the vaccine far outweighs the risk of anaphylaxis to the vaccine. Immune globulin does not adequately protect against influenza disease. Dividing the influenza vaccine into more than one dose and performing a skin test to the vaccine is no longer considered necessary, even in children who have had anaphylaxis to eggs. Vaccine hesitancy is pervasive. Rates of unvaccinated or incompletely vaccinated children are increasing yearly, leading to increased health care costs. Reasons for vaccine hesitancy are multifactorial and personal. Addressing individual concerns about vaccines requires trust and open-mindedness. The American Academy of Pediatrics Immunization Initiative provides resources on strategies to address various concerns. Termination of patients from a practice solely on the basis of vaccine refusal is generally not recommended except in extreme circumstances. Likewise, vaccine refusal is not considered medical neglect unless the child is at a significant potential risk of harm, as during an epidemic. One common target of vaccine refusal is the influenza vaccine, particularly in patients who have had an allergic reaction to eggs. Strategies for vaccinating these patients with the influenza vaccine have evolved over time. Previous guidelines have recommended that children who are allergic to eggs have skin testing performed, and if the results were positive then the influenza vaccine should be given in 2 graduated doses. The current recommendation is to administer the influenza vaccine without reservation while ensuring that patients who have had reactions to egg other than hives receive their influenza vaccine in a medical setting. In the rare event that a patient has an adverse reaction to a vaccine, the reaction should be documented in the Vaccine Adverse Event Reporting System. While any reaction thought to be related to the vaccine should be reported, reactions that are required by law to be reported are specific to each vaccine and can be found at https://vaers.hhs.gov/docs/VAERS_Table_of_Reportable_Events_Following_Vaccination.pdf. PREP Pearls • Children who have experienced an allergic reaction of any severity to eggs should still receive the influenza vaccine. • Addressing parental concerns regarding vaccine hesitation or refusal should take into consideration the individual physician-patient relationship. • Adverse reactions thought to be related to a vaccine should be reported in the Vaccine Adverse Event Reporting System. MOCA-Peds Objective • Recognize the situations in which various immunizations may be contraindicated.

American academy of pediatrics

630

PREP ® Self-Assessment

PREPSA 2020

ABP Content Specifications(s) • Recognize adverse reactions to various vaccine constituents and manage appropriately • Plan an appropriate approach to addressing the needs of the vaccine-hesitant family Suggested Readings • American Academy of Pediatrics. Immunizations. https://www.aap.org/en-us/advocacyand-policy/aap-health-initiatives/immunizations/Pages/Immunizations-home.aspx. • Centers for Disease Control and Prevention. Vaccines and immunizations. https://www.cdc.gov/vaccines/index.html.

American academy of pediatrics

631

PREP ® Self-Assessment

PREPSA 2020

Question 178 A 3-year-old boy is brought to the emergency department for increasing lethargy. He had been in good health until today. His grandmother had brought her pill box to the kitchen table to take her afternoon medications, left the room briefly, and when she returned, found her grandson sitting on the floor with multiple pills scattered around him. He denied taking any of the pills, but then became less alert and more lethargic. His grandmother called 911 when she was not able to arouse him. Upon arrival at the emergency department, the boy had a temperature of 36.4°C, heart rate of 60 beats/min, blood pressure of 72/54 mm Hg, and respiratory rate of 12 breaths/min. His physical examination findings are significant for unresponsiveness to voice or touch, pinpoint pupils, and decreased bowel sounds. His blood glucose level is 124 mg/dL (6.9 mmol/L). Of the following, the BEST treatment to administer is A. activated charcoal B. flumazenil C. naloxone D. sodium bicarbonate

American academy of pediatrics

632

PREP ® Self-Assessment

PREPSA 2020

Correct Answer: C The boy in the vignette is unresponsive and has signs of acute opioid overdose (pinpoint pupils; mental status depression; and decreased blood pressure, pulse, respiratory rate, temperature, and bowel sounds); he should be treated with naloxone. A patient with tricyclic antidepressant overdose may also present with drowsiness and hypotension but would have dilated pupils; sodium bicarbonate would be the treatment in this case. Flumazenil is an antidote for benzodiazepine overdose in nonhabituated users. Children with benzodiazepine overdose typically exhibit ataxia, normal vital signs and pupils, and nystagmus. With more significant benzodiazepine overdose, children may have altered mental status. Although activated charcoal can decrease gastrointestinal absorption of drugs, its administration is contraindicated in those with mental status depression; aspiration of activated charcoal can result in a chemical pneumonitis and respiratory distress. Opioids (eg, morphine, oxycodone) are most commonly prescribed to children for a short time to control pain and decrease anxiety and agitation for a procedure or surgery or in the intensive care unit for ongoing pain, analgesia, or sedation. Childhood exposure to opioids may also be through accidental ingestion or intentional use/abuse (eg, heroin, prescription opioid medication). Acute opioid intoxication results in euphoria, slurred speech, drowsiness, and pinpoint pupils. With more severe overdose, patients may become sedated and hypotensive, and may have respiratory depression (the main cause of death). Treatment is with naloxone, an opioid receptor antagonist. Complications that may arise from opioids depend on the method by which the opioids are used. Intravenous injection of opioids can rapidly produce euphoria, but can cause soft-tissue infections and scarred veins and increase risk of infective endocarditis. Sharing needles increases the risk of blood-borne infections such as hepatitis B, hepatitis C, and HIV. Inhaling opioids through the nose can damage nasal tissues (eg, perforation of the nasal septum). Smoking opioids (“chasing the dragon”) can cause lung-related complications such as pneumonia. Opioid use/abuse results in physiologic dependence with tolerance and withdrawal. Signs of withdrawal include rhinorrhea, yawning, diarrhea, diaphoresis, vomiting, myalgia, tremors, restlessness, agitation, insomnia, and tachycardia. The individual with opioid use disorder can have impairment in relationships, work, and social functioning; he may not fulfill major obligations, may give up important activities, and may engage in illegal behaviors (eg, burglary) to access opioids. Buprenorphine, naltrexone, and methadone are approved for the treatment of opioid use disorder. PREP Pearls • Signs of acute opioid overdose include pinpoint pupils; mental status depression; and decreased blood pressure, pulse, respiratory rate, temperature, and bowel sounds. • Respiratory depression is the main cause of death with opioid overdose. • Signs of opioid withdrawal include rhinorrhea, yawning, diarrhea, diaphoresis, vomiting, myalgia, tremors, restlessness, agitation, insomnia, and tachycardia. American academy of pediatrics

633

PREP ® Self-Assessment

PREPSA 2020

ABP Content Specifications(s) • Recognize the major behavioral consequences of opioid use/abuse • Recognize the clinical findings associated with an acute opioid overdose, and manage appropriately • Identify the major physiologic consequences associated with opioid use/abuse, including those associated with the various means of administration Suggested Readings • Galinkin J, Koh JL, Committee on Drugs and Section on Anesthesiology and Pain Medicine. Recognition and management of iatrogenically induced opioid dependence and withdrawal in children. Pediatrics 2014;133(1):152-155. doi:10.1542/peds.2013-3398. • Tobias JD. Managing acute pain in children. In: McInerny TK, Adam HM, Campbell DE, DeWitt TG, Foy JM, Kamat DM, eds. American Academy of Pediatrics Textbook of Pediatric Care. 2nd ed. Elk Grove Village, IL: American Academy of Pediatrics; 2017:379-390. Pediatric Care Online. • Wang GS, Hoyte C. Common substances of abuse. Pediatr Rev. 2018;39(8):403-414. doi:10.1542/pir.2017-0267.

American academy of pediatrics

634

PREP ® Self-Assessment

PREPSA 2020

Question 179 A previously healthy 16-year-old adolescent boy is brought to the emergency department because of the sudden onset of chest pain and difficulty breathing. He was previously in his usual state of good health with no intercurrent illnesses. The pain started spontaneously when he was sitting down watching television. It is left-sided, sharp, located in the mid-axillary line radiating to the shoulder, and worsens when he takes a deep breath. He has a temperature of 37.0°C, heart rate of 120 beats/min, respiratory rate of 40 breaths/min, blood pressure of 70/50 mm Hg, and oxygen saturation by pulse oximetry of 75% on room air. He is in pain and severe respiratory distress. Breath sounds on the left side are decreased. Breath sounds on the right side are diminished and bronchial. The jugular veins are distended. Heart sounds are rapid and muffled, with normal S1 and S2 and without rubs, murmurs, or gallops. Extremities are cool with lowamplitude pulses throughout. Capillary refill time is 5 seconds. His chest radiograph is shown in Item Q179.

American academy of pediatrics

635

PREP ® Self-Assessment

PREPSA 2020

Item Q179: Chest radiograph for the patient described in the vignette. Reprinted with permission from R. Wiggins. Of the following, the BEST next step is to A. consult a surgeon for video-assisted thoracoscopic surgery B. perform needle decompression of left hemithorax C. perform pericardiocentesis D. start intravenous dopamine infusion

American academy of pediatrics

636

PREP ® Self-Assessment

PREPSA 2020

Correct Answer: B The adolescent boy in this vignette has severe shock caused by a left-sided spontaneous pneumothorax. Of the response choices, the best next step is to perform a needle decompression of the left hemithorax. The pleural space is bordered by the visceral pleura overlying the lung and the parietal pleura, which is attached deep to the chest wall (Item C179). There is usually a small amount of pleural fluid present in the pleural space to lubricate the lungs during breathing. During normal breathing, downward movement of the diaphragm creates slightly negative intrathoracic pressure to draw air into the lungs, followed by passive exhalation caused by the elastic recoil of the lungs. Thus, the pleural space is normally under a slight vacuum, and any amount of air in the pleural space (pneumothorax) is abnormal. Pneumothorax is caused by an air leak through the parietal pleura into the pleural space. The vacuum of the pleural space favors air moving through the leak and leads to partial or total lung collapse.

American academy of pediatrics

637

PREP ® Self-Assessment

PREPSA 2020

Item C179: Chest radiograph for the patient described in the vignette. Reprinted with permission from R. Wiggins. Pneumothorax can be spontaneous (ie, without an identifiable cause) or trauma related. The overall incidence of spontaneous pneumothorax is 3.4 cases per 100,000 population 0 to 17 years of age, with a 6-fold male predominance. Spontaneous pneumothorax in tall, athletic male adolescents is particularly common. Air leaks causing pneumothorax can be due to the rupture of walled-off air collections such as blebs, bullae, and pneumatoceles. Conditions that can cause the development of blebs and pneumatoceles and predispose to spontaneous pneumothorax include asthma, cystic fibrosis, chronic lung disease of prematurity, and connective tissue disorders such as Marfan syndrome and Ehlers-Danlos syndrome. Acute conditions causing increased mean American academy of pediatrics

638

PREP ® Self-Assessment

PREPSA 2020

airway pressure or high alveolar pressure, such as status asthmaticus with air trapping or excessive positive-pressure ventilation, can also cause pneumothorax. Acute or resolving necrotizing pneumonia can erode the visceral pleura and connective tissue, leading to bronchopleural fistula and pneumothorax. Blunt trauma can cause pneumothorax due to rib fracture, esophageal rupture, or a disruption of airway structures anywhere in the tracheobronchial tree. Penetrating trauma can cause direct entry of air into the pleural space, as well as the creation of a connection from the lungs or airway into the pleural space. Pneumothorax with hemothorax is more common in penetrating trauma or with rib fractures compared to other mechanisms of blunt trauma. The clinical impact of pneumothorax depends on the degree of lung collapse and the presence of tension physiology. A small pneumothorax with minimal lung collapse may be asymptomatic. A large pneumothorax associated with significant lung collapse can lead to impaired oxygenation caused by shunting of blood past collapsed alveoli. Clinical signs of a significant pneumothorax include tachycardia, tachypnea, decreased breath sounds in the affected hemithorax, and decreased chest excursion. Ventilation can be intact despite a significant oxygenation defect because minute ventilation can be preserved by other areas of recruited lung and tachypnea. Tension pneumothorax is a life-threatening condition caused by a ball-valve effect of air entering the pleural space and a significant increase in intrapleural pressure. Tension pneumothorax is usually associated with significant oxygenation defects, because extremely high pleural pressure causes total lung collapse, and the large air collection can shift to the contralateral side, further impairing lung expansion. Hemodynamic compromise can occur due to impaired venous return and cardiac tamponade physiology, in which bowing of the intra-ventricular septum impairs left ventricular filling, leading to decreased cardiac output. Clinical signs of tension pneumothorax include tracheal deviation towards the contralateral side, over-expansion of the affected side without chest excursion, muffled heart sounds, and other signs of hemodynamic compromise. Tension pneumothorax often occurs due to trauma and is less common in spontaneous pneumothorax. Management of pneumothorax is based on the underlying etiology, symptoms, and the degree of respiratory and hemodynamic compromise, which depends on the size of the air collection and the presence of tension physiology. A small pneumothorax can be observed in stable patients. Oxygen can be provided for patients who are tachypneic or hypoxic. Placement of a chest tube is required for a pneumothorax that is large or causes significant symptoms. A pigtail catheter placed by the Seldinger technique suffices for management of a pneumothorax that is unlikely to resolve spontaneously; large-bore chest tubes are rarely required. The patient in this vignette has a tension pneumothorax, evidenced by hypotension, tachycardia, and poor perfusion. Immediate needle decompression with a large-bore angiocatheter is indicated. The needle is inserted perpendicular to the skin above the second intercostal space in the midclavicular line. After a gush of air is heard, the needle is retracted and the angiocatheter can be left in place and secured. This procedure relieves the high pressure in the intrapleural space, but it does not resolve the pneumothorax, so a chest tube will be required. This can be a lifesaving procedure to stabilize a patient, so it is important for physicians to be able to perform, even in the absence of the expertise or equipment to insert a chest tube. Although the patient in this vignette is hypotensive, increasing the inotropy of the heart with a dopamine infusion will not be helpful. American academy of pediatrics

639

PREP ® Self-Assessment

PREPSA 2020

Pericardiocentesis is indicated when tamponade is caused by pericardial effusion, but the etiology for this patient is tension pneumothorax. Video-assisted thoracoscopic surgery is not indicated because a more immediate intervention is required. PREP Pearls • Conditions that can cause the development of blebs and pneumatoceles and predispose a patient to spontaneous pneumothorax include asthma, cystic fibrosis, chronic lung disease of prematurity, and connective tissue disorders such as Marfan syndrome and EhlersDanlos syndrome. • A small pneumothorax can be observed in stable patients. • A pigtail catheter placed by the Seldinger technique suffices for management of a pneumothorax that is unlikely to resolve spontaneously. • Tension pneumothorax occurs when the air in the pleural space is under high pressure. It often causes hemodynamic compromise and requires needle decompression for stabilization prior to insertion of a chest tube. MOCA-Peds Objective • Evaluate and manage spontaneous pneumothorax. ABP Content Specifications(s) • Recognize complications associated with pneumothorax/pneumomediastinum • Recognize the natural history of spontaneous pneumothorax/pneumomediastinum • Recognize the clinical features of a pneumothorax/pneumomediastinum, and manage appropriately Suggested Readings • Cashen K, Petersen TL. Pleural effusions and pneumothoraces. Pediatr Rev. 2017;38(4):170-181. doi:10.1542/pir.2016-0088. • Dotson K, Johnson LH. Pediatric spontaneous pneumothorax. Pediatr Emerg Care. 2012;28(7):715-723. doi:10.1097/PEC.0b013e31825d2dd5. • Pasquier M, Hugli O, Carron PN. Videos in clinical medicine. Needle aspiration of primary spontaneous pneumothorax. N Engl J Med. 2013;368(19):e24. doi:10.1056/NEJMvcm1111468. • Posner K, Needleman JP. Pneumothorax. Pediatr Rev. 2008;29(2):69-70. doi:10.1542/pir.29-2-69.

American academy of pediatrics

640

PREP ® Self-Assessment

PREPSA 2020

Question 180 A 12-month-old girl is seen for evaluation of early hand preference. She was born at 39 weeks’ gestation after an uncomplicated pregnancy to a gravida 1 para 1 mother via normal spontaneous vaginal delivery. The Apgar scores were 8 and 9 with a normal neonatal course. Her mother has noticed that she always reaches for toys with her right hand and that since shortly after birth she has had a tendency to hold her left hand in a closed fist with the arm tense against her side. She rolled at 5 months of age, sat independently at 7 months, and has pulled to stand since 10 months. Her mother has noticed that her left foot does not rest flat on the floor when she stands and that she tends to drag the left leg when cruising along furniture or crawling. She has a welldeveloped pincer grasp with her right hand but predominantly grasps objects with a raking motion with her left hand. Social-emotional and language development have been normal. No developmental regression has been appreciated. Her general physical examination findings are normal. Neurological examination is significant for hypertonia and hyperreflexia in the left arm and leg with normal cranial nerve and sensory examination findings. She tends to hold her left hand in a fist with the thumb adducted across the palm. She only reaches for objects with her left hand when her right arm is restrained; in this case, she uses a raking motion associated with some tremulousness. When cruising she tends to hold her left foot in a tip-toe position and drag the leg. She is referred to a pediatric neurology clinic where magnetic resonance imaging of the brain reveals right hemispheric closed-lip schizencephaly (Item Q180). Her mother returns for follow-up, tearful, because the neurologist diagnosed her daughter with a form of cerebral palsy.

Item Q180: Brain magnetic resonance imaging for the patient described in the vignette. Courtesy of J. Goldstein

Of the following, this child’s cerebral palsy is classified as A. dyskinetic B. mixed C. spastic diplegia D. spastic hemiplegia

American academy of pediatrics

641

PREP ® Self-Assessment

PREPSA 2020

Correct Answer: D The patient in this vignette has a spastic hemiplegia secondary to schizencephaly (a congenital disorder of cortical migration). Malformations of cortical development have a broad range of clinical manifestations and severities including epilepsy, hemiparesis, early hand preference, global developmental delay, learning disability, or attention issues. In children suspected of having cerebral palsy (CP), magnetic resonance imaging (MRI) is recommended to evaluate for an underlying brain abnormality as part of the initial diagnostic evaluations. Abnormal MRI findings are seen in a majority of children with CP. Term infants are more likely to have cortical and grey matter lesions in comparison to preterm infants who are more likely to have periventricular white matter lesions. The presence of a brain malformation or normal MRI findings should prompt second-tier diagnostic testing including genetic and metabolic studies aimed at identifying an underlying etiology for the child’s CP. Cerebral palsy is a common group of conditions defined as a nonprogressive (static) genetic or acquired disorder of posture, tone, and/or movement secondary to injury or abnormality of the developing brain. The term encompasses a broad range of clinical conditions with variable presentations and severity. While there has been debate around the nomenclature and definition of CP, clinically the condition is classified by tone and limb involvement, which can allow for localization and guide diagnostic investigations aimed at identifying etiology and tailoring symptomatic management. In patients with bilateral spasticity, there are 2 types of CP conditions: spastic diplegia, in which there is predominately spasticity of the legs, and spastic quadriparesis, in which spasticity affects all extremities. Spastic diplegia is most commonly seen in premature infants due to periventricular leukomalacia and often presents with gross motor delay. Spastic quadriparesis can result from a broad range of injuries including periventricular leukomalacia, brain malformations, prenatal and postnatal infections, traumatic brain injury, or anoxic injury and presents with prominent gross motor delay and often global developmental delay. Unilateral spasticity is seen in spastic hemiplegia and is often secondary to structural abnormalities, such as brain malformations, stroke, or vascular malformations. Patients often achieve the ability to ambulate independently and have normal intelligence with increased risk for seizures. A third category of CP is the extrapyramidal or dyskinetic type, typically affecting the arms more than the legs with various abnormal movements such as bradykinesia, choreoathetosis, hemiballismus, or dystonia. Although there are a variety of causes for dyskinetic CP, the 2 most common are neonatal and include hypoxic-ischemic encephalopathy and kernicterus. The least common types of CP, hypotonic and ataxic, are a result of a heterogenous group of disorders, most commonly nonprogressive genetic-metabolic disorders such as Prader-Willi syndrome or Angelman syndrome. Classification and identification of the etiology for a child’s CP is critical for accurate diagnosis and treatment. The majority of clinicians will wait to establish the diagnosis until the child is aged 1 to 2 years to ensure the nonprogressive nature of the motor abnormality and account for normal variation in infant and child development. An accurate and complete history including a detailed developmental history and bedside assessment coupled with a thorough and complete neurological examination are useful in identifying prenatal and perinatal factors suggestive of a specific etiology. Failure to identify a specific etiology for the child’s CP from the history, American academy of pediatrics

642

PREP ® Self-Assessment

PREPSA 2020

physical examination findings, or progression of symptoms should prompt investigations for CP mimics. These mimics include inborn errors of metabolism, neurodegenerative conditions, movement disorders, neoplasm, or hydrocephalus. These conditions share features at presentation with CP but may have specific treatments. For example, Segawa disease, an inherited dopamine responsive dystonia, presents with a diurnal variation predominantly affecting the lower extremities, thus mimicking spastic diplegia; however, Segawa disease is treatable with levodopa, resulting in rapid improvement in tone and return to normal motor function. Pediatricians are uniquely positioned to assume a central role in the care of children with CP during both the diagnostic evaluation and long-term management. Through routine developmental surveillance and screening, early detection of tone abnormalities or delays in motor development can be identified, prompting further diagnostic evaluation that typically includes neuroimaging followed by genetic/metabolic testing. Once a diagnosis is established, focus shifts to initiation of intervention services such as therapies, educational services, and symptom management, allowing promotion of development. Children with CP can have associated difficulties with communication, swallowing/feeding, intellectual disability, seizures, mental health conditions, and respiratory disease. Part of their comprehensive care includes screening and identification of these common comorbidities to allow for initiation of appropriate care. PREP Pearls • Cerebral palsy is a common group of conditions defined as a nonprogressive (static) genetic or acquired disorder of posture, tone, and/or movement secondary to injury or abnormality of the developing brain. • Cerebral palsy is classified by the tone abnormality and limbs involved. Types of cerebral palsy include spastic (diplegia, quadriplegia, hemiplegia), dyskinetic, and hypotonicataxic. • Care of children with cerebral palsy includes spasticity management, establishment of intervention and educational services, and screening and treatment of common comorbidities including communication difficulties, intellectual disability, seizures, feeding difficulties, and respiratory disorders. ABP Content Specifications(s) • Recognize the clinical features associated with cerebral palsy • Understand the prenatal risk factors associated with cerebral palsy

American academy of pediatrics

643

PREP ® Self-Assessment

PREPSA 2020

Suggested Readings • Ashwal S, Russman BS, Blasco PA, et al. Practice parameter: diagnostic assessment of the child with cerebral palsy: report of the Quality Standards Subcommittee of the American Academy of Neurology and the Practice Committee of the Child Neurology Society. Neurology. 2004;62(6):851-863. doi:10.1212/01.WNL.0000117981.35364.1B. • Leach E, Shevell M, Bowden K, Stockler-Ipsiroglu S, van Karnebeek C. Treatable inborn errors of metabolism presenting as cerebral palsy mimics: systematic literature review. Orphanet J Rare Dis. 2014;9:197. doi:10.1186/s13023-014-0197-2. • Liptak G, Murphy N; Council on Children with Disabilities. Clinical report: providing a primary care medical home for children and youth with cerebral palsy. Pediatrics. 2011;128(5):e1321-e1329. doi:10.1542/peds.2011-1468. • Murphy N. Cerebral palsy. In: McInerny TK, Adam HM, Campbell DE, DeWitt TG, Foy JM, Kamat DM, eds. American Academy of Pediatrics Textbook of Pediatric Care. 2nd ed. Elk Grove Village, IL: American Academy of Pediatrics; 2017:1829-1835. Pediatric Care Online. • Noritz GH, Murphy NA; Neuromotor Screening Expert Panel. Motor delays: early identification and evaluation. Pediatrics. 2013;131(6):e2016-e2027. doi:10.1542/peds.2013-1056.

American academy of pediatrics

644

PREP ® Self-Assessment

PREPSA 2020

Question 181 A 9-year-old girl is seen for evaluation of abdominal pain and vomiting. Her symptoms began 5 months ago with onset of epigastric pain, for which she tried histamine-2 receptor antagonists for several weeks without improvement. She has lost 3 kg in 5 months. This week, she developed intermittent bilious emesis and has had an additional 2-kg weight loss. She has no other medical history and reports no medication use. She appears pale and thin (weight at the 10th percentile for age, height at the 50th percentile for age, and body mass index at the 5th percentile for age). Abdominal and rectal examination findings are normal, as is the remainder of the physical examination. Of the following, the test MOST likely to establish the cause of the bilious emesis in this girl is A. abdominal ultrasonography B. hepatobiliary scan C. magnetic resonance imaging of the brain D. upper gastrointestinal series

American academy of pediatrics

645

PREP ® Self-Assessment

PREPSA 2020

Correct Answer: D The girl in this vignette has superior mesenteric artery (SMA) syndrome, a complication of significant weight loss. It occurs after loss of adipose tissue anterior to the duodenum. Normally, adipose tissue separates the SMA from the third portion of the duodenum; however, when the loss of this tissue occurs, the SMA compresses the duodenum, creating a partial or complete small bowel obstruction. Superior mesenteric artery syndrome is diagnosed with upper gastrointestinal series, demonstrating an obstruction in the third portion of the duodenum with contrast “to and fro” flow present. Vomiting commonly occurs in the pediatric population and has a broad differential diagnosis. Initial evaluation of vomiting should include a thorough history and physical examination, paying close attention to the age of the child and the pattern of the vomiting (acute, chronic, or cyclic). Acute onset of emesis (occurring over a period of 24-48 hours) should prompt consideration for surgical causes of vomiting (small bowel obstruction, intussusception, appendicitis), foreign body or drug ingestion, infections, pancreatitis, and cholelithiasis or choledocholithiasis. Causes of chronic vomiting (occurring for days to weeks) can include gastroesophageal reflux disease, celiac disease, eosinophilic gastrointestinal disease, peptic ulcer disease, drug use (particularly marijuana), SMA syndrome, increased intracranial pressure (brain tumors), and pregnancy. Cyclic vomiting (episodic vomiting with periods of wellness in between) can be associated with malrotation and intermittent midgut volvulus, abdominal migraine, cyclic vomiting syndrome, drug use (marijuana), SMA syndrome, and eating disorders. The differential diagnosis of vomiting according to age is shown in Item C181A. Any pattern of vomiting occurring early in the morning and/or associated with neurologic or visual disturbances should prompt consideration of intracranial processes.

American academy of pediatrics

646

PREP ® Self-Assessment

PREPSA 2020

Abbreviations: CHS, cannabinoid hyperemesis syndrome; FPIES, food protein–induced enterocolitis syndrome; GERD, gastroesophageal reflux disease; IEM, inborn error of metabolism; PUD, peptic ulcer disease; SMA, superior mesenteric artery; UPJ, ureteropelvic junction; UTI, urinary tract infection. Reprinted with permission from Shields TM, Lightdale JR. Vomiting in children. Pediatr Rev. 2018;39(7):347.

In addition to the age of the child and pattern of the vomiting, the history should also include severity (forcefulness) of the vomiting and color of the vomit (bilious or bloody). Forceful nonbilious emesis in an infant around 1 month of age should raise concern for pyloric stenosis, a condition resulting from the narrowing of the gastric outlet due to a hypertrophic pyloric muscle. Dehydration and electrolyte abnormalities (hypokalemic, hypochloremic metabolic alkalosis) may be present. Pyloric ultrasonography is the recommended first radiographic test (Item C181B), although if inconclusive, upper gastrointestinal series may be diagnostic ("string" sign, representing contrast following through a narrowed pylorus). Treatment is a surgical pyloromyotomy, however electrolyte abnormalities should be corrected prior to undergoing anesthesia.

American academy of pediatrics

647

PREP ® Self-Assessment

PREPSA 2020

Item C181B: Hypertrophied pylorus on ultrasonography. Reprinted with permission from Chandran L, Chitkara M. Vomiting in children: reassurance, red flag, or referral? Pediatr Rev. 2008;39(6):189.

Bilious emesis in a neonate or infant should prompt urgent evaluation for bowel obstruction. Intestinal anomalies, including malrotation with volvulus, intestinal atresia, and Hirschsprung disease, should be considered, and emergent surgical consultation obtained. Plain abdominal radiography can be suggestive of a small bowel obstruction in midgut volvulus and a "double bubble" sign (Item C181C) in duodenal atresia (dilated stomach and duodenum). Upper gastrointestinal series can reveal malrotation with a "corkscrew" sign when volvulus is present (Item C181D). Evidence of a large bowel obstruction would be concerning for Hirschsprung disease.

American academy of pediatrics

648

PREP ® Self-Assessment

PREPSA 2020

Item C181C: “Double bubble” sign on abdominal radiograph. Reprinted with permission from Chandran L, Chitkara M. Vomiting in children: reassurance, red flag, or referral? Pediatr Rev. 2008;39(6):187.

American academy of pediatrics

649

PREP ® Self-Assessment

PREPSA 2020

Item C181D: “Corkscrew sign” on upper gastrointestinal series. Reprinted with permission from Chandran L, Chitkara M. Vomiting in children: reassurance, red flag, or referral? Pediatr Rev. 2008;39(6):187. Abdominal ultrasonography and hepatobiliary scan may be important tools to assess a child with emesis, however neither test is indicated when evaluating bilious emesis. Magnetic resonance imaging of the brain should be considered in a child with chronic vomiting, particularly when paired with neurologic symptoms (eg, headache, photophobia), but is not indicated in the evaluation of a child with acute bilious emesis.

American academy of pediatrics

650

PREP ® Self-Assessment

PREPSA 2020

PREP Pearls • Initial evaluation of a child with emesis should include a thorough history and physical examination; the patient’s age and vomiting pattern will narrow the differential diagnosis. • Forceful, projectile vomiting in a young infant should prompt consideration for pyloric stenosis and associated electrolyte abnormalities. • Bilious emesis in a neonate or infant should prompt consideration for intestinal obstruction. ABP Content Specifications(s) • Formulate an age-appropriate differential diagnosis of vomiting • Plan the appropriate management of bilious vomiting in a newborn infant • Plan appropriate management for a patient with intestinal obstruction • Plan the evaluation of projectile vomiting in a newborn infant, and manage appropriately Suggested Readings • McCollough M, Sharieff GQ. Abdominal surgical emergencies in infants and young children. Emerg Med Clin N Am. 2003;21(4):909-935. doi:10.1016/S07338627(03)00090-7. • Shields TM, Lightdale JR. Vomiting in children. Pediatr Rev. 2018;39(7):342-358. doi:10.1542/pir.2017-0053. • Ulshen MH, McGreal N. Vomiting. In: McInerny TK, Adam HM, Campbell DE, DeWitt TG, Foy JM, Kamat DM, eds. American Academy of Pediatrics Textbook of Pediatric Care. 2nd ed. Elk Grove Village, IL: American Academy of Pediatrics; 2017:16621665. Pediatric Care Online.

American academy of pediatrics

651

PREP ® Self-Assessment

PREPSA 2020

Question 182 A 13-year-old adolescent girl is brought to the emergency department 1 hour after sustaining a laceration to her right leg during a street football game. She was running alongside parked cars when she lost her footing, tripped into a car, and sustained a laceration to her right lower leg from a broken bumper. Hemostasis was achieved with pressure at the scene, and her parents brought her directly to the emergency department. She can ambulate without assistance, although she is limping. There is a 5-cm jagged laceration slightly oozing on the lateral aspect of her right lower leg. The wound is cleaned and repaired. Prophylactic antibiotics are administered, and the parents are asked about their child’s tetanus immunization status. They produce her immunization card that shows a complete primary series. The last dose of tetanus vaccine was 7 years ago. Of the following, the MOST appropriate tetanus prophylaxis for this patient is A. DTaP and tetanus immune globulin B. none C. Tdap and tetanus immune globulin D. Tdap only

American academy of pediatrics

652

PREP ® Self-Assessment

PREPSA 2020

Correct Answer: D Lacerations are a common reason for children to be brought to medical attention. Evaluation of a child with a laceration includes: obtaining a thorough history to discern as detailed a mechanism of injury as possible; considering other potential concomitant injuries; examining the wound and focusing on its dimensions (width, length, and depth) and the potential for a foreign body; determining how well the wound approximates; and assessing if there has been any compromise to neurovascular function distal to the wound. Proper management of a laceration includes: thorough cleansing of the wound with normal saline or tap water under pressure; local analgesia (and procedural sedation if necessary); repair of the wound; and consideration of the need for antibiotics and postexposure tetanus prophylaxis. Appropriate postexposure tetanus prophylaxis depends on 2 factors: if the child has received at least 3 doses of tetanus toxoid (typically from a primary series of vaccinations) and if the wound is considered to be clean or dirty. Dirty wounds include burns, bites, crush wounds, and wounds that may be contaminated with soil or dirt. If the child has previously received 3 doses of tetanus toxoid and sustained a clean wound, then no postexposure prophylaxis is indicated if the last dose of tetanus toxoid was within the previous 10 years. If the child previously received 3 doses of tetanus toxoid and sustained a dirty wound, a tetanus vaccine is indicated if the last dose was more than 5 years earlier, as described for the patient in this vignette. A child who did not have 3 previous doses of tetanus toxoid (or if it is uncertain if they did) and who sustained a clean wound requires administration of only a tetanus vaccine, while a dirty wound necessitates administration of both a tetanus vaccine and tetanus immune globulin. Children younger than 7 years who require a tetanus vaccine under this management algorithm should receive the DTaP vaccine, children 7 to 10 years old should receive Td, and children 11 years and older should receive Tdap. The 13-year-old patient in this vignette sustained a dirty wound. Although she had received her primary vaccination series, the last dose was more than 5 years before the injury, necessitating administration of a tetanus vaccine for postexposure prophylaxis. Item C182 contains a summary of the tetanus postexposure prophylaxis recommendations.

American academy of pediatrics

653

PREP ® Self-Assessment

PREPSA 2020

Abbreviations: Tdap, booster tetanus toxoid, reduced diphtheria toxoid, and acellular pertussis vaccine; DTaP, diphtheria and tetanus toxoids and acellular pertussis vaccine; Td, adult-type diphtheria and tetanus toxoids vaccine; TIG, tetanus immune globulin (human). a Such as, but not limited to, wounds contaminated with dirt, feces, soil, and saliva (eg, following animal bites); puncture wounds; avulsions; and wounds resulting from missiles, crushing, burns, and frostbite. b DTaP is used for children younger than 7 years. Tdap is preferred over Td for underimmunized children 7 years and older who have not received Tdap previously. c Immune globulin intravenous should be used when TIG is not available. d More frequent boosters are not needed and can accentuate adverse effects. Reprinted with permission from American Academy of Pediatrics. Tetanus (lockjaw). In: Kimberlin DW, Brady MT, Jackson MA, Long SS, eds. Red Book: 2018 Report of the Committee on Infectious Diseases. 31st ed. Itasca, IL: American Academy of Pediatrics; 2018:796.

Certain types of lacerations demand special consideration. Animal or human bite wounds are typically only sutured if they are on a cosmetically important area, such as the face, and should never be closed using a tissue adhesive because of the high risk for infection. Lacerations American academy of pediatrics 654

PREP ® Self-Assessment

PREPSA 2020

overlying joints necessitate sutures remaining in place for a longer period of time because the movement of the joint makes wound dehiscence more likely with early suture removal. Puncture wounds are at an especially high risk of infection and typically warrant prophylactic antibiotics. Lip lacerations that pass through the vermillion border must be repaired with utmost caution ensuring that the 2 edges of the vermillion border are perfectly aligned because even minimal malalignment can result in easily visible substandard cosmesis and possibly impaired lip function. Similarly, lacerations involving the edge of an eyelid necessitate impeccable approximation because failure to do so may result in poor cosmesis and impaired function. PREP Pearls • In a child who has received 3 or more doses of tetanus toxoid, a clean wound requires postexposure prophylaxis with a tetanus vaccine if the last dose was more than 10 years prior, and a dirty wound requires postexposure prophylaxis if the last dose was more than 5 years prior. • If a child has not received 3 or more doses of tetanus toxoid, any wound requires postexposure prophylaxis with a tetanus vaccine, while a dirty wound also warrants tetanus immune globulin. • If a tetanus vaccine is required, the preferred vaccine is DTap for children younger than 7 years, Td for children aged 7 to 10 years, and Tdap for children aged 11 years and older. ABP Content Specifications(s) • Plan the appropriate use of tetanus immune globulin • Understand the complications associated with various lacerations, including one through the vermilion border of the lip, and manage appropriately Suggested Readings • American Academy of Pediatrics. Tetanus (lockjaw). In: Kimberlin DW, Brady MT, Jackson MA, Long SS, eds. Red Book: 2018 Report of the Committee on Infectious Diseases. 31st ed. Itasca, IL: American Academy of Pediatrics; 2018:793798. https://redbook.solutions.aap.org/chapter.aspx?sectionId=189640195&bookId=2205 &resultClick=1. • Centers for Disease Control and Prevention. Tetanus. In: Hamborsky J, Kroger A, Wolfe S, eds. Epidemiology and Prevention of Vaccine-Preventable Diseases. 13th ed. Washington, DC: Public Health Foundation; 2015:341-352. • Centers for Disease Control and Prevention. Tetanus: for clinicians. https://www.cdc.gov/tetanus/clinicians.html. • Rivera RF, Fagan MJ. Laceration repair. In: Olympia RP, O’Neill RM, Silvis MR, eds. Urgent Care Medicine Secrets. Philadelphia, PA: Elsevier; 2018:270-278.

American academy of pediatrics

655

PREP ® Self-Assessment

PREPSA 2020

Question 183 An 8-year-old boy was referred to an allergist for evaluation of wheezing with viral respiratory infections and frequent cough and wheezing with exercise. He wakes up frequently at night because of his cough, requiring treatment with an albuterol inhaler before he can go back to sleep. He uses this inhaler several additional times every week. He has a long history of nasal congestion, rhinorrhea, and sneezing, especially during the spring pollen season. There is a family history of asthma and inhalant allergies. The allergist report notes that skin tests were positive for cat, dog, house dust mite, grass pollen, and maple tree pollen. The family has a dog that sleeps in the boy’s bed. His mother asks if she needs to keep the dog out of the boy’s bedroom. Of the following, the MOST accurate statement in response to the mother’s question is A. based on the boy’s symptoms, his exposure to pollen is a more likely precipitant for his asthma B. because the boy does not get acute symptoms in response to a specific exposure, it is not likely to be causing his symptoms C. regardless of obvious cause and effect related to exposure, it is appropriate to avoid known allergens D. this change would have a detrimental psychological impact on the child

American academy of pediatrics

656

PREP ® Self-Assessment

PREPSA 2020

Correct Answer: C There are multiple potential triggers for this boy’s asthma, and the first treatment measure should be avoidance of known allergens. The prevalence of nocturnal symptoms and the presence of house dust mite and dog exposures in his bedroom make these highly suspect as allergy triggers for his asthma symptoms. Spring pollen exposure may be a seasonal contributor, but it is not likely the primary trigger for his persistent asthma symptoms. An acute dose/response reaction is not necessary to identify allergens in the context of daily or continual exposure, such as occurs with indoor perennial allergens. Although viral respiratory infections are the most common precipitant for acute asthma episodes, chronic allergen exposure is a primary factor in the recurrence and persistence of asthma symptoms in sensitized children. Whether or not removing the dog from the bedroom will have a detrimental psychological effect on the boy is uncertain; however, the potential positive impact of removing this allergen exposure is significant. PREP Pearls • Indoor allergens are common precipitants for perennial asthma symptoms in allergic children. • Avoidance of known allergens is the first rule in the control of allergic asthma. ABP Content Specifications(s) • Understand the natural history of asthma and the factors that affect it Suggested Readings • Fitzpatrick AM, Bacharier LB, Guilbert TW, et al. Phenotypes of recurrent wheezing in preschool children: identification by latent class analysis and utility in prediction of future exacerbation. J All Clin Immunol Pract. 2018;7(3):915-924.e7. doi:10.1016/j.jaip.2018.09.016. • Gaffin JM, Phipatanakul W. The role of indoor allergens in the development of asthma. Curr Opin Allergy Clin Immunol. 2009;9(2):128-135. doi:10.1097/ACI.0b013e32832678b0. • Hill VL, Wood PR. Asthma epidemiology, pathophysiology, and initial evaluation. Pediatr Rev. 2009;30(9):331-335. doi:10.1542/pir.30-9-331. • Rhee H, Love T, Harrington D, Grape A. Common allergens in urban adolescents and their relationships with asthma control and healthcare utilization. Allergy Asthma Clin Immunol. 2018;14:33. doi:10.1186/s13223-018-0260-y. • Salo PM, Cohn RD, Zelden DC. Bedroom allergen exposure beyond house dust mites. Curr Allergy Asthma Rep. 2018;18(10):52. doi:10.1007/s11882-018-0805-7.

American academy of pediatrics

657

PREP ® Self-Assessment

PREPSA 2020

Question 184 A 6-year-old boy is seen for evaluation of ear pain and fever. He has had pain in the right ear for 2 days and fever as high as 39.5°C. He has had 3 previous episodes of otitis media. In the medical record, penicillin is listed as an allergy with a description of a rash for the allergic reaction. He has a temperature of 38.8°C, blood pressure of 97/57 mm Hg, heart rate of 110 beats/min, and respiratory rate of 20 breaths/min. Examination of the right ear is shown in Item Q184. The remainder of the physical examination findings are normal.

Item Q184: Examination findings for the right ear of the boy in the vignette Reprinted with permission from Siegel RM, Bien JP. Pediatr Rev. 2004;25(6):189.

Of the following, the antibiotic that is BEST suited for this boy’s infection is A. amoxicillin-clavulanate B. azithromycin C. cefdinir D. trimethoprim-sulfamethoxazole

American academy of pediatrics

658

PREP ® Self-Assessment

PREPSA 2020

Correct Answer: C The antibiotic that is best suited for the boy in this vignette is cefdinir. The American Academy of Pediatrics guidelines for the management of otitis media list several cephalosporins, including cefdinir, as acceptable alternatives in the setting of penicillin allergy. Only 2% of individuals with a penicillin allergy are expected to react to cephalosporins, and less than 1% will react to carbapenems. In the United States, up to 10% of the population is labeled as penicillin allergic. However, after allergy testing or challenge, less than 10% of individuals that report an antibiotic allergy are confirmed to be allergic. When a reported allergy is encountered, it is prudent to solicit detailed information of the purported reaction. Most rashes attributed to antibiotics are in fact viral in origin or caused by a drug-virus interaction. Patients that are labelled with drug allergies have higher drug prescriptions costs and higher rates of antibiotic-resistant infections as a result of being exposed to more broad-spectrum agents. Penicillins and cephalosporins are broadly characterized as β-lactam antibiotics. Other β-lactam antibiotics include carbapenems, monobactams, and β-lactamase inhibitors. β-Lactam refers to a structural feature, a 4-membered β-lactam ring, shared by all antibiotics in the class. There is a side chain that rises from the β-lactam ring called the R1 side chain. Penicillins and cephalosporins have R1 side chains, and the R1 side chain is a major determinant of crossreactivity. In the setting of a penicillin allergy, as in the vignette, the risk of cross-reacting is minimized by choosing a cephalosporin with an R1 side chain that is different from the R1 side chain on the penicillin drug to which there is an allergy. β-Lactam antibiotics inhibit enzymes used by bacteria for cell wall synthesis. Adverse effects of β-lactam antibiotics include gastrointestinal upset, diarrhea, and rashes. IgE-mediated hypersensitivity can occur, as well as delayed-onset reactions including Stevens-Johnson syndrome, acute interstitial nephritis, serum sickness, and drug-induced cytopenias. Amoxicillin-clavulanate is in the penicillin class and should be avoided if a drug allergy is suspected. Because of decreased susceptibility in Streptococcus pneumoniae isolates, azithromycin and trimethoprim-sulfamethoxazole should not be used to treat otitis media. Additionally, azithromycin has limited activity against Haemophilus influenzae, which is the causative agent in a significant number of otitis media cases. PREP Pearls • β-Lactam antibiotics, which include penicillins, cephalosporins, and carbapenems, inhibit enzymes used by bacteria for cell wall synthesis. • β-Lactam antibiotics can cause gastrointestinal upset, diarrhea, rashes, delayed-onset reactions (Stevens-Johnson syndrome, acute interstitial nephritis, serum sickness, and drug-induced cytopenias), and IgE-mediated hypersensitivity. • For a patient with penicillin allergy, use of a cephalosporin with a different R1 side chain minimizes the risk of a cross-reaction. ABP Content Specifications(s) • Know the mechanism of action of penicillin and other beta-lactam antibiotics • Recognize the adverse effects associated with the use of various antibiotic drugs American academy of pediatrics

659

PREP ® Self-Assessment

PREPSA 2020

Suggested Readings • Lieberthal A, Carroll A, Chonmaitree T, et al. The diagnosis and management of acute otitis media. Pediatrics. 2013;131(3):e964-e999. doi:10.1542/peds.2012-3488. • Norton A, Konvinse K, Phillips E, Broyles A. Antibiotic allergy in pediatrics. Pediatrics. 2018;141(5):e20172497. doi:10.1542/peds.2017-2497. • Rosa-Olivares J, Porro A, Rodriguez-Varela M, Riefkohl G, Niroomand-Rad I. Otitis media: to treat, to refer, to do nothing: a review for the practitioner. Pediatr Rev. 2015;36(11):480-488. doi:10.1542/pir.36-11-480.

American academy of pediatrics

660

PREP ® Self-Assessment

PREPSA 2020

Question 185 A 16-year-old adolescent boy undergoes a follow-up evaluation after a recent hospital stay for a deep venous thrombosis of his right lower extremity. He is taking anticoagulation medication. He has a history of mild intellectual disability, severe myopia, and downward dislocation of the lens. He has tall stature, long limbs, pectus excavatum, moderate levothoracic scoliosis, higharched palate, and arachnodactyly. He has no joint hypermobility. Complete blood cell count is unremarkable. Findings of a coagulation evaluation were unremarkable, other than an abnormal plasma amino acid analysis that showed a methionine level of 70 μmol/L (reference range 10-40 μmol/L). A genetic disorder is suspected. Of the following, the BEST test to determine this patient's diagnosis is A. FBN1 analysis B. NSD1 analysis C. total plasma homocysteine D. urine S-sulfocysteine

American academy of pediatrics

661

PREP ® Self-Assessment

PREPSA 2020

Correct Answer: C The history, physical examination, and laboratory findings for the patient in the vignette suggest that he has homocystinuria, an amino acid disorder. A markedly increased total plasma homocysteine level, in conjunction with an elevated methionine level on serum amino acid analysis, would be biochemically consistent with a diagnosis of homocystinuria, also known as cystathionine β-synthase (CBS) deficiency. The diagnosis can be further confirmed with the detection of biallelic pathogenic mutations in CBS because this is an autosomal recessive disorder. It is identified on newborn screening via tandem mass spectrometry which detects most but not all cases. Homocystinuria characteristically presents with a marfanoid habitus (tall stature, pectus excavatum, long limbs, and scoliosis), eye findings (severe myopia and risk of downward ectopia lentis), intellectual disability/developmental delay, and vascular thromboembolic events. Downward dislocation of the eye lens is typical rather than the upward dislocation seen in Marfan syndrome. Vascular events are the primary cause of morbidity and mortality in CBS deficiency. Management of homocystinuria involves prevention of thromboembolic events and normalization or near-normalization of the plasma homocysteine concentration. Treatment includes vitamin B6 (pyridoxine) supplementation for those who are B6 responsive, a methionine-restricted diet, betaine therapy, and folate and vitamin B12 supplementation. Betaine therapy yields an alternate remethylation pathway for the conversion of homocysteine to methionine and is helpful in preventing thrombosis. Amino acid disorders typically involve impairment in the breakdown of amino acids or in the transport of amino acids into the cells, leading to a buildup of harmful substances in the body that can lead to serious or life-threatening clinical presentations. Other amino acid disorders include isovaleric acidemia, propionic acidemia, maple syrup urine disease, phenylketonuria, methylmalonic acidemia, and tyrosinemia. Many of these conditions present with neonatal onset of irritability, poor feeding, seizures, and worsening metabolic encephalopathy; others, such as phenylketonuria, present with progressive and irreversible intellectual disability because of the accumulation of a specific harmful amino acid. Most amino acid disorders are included in statewide newborn screening programs because of the importance of early intervention and treatment in optimizing clinical outcomes. FBN1 analysis would be an appropriate test if Marfan syndrome were highly suspected; however, the elevated methionine and homocysteine combined with the downward ectopia lentis and mild intellectual disability would make homocystinuria more likely in this clinical vignette. NSD1 mutations cause Sotos syndrome, which presents as an overgrowth syndrome with learning disability/developmental delay and a distinctive facial appearance (sparse frontotemporal hair, downslanting palpebral fissures, prominent forehead, and elongated face). Isolated sulfite oxidase deficiency (ISOD) manifests with elevated urine S-sulfocysteine levels and reduced levels of total plasma homocysteine. Clinically ISOD presents from birth to 18 months of age with ectopia lentis, seizures, feeding difficulties, episodic encephalopathy, developmental delay/regression, and an evolving movement disorder (dystonia, choreoathetosis, and ataxia) without evidence of overgrowth or tall stature. American academy of pediatrics

662

PREP ® Self-Assessment

PREPSA 2020

PREP Pearls • Homocystinuria, also known as cystathionine β-synthase deficiency, characteristically presents with a marfanoid habitus (tall stature, pectus excavatum, long limbs, and scoliosis), eye findings (severe myopia and risk of downward ectopia lentis), intellectual disability/developmental delay, and vascular thromboembolic events. • Cardinal biochemical features of homocystinuria include a markedly elevated total plasma homocysteine and elevated methionine level on serum amino acid analysis. • Many amino acid disorders are detectable on newborn screening via tandem mass spectrometry; early intervention, both dietary and via medical therapies, will optimize patient outcomes and save lives. ABP Content Specifications(s) • Recognize the clinical features associated with a disorder of amino acid metabolism other than phenylketonuria Suggested Readings • Baric I, Staufner C, Augoustides-Savvopoulou P, et al. Consensus recommendations for the diagnosis, treatment and follow-up of inherited methylation disorders. J Inherit Metab Dis. 2017;40:5-20. doi:10.1007/s10545-016-9972-7. • Morris AA, Kožich V, Santra S, et al. Guidelines for the diagnosis and management of cystathionine ß-synthase deficiency. J Inherit Metab Dis. 2017;40:49-74. doi:10.1007/s10545-016-9979-0. • Rios A, Adams DJ. Specific congenital metabolic diseases. In: McInerny TK, Adam HM, Campbell DE, DeWitt TG, Foy JM, Kamat DM, eds. American Academy of Pediatrics Textbook of Pediatric Care. 2nd ed. Elk Grove Village, IL: American Academy of Pediatrics; 2017:938-963. Pediatric Care Online. • Sacharow SJ, Picker JD, Levy HL. Homocystinuria caused by cystathionine betasynthase deficiency. GeneReviews. https://www.ncbi.nlm.nih.gov/books/NBK1524/. • Saronwala A, Kubendran S, Kahler SG. Screening for genetic-metabolic diseases. In: McInerny TK, Adam HM, Campbell DE, DeWitt TG, Foy JM, Kamat DM, eds. American Academy of Pediatrics Textbook of Pediatric Care. 2nd ed. Elk Grove Village, IL: American Academy of Pediatrics; 2017:210-225. Pediatric Care Online.

American academy of pediatrics

663

PREP ® Self-Assessment

PREPSA 2020

Question 186 A 3-month-old female infant is brought to the emergency department by her mother because the infant has been eating less and breathing faster for the past 2 days. She has had no fever, rash, rhinorrhea, congestion, or vomiting. Her temperature is 37.0°C, heart rate is 170 beats/minute, respiratory rate is 60 breaths/min, room air oxygen saturation is 95%, and blood pressure is 85/55 mm Hg. She is alert and somewhat irritable. There are mild subcostal retractions. Her heart has a regular rhythm with a gallop, but no murmur is heard on auscultation, and the liver edge is palpable 3 cm below the right costal margin. Pulses are strong; the infant is warm centrally and cool to the touch at the toes and fingers. Chest radiography (Item Q186A) and electrocardiography (Item Q186B) are performed.

Item Q186A: Chest radiograph for the infant described in the vignette. Reprinted with permission from Harper BD. Pediatr Rev. 2017;38(7):338.

American academy of pediatrics

664

PREP ® Self-Assessment

PREPSA 2020

Item Q186B: Electrocardiogram for the infant described in the vignette. Reprinted with permission from Harper BD. Pediatr Rev. 2017;38(7):340. Of the following, the MOST appropriate therapy to correct this infant’s primary condition is A. cardiac surgery B. digoxin C. furosemide D. increasing caloric density of feeds

American academy of pediatrics

665

PREP ® Self-Assessment

PREPSA 2020

Correct Answer: A The infant in the vignette has respiratory distress and physical examination findings that include a gallop and hepatomegaly. These findings are suggestive of congestive heart failure (CHF) and prompted the provider to obtain a chest radiograph, which demonstrates cardiomegaly and pulmonary edema. Electrocardiography (ECG) demonstrates deep Q waves in the inferior leads (II, III, AVF). The child is in compensated cardiogenic shock with evidence of decreased perfusion to the distal extremities. This constellation of findings, particularly the ECG abnormality, is suggestive of an anomalous left coronary artery from the pulmonary artery (ALCAPA). This anomaly requires prompt surgical repair which is the definitive therapy. Many children will have full recovery of myocardial function after repair. Although digoxin, furosemide, and diet may improve her clinical status, surgery would be required to correct this infant’s primary condition. When the left coronary artery arises from the pulmonary artery and not the aorta, it leads to an ALCAPA. At birth, the condition is typically asymptomatic. The natural history of pulmonary vascular resistance is that it begins to drop in the first few months after birth. As the pulmonary vascular resistance begins to drop, and consequently, the pulmonary artery pressure begins to drop, the perfusion pressure of this coronary artery also drops, resulting in myocardial ischemia and necrosis. In some children who develop collateral vessels from the right coronary artery, this condition will manifest later than in the infant in the vignette. Other etiologies should be considered when CHF presents outside the neonatal period. In addition to ALCAPA, systolic dysfunction is also seen in myocarditis and cardiomyopathy. Children with myocarditis may recover function. If they do not, as with cardiomyopathy and decreased function, they may need oral medications and some will need cardiac transplantation. Although cardiac function is normal in large left-to-right shunts (eg, ventricular septal defect), the increased pulmonary blood flow can result in increased work of breathing and respiratory distress. Ultimately these children will need cardiac surgery, but until then, they can often be treated with diuretics and increased caloric density feeds. Other etiologies of CHF include acquired heart diseases such as Kawasaki disease, endocarditis, and rheumatic heart disease. Kawasaki disease with coronary involvement can result in ischemia and decreased cardiac function. Endocarditis and rheumatic heart disease can result in valve pathology that causes stenosis, regurgitation, or both, and can also present with heart failure. PREP Pearls • Physical examination findings of respiratory distress, cardiac gallop, and hepatomegaly are suggestive of congestive heart failure. • Diagnostic evaluation for congestive heart failure includes a chest radiograph (which may demonstrate cardiomegaly and pulmonary edema) and electrocardiography. • An infant with congestive heart failure and electrocardiogram showing deep Q waves in the inferior leads (II, III, AVF) should be evaluated for an anomalous left coronary artery from the pulmonary artery (ALCAPA).

American academy of pediatrics

666

PREP ® Self-Assessment

PREPSA 2020

ABP Content Specifications(s) • Plan the appropriate initial diagnostic evaluation of congestive heart failure in children of various ages • Plan the appropriate initial management of congestive heart failure in children of various ages Suggested Readings • Harper BD. Acute-onset respiratory failure in a 4-month-old girl. Pediatr Rev. 2017;38(7):338-341. doi:10.1542/pir.2016-0093. • Madriago E, Silberbach M. Heart failure in infants and children. Pediatr Rev. 2010;31(1):4-12. doi:10.1542/pir.31-1-4. • Rusconi PG, Harmon W, Wilkinson JD, Lipshultz SE. Heart failure. In: McInerny TK, Adam HM, Campbell DE, DeWitt TG, Foy JM, Kamat DM, eds. American Academy of Pediatrics Textbook of Pediatric Care. 2nd ed. Elk Grove Village, IL: American Academy of Pediatrics; 2017:2868-2878. Pediatric Care Online.

American academy of pediatrics

667

PREP ® Self-Assessment

PREPSA 2020

Question 187 A term neonate who was born with “a rash” is being evaluated. He was the product of an uncomplicated full-term pregnancy, labor, and delivery. The infant has normal vital signs and appears well. The physical examination findings are remarkable only for skin lesions. The eruption involves the face, trunk, and extremities (Item Q187A and Item Q187B).

Item Q187A: Eruption on the face of the neonate in the vignette. Courtesy of L. VonCannon

American academy of pediatrics

668

PREP ® Self-Assessment

PREPSA 2020

Item Q187B: Eruption on the arm of the neonate in the vignette. Courtesy of L. VonCannon

Of the following, microscopic examination of the contents of the described lesions is MOST likely to demonstrate A. eosinophils B. gram-positive cocci C. multinucleated giant cells D. neutrophils

American academy of pediatrics

669

PREP ® Self-Assessment

PREPSA 2020

Correct Answer: D The infant in the vignette has pustules without surrounding erythema concentrated on the face and extremities and small round hyperpigmented macules, some of which are surrounded by scale. These findings are consistent with a diagnosis of transient neonatal pustular melanosis (TNPM). Microscopic examination of pustule contents would reveal neutrophils but no organisms. Transient neonatal pustular melanosis is a self-limited disorder of unknown cause that occurs most often in African American infants. The condition begins in utero as sterile pustules that may rupture or remain intact after delivery. When pustules rupture, the infant is left with round hyperpigmented macules (the bases of pustules) that often are surrounded by a rim or collarette of scale (the remnants of pustule roofs) (Item C187A). The lesions of TNPM may be widespread but tend to be concentrated on the forehead, chin, neck, lower back, and shins. Pustules resolve 24 to 48 hours after birth and hyperpigmented macules fade in several weeks to months.

Item C187A: When the pustules of transient neonatal pustular melanosis rupture they leave small hyperpigmented macules surrounded by a collarette of scale. Courtesy of D. Krowchuk

The differential diagnosis of vesicular or pustular lesions in a neonate includes a number of disorders, including: • Erythema toxicum: A benign, self-limited disorder of unknown cause. It occurs commonly in term infants but is unusual in those who are preterm. The onset is usually at 24 to 48 hours after birth but may be delayed to 10 days after birth. The lesions usually resolve within weeks. Lesions are erythematous macules that develop a yellow or white central papule, vesicle, or pustule (Item C187B and Item C187C). Vesicles and pustules contain eosinophils (Item C187D). • Impetigo neonatorum: Caused by Staphylococcus aureus infection, impetigo neonatorum appears as pustules or bullae on erythematous bases. Lesions are often concentrated in the diaper area, axillae, and neck folds. When lesions rupture, erosions appear, not American academy of pediatrics

670

PREP ® Self-Assessment •

PREPSA 2020

hyperpigmented macules (Item C187E). A Gram stain of lesional fluid would reveal gram-positive cocci. Neonatal herpes simplex virus infection: Typically, lesions are grouped vesicles on an erythematous base. A Tzanck smear performed on material scraped from the base of a ruptured vesicle would reveal multinucleated giant cells.

Item C187B: Erythematous macules with a central papule are characteristic of erythema toxicum. Reprinted with permission from Mancini AJ, Krowchuk DP, eds. Pediatric Dermatology: A Quick Reference Guide. 3rd ed. Elk Grove Village, IL: American Academy of Pediatrics; 2016:581.

Item C187C: Erythema toxicum involving the knee. Each lesion has a white or yellow central papule. Reprinted with permission from P. Sagerman.

American academy of pediatrics

671

PREP ® Self-Assessment

PREPSA 2020

Item C187D: Wright stain of fluid from a vesicle in erythema toxicum demonstrating eosinophils (arrows) containing orange-red granules stained with eosin. Courtesy of D. Krowchuk

Item C187E: Fragile bullae and erosions are characteristic of impetigo in the neonatal period. Reprinted with permission from Mancini AJ, Krowchuk DP, eds. Pediatric Dermatology: A Quick Reference Guide. 3rd ed. Elk Grove Village, IL: American Academy of Pediatrics; 2016:572.

PREP Pearls • Transient neonatal pustular melanosis is present at birth and usually appears as small, round hyperpigmented macules with a rim or collarette of scale. In a minority of infants, pustules contain polymorphonuclear neutrophils. • Erythema toxicum usually appears at 24 to 48 hours of age as erythematous macules with a central yellow or white papule, vesicle, or pustule. The vesicles contain eosinophils.

American academy of pediatrics

672

PREP ® Self-Assessment

PREPSA 2020

MOCA-Peds Objective • Evaluate and manage common neonatal skin disorders. ABP Content Specifications(s) • Recognize the clinical and cytologic findings associated with erythema toxicum • Differentiate the laboratory findings associated with transient neonatal pustular melanosis from those of staphylococcal pustules • Recognize the clinical findings associated with transient neonatal pustular melanosis Suggested Readings • Kaur H, Campbell DE. Physical examination of the newborn. In: McInerny TK, Adam HM, Campbell DE, DeWitt TG, Foy JM, Kamat DM, eds. American Academy of Pediatrics Textbook of Pediatric Care. 2nd ed. Elk Grove Village, IL: American Academy of Pediatrics; 2017:802-819. Pediatric Care Online. • Section on Dermatology, American Academy of Pediatrics. Transient neonatal pustular melanosis. In: Mancini AJ, Krowchuk DP, eds. Pediatric Dermatology: A Quick Reference Guide. 3rd ed. Elk Grove Village, IL: American Academy of Pediatrics; 2016:597-602.

American academy of pediatrics

673

PREP ® Self-Assessment

PREPSA 2020

Question 188 A 15-year-old girl is seen for her health supervision visit. Her mother is concerned that she may have started smoking cigarettes. With her mother out of the room, the patient discloses that she started smoking cigarettes approximately 3 months ago. She smokes half a pack per week, and is surprised that her mother notices the smell of smoke in her clothes because her mother also smokes cigarettes. The girl states that she smokes with her friends after school as a stress reliever, and that she can stop smoking at anytime; she knows she would never become dependent on cigarettes. She has not tried any illicit drugs or alcohol and denies any other highrisk behaviors. Of the following, the BEST next management step for this girl is to A. discuss the long-term health consequences associated with smoking tobacco B. order a urine toxicology screen C. recommend the use of electronic cigarettes D. use motivational interviewing to counsel the girl regarding smoking

American academy of pediatrics

674

PREP ® Self-Assessment

PREPSA 2020

Correct Answer: D The girl in the vignette is engaging in behavior that she knows her mother would not approve of; she has decided to experiment with smoking cigarettes. The best next management step for her would be a discussion using motivational interviewing to determine if she is at a stage of readiness for behavior change, and if so, to help her move forward. Discussing the long-term health consequences associated with smoking tobacco is not likely to make a strong impact, especially if this girl is still predominantly a concrete thinker; it is unlikely that she will be able to appreciate any health risks in the future while she is focused on the present. Ordering a urine toxicology screen is not likely to yield any useful information unless there is a concern regarding the use of other substances. Recommending the use of electronic cigarettes would not be appropriate, because they have many of the same health consequences as smoking regular cigarettes and do not aid in tobacco cessation. Adolescence is a time of transition from childhood to adulthood and includes biological, psychological, and social changes. Adolescence can be divided into early (10-13 years), middle (14-17 years), and late stages (18-21 years), and each stage has hallmark characteristics. Puberty marks the beginning of the biological and physical changes that occur during adolescence. Physical changes are initiated because of maturation of the hypothalamic-pituitarygonadal axis and hypothalamic-pituitary-adrenal axis, resulting in an increase in growth hormone, and stimulates reproductive function and the development of secondary sexual characteristics. In girls, puberty begins between 8 and 13 years of age, and in boys, between 9 and 14 years of age. Hormonal changes lead to weight gain, growth in height, and change in shape. The hallmark of puberty in girls is usually breast development, followed by the development of pubic hair, more rapid growth in height, and then menarche. Menarche occurs approximately 2 to 2.5 years after breast development, at an average age of 12 to 12.5 years. The hallmark of puberty in boys is usually testicular enlargement, followed by the development of pubic hair, penile growth, and more rapid growth in height. The physical changes that adolescents are adjusting to coincide with substantial psychological and social changes. This is also a time of significant growth in brain development in the domains of decision making, executive functioning, and response inhibition. The fact that the prefrontal cortex, important for executive functioning, matures late in adolescence may help explain why adolescents often engage in high-risk behaviors. Certain behaviors and characteristics are associated with the different stages (early, middle, and late) of adolescence (Item C188). The HEADDSS (home, education, activities, drugs and alcohol, depression/suicide, sex, safety/violence) examination is an important tool for providers who care for adolescents. It provides insight into adolescents’ stage of cognitive development, how they are adjusting to and managing new-found independence, and if they are engaging in any high-risk behaviors. Providers should use this tool at each adolescent health supervision visit.

American academy of pediatrics

675

PREP ® Self-Assessment

American academy of pediatrics

PREPSA 2020

676

PREP ® Self-Assessment

PREPSA 2020

PREP Pearls • Motivational interviewing is a useful approach to determine whether adolescents are at a stage of readiness for behavior change, and if so, to help them move forward. • Adolescent development has distinct biological, psychological, and social phases: early (10-13 years); middle (14-17 years); and late (18-21 years). • The prefrontal cortex, responsible for executive functioning, matures in the late stages of adolescent development, which may explain why adolescents engage in more high-risk behaviors. • The HEADDSS examination is a useful screening tool to assess for high-risk behaviors in adolescents. ABP Content Specifications(s) • Recognize the sequence of emotional and cognitive development and physical maturation • Recognize the tasks and features of early, mid, and late adolescent socioemotional development • Understand the importance of routinely reviewing behavioral risk factors (eg, school, extracurricular activities, diet, exercise, substance use, sexuality, stress, personal safety, driving, sleep) in adolescents • Recognize how thought processes in early, middle, and late adolescence influence problem solving and risk taking Suggested Readings • American Academy of Pediatrics. Promoting healthy development. In: Hagan JF, Shaw JS, Duncan PM, eds. Bright Futures: Guidelines for Health Supervision of Infants, Children, and Adolescents. 4th ed. Elk Grove Village, IL: American Academy of Pediatrics; 2017:77-114. Pediatric Care Online. • Sanders RA. Adolescent psychosocial, social, and cognitive development. Pediatr Rev. 2013;34(8):354-359. doi:10.1542/pir.34-8-354. • Schaefer MR, Kavookjian J. The impact of motivational interviewing on adherence and symptom severity in adolescents and young adults with chronic illness: a systematic review. Patient Educ Counsel. 2017;100(12):2190-2199. doi:10.1016/j.pec.2017.05.037. • Wolf RM, Long D. Pubertal development. Pediatr Rev. 2016;37(7):292-300. doi:10.1542/pir.2015-0065.

American academy of pediatrics

677

PREP ® Self-Assessment

PREPSA 2020

Question 189 The mother of a 4-month-old boy was recently diagnosed with shingles on her lower abdomen. The boy was born at term and has been growing and developing normally with no intercurrent illnesses. His mother is wondering if she can continue to breastfeed. Of the following, the MOST accurate response is that she should A. continue breastfeeding without interruption B. continue breastfeeding after vaccinating the infant against varicella C. pump and discard her milk until the lesions heal D. stop breastfeeding immediately

American academy of pediatrics

678

PREP ® Self-Assessment

PREPSA 2020

Correct Answer: A The mother described in the vignette has shingles, or herpes zoster infection, on her lower abdomen caused by reactivation of varicella-zoster virus. This virus can be transmitted from direct contact with herpes zoster lesions. However, because the mother can and should cover the lesions to prevent her breastfeeding infant from coming into contact with them, she may continue to breastfeed without interruption. At 4 months of age, this infant is too young to receive the varicella vaccine. Active shingles is not an indication for a mother to stop breastfeeding or pump and discard her milk. Breastfeeding is the optimal feeding method for neonates and infants. It confers significant health benefits for both the mother and child. There are very few contraindications to breastfeeding; these include a limited number of maternal infections, maternal medications, and infant conditions (Item C189).

It is important to note that maternal infection with hepatitis B, hepatitis C, or cytomegalovirus are not contraindications to breastfeeding. Mothers with herpes simplex virus infection may continue to breastfeed unless the lesions are on the nipple or breast; in this situation, a mother should be advised to cover the lesions and feed temporarily only from the unaffected breast. The National Institutes of Health maintains a free, online database of medication safety during lactation, LactMed (https://toxnet.nlm.nih.gov/newtoxnet/lactmed.htm). Mothers with substance use disorders who receive long-term opiate replacement maintenance therapy (eg, methadone) should be encouraged to breastfeed. Neonates and infants with phenylketonuria or metabolic American academy of pediatrics

679

PREP ® Self-Assessment

PREPSA 2020

disorders other than classic galactosemia can typically breastfeed. These cases often require supplementation with medical formulas and close laboratory monitoring. PREP Pearls • There are very few contraindications to breastfeeding, including: o Specific maternal infections (untreated active tuberculosis, HIV in developed countries, human T-cell lymphotropic virus type I or II, untreated brucellosis, and ebola virus) o A limited number of maternal medications (illicit drugs, chemotherapy, radioactive medications) o galactosemia in the infant • The National Institutes of Health maintain a free, online database of medication safety during lactation, LactMed (https://toxnet.nlm.nih.gov/newtoxnet/lactmed.htm). ABP Content Specifications(s) • Recognize when breast-feeding should be interrupted because of maternal infection Suggested Readings • American Academy of Pediatrics, Committee on Drugs. The transfer of drugs and therapeutics into human breast milk: an update on selected topics. Pediatrics. 2013;132(3):e796-e809. doi:10.1542/peds.2013-1985. • American Academy of Pediatrics, Section on Breastfeeding. Breastfeeding and the use of human milk. Pediatrics. 2012;129(3):e827-e841. doi:10.1542/peds.2011-3552. • American Academy of Pediatrics. Varicella-zoster virus infection. In: Kimberlin DW, Long SS, Brady MT, Jackson MA, eds. Red Book 2018 Report of the Committee on Infectious Diseases. 31st ed. Itasca, IL: American Academy of Pediatrics; 2018:869-882. • Centers for Disease Control and Prevention. Contraindications to breastfeeding or feeding expressed breast milk to infants. https://www.cdc.gov/breastfeeding/breastfeeding-specialcircumstances/contraindications-to-breastfeeding.html.

American academy of pediatrics

680

PREP ® Self-Assessment

PREPSA 2020

Question 190 A multi-institutional study examines the impact of a web-based intervention for weight loss in adolescents who are obese. The study requires participation in an interactive internet-based intervention; measurement of height, weight, body mass index, and abdominal girth every month; and blood tests every 6 months. A 17-year-old adolescent boy seen in the local investigator’s practice meets all eligibility criteria, and he is approached for participation in the study. Of the following, study enrollment requires A. informed consent from the adolescent and assent from his parents B. informed consent from the adolescent and his parents C. informed consent from the adolescent, but no consent from his parents D. informed consent from the parents and assent from the adolescent

American academy of pediatrics

681

PREP ® Self-Assessment

PREPSA 2020

Correct Answer: D Patient involvement in research studies requires the provision of informed consent through a 3step process that includes: 1. Disclosure of information regarding the research to the patient or their surrogate 2. Assessment of their understanding of the information and their capacity for medical decision-making 3. An agreement to participate in the research formalized through the signage of a consent document Inclusion of minors (defined in most states as people younger than 18 years) in research is somewhat more complex, as minors have limited autonomy and are reliant on proxies (parents or legal guardians) to provide permission for their participation in research. As such, they are considered a vulnerable population and have been granted extra legal protections (Subpart D of the Common Rule [Title 45, CFR 46] and Food and Drug Administration [Title 21, Part 50]). In addition to an informed consent process, Subpart D 408 establishes the need for assent when approaching minors who are old enough to have an understanding of their involvement in research studies. Assent requires that minors receive an explanation of the research in an ageappropriate manner and then agree to participate. Many institutional review boards require minors to provide written assent, in addition to a formal consent provided by their proxies. While the minimum age of assent varies, children as young as 7 years have developed limited logical thought processes and have the capacity for reasoned decision-making. The 17-year-old adolescent boy in the vignette is considered a minor. As such, he needs to provide assent in addition to the consent provided by his parents. The provision of informed consent must come from a patient or proxy of the age of majority. As the adolescent boy in the vignette is not of the age of majority, he is not able to provide informed consent. Informed consent must come from his parents. PREP Pearls • Informed consent for participation in research involves the provision of information about the research, assessment of the participant’s understanding of the research and capacity for medical decision-making, and the signing of a formal consent document. • Involvement of minors (people younger than 18 years) in research requires informed consent from their adult proxies (parents or legal guardians), as well as assent from the minors themselves if they are old enough to understand their participation. • While the minimum age of assent varies, children as young as 7 years have developed limited logical thought processes and have the capacity for reasoned decision-making. ABP Content Specifications(s) • Understand the difference between informed consent and assent • Recognize and apply ethical principles involved in the patient-parent-pediatrician relationship regarding issues of informed consent/dissent/assent

American academy of pediatrics

682

PREP ® Self-Assessment

PREPSA 2020

Suggested Readings • Committee on Bioethics. Informed consent in decision-making in pediatric practice. Pediatrics. 2016;138(2):e20161484. doi:10.1542/peds.2016-1484. • Katz AL, Webb SA; Committee on Bioethics. Informed consent in decision-making in pediatric practice. Pediatrics. 2016;138(2):e20161485. doi:10.1542/peds.2016-1485. • Rose CD. Ethical conduct of research in children: pediatricians and their IRB (part 2 of 2). Pediatrics. 2017;139(6):e20163650. doi:10.1542/peds.2016-3650.

American academy of pediatrics

683

PREP ® Self-Assessment

PREPSA 2020

Question 191 A 10-year-old African American boy with sickle cell anemia is admitted to the hospital for intravenous fluids and pain control because of dehydration and acute pain crisis. After 12 hours in the hospital he is found to be acutely unresponsive. Naloxone is administered with no change in his clinical status, and he is emergently transferred to the pediatric intensive care unit. He has a temperature of 38.1°C, heart rate of 160 beats/min, respiratory rate of 16 breaths/min, and blood pressure of 60/30 mm Hg. His general physical examination is notable for a rigid neck, tacky mucous membranes, and tachycardia with a flow murmur. His lungs are clear. He is obtunded and groans to painful stimuli without localizing it. His pupils are equal, round, and reactive to light, with intact extraocular movements and symmetric facies. He moves all extremities equally to noxious stimuli. Deep tendon reflexes are 2+ throughout with toes downgoing to plantar stimulation. Because of his deteriorating condition, he is intubated for airway protection, begun on fluid resuscitation, and placed on broad-spectrum antibiotics after obtaining blood cultures. Noncontrast computed tomography of the head performed as part of a stroke evaluation has normal results. Lumbar puncture is performed with an opening pressure of 61 cm H2O. The cerebrospinal fluid is cloudy, and laboratory data from its analysis are shown: Laboratory Test White blood cell count Neutrophils Lymphocytes Monocytes Red blood cell count Protein Glucose

Result 1,300/µL 70% 25% 5% 0/µL 145 mg/dL 10 mg/dL (0.6 mmol/L)

Of the following, the MOST likely cause of this child’s meningitis is A. Haemophilus influenzae B. herpes simplex virus C. Salmonella D. Streptococcus pneumoniae

American academy of pediatrics

684

PREP ® Self-Assessment

PREPSA 2020

Correct Answer: D The patient in this vignette has bacterial meningitis most likely caused by Streptococcus pneumoniae. Children with sickle cell disease are predisposed to invasive bacterial infections from encapsulated organisms such as S pneumoniae, Haemophilus influenzae type b (Hib), and nontyphoidal Salmonella due to functional asplenia, which results in diminished antibodymediated phagocytosis. Of these organisms, infection with S pneumoniae occurs most frequently. Clinical manifestations of encapsulated bacteria vary in severity in both healthy and immunocompromised hosts and include bacteremia, pneumonia, otitis media, upper respiratory tract infections, and meningitis. Prophylactic antibiotics and immunization are critical aspects of preventive care in patients with sickle cell disease. However, a high index of suspicion is warranted, despite preventive measures, because these invasive bacterial infections can be rapidly progressive with significant morbidity and mortality. Bacterial meningitis is a life-threatening infection that often presents with a relatively rapid onset of signs and symptoms. The condition is clinically characterized by fever, lethargy, and nuchal rigidity progressing to seizures, coma, and signs and symptoms of elevated intracranial pressure. Prompt recognition and initiation of treatment is crucial because the long-term neurological sequelae can be significant. In children suspected of having bacterial meningitis, a rapid assessment focused on stabilization and airway protection should be undertaken, especially when altered mental status is present (Item C191A). At the time of initial presentation, urgent head imaging is warranted if papilledema, focal neurological deficits, immune deficiency, or a preexisting structural central nervous system condition is present, but imaging can otherwise be deferred. Lumbar puncture should be performed unless there is evidence of increased intracranial pressure, coagulopathy, or hemodynamic instability. Empiric antibiotic therapy should be initiated as quickly as possible, even prior to diagnostic testing if there will be any anticipated delay. Dexamethasone has been used as adjunct therapy to diminish the inflammatory response to the infection and reduce potential long-term sequelae of hearing loss. Although its use is controversial in the pediatric population, the American Academy of Pediatrics Committee on Infectious Disease has noted a potential benefit of dexamethasone in reducing hearing loss in Hib meningitis. If administered, steroids should be given with the initial dose of antibiotics because no benefit has been seen if given more than 1 hour after antibiotics. Caution is recommended in using corticosteroids in patients with sickle cell disease.

American academy of pediatrics

685

PREP ® Self-Assessment

American academy of pediatrics

PREPSA 2020

686

PREP ® Self-Assessment

PREPSA 2020

Cerebrospinal fluid (CSF) analysis is a key diagnostic study that provides critical information on the likely source of infection as well as potential isolation of the pathogen (Item C191B). In bacterial meningitis, the CSF will demonstrate a neutrophilic pleocytosis with an elevated protein level and low glucose level. Antibiotic pretreatment reduces the likelihood of isolating a pathogen on culture, however it will not affect the CSF cell counts or identification of the organism by molecular methods and is unlikely to obscure the diagnosis.

Item C191B: Usual Cerebrospinal Fluid Findings in Healthy Children and Those With Meningitis Caused by Selected Pathogen. Abbreviations: N, normal; NA, not applicable. aValues should not be used in isolation because there can be significant overlap in each of the categories. bGram, silver, or acid-fast bacilli staining for bacteria, fungi, and mycobacteria, respectively. cNeutrophil predominance can be seen in the early stages of meningitis. dPresence of xanthochromia can be seen in herpes simplex virus meningoencephalitis. Adapted and reprinted with permission from Swanson D. Meningitis. Pediatr Rev. 2015;36(12):519. Adapted and reprinted with permission from Swanson D. Meningitis. Pediatr Rev. 2015;36(12):519.

Haemophilus influenzae is an encapsulated organism that is a common cause of bacterial meningitis in children and can have a similar presentation to S pneumoniae, making it difficult to differentiate solely on a clinical basis. While the patient in the vignette is at risk for both, S pneumoniae is the more likely bacterial pathogen to cause childhood meningitis since the introduction of Hib vaccination. Herpes simplex virus can result in a devastating meningoencephalitis with a predilection for the temporal lobes, which often presents with rapid development of headache, lethargy, refractory seizures, and personality changes. Magnetic resonance imaging can demonstrate flair hyperintensities in the bilateral temporal lobes, with electroencephalography showing periodic lateralizing epileptiform discharges. Cerebrospinal fluid analysis demonstrates a lymphocytic pleocytosis with xanthochromia and identification of the virus by polymerase chain reaction testing. Acyclovir is the antiviral agent of choice. Children with sickle cell disease remain at risk for Salmonella, which typically causes gastroenteritis and bacteremia but in severe cases can result in meningitis and other invasive bacterial infections. Invasive disease is more commonly seen in infants younger than 3 months. American academy of pediatrics

687

PREP ® Self-Assessment

PREPSA 2020

In addition to early empiric broad-spectrum antibiotic treatment, management of bacterial meningitis is supportive and is frequently provided in the intensive care unit setting. Children should be closely monitored for decline in neurological status, the development of seizures, syndrome of inappropriate secretion of antidiuretic hormone, Cushing’s triad, and cerebral infarction. Up to one-third of children with bacterial meningitis develop subdural effusions that do not require surgical intervention unless they develop into subdural empyema. Prognosis varies with pathogen and certain clinical features. Poorer prognosis is suggested by a younger age, large bacterial burden, decreased level of consciousness at presentation, and infection with S pneumoniae. Mortality rates range from 5% to 15%. Long-term neurological sequelae in survivors can include hearing impairment (in particular with S pneumoniae and Hib), vestibular injury, developmental delay, epilepsy, hydrocephalus, hemi- or quadriparesis, hypothalamic dysfunction, or cortical blindness. Recovery occurs over weeks to months and requires supportive rehabilitative and educational services as well as close long-term follow up. PREP Pearls • Children with sickle cell disease are predisposed to invasive bacterial infections from encapsulated organisms, such as Streptococcus pneumoniae, that can be rapidly progressive if unrecognized. • Bacterial meningitis is a life-threatening, rapidly progressive, neurological emergency that presents with fever, lethargy, nuchal rigidity, and headache. Even with early treatment, it carries high morbidity and mortality. ABP Content Specifications(s) • Plan the appropriate diagnostic evaluation of meningitis of various etiologies • Recognize the clinical findings associated with meningitis and manage appropriately • Understand the etiology of meningitis in patients of various ages Suggested Readings • Curtis S, Stobart K, Vandermeer B, Simel DL, Klassen T. Clinical features suggestive of meningitis in children: a systematic review of prospective data. Pediatrics. 2010;126(5):952-960. doi:10.1542/peds.2010-0277. • Maraqa N. Pneumococcal infections. Pediatr Rev. 2014;35(7):299-310. doi:10.1542/pir.35-7-299. • Swanson D. Meningitis. Pediatr Rev. 2015;36(12):514-524. doi:10.1542/pir.36-12-514.

American academy of pediatrics

688

PREP ® Self-Assessment

PREPSA 2020

Question 192 A 26-hour-old male neonate, born at 36 weeks of gestation to a 24-year-old gravida 1, para 0 woman, is being evaluated. The mother has a history of polycystic ovarian syndrome and breast reduction surgery. He was delivered via cesarean section because of breech presentation and had a birthweight of 2.9 kg. He has been both breast and bottle feeding. He has had 3 wet diapers since birth, but has not passed stool. His weight today is 2.8 kg. Physical examination reveals a comfortable neonate with no dysmorphic features, no murmur, and a soft abdomen with mild distention. An abdominal radiograph (Item Q192A) and a barium enema test (Item Q192B) are performed.

Item Q192A: Abdominal radiograph for the neonate described in the vignette. Courtesy of M. Liszewski

American academy of pediatrics

689

PREP ® Self-Assessment

PREPSA 2020

Of the following, the neonate’s MOST likely diagnosis is A. anal stenosis B. Hirschsprung disease C. inadequate oral intake D. small left colon syndrome

Incorrect American academy of pediatrics

690

PREP ® Self-Assessment

PREPSA 2020

Correct Answer: B The neonate in the vignette failed to pass meconium in the first 24 hours after birth. No abdominal distention is noted. Abdominal radiograph (Item C192A) reveals mild intestinal dilation. Barium enema (Item C192B) shows a segment of narrowed bowel consistent with Hirschsprung disease.

Item C192A: Abdominal radiograph reveals mild intestinal dilation. Courtesy of M. Liszewski

American academy of pediatrics

691

PREP ® Self-Assessment

PREPSA 2020

Hirschsprung disease affects approximately 1 in 5,000 live births and is more common among boys. Hirschsprung disease may occur in isolation but may also be associated with syndromes such as trisomy 21, Joubert syndrome, Goldberg Shprintzen syndrome, and Smith-Lemli-Opitz syndrome. It occurs because of an arrest in migration of ganglionic cells to the rectosigmoid region. Without adequate innervation, the colonic muscles do not relax properly. Affected neonates typically exhibit failure to pass meconium with or without abdominal distention. Abdominal radiography may show mild bowel dilation. Typically, digital rectal examination will result in a large foul-smelling bowel movement. Toxic megacolon, a rare complication of Hirschsprung disease, is caused by increased luminal pressure, which compromises perfusion of the bowel wall causing bacterial translocation. On physical examination, affected neonates have abdominal tenderness, distention, and erythema. A barium enema is not diagnostic for Hirschsprung disease, but will show evidence of a transition zone of aganglionic cells in the sigmoid rectum. The diagnosis must be confirmed with suction rectal biopsy. Surgical repair is typically performed within the first week after birth and generally results in fecal continence. Children with surgically repaired Hirschsprung disease may develop constipation, fecal soiling, prolapse, and stricture. Neonates with anal stenosis will have severe abdominal distention and require emergent surgical treatment. Small left colon syndrome has been associated with maternal diabetes; affected neonates have feeding intolerance and may have bilious emesis. A barium enema will reveal a American academy of pediatrics

692

PREP ® Self-Assessment

PREPSA 2020

left colon of reduced caliber. Because this neonate had 3 wet diapers in the first 24 hours, inadequate oral intake is not the correct response. PREP Pearls • Hirschsprung disease is caused by failed migration of ganglion cells to the rectosigmoid region of the colon. • Toxic megacolon, a rare complication of Hirschsprung disease, is characterized by increased luminal pressure, which compromises bowel wall perfusion, causing bacterial translocation; physical examination findings include abdominal distention, tenderness, and erythema. • Most neonates achieve fecal continence after surgical repair of Hirschsprung disease. ABP Content Specifications(s) • Recognize disorders associated with delayed or absent passage of meconium • Plan the appropriate evaluation of delayed urination in a newborn infant Suggested Readings • Maqbool A, Fiorino KN, Liacouras CA. Motility disorders and Hirschsprung disease. In: Kliegman RM, St Geme JW III, Blum NJ, Shah SS, Tasker RC, Wilson KM, eds. Nelson Textbook of Pediatrics. 21st ed. Philadelphia, PA: Elsevier; 2020:1955-1965. • Warren JB, Phillipi CA. Care of the well newborn. Pediatr Rev. 2012;33:4. doi:10.1542/pir.33-1-4.

American academy of pediatrics

693

PREP ® Self-Assessment

PREPSA 2020

Question 193 A 17-year-old soccer player who was diagnosed with a concussion 3 weeks ago is evaluated for clearance to return to sport. She has had full resolution of symptoms and remains asymptomatic with walking and light strength training. She has returned to her previous level of academic achievement in school. This is her third concussion. Her first concussion was at 14 years of age, and her second concussion was 6 months ago. She recovered uneventfully from both of these injuries and is otherwise healthy. She is eager to return to practice and would like to be cleared for sports participation, but her mother is concerned about the risk of another concussion. Her neurologic examination findings are unremarkable. Of the following, the MOST appropriate advice for this patient is that she A. B. C. D.

has met criteria for full recovery and can be cleared for unrestricted sport participation needs computerized neuropsychological testing to confirm full cognitive recovery needs to perform a trial of high-intensity exercise and report any return of symptoms should retire from contact sports based on current recommendations

American academy of pediatrics

694

PREP ® Self-Assessment

PREPSA 2020

Correct Answer: C Medical clearance for return to contact sports requires: full symptom resolution, return to prior levels of academic performance, and no symptom recurrence with physical activity. The patient in this vignette needs to remain asymptomatic during a trial of high-intensity activity as the next step back to full participation. Item C193 outlines current recommendations for progressive return to physical activity and sport after concussive injury.

Item C193: Return to Sports After Concussiona. a

Recommend 48 h of relative physical and cognitive rest before beginning the program. No more than 1 step should be completed per day. If any symptoms worsen during exercise, the athlete should return to the previous step. Consider prolonging and/or altering the return-to-sport program for any pediatric and/or adolescent patient with symptoms over 4 weeks. Reprinted with permission from Halstead ME, Walter KD, Moffatt K; Council on Sports Medicine and Fitness. Sport-related concussion in children and adolescents. Pediatrics. 2018;142(6):e20183074.

American academy of pediatrics

695

PREP ® Self-Assessment

PREPSA 2020

Concussion management has evolved significantly over the past decade, and evidence currently suggests that appropriate levels of physical activity seem to facilitate symptom improvement. Immediately after concussion, several days of relative rest are reasonable, but prior recommendations for longer periods of physical inactivity are no longer considered appropriate. Patients should be encouraged to return to activities of daily living as soon as tolerated. A progression of subsymptomatic levels of noncontact activity can then be implemented, starting with brisk walking. This activity can be self-regulated, and quantitative titration of exercise intensity is readily accomplished with any portable device that monitors heart rate. Sample patient instructions may include: 1. Start with low-intensity cardiovascular exercise (stationary bicycling, walking, or easy jogging). 2. Gradually increase activity, and take note of the heart rate at the point of symptom onset or intensification. 3. Calculate 85% to 90% of this heart rate to determine a ceiling heart rate for physical activity. Noncontact physical exertion can then be performed up to this level of intensity. 4. Recalculate the ceiling heart rate every 4 to 7 days until maximum intensity is tolerated. 5. Some athletes may prefer implementation of a more structured program by a certified athletic trainer or physical therapist with experience in sport-related concussion. Athletes with vestibular or oculomotor symptoms after concussion may find even very low levels of activity problematic, and in these cases dedicated rehabilitation may be beneficial. Once the patient is asymptomatic with activity and back to preinjury academic performance, results of computerized neuropsychological testing (CNT) may provide additional information regarding return-to-play decision-making, particularly if there is a preinjury baseline for comparison. However, CNT results are not to be used as a sole determinant for return to sport. The presence of symptoms or ongoing cognitive dysfunction supersedes CNT results when making a decision regarding return to sport. The long-term cognitive effects of concussion in youth sport is an area of active research and some controversy. The current state of knowledge is outlined in the American Academy of Pediatrics clinical report on concussion published in 2018 (http://pediatrics.aappublications.org/content/142/6/e20183074#ref-210). Despite evolving information on the effects of concussions, families should be advised of the following: • Fewer concussions are always better. • Previous concussion is the biggest risk factor for future concussion. • As much time as possible should be put between the last concussion and the next one. With repeat concussive injury, some athletes may reconsider their participation in contact sports. There is a widespread belief that there is a “3 strikes and out” approach in regards to concussion and retirement from contact sports. However, there are no evidence-based guidelines to help families and providers with this decision. Several factors may indicate an athlete who is at higher risk for consequences of repeat injury and may prompt providers to raise the possibility of temporary or permanent cessation of participation in contact sports: • Pattern of repeat concussion with prolonged periods of recovery American academy of pediatrics

696

PREP ® Self-Assessment • •

PREPSA 2020

Progressively lowered injury thresholds with repeat concussive injury (ie, clinically significant injury occurs with relatively trivial mechanism) Children and adolescents with attention-deficit/hyperactivity disorder or learning disabilities

Expert opinion suggests that athletes with repeat concussion in a single competitive season may benefit from discontinuing participation for the remainder of the season. Permanent retirement from contact sports is a complex decision, and consultation with a provider with experience in sport-related concussion (typically a sports medicine specialist or pediatric neurologist) is recommended. Anytime athletes are considering temporary or permanent withdrawal from contact sport, the physical and mental benefits of maintaining involvement in noncontact sport and physical activity should be emphasized. PREP Pearls • Return to sports after concussion requires resolution of symptoms at rest and with exertion, as well as return to baseline level of cognitive achievement. • Recommendations regarding retirement from contact sports after repeat concussion should be made on an individualized basis. ABP Content Specifications(s) • Understand the criteria for return to play in sports after a head injury Suggested Readings • Halstead ME, Walter KD, Moffatt K; Council on Sports Medicine and Fitness. Sportrelated concussion in children and adolescents. Pediatrics. 2018;142(6):e20183074. doi:10.1542/peds.2018-3074. • Kannikeswaran N, Suresh S. Sports musculoskeletal injuries. In: McInerny TK, Adam HM, Campbell DE, DeWitt TG, Foy JM, Kamat DM, eds. American Academy of Pediatrics Textbook of Pediatric Care. 2nd ed. Elk Grove Village, IL: American Academy of Pediatrics; 2017:2675-2686. Pediatric Care Online. • McCrory P, Meeuwisse W, Dvorak J, et al. Consensus statement on concussion in sport—the 5th International Conference on Concussion in Sport held in Berlin, October 2016. Br J Sports Med. 2017;51(11):838-847. doi:10.1136/bjsports-2017-097699.

American academy of pediatrics

697

PREP ® Self-Assessment

PREPSA 2020

Question 194 An 8-month-old girl is being evaluated for a droopy left eyelid. In addition, her left pupil has become noticeably smaller than the right. Her parents are unsure of when these symptoms began. She has no history of fever, poor feeding, or recent illness. She was born via an uncomplicated vaginal delivery at term and has been otherwise healthy. The girl has been growing and developing normally. Physical examination findings are significant for left ptosis and pupillary anisocoria, with left miosis and preservation of response to light. The remainder of her physical examination findings are normal. Of the following, the MOST likely laboratory abnormality(ies) in this infant is(are) A. serum anti-acetylcholine receptor antibody B. serum lactate and pyruvate levels C. stool Clostridium botulinum toxin D. urinary homovanillic and vanillylmandelic acid levels

American academy of pediatrics

698

PREP ® Self-Assessment

PREPSA 2020

Correct Answer: D The infant in the vignette has acquired left ptosis in the setting of Horner syndrome (unilateral ptosis, miosis, and anhidrosis) because of neuroblastoma. The tumor arises from primitive sympathetic ganglion cells that have the capacity to synthesize and secrete catecholamines. Neuroblastoma may be diagnosed by measuring urinary homovanillic (HVA) and vanillylmandelic acid (VMA) levels, which are catecholamine metabolites secreted by neuroblastoma cells; these are elevated in 90% to 95% of affected children. Neuroblastomas that involve the cervical paravertebral sympathetic chain and inferior cervical (stellate) ganglion may result in Horner syndrome, though anhidrosis may be absent or unnoticeable. Horner syndrome is caused by a disruption in the 3-neuron oculosympathetic chain. Young children with isolated Horner syndrome should undergo careful examination for cervical and abdominal masses, measurement of urinary HVA and VMA levels, and radiologic imaging of the head, neck, and chest. Ptosis refers to drooping of the upper eyelid that usually results from a congenital or acquired abnormality of the muscles or nerves that elevate the eyelid. The facial or seventh cranial nerve innervates the circumferential orbicularis oculi muscle to close the upper and lower eyelids, while the oculomotor or third cranial nerve innervates the levator palpebrae superioris to elevate the upper eyelid. In addition to Horner syndrome, causes of ptosis include congenital ptosis, third nerve palsy, neuromuscular junction disorders (eg, myasthenia gravis and botulism), mitochondrial myopathy, and mechanical causes (Item C194).

American academy of pediatrics

699

PREP ® Self-Assessment

PREPSA 2020

Item C194: Features of Congenital and Acquired Causes of Ptosis. Courtesy of K. Montez PREP Pearls • Ptosis of the eyelid results from a congenital or acquired abnormality of the muscles or nerves that elevate the eyelid. • Causes of ptosis include congenital ptosis, Horner syndrome, third nerve palsy, neuromuscular junction disorders, mitochondrial myopathy, and mechanical causes. • Young children with isolated Horner syndrome should undergo careful evaluation for cervical and abdominal masses, and measurement of urinary homovanillic and vanillylmandelic acid levels to assess for the possibility of neuroblastoma. ABP Content Specifications(s) • Differentiate the clinical findings associated with congenital ptosis from those of acquired ptosis American academy of pediatrics

700

PREP ® Self-Assessment

PREPSA 2020

Suggested Readings • Guercio JR, Martyn LJ. Congenital malformations of the eye and orbit. Otolaryngol Clin North Am. 2007;40(1):113-140. • Pollard ZF, Greenberg MF, Bordenca M, Lange J. Atypical acquired pediatric Horner syndrome. Arch Ophthalmol. 2010;128(7):937-940. doi:10.1001/archophthalmol.2010.119. • Sharma R, Mer J, Lion A, Vik TA. Clinical presentation, evaluation, and management of neuroblastoma. Pediatr Rev. 2018;39(4):194-203. doi: 10.1542/pir.2017-0087.

American academy of pediatrics

701

PREP ® Self-Assessment

PREPSA 2020

Question 195 A 2-year-old girl is brought to the emergency department with a 2-day history of fever, irritability, malaise, and emesis. Five days before presentation, she completed a 10-day course of amoxicillin for acute otitis media. On physical examination, she is ill-appearing, with a temperature of 39.4°C, heart rate of 136 beats/min, respiratory rate of 34 breaths/min, and blood pressure of 104/54 mm Hg. There is redness and swelling above the right ear, with downward and outward displacement of the pinna. Her right tympanic membrane is bulging, immobile, and opaque. The remainder of her physical examination findings are normal. Laboratory findings are notable for leukocytosis and an elevated C-reactive protein. Tympanocentesis is performed and specimens are sent for culture. Of the following, the MOST likely pathogen to be isolated in this girl is A. Pseudomonas aeruginosa B. Staphylococcus aureus C. Streptococcus pneumoniae D. Streptococcus pyogenes

American academy of pediatrics

702

PREP ® Self-Assessment

PREPSA 2020

Correct Answer: C The child described in the vignette has a clinical picture suggestive of acute mastoiditis following a recent episode of acute otitis media (AOM). Acute mastoiditis is an unusual complication of otitis media in the current antibiotic era, with an incidence of 1.2 to 4.2 cases per 100,000 children each year. Mastoiditis can result in serious intracranial and extracranial complications. Mastoiditis can be acute (3 weeks’ duration of chronic suppurative otitis media). Similar to the microbiology of AOM, the most common etiologic agent of acute mastoiditis is Streptococcus pneumoniae, even in the era of the pneumococcal conjugate vaccine (PCV). Other bacterial causes include Staphylococcus aureus (including methicillin-resistant S aureus), group A Streptococcus, nontypeable Haemophilus influenzae, Moraxella catarrhalis, and Streptococcus anginosus. In chronic mastoiditis, infection is often polymicrobial; common bacterial pathogens include S aureus, anaerobes (eg, Fusobacterium necrophorum, Peptostreptococcus, Prevotella, and Bacteroides), Pseudomonas aeruginosa, and Enterobacteriaceae. The incidence and frequency of AOM caused by S pneumoniae has declined since the introduction of pneumococcal conjugate vaccines PCV-7 and PCV-13. Acute mastoiditis occurs most commonly in children younger than 2 years. The middle ear cavity is connected to the mastoid air cells via the aditus ad antrum. After AOM, there is inflammation of the middle ear and mastoid mucosa which typically resolves with antibiotic therapy. In rare instances, obstruction of the aditus ad antrum may result in accumulation of serous and then purulent fluid in the mastoid cavity, resulting in periostitis of the bony septa. Destruction of the thin trabecular system leads to coalescent mastoiditis and formation of a subperiosteal abscess. Infection can extend beyond the mastoid cavity and cause various intracranial (eg, lateral or sigmoid sinus thrombosis, meningitis, epidural, subdural, or brain abscess) and extracranial complications (eg, Bezold abscess along sternocleidomastoid muscle sheath, Bell palsy, labyrinthitis, and occipital osteomyelitis). The clinical manifestations of acute mastoiditis include evidence of AOM on otoscopy with associated redness, edema, and tenderness over the mastoid area. Swelling over the mastoid may result in ear proptosis (displacement of the ear down and out in infants or up and out in older children). The diagnosis of acute mastoiditis is clinical. Imaging is not indicated in uncomplicated cases. Computed tomography (CT) of the temporal bone shows destruction of the bony septae between the mastoid air cells, sometimes associated with soft tissue abscess. Radiographic findings of haziness of the mastoid bone without evidence of bony destruction may be noted in AOM and does not represent acute mastoiditis in the absence of clinical findings. Contrast-enhanced CT scan or magnetic resonance imaging of the brain may be indicated in suspected cases of complicated mastoiditis such as intracranial abscess or sinus venous thrombosis. Laboratory data may show leukocytosis and elevated C-reactive protein level. Blood cultures may be positive in 3% to 6% of cases. Treatment of mastoiditis includes systemic antibiotics and, in many cases, surgical drainage. An otolaryngology consultation should be obtained. Conservative management may be appropriate in uncomplicated cases of acute mastoiditis, especially in young children. It is prudent to obtain a American academy of pediatrics

703

PREP ® Self-Assessment

PREPSA 2020

middle ear fluid specimen by tympanocentesis for culture and antimicrobial sensitivities. Intravenous antimicrobial agents should be targeted against pathogens causing AOM, especially S pneumoniae and S aureus (including methicillin-resistant S aureus), for example, vancomycin plus ampicillin-sulbactam. In the setting of recurrent AOM and recent antibiotic use, some experts prefer a combination of vancomycin plus ceftazidime or cefepime (to cover P aeruginosa). If complicated disease, including intracranial complications, is suspected, empiric treatment with vancomycin and meropenem, or vancomycin plus ceftriaxone and metronidazole, is recommended. A 4- to 7-day course of parenteral antibiotic therapy followed by an oral antibiotic regimen for a total of 14 to 21 days is recommended for uncomplicated cases of acute mastoiditis. Longer durations of intravenous antibiotic therapy (3-6 weeks) may be necessary for intracranial complications of mastoiditis (such as venous thrombosis or brain abscess). Medical management alone is successful in 60% to 87% of cases. In many institutions, empiric antimicrobial therapy with or without myringotomy and placement of a tympanostomy tube to facilitate continued drainage and relieve pain and pressure is often the initial management strategy. If there is no clinical improvement after 48 hours of intravenous antibiotic therapy, a contrast-enhanced CT scan of the temporal bone is recommended to evaluate for suppurative complications. Subperiosteal abscess may be drained by simple mastoidectomy procedure. Retroauricular incision and drainage of subperiosteal abscess under local anaesthesia is associated with a high success rate (92%). Radical mastoidectomy may be indicated in cases of mastoiditis complicated by suppurative intracranial complications. PREP Pearls • Acute mastoiditis is an unusual complication of otitis media in children. • The most common etiologic agent of acute mastoiditis is Streptococcus pneumoniae followed by other bacteria: Staphylococcus aureus, group A Streptococcus, nontypeable Haemophilus influenzae, Moraxella catarrhalis, and Streptococcus anginosus. • Conservative (medical) management may be appropriate in uncomplicated cases of acute mastoiditis, especially in young children. ABP Content Specifications(s) • Plan the appropriate diagnostic evaluation of mastoiditis • Plan the appropriate management of mastoiditis • Recognize the clinical findings associated with mastoiditis • Identify the etiology of mastoiditis Suggested Readings • Anne S, Schwartz S, Ishman SL, Cohen M, Hopkins B. Medical versus surgical treatment of pediatric acute mastoiditis: a systematic review. Laryngoscope. 2019;129(3):754-760. doi:10.1002/lary.27462. • Chesney J, Black A, Choo D. What is the best practice for acute mastoiditis in children? Laryngoscope. 2014;124(5):1057-1058. doi:10.1002/lary.24306. American academy of pediatrics

704

PREP ® Self-Assessment •

• •

PREPSA 2020

Koutouzis E, Michos A, Koutouzi F, et al. Pneumococcal mastoiditis in children before and after the introduction of conjugate pneumococcal vaccines. Pediatr Infect Dis J. 2016;35(3):292-296. doi:10.1097/INF.0000000000000995. Kynion R. Mastoiditis. Pediatr Rev. 2018;39(5):267-269. doi:10.1542/pir.2017-0128. Tharpe AM, Sladen DP, Rothpletz A. Hearing loss. In: McInerny TK, Adam HM, Campbell DE, DeWitt TG, Foy JM, Kamat DM, eds. American Academy of Pediatrics Textbook of Pediatric Care. 2nd ed. Elk Grove Village, IL: American Academy of Pediatrics; 2017:1408-1412. Pediatric Care Online.

American academy of pediatrics

705

PREP ® Self-Assessment

PREPSA 2020

Question 196 A 12-year-old boy with a 2-month history of headache is seen for evaluation. His pain is episodic, located in the back of the head, moderate in intensity, and associated with flushing. He has no blurry vision, dizziness, or vomiting. He has a heart rate of 104 beats/min, respiratory rate of 16 breaths/min, and blood pressure of 146/92 mm Hg. His palms are sweaty, and he has multiple hyperpigmented macules (Item Q196) on his chest and back. His abdominal examination has normal findings, and no bruit is heard. The rest of the examination findings are unremarkable.

Item Q196: Hyperpigmented macules on the adolescent boy described in the vignette. Reprinted with permission from Krowchuk D. Rash. In: McInerny TK, Adam HM, Campbell DE, DeWitt TG, Foy JM, Kamat DM, eds. American Academy of Pediatrics Textbook of Pediatric Care. 2nd ed. Elk Grove Village, IL: American Academy of Pediatrics; 2017:1547.

Of the following, the MOST likely diagnosis for this child is A. anxiety disorder B. hyperthyroidism C. pheochromocytoma D. renal artery stenosis

American academy of pediatrics

706

PREP ® Self-Assessment

PREPSA 2020

Correct Answer: C The patient in this vignette has multiple café-au-lait spots, favoring a diagnosis of neurofibromatosis type 1 (NF-1). The associated headaches, flushing, sweating, tachycardia, and hypertension suggest a high level of catecholamine production. Together, these signs and symptoms make pheochromocytoma the most likely diagnosis. Pheochromocytomas are catecholamine-secreting tumors that develop from the chromaffin cells of the adrenal medulla and are known to occur in patients with NF-1. The characteristic symptoms and signs of these tumors are caused by overproduction of norepinephrine, epinephrine, dopamine, and their metabolites. The classical triad of symptoms is episodic headaches, sweating, and tachycardia. Hypertension is also a common feature. In children, unlike adults, elevated blood pressure is more often sustained than paroxysmal. Other uncommon presenting features include pallor, constipation, panic attacks, blurry vision, weight loss, polyuria, and polydipsia. Pheochromocytoma is sometimes asymptomatic and detected as an incidental adrenal mass on imaging studies. The differential diagnosis of pheochromocytoma includes other conditions with increased sympathetic activity, such as panic disorder and attention-deficit/hyperactivity disorder. Sympathomimetic drugs (eg, phenylephrine, phenylpropanolamine, amphetamines, cocaine) and mercury intoxication can have symptoms similar to pheochromocytoma. Children with anxiety disorder and hyperthyroidism can have features of headaches, sweating, tachycardia, and sometimes elevated blood pressures. However, multiple café-au-lait macules in the patient in this vignette strongly suggest a diagnosis of NF-1, which is associated with renal artery stenosis as well as pheochromocytoma. The absence of bruit on examination and presence of symptoms of catecholamine excess make a clinical diagnosis of renal artery stenosis less likely in this patient. Children, in comparison to adults with pheochromocytoma, are at increased risk of having bilateral presentation, extra-adrenal location (paraganglioma), recurrent tumors, and familial disease. Von Hippel-Lindau syndrome, multiple endocrine neoplasia type 2, NF-1, and other germline mutations (succinate dehydrogenase complex) are commonly associated with pheochromocytoma. All children diagnosed with pheochromocytoma should undergo genetic testing because of the association with these familial syndromes. PREP Pearls • Episodic headache, sweating, tachycardia, and hypertension are common presenting features in a child with pheochromocytoma caused by excessive catecholamine secretion. • Pheochromocytoma is commonly associated with familial syndromes such as von HippelLindau syndrome, multiple endocrine neoplasia type 2, and neurofibromatosis type 1. MOCA-Peds Objective • Recognize and evaluate neurocutaneous disorders. ABP Content Specifications(s) • Recognize disorders commonly associated with pheochromocytoma • Recognize the clinical findings associated with pheochromocytoma

American academy of pediatrics

707

PREP ® Self-Assessment

PREPSA 2020

Suggested Readings • Bholah R, Bunchman TE. Review of pediatric pheochromocytoma and paraganglioma. Front Pediatr. 2017;5:155. doi:10.3389/fped.2017.00155. • Jain A, Baracco R, Kapur G. Pheochromocytoma and paraganglioma—an update on diagnosis, evaluation, and management. Pediatr Nephrol. 2019. doi:10.1007/s00467-0184181-2. • Speiser PW. Adrenal dysfunction. In: McInerny TK, Adam HM, Campbell DE, DeWitt TG, Foy JM, Kamat DM, eds. American Academy of Pediatrics Textbook of Pediatric Care. 2nd ed. Elk Grove Village, IL: American Academy of Pediatrics; 2017:16921701. Pediatric Care Online .

American academy of pediatrics

708

PREP ® Self-Assessment

PREPSA 2020

Question 197 A 4-month-old male infant is referred to the emergency department by his pediatrician for weakness in his legs. He was born full term via vaginal delivery following an uncomplicated pregnancy. He was well with normal feeding, growth, and development until 3 weeks ago. At that time, his mother noticed that he would not push his feet against her hands as hard as he used to. A video from the mother taken before that time shows the infant jumping in a bouncer, using both legs to push off. The day after the video was taken, he received his 4-month vaccines in both legs, and since that time he stopped planting his feet on the ground when held up, and he has had decreasing spontaneous movement of his lower extremities. His upper body movements and strength have been normal. He has had no fever, weight loss, irritability, sick contacts, or exposure to honey. He has a temperature of 37.2°C, heart rate of 127 beats/min, blood pressure of 98/50 mm Hg, respiratory rate of 44 breaths/min, and oxygen saturation of 99% on room air. He appears happy and playful, reaching for and pulling his mother’s hair. His physical examination is remarkable for flaccid paralysis of both lower extremities, with clonus in both feet. He has normal strength and activity in his arms, hands, and head. Other than the lower extremity neurologic examination findings, there are no other abnormal findings. Of the following, the MOST appropriate next step is to A. administer the botulism antitoxin immediately B. admit to the neurology service for observation C. consult a neurologist to perform electromyography D. perform emergent magnetic resonance imaging of the spine

American academy of pediatrics

709

PREP ® Self-Assessment

PREPSA 2020

Correct Answer: D The rapid recognition, diagnosis, and treatment of spinal cord compression is essential to provide the best opportunity for recovery of neurologic function. The infant in this vignette has flaccid paralysis of both legs with clonus but no other abnormal physical findings. His mother has provided video evidence of previously normal lower extremity function, suggesting that the paralysis is acquired and of relatively recent onset. The bilateral nature of the neurologic deficit strongly suggests a lesion of the spinal cord, likely in the thoracic cord. The differential diagnosis includes diseases intrinsic to the spinal cord, such as transverse myelitis, as well as external compression. While the treatment of transverse myelitis is urgent, decompression of the spinal cord when impinged by an external lesion is emergent. Every minute that passes with the cord compressed decreases the likelihood of achieving a neurologic recovery. When spinal cord compression is suspected, it is imperative to obtain emergent magnetic resonance imaging so that action can be taken to decompress the cord. Thus, the most appropriate next step for the infant in the vignette is emergent magnetic resonance imaging of the spine. In an otherwise well infant with new-onset bilateral lower extremity weakness, a neuroblastoma should be suspected. Neuroblastoma is the most common extracranial solid tumor in children. It is a tumor of the sympatico-adrenal axis and can arise from the paraspinal ganglia. It can invade through the spinal neural foramina and create an epidural mass causing spinal cord compression (Item C197). While decompression of the spinal cord can be accomplished through surgery, paraspinal neuroblastomas in infants often span multiple spinal levels and would require a multilevel laminectomy to decompress. As neuroblastoma typically responds very quickly to chemotherapy, it would be appropriate to emergently treat a child with this type of tumor with emergent chemotherapy.

Item C197: Spinal cord compression from neuroblastoma. Courtesy of J. Fish

Botulism toxin exposure leads to a systemic impairment of neurologic function and would not present as isolated flaccid paralysis of the lower extremities in an otherwise well infant. The classic triad of botulinum toxin includes bulbar palsy, a descending (not ascending) paralysis, and clear sensorium. Admission of the infant to the hospital is appropriate, however observation alone without imaging would not be appropriate because emergent action may be needed to decompress the spinal cord. Electromyography would not be appropriate because the first step in the infant’s care would be to rule out spinal cord compression. American academy of pediatrics

710

PREP ® Self-Assessment

PREPSA 2020

PREP Pearls • Flaccid, bilateral paralysis of the lower extremities should raise immediate concern for spinal cord compression. • In cases of suspected spinal cord compression, emergent magnetic resonance imaging of the spinal level of concern is the most appropriate first step in management. • The classic triad of botulinum toxin includes bulbar palsy, a descending (not ascending) paralysis, and clear sensorium. ABP Content Specifications(s) • Recognize the clinical findings associated with spinal cord compression (eg, from a tumor, from myelopathy), and the need for prompt evaluation Suggested Readings • Francel PC. Extrinsic spinal cord mass lesions. Pediatr Rev. 1998;19(11):389-394. doi:10.1542/pir.19-11-389. • Henry M, Sung L. Supportive care in pediatric oncology: oncologic emergencies and management of fever and neutropenia. Pediatr Clin North Am. 2015;62(1):27-46. doi:10.1016/j.pcl.2014.09.016. • Khan UA, Shanholtz CB, McCurdy MT. Oncologic mechanical emergencies. Emerg Med Clin North Am. 2014;32(3):495-508. doi:10.1016/j.emc.2014.04.001.

American academy of pediatrics

711

PREP ® Self-Assessment

PREPSA 2020

Question 198 A 9-year-old boy is seen for evaluation of a rash. He was diagnosed with focal epilepsy 2 weeks ago after he was brought to the hospital for a first-time focal seizure. Evaluations at the time included an electroencephalogram that showed focal epileptiform discharges arising from the left frontal region. Magnetic resonance imaging of the brain had normal findings. Oxcarbazepine was initiated with immediate seizure control, and he was discharged home with a titration to a goal starting dose. One week ago, his mother noticed a new pruritic rash over his arms, legs, and torso. He has had temporary relief with antihistamines. No new environmental exposures have been identified. His vital signs are within normal limits. He has a fine maculopapular rash over his torso, arms, and legs with areas of excoriation but no bullae, blistering, or sloughing. His mucosal membranes and posterior pharynx have no lesions and are nonerythematous. He is breathing comfortably, in no distress with good air exchange. Neurological examination has normal findings. His neurologist is consulted, and it is determined that his anticonvulsant should be held with close outpatient monitoring and symptomatic relief with antihistamines. Within 1 week his rash has completely resolved. Of the following, the MOST likely diagnosis for this boy’s rash is A. drug reaction with eosinophilia and systemic symptoms B. exanthematous drug reaction C. Stevens-Johnson syndrome D. toxic epidermal necrolysis

American academy of pediatrics

712

PREP ® Self-Assessment

PREPSA 2020

Correct Answer: B The rash described in this vignette is a hypersensitivity reaction to oxcarbazepine, a common reaction seen with antiepileptic drug (AED) use. The clinical spectrum of hypersensitivity reactions with AED use is broad, ranging from mild rashes to severe, life-threatening cutaneous reactions. Hypersensitivity reactions can include exanthematous drug eruptions, such as described in the vignette, in which a diffuse maculopapular rash develops during the initial 5 to 14 days of starting a medication (Item C198A, Item C198B). Antiepileptic medications that can commonly cause hypersensitivity reactions include phenytoin, carbamazepine, oxcarbazepine, and phenobarbital, although any AED could result in a hypersensitivity reaction. Treatment is supportive with discontinuation of the AED and symptomatic relief with antihistamines.

Item C198A: Maculopapular exanthematous drug eruption Reprinted with permission from Segal A, Doherty K, Leggott J, Zlotoff B. Cutaneous reactions to drugs in children. Pediatrics. 2007;120(4):e1086.

American academy of pediatrics

713

PREP ® Self-Assessment

PREPSA 2020

Item C198B: Morbilliform Exanthematous drug eruption Reprinted with permission from Segal A, Doherty K, Leggott J, Zlotoff B. Cutaneous reactions to drugs in children. Pediatrics. 2007;120(4):e1087.

Antiepileptic medications are a broad category of drugs with varied adverse effect profiles and adverse reactions. The 2 most common adverse drug reactions are type A and type B reactions. Type A reactions are acute, related to the pharmacologic properties of the drug, occur in a predictable, dose-dependent fashion, and are typically mild to moderate in severity. Type B reactions are idiosyncratic and unpredictable, occur unrelated to the pharmacologic properties of the medication, and can be severe. Type A reactions include common adverse effects (eg, nausea, vomiting, fatigue, appetite effects, sleep-related difficulties, cognitive and behavioral changes). When selecting a specific AED, potential adverse effects should be taken into consideration in addition to the seizure type American academy of pediatrics 714

PREP ® Self-Assessment

PREPSA 2020

targeted. Common and severe adverse reactions or side effects should be discussed prior to the initiation of any AED, as well as any potential benefit or dual effect from a specific medication. For example, in a patient with generalized epilepsy and migraine headaches, topiramate may be chosen for dual effect as both an anticonvulsant and migraine preventative therapy. Type B reactions are hypersensitivity reactions with a wide range of clinical presentation and severity. Several genetic risk factors predispose specific populations to development of these idiosyncratic reactions. Patients of Asian descent who have the HLA-B*1502 allele are at risk for serious hypersensitivity reactions, such as Stevens-Johnson syndrome or toxic epidermal necrolysis (TEN), when treated with oxcarbazepine. This risk has prompted the US Food and Drug Administration to revise the initial labeling to recommend HLA testing in at-risk populations prior to initiation of oxcarbazepine, carbamazepine, or phenytoin. Laboratory monitoring varies with AED choice. Hepatic function panels are recommended for AEDs metabolized through the liver, such as valproic acid, oxcarbazepine, and phenytoin. Complete blood cell counts are recommended for AEDs such as valproic acid that cause blood dyscrasias, commonly leukopenia or thrombocytopenia. The levels of AEDs are measured in some situations to aid in dosing, ensure therapeutic level, or evaluate for adherence. Hepatically metabolized drugs can act as either inducers or inhibitors of the cytochrome P450 system, resulting in drug-drug interactions with other hepatically metabolized AEDs, certain antibiotics, and oral contraceptive pills. Drug reaction with eosinophilia and systemic symptoms (DRESS) is a rare, potentially lifethreatening condition characterized by rash, hematologic abnormalities (eosinophilia), lymphadenopathy, and internal organ involvement that develops typically 2 to 6 weeks after starting the causative medication. Antiepileptic drugs, in particular carbamazepine, lamotrigine, phenytoin, and phenobarbital, are the most common cause of DRESS. Treatment of DRESS is supportive with use of systemic corticosteroids or cyclosporine in severe cases with lung and kidney involvement. Stevens-Johnson syndrome is a severe drug reaction that appears up to 3 weeks after starting a medication and is characterized by mucosal lesions at 2 or more sites and widespread targetoid or macular skin lesions preceded by a prodrome of fever, sore throat, and cough (Item C198C). Toxic epidermal necrolysis presents in a similar time frame, with mucosal involvement and a morbilliform rash that rapidly blisters and exfoliates involving more than 30% of the cutaneous surface (Item C198D). A positive Nikolsky sign, where the epidermis detaches with pressure from the finger, is consistent with TEN. Stevens-Johnson syndrome and TEN represent a disease continuum, with more extensive cutaneous involvement seen in TEN. Supportive care is provided emergently in an intensive care or burn unit setting with attention to fluids, pain control, and wound care.

American academy of pediatrics

715

PREP ® Self-Assessment

PREPSA 2020

Item C198C: Stevens-Johnson syndrome. Reprinted with permission from A, Doherty K, Leggott J, Zlotoff B. Cutaneous reactions to drugs in children. Pediatrics. 2007;120(4):e1092

American academy of pediatrics

716

PREP ® Self-Assessment

PREPSA 2020

Item C198D: Toxic epidermal necrolysis. Reprinted with permission from Segal A, Doherty K, Leggott J, Zlotoff B. Cutaneous reactions to drugs in children. Pediatrics. 2007;120(4):e1092.

Potential adverse reactions and side effects of AEDs are broad. Monitoring practices, adverse effects, and drug-drug interactions vary with AED choice. Adverse cutaneous reactions can range from mild to severe and require prompt medical evaluation and discontinuation of the causative medication.

PREP Pearls • The clinical spectrum of hypersensitivity reactions with antiepileptic drug use is broad, ranging from mild rashes, such as exanthematous cutaneous eruptions, to severe, lifethreatening cutaneous reactions, such as Stevens-Johnson syndrome or toxic epidermal necrolysis. • Acute, dose-dependent, predictable reactions related to the pharmacologic properties of a medication include adverse effects, such as nausea, fatigue, appetite effects, and cognitive and behavioral changes, and are unique to each antiepileptic drug. • Laboratory monitoring for patients on antiepileptic drugs typically includes serum antiepileptic drug levels, hepatic function panels for drugs metabolized through the liver, and complete blood cell counts for medications associated with blood dyscrasias.

American academy of pediatrics

717

PREP ® Self-Assessment

PREPSA 2020

ABP Content Specifications(s) • Recognize laboratory abnormalities associated with anticonvulsant drug therapy • Recognize side effects and toxicities associated with anticonvulsant drugs • Plan the appropriate evaluation of serum anticonvulsant drug concentrations, including limitations and timing Suggested Readings • Guvenir H, Dibek Misirlioglu E, Civelek E, et al. The frequency and clinical features of hypersensitivity reactions to antiepileptic drugs in children: a prospective study. J Allergy Clin Immunol Pract. 2018;6(6):2043-2050. doi:10.1016/j.jaip.2018.02.018. • Henderson L, Williams JV. Drug eruptions, erythema multiforme, Stevens-Johnson syndrome. In: McInerny TK, Adam HM, Campbell DE, DeWitt TG, Foy JM, Kamat DM, eds. American Academy of Pediatrics Textbook of Pediatric Care. 2nd ed. Elk Grove Village, IL: American Academy of Pediatrics; 2017:1984-1992. Pediatric Care Online. • Perucca P, Gilliam FG. Adverse effects of antiepileptic drugs. Lancet Neurol. 2012;11(9):792-802. doi:10.1016/S1474-4422(12)70153-9. • Segal A, Doherty K, Leggott J, Zlotoff B. Cutaneous reactions to drugs in children. Pediatrics. 2007;120(4):e1082-e1096. doi:10.1542/peds.2005-2321.

American academy of pediatrics

718

PREP ® Self-Assessment

PREPSA 2020

Question 199 A 15-month-old boy is seen in clinic as a new patient. He is a refugee from the Democratic Republic of Congo who recently immigrated to the United States. Medical records provided by his mother are reviewed. His date of birth is May 6, 2017. He had health supervision visits at 6 weeks, 10 weeks, 14 weeks, and 9 months of age. His immunization record is shown: Vaccine Date BCG 06/05/2017 OPV 20/06/2017, 17/07/2017, 16/08/2017 DTP-Hib-HepB 20/06/2017, 17/07/2017, 16/08/2017 IPV 16/08/2017 Measles 08/02/2018 VAA (vaccine anti-amaril) 08/02/2018 Pneumo 20/06/2017, 17/07/2017, 16/08/2017 He does not have any major medical conditions. He has been treated in the past for otitis media twice, and he developed fever and a measles-like rash after receiving measles vaccine. Of the following, the BEST approach for measles vaccination in this patient is to A. administer measles vaccine at 4 years of age B. assume immunity given reported measles history C. provide measles vaccine at this visit D. verify measles immunity with serology

American academy of pediatrics

719

PREP ® Self-Assessment

PREPSA 2020

Correct Answer: C For the boy in this vignette, the best approach for measles vaccination is to provide measles vaccine at this visit. For refugees, immunizations received abroad are considered valid in the United States if they were administered according to the Advisory Committee on Immunizations Practices (ACIP) schedule. The boy received a measles-containing vaccine at 9 months of age, which is acceptable according to World Health Organization recommendations. However, in the United States any measles vaccination administered prior to 12 months of age does not count towards the 2 total recommended doses. Therefore, he needs a dose of a measles-containing vaccine at this visit. Refugees are not required to have any vaccinations prior to arrival in the United States. As a result of this policy and because access to vaccines prior to arrival can vary, providers may encounter wide variations regarding completion of schedules. Vaccine needs should be evaluated during the first medical evaluation after arrival. Written documentation should be the only means of verifying previous vaccination. Documentation is likely to occur in foreign languages, thus providers should use online resources (https://www.cdc.gov/immigrantrefugeehealth/guidelines/domestic/immunizationsguidelines.html) for language translations of vaccine names. To be valid, dates of administration, intervals between doses, and age of administration should be comparable to that recommended in the ACIP schedule. In many countries dates are listed as Day/Month/Year, as seen on the immunization card in the vignette, as opposed to Month/Day/Year in the United States. Serologic immunity can be pursued if there is a strong suspicion that an individual had disease that would induce immunity or that an individual was appropriately vaccinated but has insufficient written documentation. The immunizations received in the Democratic Republic of Congo by the patient in the vignette reflect the vaccine schedule recommended by the World Health Organization including: BCG at birth; polio, DTP-containing vaccine, HiB, and pneumococcal vaccine starting at 6 weeks of age with an interval of 4 weeks between doses; yellow fever vaccine (in endemic regions) between 9 and 12 months of age; and measles vaccine at 9 or 12 months of age. The next dose of measles vaccine should not be deferred until 4 years of age because he has yet to receive a dose after 12 months of age. A measles-like rash after vaccination does not confirm immunity. Antibody levels could be appropriate if the clinical history strongly suggested an illness due to measles. PREP Pearls • Refugees are not required to have any vaccinations prior to arrival in the United States; thus, vaccine needs should be evaluated during the first medical encounter. • Written documentation should be the only means of verifying previous vaccination. • Serologic immunity can be pursued if there is a strong suspicion that an individual had disease that would induce immunity or that an individual had appropriate vaccination but has insufficient written documentation.

American academy of pediatrics

720

PREP ® Self-Assessment

PREPSA 2020

ABP Content Specifications(s) • Plan an immunization schedule for a child or adolescent who begins receiving immunizations late or whose immunizations are delayed Suggested Readings • American Academy of Pediatrics. Medical evaluation for infectious diseases for internationally adopted, refugee, and immigrant children. In: Kimberlin DW, Brady MT, Jackson MA, Long SS, eds. Red Book: 2018 Report of the Committee on Infectious Diseases. 31st ed. Itasca, IL: American Academy of Pediatrics; 2018:176178. https://redbook.solutions.aap.org/chapter.aspx?sectionId=189640017&bookId=2205 &resultClick=1. • Centers for Disease Control and Prevention. Guidelines for evaluating and updating immunizations during the domestic medical examination for newly arrived refugees. 2015. https://www.cdc.gov/immigrantrefugeehealth/guidelines/domestic/immunizationsguidelines.html. • Shetgiri R, Magge HN, Geltman PL, Flores G. Caring for families new to the United States. In: McInerny TK, Adam HM, Campbell DE, DeWitt TG, Foy JM, Kamat DM, eds. American Academy of Pediatrics Textbook of Pediatric Care. 2nd ed. Elk Grove Village, IL: American Academy of Pediatrics; 2017:575-589. Pediatric Care Online. • Shetty VU, Chaudhury P, Sabella C. Rationale for the immunization schedule: why is it the way it is? Pediatr Rev. 2019;40(1):36-36. doi:10.1542/pir.2018-0033. • World Health Organization. WHO recommendations for routine immunization–summary tables. 2018. https://www.who.int/immunization/policy/immunization_tables/en/.

American academy of pediatrics

721

PREP ® Self-Assessment

PREPSA 2020

Question 200 A 17-year-old adolescent girl with anorexia nervosa is admitted to the hospital for inpatient psychiatric support and placement of nasogastric feeding tube. She has had persistent weight loss, restrictive eating behaviors, and laxative abuse, despite maximizing outpatient therapy. She describes frequent nausea and daily episodes of nonbilious nonbloody emesis. Over the last 6 months, she has lost 11 kg, and her body weight at the time of admission is 38 kg (body mass index, 14.1 kg/m2). She reports that she eats small amounts of food once or twice daily. Her last menstrual period was 6 months ago. Laboratory data are shown: Laboratory Test Results at Admission Sodium 137 mEq/L (137 mmol/L) Potassium 4.1 mEq/L (4.1 mmol/L) Carbon dioxide 22 mEq/L (22 mmol/L) Chloride 99 mEq/L (99 mmol/L) Blood urea nitrogen 14 mg/dL (5.0 mmol/L) Creatinine 0.68 mg/dL (52 µmol/L) Calcium, calculated 9.7 mg/dL (2.43 mmol/L) Albumin 4.4 g/dL (44 g/L) Phosphorus 3.3 mg/dL (1.07 mmol/L) Magnesium 1.7 mEq/L (0.86 mmol/L) Hemoglobin 13.1 mg/dL (131 mg/L) Vitamin D, 25-hydroxy 19.2 ng/mL (48 nmol/L) Ferritin 9 ng/mL (20 pmol/L)

Results After 1 Day of Feeding 138 mEq/L (138 mmol/L) 3.8 mEq/L (3.8 mmol/L) 27 mEq/L (27 mmol/L) 100 mEq/L (100 mmol/L) 7 mg/dL (2.5 mmol/L) 0.57 mg/dL (43 µmol/L) 9.4 mg/dL (2.35 mmol/L) 4.2 g/dL (42 g/L) 1.9 mg/dL (0.61 mmol/L) 1.8 mEq/L (0.90 mmol/L) Not repeated Not repeated Not repeated

Of the following, the intervention MOST indicated in this patient is A. intravenous iron sucrose B. intravenous potassium C. oral magnesium D. oral phosphate

American academy of pediatrics

722

PREP ® Self-Assessment

PREPSA 2020

Correct Answer: D The adolescent girl in this vignette has refeeding syndrome, as evidenced by metabolic changes following enteral nutrition supplementation via a nasogastric feeding tube. She is hypophosphatemic and requires oral phosphate therapy to avoid complications of hypophosphatemia. Underlying medical conditions resulting in increased risk for refeeding syndrome during nutritional rehabilitation include psychiatric diseases (anorexia nervosa, alcohol or drug use), failure to thrive, gastrointestinal diseases (inflammatory bowel disease, short gut syndrome, cystic fibrosis), and social risk factors (neglect, homelessness). Thus, patients at high risk for refeeding syndrome should be carefully monitored as nutritional therapy is administered. During starvation or malnutrition, fat and proteins are broken down, allowing gluconeogenesis to occur. Insulin levels are low, and already depleted levels of electrolytes shift to maintain homeostasis. Once refeeding occurs, hyperglycemia results (caused by low insulin levels) and osmolar shifts occur. Intracellular shifts of magnesium and potassium occur, resulting in hypomagnesemia and hypokalemia. Hypophosphatemia occurs due to increased utilization of phosphate for production of ATP. Increased metabolic thiamine requirements result in thiamine deficiency. Hypophosphatemia, the most common complication of refeeding syndrome, can result in cardiac arrhythmia, respiratory failure, weakness, hemolysis, and mental status changes. Hypokalemia and hypomagnesemia can cause cardiac arrhythmia, respiratory failure, weakness, nausea, vomiting, changes in stool pattern, and neurologic symptoms. To avoid sequelae of refeeding syndrome, nutritional supplementation is slowly advanced over several days before achieving goal caloric supplementation. In addition, careful monitoring for refeeding syndrome should occur, including continuous cardiac monitoring, daily neurologic and cardiac examinations, calorie counts, daily weights, and baseline/daily laboratory studies including phosphorus, magnesium, potassium, sodium, glucose, and renal function. Replacement of vitamins and minerals (thiamine supplementation and multivitamin administration) should occur, and electrolyte disorders should be corrected (Item C200).

American academy of pediatrics

723

PREP ® Self-Assessment

PREPSA 2020

Intravenous iron sucrose is not indicated because the hemoglobin of the patient in this vignette is within normal limits. Intravenous potassium is not indicated as the potassium levels are within the normal range. Oral magnesium is not indicated as the magnesium level is within normal limits. PREP Pearls • Children and adolescents at high risk for refeeding syndrome should be closely monitored during institution of nutritional rehabilitation. • Hypophosphatemia, hypomagnesemia, and hypokalemia can occur due to intracellular electrolyte shifts as refeeding begins. MOCA-Peds Objective • Screen for medical complications related to eating disorders. ABP Content Specifications(s) • Identify the clinical and laboratory features associated with refeeding syndrome

American academy of pediatrics

724

PREP ® Self-Assessment

PREPSA 2020

Suggested Readings • Pulcini CD, Zettle S, Srinath A. Refeeding syndrome. Pediatr Rev. 2016;37(12):516-523. doi:10.1542/pir.2015-0152. • Rocks T, Pelly F, Wilkinson P. Nutrition therapy during initiation of refeeding in underweight children and adolescent inpatients with anorexia nervosa: a systematic review of the evidence. J Acad Nutr Diet. 2014;114(6):897-907. doi:10.1016/j.jand.2013.11.022. • Schneider M, Fisher M. Anorexia nervosa, bulimia nervosa, and other eating disorders. In: McInerny TK, Adam HM, Campbell DE, DeWitt TG, Foy JM, Kamat DM, eds. American Academy of Pediatrics Textbook of Pediatric Care. 2nd ed. Elk Grove Village, IL: American Academy of Pediatrics; 2017:1719-1728. Pediatric Care Online.

American academy of pediatrics

725

PREP ® Self-Assessment

PREPSA 2020

Question 201 A 10-year-old boy is brought to the emergency department for the evaluation of sudden onset of fever to 39°C after a viral upper respiratory illness. He has a wet cough and complains of right chest pain. On physical examination, he is ill appearing. He has a respiratory rate of 20 breaths/min and oxygen saturation of 94% on room air. He complains of pleuritic pain on the right side with deep inspiration and splints his chest to the right. Breath sounds are decreased in the right midlung field, but no rales, wheezes, or rubs are heard. He is able to produce a sputum sample that shows gram-positive lancet-shaped diplococci and demonstrates the pattern of hemolysis shown in the image (Item Q201).

Item Q201: Sputum culture showing 24-hour growth on sheep blood agar. Reprinted with permission from R Jerris and Kimberlin DW, et al, eds. Red Book Online. Itasca, IL: American Academy of Pediatrics; 2018.

Of the following, this boy would be at INCREASED risk for complications if he had a history of A. allergy to house dust mite and cat B. atrial septal defect C. parental smoking in the home D. sickle cell disease

American academy of pediatrics

726

PREP ® Self-Assessment

PREPSA 2020

Correct Answer: D The boy in the vignette has pneumococcal pneumonia, characterized by lancet-shaped diplococci on Gram stain with α-hemolysis on culture plate. The presence of sickle cell disease would predispose him to complications of invasive pneumonia or septicemia. Allergy to house dust mite and cat would not put this boy at increased risk for an adverse outcome from pneumococcal pneumonia, though children with asthma are at greater risk for invasive disease. Atrial septal defect and environmental tobacco smoke are not risk factors for invasive pneumococcal disease, though they are associated with an overall increased risk for poor outcome from pneumonia of any kind. Risk for invasive pneumococcal pneumonia is increased in children with HIV disease, primary immunodeficiency, immunosuppression related to medical therapy for cancer or inflammatory processes, cochlear implants, or sickle cell disease. Introduction of the conjugated pneumococcal vaccine has significantly decreased the incidence of clinically significant pneumococcal disease in all children and invasive disease in children at risk. Routine penicillin prophylaxis for children with sickle cell disease was started in the 1980s, and resulted in a great decrease in severe pneumonias and other complications of pneumococcal disease. Introduction of the 23-valent polysaccharide vaccine for children older than 2 years with sickle cell disease further decreased the incidence of invasive pneumococcal infections, but there has been even greater benefit with the introduction of the conjugated 13-valent vaccine even when given after the polysaccharide vaccine. The current practice of administering the conjugated vaccine first followed by the polysaccharide vaccine is expected to show additional benefit. Most respiratory infections in children are caused by viruses and specific pathogens are never identified. For viral pneumonias there is no specific treatment other than support. However, many children with viral pneumonias are treated with antibiotics as if they had bacterial pathogens because no specific data are available to rule out bacterial infection. Availability of easily accessible and cost-effective respiratory viral antigen panels performed on nasopharyngeal swabs may help to improve decision making in this area. The predominance of communityacquired pneumonias for which a bacterial organism is identified are caused by pneumococcus. Staphylococcus is the other bacterium that may be implicated, and particularly methicillinresistant Staphylococcus when there are complications. Complications of pneumococcal pneumonia occur in about 50% of children who require hospitalization for the pneumonia. These may include pleural effusion, empyema, necrotizing pneumonia, lung abscess, atelectasis, pneumatocele, or pneumothorax. Invasive staphylococcal pneumonia may result in empyema, necrotizing parenchymal disease, abscess, or pneumatocele.

American academy of pediatrics

727

PREP ® Self-Assessment

PREPSA 2020

PREP Pearls • Children with sickle cell disease, immunodeficiency, HIV disease, and cochlear implants are at risk for invasive pneumococcal pneumonia. • Children with congenital heart disease, asthma, and environmental tobacco smoke exposure are at greater risk for the development of pneumonia of any cause than their peers, but not at greater risk for invasive pneumococcal disease. • Sequential administration of the 13-valent conjugated pneumococcal vaccine and the 23valent polysaccharide vaccine (at least 8 weeks later) will decrease the incidence of invasive pneumococcal disease for children at risk. ABP Content Specifications(s) • Plan the appropriate diagnostic evaluation for pneumonias of various etiologies • Plan appropriate management of the different types of pneumonia • Identify the major acute and chronic complications of pneumonia • Recognize the clinical features of pneumonias of various etiologies and the associated sequelae Suggested Readings • Farnham AC, Zimmerman CM, Papadouka V, et al. Invasive pneumococcal disease following introduction of 13-valent conjugate vaccine in children in New York City from 2007-2012. JAMA Pediatr. 2015;169:646-652. JAMA. • Light M. Pneumonia. In: McInerny TK, Adam HM, Campbell DE, DeWitt TG, Foy JM, Kamat DM, eds. American Academy of Pediatrics Textbook of Pediatric Care. 2nd ed. Elk Grove Village, IL: American Academy of Pediatrics; 2017:2510-2522. Pediatric Care Online. • Obaro SK, Tam PYI. Preventing infections in sickle cell disease: the unfinished business. Pediatr Blood Cancer. 2016;63:781-785. doi:10.1002/pbc.25911. • Ricketson LJ, Conradi NG, Vanderkooi OG, Kellner JD. Changes in the nature and severity of invasive pneumococcal disease in children before and after the seven-valent and thirteen-valent pneumococcal conjugate vaccine programs in Calgary, Canada. Pediatr Infect Dis J. 2018;37:2227. https://insights.ovid.com/pubmed?pmid=28737622.

American academy of pediatrics

728

PREP ® Self-Assessment

PREPSA 2020

Question 202 A 3-month-old female infant is seen for a bump on her upper eyelid. Her mother first noticed an area of redness on the eyelid 3 weeks ago, which has progressively become darker and is now raised and firm. The infant was born at term without complications, has been growing appropriately, and has no other systemic symptoms. She is alert and has normal tone and neurological examination findings. There is a red, well-demarcated, raised plaque on the upper eyelid, which is causing mild ptosis (Item Q202). Pupils are round, equal, and reactive to light with a normal red reflex. Extraocular movements are intact.

Item Q202: Infant described in the vignette. Reprinted with permission from Darrow D, Greene A, Mancini A, Nopper A. Pediatrics. 2015;136(4):e1085.

Of the following, the MOST likely diagnosis is A. hemangioma B. orbital cyst C. orbital lymphangioma D. rhabdomyosarcoma

American academy of pediatrics

729

PREP ® Self-Assessment

PREPSA 2020

Correct Answer: A The infant in the vignette has an infantile hemangioma. This is the most common tumor in children, affecting about 5% of the population. It is more common in premature infants (especially those weighing < 1 kg at birth), female infants, and white infants. Infantile hemangiomas are unique given that they usually involute on their own but can lead to complications. In this vignette, the location of the hemangioma can lead to visual compromise and requires treatment. Hemangiomas that are larger than 1 cm, located on the medial aspect of the periorbital area, or cause displacement of the globe or ptosis are more likely to lead to visual impairment. Infantile hemangiomas usually begin growing prior to 1 month of age and continue to grow until approximately 3 to 5 months. By 12 months of age involution begins and is usually complete by 4 years of age but can leave the area distorted. Between 6 and 12 months of age the proliferation and involution often occur at equal rates, leading to little change in the hemangioma during this time. The first-line medical treatment for hemangioma is currently propranolol, a β-blocker. Indications for treatment of hemangioma are outlined by Darrow et al. The effectiveness of propranolol was first discovered in 2008 when it was observed that infants with infantile hemangiomas who received propranolol for cardiac abnormalities had improvement of their hemangiomas compared to infants who did not receive propranolol. β-Blockers likely decrease the proliferation of cells within the tumor and lead to lightening and softening of the hemangioma. The changes frequently begin within the first few doses. An orbital cyst is subcutaneous and would not be red and darkening. Orbital lymphangiomas are less circumscribed and can grow rapidly leading to pain and proptosis. Rhabdomyosarcomas also can develop quickly leading to proptosis as the tumor grows and pushes the globe forward. PREP Pearls • Infantile hemangiomas in the periorbital area leading to visual changes require treatment. • Propranolol is the first-line medical treatment for infantile hemangiomas. • Infantile hemangiomas grow over the first few months and then involute over several years. ABP Content Specifications(s) • Identify conditions that may lead to the development of amblyopia • Recognize the visual consequences of a tumor or hemangioma in the periorbital area

American academy of pediatrics

730

PREP ® Self-Assessment

PREPSA 2020

Suggested Readings • Arneja JS, Benson A, Gilardino MS. Hemangiomas. In: McInerny TK, Adam HM, Campbell DE, DeWitt TG, Foy JM, Kamat DM, eds. American Academy of Pediatrics Textbook of Pediatric Care. 2nd ed. Elk Grove Village, IL: American Academy of Pediatrics; 2017:2110-2117. Pediatric Care Online. • Darrow D, Greene A, Mancini A, Nopper A. Diagnosis and management of infantile hemangioma. Pediatrics. 2015;136(4):e1060-e1104. doi:10.1542/peds.2015-2485. • Drolet BA, Frommelt PC, Chamlin SL, et al. Initiation and use of propranolol for infantile hemangioma: report of a consensus conference. Pediatrics. 2013;131(1):128140. doi:10.1542/peds.2012-1691. • Frieden IJ, Mancini AJ. AAP infantile hemangioma [webinar]. Pediatric Care Online. • Krowchuk DP, Frieden IJ, Mancini AJ; Subcommittee on the Management of Infantile Hemangiomas. Clinical practice guideline for the management of infantile hemangiomas. Pediatrics. 2019;143(1):e20183475. doi:10.1542/peds.2018-3475.

American academy of pediatrics

731

PREP ® Self-Assessment

PREPSA 2020

Question 203 A 6-year-old girl is being evaluated for developmental delay. She is quite outgoing and talkative during the visit; however, her cognitive skills are at the level of a 3 year old. Her full-scale IQ was 50 on a recent educational psychology assessment. Her verbal IQ was significantly higher than her visual-spatial score. She has a history of idiopathic hypercalcemia and mild hypotonia. On physical examination, she has facial dysmorphology most notable for bitemporal narrowing, periorbital fullness, long philtrum, short nose, wide mouth with full lips, and stellate blue eyes (Item Q203). A crescendo-decrescendo low-pitched systolic murmur is noted at the cardiac base with radiation to the right carotid artery. The precordium is hyperdynamic and carotid pulses are decreased to the left upper limb. Microarray testing reveals a contiguous gene deletion of 7q11.23.

American academy of pediatrics

732

PREP ® Self-Assessment

PREPSA 2020

Item Q203: Girl described in the vignette. Reprinted with permission from Lyons M. Specific genetic conditions. In: Saul RA, ed. Medical Genetics in Pediatric Practice. Itasca, IL: American Academy of Pediatrics; 2013; chap 9:225.

Of the following, the MOST likely cardiac defect in this girl is A. atrial septal defect B. supravalvar aortic stenosis C. tetralogy of Fallot D. ventricular septal defect

American academy of pediatrics

733

PREP ® Self-Assessment

PREPSA 2020

Correct Answer: B The girl in the vignette has Williams syndrome resulting from a contiguous gene deletion in 7q11.23 that encompasses the elastin gene (ELN). Penetrance is 100% while expression is variable. The most common cardiovascular finding in Williams syndrome is supravalvar aortic stenosis, which is present in 75% of individuals. Other cardiovascular manifestations include elastin arteriopathy, peripheral pulmonary stenosis, and hypertension. Distinctive facies include bitemporal narrowing, periorbital fullness, stellate irides, long philtrum, upper/lower lips with a thickened vermilion, wide mouth, small jaw, large ear lobes, and a broad forehead. Individuals commonly possess a specific neurocognitive profile with overfriendliness, generalized anxiety, attention-deficit/hyperactivity disorder, strengths in verbal short-term memory and language, and weakness in visual-spatial construction. Most have intellectual disability, typically mild. Growth abnormalities are characterized by prenatal growth deficiency and slow weight gain and linear growth in the first 4 years, and a brief pubertal growth spurt resulting in short stature. Endocrine abnormalities can include idiopathic hypercalciuria and hypothyroidism. The diagnosis of Williams syndrome is confirmed via chromosomal microarray or targeted deletion analysis using fluorescence in situ hybridization. A contiguous gene deletion syndrome is caused by a microdeletion that encompasses 2 or more genes in tandem position along a chromosome. By virtue of the fact that several genes are involved, contiguous gene syndromes often affect multiple systems of the body. Thus, one must assess the function of the specific genes involved within the deletion and thoroughly examine the patient for involvement for those specified regions (heart, kidney, brain, etc). Other common contiguous gene deletions include 22q11.2 deletion syndrome, 1p36 deletion, Smith-Magenis syndrome, and Cri-du-chat syndrome. Ventricular septal defect and atrial septal defect are common congenital heart defects, but not the most common ones noted in an individual with Williams syndrome. Tetralogy of Fallot is typically associated with 22q11.2 deletion syndrome. PREP Pearls • A contiguous gene deletion syndrome is caused by a microdeletion that encompasses 2 or more genes in tandem position along a chromosome. • Williams syndrome is a contiguous gene deletion syndrome arising from a recurrent deletion in the 7q11.23 region encompassing the elastin gene (ELN). • Williams syndrome is characterized by cardiovascular disease, typical facies, connective tissue abnormalities, intellectual disability, idiopathic hypercalciuria/hypothyroidism, growth issues, and a specific neurocognitive profile; the most common cardiovascular finding is supravalvar aortic stenosis. ABP Content Specifications(s) • Recognize the clinical and laboratory findings associated with contiguous gene syndromes • Recognize the inheritance pattern associated with contiguous gene syndromes American academy of pediatrics

734

PREP ® Self-Assessment

PREPSA 2020

Suggested Readings • American Academy of Pediatrics, Committee on Genetics. Health care supervision for children with Williams syndrome. Pediatrics. 2001;107(5):11921204. https://pediatrics.aappublications.org/content/107/5/1192. • Lacroix A, Famelart N, Guidetti M. Language and emotional abilities in children with Williams syndrome and children with autism spectrum disorder: similarities and differences. Pediatr Health Med Ther. 2016;7:89-97. doi:10.2147/PHMT.S66347. • McCulloch MA, Gajarski RJ. Congenital and acquired heart disease. In: McInerny TK, Adam HM, Campbell DE, DeWitt TG, Foy JM, Kamat DM, eds. American Academy of Pediatrics Textbook of Pediatric Care. 2nd ed. Itasca, IL: American Academy of Pediatrics; 2016:1883-1917. Pediatric Care Online. • Morris CA. Williams syndrome. GeneReviews. https://www.ncbi.nlm.nih.gov/books/NBK1249/.

American academy of pediatrics

735

PREP ® Self-Assessment

PREPSA 2020

Question 204 A 6-month-old infant who was born at 27 weeks’ gestation has feeding difficulties. A gastrostomy tube has been placed through which she receives most of her nutrition. The parents seek advice on what they need to consider if tube feedings are to continue when starting daycare. Of the following, the MOST accurate statement about this infant’s feeding is that A. daycare centers are considered private entities, therefore they are not required to accept children with this condition B. in all states, this is considered a medical procedure and can only be administered by a nurse or medical professional C. the child should only attend a medical or special needs daycare D. this would be covered under the Americans with Disabilities Act; daycare centers must make reasonable accommodations

American academy of pediatrics

736

PREP ® Self-Assessment

PREPSA 2020

Correct Answer: D The premature infant in the vignette receives most of her nutrition via gastrostomy tube, which would be considered a disability under federal law. The Americans with Disabilities Act (ADA) is a federal civil rights law originally passed in 1990 that, among other things, prohibits discrimination by child care centers and family child care providers against individuals of all ages with disabilities. Individuals of all ages receive protection under the ADA who have a history of, who are regarded as having, or who have a physical or mental impairment that substantially limits 1 or more major life activities (seeing, hearing, eating, etc). Privately run child care centers are considered "public accommodations," similar to private schools, recreation centers, restaurants, hotels, movie theaters, etc, and must comply with the ADA, which states that centers: • Cannot exclude children with disabilities from their programs unless their presence would pose a direct threat to the health or safety of others or require a fundamental alteration of the program • Must make reasonable modifications to policies and practices to integrate children with disabilities into their programs unless doing so would constitute a fundamental alteration • Must provide appropriate auxiliary aids and services needed for effective communication with children with disabilities when doing so would not constitute an undue burden • Must generally make facilities accessible to children with disabilities Child care services facilitated by government agencies, such as Head Start, must also comply with the ADA. Child care centers that are provided by religious entities, such as churches, mosques, or synagogues, are exempt. Although a day care center may make an individualized assessment about whether it can meet the particular needs of a child without fundamentally altering its program, case law has demonstrated that children who require diabetes-related care, for example, can be integrated into these settings. Furthermore, most community care licensing agencies do not consider the administration of routine gastrostomy tube feedings or administering liquid medication through gastrostomy tubes to be a medical procedure. However, day care personnel must be properly trained and instructed on gastrostomy tube care. Personnel are usually required to follow specific written instructions from the child’s health care provider including, but not limited to the exact steps necessary to provide feedings or administer medications; frequency, amount, and type of formula, liquid, or medication; proper cleaning procedures; potential side effects; and emergency contact and procedures should the gastrostomy tube become dislodged. A child who feeds via gastrostomy tube does not need to attend a medical or special needs–only day care center. In the realm of educational access, children with special needs receive additional protections under 2 federal laws, the Individuals with Disabilities Education Act (IDEA) and Section 504 of the Rehabilitation Act (Item C204).

American academy of pediatrics

737

PREP ® Self-Assessment

American academy of pediatrics

PREPSA 2020

738

PREP ® Self-Assessment

PREPSA 2020

Rehabilitative services may be provided to children as part of IDEA (Individualized Education Program/Individualized Family Service Plan) or Section 504 (504 plan). These include occupational, physical, and speech/language therapy. Occupational therapy assists children to improve their physical, sensory, and fine motor skills. Occupational therapy addresses psychological, social, and environmental factors that can affect functioning, such as play, school performance, and daily activities. Physical therapy might assist with developing or normalizing range of motion, strength, flexibility, functional stability, proprioception, and gross motor skills. Speech and language therapy addresses disorders related to articulation, fluency, voice, reception, expression, cognitive-communication, dysphagia, and oral feeding. PREP Pearls • The Americans with Disabilities Act is a federal civil rights law that, among other things, prohibits discrimination by child care centers and family child care providers against individuals of all ages with disabilities, including children with gastrostomy tubes. These providers are considered public entities except for those governed by religious organizations. • The Individuals with Disabilities Education Act requires special educational programs for children with disabilities; provides funding for free appropriate public education for children aged 3 to 22 years with disabilities; and authorizes services for early intervention programs for infants and children younger than 3 years. • Section 504 of the Rehabilitation Act prohibits discrimination on the basis of disability in programs that receive federal funding; affords rights of equal access to free appropriate public education via a wide range of services in the least restrictive environment for school-aged children and reasonable accommodations for children in child care or college. ABP Content Specifications(s) • Recognize the value and limitations of language, occupational, and physical therapy • Understand the provisions of current legislation for patients of various ages who have educational or physical disabilities Suggested Readings • Lipkin PH, Okamoto J; Council on Children with Disabilities, Council on School Health. The Individuals with Disabilities Education Act (IDEA) for children with special healthcare needs. Pediatrics. 2015;136(6):e1650-e1662. doi:10.1542/peds.2015-3409. • Section 504 of the Rehabilitation Act. 29 USC §794, Title 29. Labor. Chapter 16: Vocational Rehabilitation and Other Rehabilitation Services, Subchapter V; Rights and Advocacy, section 794, nondiscrimination under federal grants and programs; 2011. • The Americans with Disabilities Act. 42 USC §12101, Title 42. The Public Health and Welfare, Chapter 126, Equal Opportunity for Individuals with Disabilities, section 12101, findings and purpose; 1990. • The Individuals with Disabilities Education Act. 20 USC §1400, Title 20. Education, Education of Children with Disabilities, Subchapter I, Definitions, section 1400, short title, findings, purposes; 2011.

American academy of pediatrics

739

PREP ® Self-Assessment

PREPSA 2020

Question 205 A term neonate is born via precipitous vaginal delivery to a 19-year-old woman who received no prenatal care. She lives with her boyfriend and reports using marijuana and alcohol early in her pregnancy. A rapid HIV test and rapid plasma reagin test performed on the mother soon after delivery were negative. Physical examination reveals a 2.2-kg neonate with microcephaly, hepatosplenomegaly, and petechiae. He is alert, has a good cry, and normal tone and activity. The remainder of his physical examination findings are unremarkable. Laboratory data are notable for thrombocytopenia and hepatitis. Computed tomography scan of the head reveals periventricular calcifications. Evaluation by the ophthalmologist reveals bilateral chorioretinitis. Polymerase chain reaction testing of the neonate’s urine specimen on day 2 after birth is positive for cytomegalovirus infection. Of the following, the MOST appropriate antiviral therapy for this neonate is A. intravenous ganciclovir for 6 weeks B. intravenous ganciclovir for 4 weeks C. oral valganciclovir for 6 weeks D. oral valganciclovir for 6 months

American academy of pediatrics

740

PREP ® Self-Assessment

PREPSA 2020

Correct Answer: D The neonate described in the vignette with microcephaly, intracranial calcifications, hepatosplenomegaly, petechial rash, thrombocytopenia, and hepatitis has congenital cytomegalovirus (CMV) infection. Antiviral treatment with oral valganciclovir for 6 months is the recommended treatment for term neonates with symptomatic congenital CMV infection with and without central nervous system disease. Therapy must be started by 4 weeks of age. Cytomegalovirus is the most common congenital viral infection in the United States, occurring in 0.5% to 1% of all live births. Mother-to-infant transmission of CMV can occur during pregnancy through transplacental spread of maternal virus, during delivery via exposure to infected maternal cervical secretions, or postnatally through breast milk or by receiving blood products from CMV-positive donors. The rate of intrauterine transmission is 30% to 40% in women with primary CMV infection (ie, mothers with no preexisting immunity) but is only 1% in women with nonprimary CMV infection (ie, CMV reactivation or reinfection with a different viral strain in mothers with preexisting immunity). Congenital CMV infection with sequelae can occur throughout pregnancy; however, severe neonatal sequelae are more likely to occur with primary maternal infection during the early stages of gestation. In resource-rich countries, up to 50% of women of reproductive age lack immunity to CMV, making them susceptible to primary infection during pregnancy. Symptomatic congenital CMV infection can also occur after nonprimary maternal infection. In the United States, it is estimated that maternal nonprimary infection contributes to more than two-thirds of all infants born with congenital CMV infection. Of neonates with congenital CMV infection, 90% are asymptomatic at birth. Of these, about 15% develop sensorineural hearing loss (SNHL). When CMV infection is clinically apparent at birth, there may be a wide spectrum of multiorgan involvement. Manifestations include intrauterine growth restriction, microcephaly, intracranial calcifications, chorioretinitis, jaundice, hepatosplenomegaly, petechial/purpuric rash (because of thrombocytopenia), “blueberry muffin” lesions (because of extramedullary hematopoiesis), and elevated transaminase concentrations. Three percent to 10% of symptomatic neonates die as a result of the infection; 40% to 90% of survivors have some degree of neurologic sequelae, including SNHL, intellectual deficits, and visual impairment. Sensorineural hearing loss is the most frequent complication, occurring in up to 50% of infants with symptomatic congenital CMV infection. Sensorineural hearing loss may not be apparent during the neonatal period, with progressive hearing loss developing over the next several years. The diagnosis of congenital CMV is confirmed by detection of the virus in urine, saliva, respiratory tract secretions, blood, or CSF using polymerase chain reaction (PCR) or culture within the first 2 to 4 weeks after birth. Real-time PCR is the preferred diagnostic test for detecting CMV because of its superior sensitivity and specificity, quick turnaround time, and lower cost compared with viral culture and other methods. Serology is not recommended for the diagnosis of congenital CMV infection because of low specificity and frequent false-positive results. Ophthalmologic evaluation may detect chorioretinitis; computed tomography imaging of the brain may detect periventricular calcifications suggestive of congenital CMV infection. American academy of pediatrics

741

PREP ® Self-Assessment

PREPSA 2020

Infants and children with congenital CMV infection must have close follow-up with audiology, ophthalmology, and neurodevelopment services. Treatment with antiviral agents is not recommended for neonates with asymptomatic congenital CMV infection. Oral valganciclovir (16 mg/kg per dose administered twice daily for 6 months) is recommended for term neonates with symptomatic congenital CMV infection with and without central nervous system disease; therapy must be started by 4 weeks of age. Adverse effects of oral valganciclovir therapy include significant neutropenia (in 20% of infants) and hepatotoxicity. Consultation with a pediatric infectious disease physician is recommended when considering antiviral treatment of symptomatic congenital CMV infection. Intravenous ganciclovir may be considered in neonates with symptomatic congenital CMV infection who have necrotizing enterocolitis or other intestinal problems limiting their ability to absorb oral medication, a situation not noted for the neonate in the vignette. Significant myelosuppression (in 66% of infants) is a major concern with intravenous ganciclovir therapy; in addition, catheter-related complications may occur from prolonged intravenous access. If intravenous ganciclovir therapy is administered, close monitoring is recommended. PREP Pearls • Oral valganciclovir therapy for 6 months has demonstrated modest benefit in improving audiologic and some neurodevelopmental outcomes in neonates and infants with symptomatic congenital cytomegalovirus infection and is the recommended treatment. • Cytomegalovirus is the most common congenital viral infection in the United States, occurring in 0.5% to 1% of all live births. • About 10% of neonates with congenital cytomegalovirus infection are symptomatic at birth with a wide spectrum of multiorgan involvement. MOCA-Peds Objective • Evaluate a child with possible vertically transmitted infection. ABP Content Specifications(s) • Plan the diagnostic evaluation for congenital or acquired cytomegalovirus infection in normal and immunocompromised children of various ages • Recognize the signs and symptoms of symptomatic congenital cytomegalovirus infection • Recognize the various routes by which cytomegalovirus infection can be acquired in a newborn infant • Understand how cytomegalovirus is transmitted beyond the neonatal period

American academy of pediatrics

742

PREP ® Self-Assessment

PREPSA 2020

Suggested Readings • American Academy of Pediatrics. Cytomegalovirus infection. In: Kimberlin DW, Brady MT, Jackson MA, Long SS, eds. Red Book: 2018 Report of the Committee on Infectious Diseases. 31st ed. Itasca, IL: American Academy of Pediatrics; 2018:310317. https://redbook.solutions.aap.org/chapter.aspx?sectionId=189640079&bookId=2205 &resultClick=1. • Kimberlin DW, Jester PM, Sánchez PJ, et al. Valganciclovir for symptomatic congenital cytomegalovirus disease. N Engl J Med. 2015;372(10):933-943. doi:10.1056/NEJMoa1404599. • Kojaoghlanian T. The newborn at risk for infection. In: McInerny TK, Adam HM, Campbell DE, DeWitt TG, Foy JM, Kamat DM, eds. American Academy of Pediatrics Textbook of Pediatric Care. 2nd ed. Elk Grove Village, IL: American Academy of Pediatrics; 2017:899-909. Pediatric Care Online. • Pinninti S, Hough-Telford C, Pati S, Boppana S. Cytomegalovirus and Epstein-Barr virus infections. Pediatr Rev. 2016;37(6):223-234. doi:10.1542/pir.2015-0072.

American academy of pediatrics

743

PREP ® Self-Assessment

PREPSA 2020

Question 206 A healthy 9-month-old female infant is seen for a routine health supervision visit. On physical examination, a membrane connecting the labia minora that seems to cover most of the vaginal opening is seen (Item Q206). The parents do not report any symptoms and have been able to see a strong urine stream.

Item Q206 : Image of the lesion described in the vignette. Reprinted with permission from American Academy of Pediatrics Textbook of Pediatric Care. 2nd ed. Elk Grove Village, IL: American Academy of Pediatrics; 2017:2241.

Of the following, the BEST approach to the management of the examination finding is A. manual separation in the office B. reassurance that this is self-limited C. referral to a urologist or gynecologist D. topical estrogen cream

American academy of pediatrics

744

PREP ® Self-Assessment

PREPSA 2020

Correct Answer: B Manual separation, referral to a specialist, and topical estrogen cream are not necessary at this time because the infant in this vignette has been asymptomatic. Her parents should be given reassurance that her condition is self-limited. Labial adhesions in prepubertal females are most likely the result of chronic irritation of the labia minora. Chronic irritation may result from poor hygiene or from infections such as Candida albicans. Girls younger than 5 years are most susceptible to developing labial adhesions because of difficulties with hygiene. Increasing estrogen levels in older prepubertal females also help protect against the formation of adhesions. Labial adhesions in pubertal females are very rare because of the higher estrogen levels but may still may occur in patients with herpes infections or patients receiving chemotherapy. Nearly all children with labial adhesions will have complete self-resolution within 18 months, therefore asymptomatic individuals should receive reassurance. Symptomatic individuals (those with urinary problems such as urinary tract infections or urinary retention) should first undergo a trial of twice daily topical estrogen cream. Adverse effects of estrogen cream, such as vulvar hyperpigmentation or breast budding, will reverse once the cream is discontinued. If adverse effects become unacceptable or if adhesions are still present after 8 weeks of topical treatment, manual separation in the office may be performed under local anesthetic with postprocedural care involving daily application of a lubricant for several months to prevent reformation of adhesions. Referral to a specialist for surgical lysis may be necessary for patients who are unresponsive to conventional treatments. PREP Pearls • Asymptomatic children with labial adhesions do not require treatment. • Symptomatic children with labial adhesions should first undergo a trial of a topical estrogen cream. • Referral to a specialist for surgical lysis may be necessary for symptomatic labial adhesions that do not respond to conventional therapies. ABP Content Specifications(s) • Recognize the clinical findings associated with labial adhesions, and manage appropriately Suggested Readings • Nield LS. Labial adhesions. In: McInerny TK, Adam HM, Campbell DE, DeWitt TG, Foy JM, Kamat DM, eds. American Academy of Pediatrics Textbook of Pediatric Care. 2nd ed. Elk Grove Village, IL: American Academy of Pediatrics; 2017:22402242. Pediatric Care Online. • Oquendo Del Toro HM, Hoefgen HR. Vulvovaginitis. In: Kliegman RM, St Geme JW III, Blum NJ, Shah SS, Tasker RC, Wilson KM, eds. Nelson Textbook of Pediatrics. 21st ed. Philadelphia, PA: Elsevier; 2020:2844-2851.

American academy of pediatrics

745

PREP ® Self-Assessment

PREPSA 2020

Question 207 A 3-year-old boy is receiving a health supervision evaluation. He is an only child with no significant medical history. His teacher at the preschool he has attended for 9 months mentioned that the boy rarely interacts with other children in the classroom and only engages in solo play. He becomes upset when the teacher attempts to involve him in circle time or other group activities. The boy’s mother describes him as "shy." He speaks in 3- to 4-word phrases but not all of his words can be understood by a stranger. He follows complex verbal commands, can fill his water glass from a small pitcher, and draws rough circles. He cannot yet draw a figure, and when he dresses himself, he sometimes puts his shirt on backwards or his shoes on the wrong foot. He jumps, climbs, and can walk on tiptoe. He is not yet fully toilet trained. Of the following, the MOST accurate assessment of this boy’s development is A. expressive language delay B. global developmental delay C. normal development for age D. social-emotional delay

American academy of pediatrics

746

PREP ® Self-Assessment

PREPSA 2020

Correct Answer: D The 3-year-old boy in the vignette rarely interacts with other children and only engages in solo play; his social-emotional development is delayed. Other realms of his development are ageappropriate. Skills seen in typically developing 3-year-old children are noted in Item C207A.

Social-emotional development does not occur in isolation; normal social-emotional development requires that children concurrently acquire language and fine motor skills. Red flags in socialemotional development are shown in Item C207B.

American academy of pediatrics

747

PREP ® Self-Assessment

PREPSA 2020

The American Academy of Pediatrics recommends that pediatric providers conduct developmental surveillance at each health supervision visit, with the incorporation of standardized developmental screening tests if concerns arise. In addition, providers should American academy of pediatrics

748

PREP ® Self-Assessment

PREPSA 2020

administer developmental screening tests to all children at the 9-, 18-, and either 24- or 30-month visits, and autism spectrum disorder-specific screening tests at the 24-month visit. PREP Pearls • Typically developing 3-year-old children can pedal a tricycle, walk on tiptoe, walk up stairs with alternating feet, catch a ball, use a fork and spoon, pour water from a small pitcher into a cup, imitate a circle, speak in 3+ word sentences with ¾ of their language understood by a stranger, follow complex commands, and engage in make-believe play and turn taking. • Pediatric providers should conduct developmental surveillance at all health supervision visits, and should administer developmental screening tests to all children at the 9-, 18-, and either 24- or 30-month visits and autism spectrum disorder–specific screening tests at the 24-month visit. MOCA-Peds Objective • Evaluate the developmental status of children ages 3 to 5 years of age. ABP Content Specifications(s) • Evaluate the cognitive and behavioral developmental progress/status of a child at 36 months of age, including recognition of abnormalities • Evaluate the motor developmental progress/status of a child at 36 months of age Suggested Readings • American Academy of Pediatrics. 3 year visit. In: Hagan JF, Shaw JS, Duncan PM, eds. Bright Futures: Guidelines for Health Supervision of Infants, Children, and Adolescents. 4th ed. Elk Grove Village, IL: American Academy of Pediatrics; 2017:603624. • Gerber RJ, WIlks T, Erdie-Lalena C. Developmental milestones 3: social-emotional development. Pediatr Rev. 2011;32(12):533-536. doi:10.1542/pir.32-12-533. • Scharf RJ, Scharf GJ, Stroustrup A. Developmental milestones. Pediatr Rev. 2016;37(1):25-37. doi:10.1542/pir.2014-0103.

American academy of pediatrics

749

PREP ® Self-Assessment

PREPSA 2020

Question 208 A 17-year-old adolescent boy, who was pitching for his high school baseball team, is brought to the emergency department for evaluation after being hit in the face by a line-drive several hours earlier. After sustaining the trauma, he fell immediately to the ground but did not lose consciousness. There was bleeding from both nares, which resolved after 15 minutes of pressure to the nasal bridge. In the emergency department, he appears well, alert, and oriented and has normal vital signs. His pupils are equal, round, and reactive to light. There is no periorbital swelling or ecchymosis and no subconjunctival hemorrhages. There is mild to moderate swelling of the nasal bridge, but the nasal dorsum is not flattened. He can breathe through each nare when the contralateral nare is temporarily occluded. Protruding bilateral reddish masses are seen in the nares. There is no malocclusion or loose teeth. Of the following, the BEST next step in management is A. computed tomography of the facial bones B. intravenous antibiotics C. nasal packing D. otorhinolaryngologist consultation

American academy of pediatrics

750

PREP ® Self-Assessment

PREPSA 2020

Correct Answer: D The adolescent in this vignette has bilateral septal hematomas after sustaining midface trauma. The hallmark finding of a septal hematoma on direct rhinoscopy is a reddish or bluish protuberance emanating from the nasal septum. Other common symptoms include pain and nasal obstruction. Management of a nasal septal hematoma includes an emergent consultation with an otorhinolaryngologist for emergent drainage. Failure to expeditiously drain a septal hematoma may result in necrosis of the nasal septum in as little as 72 to 96 hours, secondary to obstruction of blood flow from the mucoperichondrium to the cartilage. This can lead to a permanent deformity (typically a saddle-nose deformity). Nasal septal hematomas result from arterial injury between the cartilaginous nasal septum and the overlying mucoperichondrium. Adequate examination of the nasal cavity is crucial after any degree of midface trauma in children as septal hematomas can occur even after minor trauma. If the antecedent trauma also caused epistaxis it may be necessary to suction the nasal cavity to ensure adequate evaluation of the nasal septum. Use of a topical vasoconstrictor (eg, oxymetazoline) may facilitate visualization of the nasal septum. In a cooperative adolescent, drainage of a nasal septal hematoma can be accomplished under local analgesia; however, younger patients may require general sedation and drainage in an operating room. There is no role for imaging in the diagnosis of a nasal septal hematoma. Although direct facial trauma can also cause nasal or other midface fractures, imaging is typically not indicated for evaluation of an isolated nasal fracture. If the history or physical examination findings raise concern for additional midface or orbital fractures, computed tomography is the imaging modality of choice. Oftentimes there is significant soft tissue swelling immediately after the trauma, making identification and reduction of a displaced fracture extremely difficult. Optimal timing for reduction of nasal fractures is debatable, with conflicting evidence in the literature. Patients who do not have immediate closed reduction of a nasal fracture should follow-up with a surgical subspecialist within 3 to 5 days to allow swelling to decrease. At that time the need for reduction can be assessed and undertaken, if deemed necessary. After 5 to 6 days a nasal fracture has started to heal and manipulation becomes more difficult. If closed reduction is not performed prior to this time, open reduction may be necessary for optimal cosmetic outcomes. Intravenous antibiotics are not indicated in the initial management of a nasal septal hematoma; however, oral antibiotics should be administered if there is suspicion of an infection (eg, if presentation is not immediately after the trauma), if nasal packing is inserted, or in the setting of midface trauma if there is concurrent sinus injury. In addition to its use after incision and drainage of a nasal septal hematoma, nasal packing is indicated for an anterior source of epistaxis that does not resolve with direct pressure or cautery, if a direct source of bleeding is visualized. PREP Pearls • A reddish or bluish protuberance from the nasal septum after trauma is indicative of a nasal septal hematoma. • Nasal septal hematomas should be drained expeditiously to prevent permanent cosmetic defects. • Nasal fractures should be reduced prior to commencement of fracture healing, which occurs approximately 5 to 6 days after the trauma. American academy of pediatrics

751

PREP ® Self-Assessment

PREPSA 2020

ABP Content Specifications(s) • Understand the importance of early referral for surgical correction of displacement of the nasal bones • Recognize the physical findings associated with a hematoma of the nasal septum, and manage appropriately Suggested Readings • Aaronson PL, Neuman MI. Facial trauma. In: Shaw KN, Bachur RG, eds. Fleisher & Ludwig’s Textbook of Pediatric Emergency Medicine. 7th ed. Philadelphia, PA: Wolters Kluwer; 2016:1151-1159. • Kass JI, Ferguson BJ. Videos in clinical medicine. Treatment of hematoma of the nasal septum. N Engl J Med. 2015;372:e28. doi:10.1056/NEJMvcm1010616. • Sanyaolu LN, Farmer SE, Cuddihy EJ. Nasal septal haematoma. BMJ. 2014;349:g6075. doi:10.1136/bmj.g6075.

American academy of pediatrics

752

PREP ® Self-Assessment

PREPSA 2020

Question 209 A 7-year-old girl is seen for evaluation of weakness. The girl’s parents have noticed a purple rash on and around her eyelids over the last few weeks. For the past week she has been having difficulty getting up from a chair. They also noticed that she had difficulty brushing her hair this morning. She has normal vital signs and is alert and oriented. When asked to stand from the chair and climb on the examination table, she needs assistance because of weakness in her upper legs and shoulders. There is a purple rash around her eyelids and hyperkeratotic, flat, red papules on the knuckles of her hands (Item Q209). Heart and lung examination findings are normal.

Item Q209: Rash on the face and hands of the girl described in the vignette. Reprinted with permission from Vehe R, Riskalla M. Pediatr Rev. 2018;39(10):508.

Of the following, the MOST likely diagnosis is A. dermatomyositis B. Duchenne muscular dystrophy C. Guillain-Barré syndrome D. viral myositis

American academy of pediatrics

753

PREP ® Self-Assessment

PREPSA 2020

Correct Answer: A Of the response choices, dermatomyositis is the most likely diagnosis for the girl in the vignette who has proximal muscle weakness and an associated rash. Dermatomyositis is a small vessel vasculitis with affinity for the skin and muscles, leading to its unique clinical findings. Dermatomyositis often first manifests with exercise intolerance and fatigue followed by proximal muscle weakness. Activities that require the proximal lower extremities, such as getting up off the floor, climbing up stairs, getting into a vehicle, or standing from a seated position, become difficult. Upper extremity weakness can make reaching above the head to wash or comb hair or reaching in a locker equally difficult. Severe cases can include weakness of the neck muscles leading to voice changes or aspiration. Assessment of the hip flexors by having the patient raise their legs off the table while in a supine position or doing a sit-up is a useful and easy maneuver. The neck muscles can be evaluated by having the patient raise their head off the table while in a supine position and hold it up. Skin findings related to the vasculitis are another clinical feature of dermatomyositis. A heliotrope rash, described as a purplish facial rash, especially in the periorbital region, that includes the eyelids and bridge of the nose, is pathognomonic for the condition. The extensor surfaces of the fingers, elbows, and knees can have a scaly dermatitis with or without smooth flat-topped papules. Parents often describe a sunburn or contact sensitivity that is more severe or prolonged than expected. Dermatomyositis affects girls slightly more often than boys. Both adults and children can be affected, especially school-aged children. The diagnosis of dermatomyositis is made clinically based on examination findings of proximal muscle weakness and dermatologic findings as previously described. Laboratory evaluation showing evidence of muscle breakdown, including elevated levels of creatinine kinase, lactate dehydrogenase, and aldolase, is consistent with dermatomyositis. Results of a complete blood cell count and basic metabolic panel are typically within normal limits. A lack of myositisspecific antibodies does not exclude the diagnosis, especially in children. Magnetic resonance imaging of the proximal muscles can be useful for diagnosing and monitoring the illness. Children with Duchenne muscular dystrophy do not have an associated rash, and progression of the illness is often more prolonged. Children with Guillain-Barré syndrome have absent reflexes and typically have an ascending progression of weakness rather than upper and lower extremity proximal weakness, as seen in the girl in this vignette. The onset of symptoms in dermatomyositis is usually more insidious and less abrupt than that seen in viral myositis. Children with dermatomyositis also typically lack calf muscle tenderness on examination. PREP Pearls • Heliotrope rash is pathognomonic for dermatomyositis. • Proximal muscle weakness associated with a rash is consistent with dermatomyositis. • Laboratory evaluation for muscle breakdown, including creatinine kinase, lactate dehydrogenase, and aldolase levels, should be obtained when suspecting dermatomyositis. American academy of pediatrics

754

PREP ® Self-Assessment

PREPSA 2020

ABP Content Specifications(s) • Plan the appropriate laboratory evaluation for dermatomyositis • Recognize the clinical findings associated with dermatomyositis Suggested Readings • Vehe R, Riskalla M. Collagen vascular diseases: SLE, dermatomyositis, scleroderma and MCTD. Pediatr Rev. 2018;39(10):501-515. doi:10.1542/pir.2017-0262.

American academy of pediatrics

755

PREP ® Self-Assessment

PREPSA 2020

Question 210 A 3-day-old male neonate is admitted to the neonatal intensive care unit (NICU) for hypoglycemia. He was born at 39 weeks of gestation via spontaneous vaginal delivery with a birthweight of 4.4 kg. The pregnancy was uncomplicated; there was no history of gestational diabetes. Routine monitoring of blood glucose, because of his large-for-gestational age status, revealed several results in the 20 to 30 mg/dL (1.1-1.7 mmol/L) range that were confirmed on laboratory testing. The neonate initially breastfed vigorously, then became increasingly lethargic. His blood glucose level did not normalize with supplementation of standard infant formula. In the NICU, he has required a glucose infusion rate of 14 mg/kg per minute to maintain normal levels. Significant physical examination findings include generalized macrosomia and increased subcutaneous fat. He has no dysmorphic features, no evidence of any congenital anomalies, and normal male genitalia with bilaterally descended testes. Of the following, the MOST likely cause of this neonate’s hypoglycemia is (a) A. fatty acid oxidation disorder B. glycogen storage disease C. hyperinsulinism D. hypopituitarism

American academy of pediatrics

756

PREP ® Self-Assessment

PREPSA 2020

Correct Answer: C The neonate described in the vignette has hyperinsulinism. Clinical features consistent with hyperinsulinism include his large-for-gestational age size at birth, macrosomia, persistent hypoglycemia despite supplemental formula feedings, and high glucose infusion rate (>8 mg/kg per minute) required to maintain normal glucose levels. The large-for-gestational age size at birth and macrosomia are because of the intrauterine insulin effect on growth. This neonate’s presentation is most consistent with congenital hyperinsulinism resulting from a genetic mutation causing abnormally excessive insulin secretion. More common causes of hyperinsulinism include maternal diabetes and perinatal stress. Beckwith-Wiedemann syndrome is also associated with hyperinsulinism. Guidelines recommend obtaining a “critical sample” of blood at the time of hypoglycemia (glucose 1.7 mmol/L) after glucagon administration is consistent with hyperinsulinism. In other causes of hypoglycemia, liver glycogen is depleted, therefore glucagon has a minimal effect on plasma glucose. It is important to recognize hyperinsulinism for proper treatment and to avoid the adverse outcomes of persistent hypoglycemia. Immediate treatment of hypoglycemia includes an intravenous bolus of 2 mL/kg of 10% dextrose followed by continuous infusion of dextrose. High glucose infusion rates are usually required in cases of hyperinsulinism. Frequent feeding can help maintain normal glucose levels; glucagon can be used as adjunctive therapy. Hyperinsulinism may be transient or permanent. Diazoxide is the first-line medication for longterm management of permanent hyperinsulinism. Surgery (removal of a focal affected area of the pancreas or near-total pancreatectomy for diffuse disease) is indicated for permanent hyperinsulinism not responsive to medical therapy. Fatty acid oxidation disorders, glycogen storage disease, and hypopituitarism also cause hypoglycemia, but are not associated with large-for-gestational age size at birth, macrosomia, or a requirement for high glucose infusion rates. Fatty acid oxidation disorders usually present after the neonatal period, given the frequency of feedings in neonates. A longer fasting period is required before hypoglycemia resulting from a fatty acid oxidation disorder occurs. Glycogen storage disease is usually associated with hepatomegaly. Hypopituitarism may be associated with wandering nystagmus (because of optic nerve hypoplasia), midline defects, and micropenis in boys (resulting from gonadotropin deficiency). American academy of pediatrics

757

PREP ® Self-Assessment

PREPSA 2020

PREP Pearls • Clinical features consistent with hyperinsulinism include large-for-gestational age size at birth, macrosomia, persistent hypoglycemia, and requirement for a high glucose infusion rate (>8 mg/kg per minute) to maintain normal glucose levels. • At the time of hypoglycemia, a detectable insulin level, low β-hydroxybutyrate and free fatty acid levels, and an increase in plasma glucose by more than 30 mg/dL (>1.7 mmol/L) after glucagon administration are consistent with hyperinsulinism. ABP Content Specifications(s) • Recognize the clinical features associated with hyperinsulinism • Plan the appropriate immediate and long-term management of hyperinsulinism, while considering the long-term prognosis Suggested Readings • Demirbilek H, Rahman SA, Buyukyilmaz GG, et al. Diagnosis and treatment of hyperinsulinaemic hypoglycaemia and its implications for paediatric endocrinology. Int J Pediatr Endocrinol. 2017;2017:9. doi:10.1186/s13633-017-0048-8. • Gandhi K. Approach to hypoglycemia in infants and children. Transl Pediatr. 2017;6(4):408-420. doi:10.21037/tp.2017.10.05. • Rios A, Adams DJ. Specific congenital metabolic diseases. In: McInerny TK, Adam HM, Campbell DE, DeWitt TG, Foy JM, Kamat DM, eds. American Academy of Pediatrics Textbook of Pediatric Care. 2nd ed. Elk Grove Village, IL: American Academy of Pediatrics; 2017:938-963. Pediatric Care Online. • Thompson-Branch A, Havranek T. Neonatal hypoglycemia. Pediatr Rev. 2017;38(4):147-157. doi:10.1542/pir.2016-0063. • Thornton PS, Stanley CA, De Leon DD, et al; Pediatric Endocrine Society. Recommendations from the Pediatric Endocrine Society for evaluation and management of persistent hypoglycemia in neonates, infants, and children. J Pediatr. 2015;167(2):238-245. doi:10.1016/j.jpeds.2015.03.057.

American academy of pediatrics

758

PREP ® Self-Assessment

PREPSA 2020

Question 211 A 16-year-old adolescent boy is brought to the clinic for follow up after a recent diagnosis of Salmonella gastroenteritis. He was treated at home with supportive care including ondansetron for nausea and oral rehydration for diarrhea. The diarrhea and nausea have resolved. He is afebrile and has been eating and drinking well. He reports right knee pain that began 2 days ago. His vital signs are within normal limits. Both eyes are mildly injected with associated discharge, which the boy states started yesterday. His right knee examination is significant for a small effusion, mild warmth, and pain with passive range of motion. The remainder of his physical examination, including examination of his other joints, has normal findings. Of the following, the BEST next step in treatment is A. amoxicillin B. ceftriaxone C. ibuprofen D. prednisone

American academy of pediatrics

759

PREP ® Self-Assessment

PREPSA 2020

Correct Answer: C The adolescent boy in this vignette has reactive arthritis, which is treated supportively. Of the response choices, ibuprofen is the best next step in his management. Reactive arthritis usually affects large joints in an asymmetric pattern with pain, limited range of motion, limping, or refusal to walk. It can involve one or multiple joints. Other common associated symptoms include conjunctivitis and urethritis. The triad of arthritis, conjunctivitis, and urethritis is referred to as Reiter syndrome. Additional symptoms can include fever, axial spine tenderness, and corneal ulcerations. Reactive arthritis is more common in adults but can occur in children. It is more common in males with a 3:1 predominance. The pathophysiology is not completely understood, but symptoms typically occur days to 6 weeks after an infection of the genitourinary, gastrointestinal, or upper respiratory system. In the United States, Chlamydia trachomatis is the most commonly associated organism. Other known associated organisms include: Neisseria gonorrhoeae, Shigella, Salmonella, Yersinia, Campylobacter, and Streptococcus pyogenes. Approximately 75% of individuals with reactive arthritis have HLA-B27 and are more likely to develop more severe symptoms. Management is primarily supportive with nonsteroidal anti-inflammatory medications, such as ibuprofen, cold packs to the affected joints, and rest. Prednisone is not used to treat this condition unless it becomes chronic. Antibiotics such as amoxicillin or ceftriaxone are used only if indicated for the underlying infection. The patient in this vignette has improved from his gastrointestinal infection with Salmonella and does not require antibiotics. PREP Pearls • Reactive arthritis requires symptomatic treatment. • Reactive arthritis usually occurs a few days to 6 weeks after an infection. • Reiter syndrome consists of postinfectious arthritis, conjunctivitis, and urethritis. ABP Content Specifications(s) • Recognize the clinical findings associated with reactive arthritis and manage appropriately • Identify illnesses commonly associated with postinfectious arthritis • Recognize the clinical findings associated with postinfectious arthritis Suggested Readings • John J, Chandran L. Arthritis in children and adolescents. Pediatric Rev. 2011;32(11):470-480. doi:10.1542/pir.32-11-470. • Weiss PF, Colbert RA. Reactive and postinfectious arthritis. In: Kliegman RM, St Geme JW III, Blum NJ, Shah SS, Tasker RC, Wilson KM, eds. Nelson Textbook of Pediatrics. 21st ed. Philadelphia, PA: Elsevier; 2020:1272-1273.

American academy of pediatrics

760

PREP ® Self-Assessment

PREPSA 2020

Question 212 A 37-week gestation neonate has undergone intubation in the delivery room for poor respiratory effort. He is limp and unresponsive. Endotracheal tube placement has been confirmed with a carbon dioxide detector and adequate chest rise. Heart rate by auscultation is estimated to be 50 beats/min. A pulse oximeter has been placed on the right hand. Of the following, the BEST indication for initiation of chest compressions for this neonate is A. heart rate less than 60 beats/min B. heart rate less than 75 beats/min C. right hand oxygen saturation less than 40% D. severe truncal hypotonia

American academy of pediatrics

761

PREP ® Self-Assessment

PREPSA 2020

Correct Answer: A For most neonates, once the lungs are inflated after delivery, pulmonary vascular resistance drops rapidly, resulting in increased blood return to the left ventricle and systemic circulation. Most commonly, resuscitation after birth involves supporting respiration. If a neonate does not breathe immediately after delivery, initial steps are to warm, dry, and stimulate. Warming consists of placing the neonate skin to skin on the mother’s abdomen with a dry blanket on top of the neonate. Alternatively, the neonate may be placed under a radiant warmer. Dry towels can be used to remove moisture, which also provides stimulation. If these efforts are unsuccessful, and the heart rate remains less than 100 beats/min and/or the neonate is apneic or gasping, positive pressure ventilation (PPV) should be initiated. Ideally, PPV should be performed using a T-piece connector. A self-inflating or flow-inflating bag may be used as well. While providing PPV, close attention should be paid to changes in lung compliance. Pulse oximetry should be used to monitor heart rate and oxygen saturation when providing PPV. If the heart rate remains below 100 beats/min with adequate chest rise, the provider should perform steps using the Neonatal Resuscitation Program (NRP) algorithm mnemonic, MRSOPA: M-adjust the mask, R-reposition the head, S-suction the mouth then nose, O-open the mouth and lift jaw forward, P-gradually increase the pressure, A-consider airway alternative (endotracheal tube). If the heart rate remains less than 60 beats/min despite adequate ventilation, chest compressions should be initiated. Chest compressions should be given using the 2-thumb technique with the hands encircling the chest. Compressions should be at a depth of one-third of the chest and given at a 3:1 ratio with assisted breaths. No neonatal resuscitation actions should be based solely on oxygen saturation values. Although poor muscle tone is used in the initial assessment of a neonate at the time of delivery, severe truncal hypotonia in the absence of apnea and bradycardia is not an indication for chest compressions. PREP Pearls • The initial steps of neonatal resuscitation include warming, drying, and stimulating. • For neonates whose heart rate remains below 100 beats/min with poor respiratory effort after initial resuscitation steps, positive pressure ventilation should be initiated. • If the heart rate is below 60 beats/min despite adequate ventilation (assessed with chest rise), chest compressions should be started. ABP Content Specifications(s) • Recognize the indications for external cardiac massage during resuscitation of a newborn infant, and institute appropriately

American academy of pediatrics

762

PREP ® Self-Assessment

PREPSA 2020

Suggested Readings • Brady JM, Kamath-Rayne BD. Neonatal resuscitation and delivery room emergencies. In: Kliegman RM, St Geme JW III, Blum NJ, Shah SS, Tasker RC, Wilson KM, eds. Nelson Textbook of Pediatrics. 21st ed. Philadelphia, PA: Elsevier; 2020:925-929. • Pinheiro JMB. Assessment and stabilization at delivery. In: McInerny TK, Adam HM, Campbell DE, DeWitt TG, Foy JM, Kamat DM, eds. American Academy of Pediatrics Textbook of Pediatric Care. 2nd ed. Elk Grove Village, IL: American Academy of Pediatrics; 2017:987-1001. Pediatric Care Online. • Wyckoff MH, Aziz K, Escobedo MB, et al. Part 13: Neonatal resuscitation: 2015 American Heart Association guidelines. update for cardiopulmonary resuscitation and emergency cardiovascular care. Pediatrics. 2015;136:S196. doi:10.1542/peds.20153373G.

American academy of pediatrics

763

PREP ® Self-Assessment

PREPSA 2020

Question 213 A 7-year-old boy is being evaluated for concerns about his development. His mother reports that he has always been delayed in achieving developmental milestones. He spoke his first words and walked at age 2 years, began to speak in sentences at age 4 years, and was toilet trained at age 5½ years. He is in a special education classroom for the majority of the school day. His mother inquires about the severity of her son’s condition and what support he may require as an adult. His most recent psychoeducational test result is reviewed. Of the following, the assessment that would BEST provide the information needed to address the mother’s question is A. achievement B. adaptive function C. intelligence D. processing speed

American academy of pediatrics

764

PREP ® Self-Assessment

PREPSA 2020

Correct Answer: B Psychoeducational assessments use various measures to obtain a picture of a child’s learning. Achievement tests measure academic performance in areas such as mathematics, written language, and reading. Intelligence tests measure cognition and problem-solving skills. Tests of processing speed measure the time it takes to absorb, understand, and use the information presented. Adaptive functioning tests assess an individual’s ability to perform tasks needed for communication, daily living, and socialization. The level of adaptive functioning determines severity of impairment and is correlated with the amount of support that a person needs in the future. A measure of adaptive functioning would best address this mother’s question. Adaptive skills allow for self-sufficiency and independence in everyday living. They involve a child’s abilities around personal care, health, safety, interpersonal relationships, home and community living, vocation, and recreation. Domains of adaptive functioning include conceptual (eg, reasoning, knowledge), social (eg, social judgment, communication skills), and practical (eg, self-care, work skills). Commonly used standardized measures of adaptive behaviors include the Vineland Adaptive Behavior Scales (VABS) and the Adaptive Behavior Assessment System (ABAS). These rating scales gather information from parents, caregivers, and teachers. The results can be used to assist with diagnosis, identify the child’s strengths and weaknesses, monitor progress, and develop treatment plans. Real-life functioning is not always aligned with intellectual ability. Intellectual disability is defined by significantly below-age-level expectations in both cognitive and adaptive functioning. The level of adaptive functioning determines the severity of impairment and is related to the amount of care and supervision that a person needs in the future. Children who maintain mild deficits in adaptive functioning as adults are typically autonomous in their self-care and may live and work self-sufficiently with possible need for intermittent support. Those with moderate adaptive deficits often need prompts for their activities of daily living and require supportive living and supervised work situations. Those with severe adaptive deficits will need assistance for their personal care and activities of daily living. Understanding information about adaptive functioning can assist primary care providers in anticipating the potential needs of their patients and in providing guidance to their families. PREP Pearls • Adaptive skills allow for self-sufficiency and independence in everyday living, and involve abilities around personal care, health, safety, interpersonal relationships, home and community living, vocation, and recreation. • Real-life functioning is not always aligned with intellectual ability. The level of adaptive functioning determines the severity of impairment and is correlated with the amount of support that a person needs in the future. ABP Content Specifications(s) • Understand the utility of an adaptive behavioral assessment

American academy of pediatrics

765

PREP ® Self-Assessment

PREPSA 2020

Suggested Readings • American Psychiatric Association. Diagnostic and Statistical Manual of Mental Disorders, Fifth Edition. Arlington, VA: American Psychiatric Association; 2013. • Phelps RA, Cohen WI. Intellectual disability. In: McInerny TK, Adam HM, Campbell DE, DeWitt TG, Foy JM, Kamat DM, eds. American Academy of Pediatrics Textbook of Pediatric Care. 2nd ed. Elk Grove Village, IL: American Academy of Pediatrics; 2017:2208-2217. Pediatric Care Online. • Purugganan O. Intellectual disabilities. Pediatr Rev. 2018;39(6):299-309. doi:10.1542/pir.2016-0116. • Shea SE. Intellectual disability (mental retardation). Pediatr Rev. 2012;33(3):110-121. doi:10.1542/pir.33-3-110.

American academy of pediatrics

766

PREP ® Self-Assessment

PREPSA 2020

Question 214 A group of first-year medical students is attending a lecture on adolescent pregnancy. They review the case of a 16-year-old sexually active adolescent girl in foster care who has not had her menstrual period in 3 months. She is in the 10th grade and attends a school that teaches "abstinence-only" in health class. The result of her pregnancy test is positive, and she is excited about the news. The students are asked to brainstorm about the socioeconomic problems associated with adolescent pregnancy, and formulate a plan to provide assistance to the patient described in the case. Of the following, the MOST likely consequence for an adolescent girl in this situation is that she will A. B. C. D.

drop out of high school to take care of her child experience premature labor because of complications from lack of prenatal care graduate from high school a year off schedule and then go to college marry the father of her child at a young age, but be unable to sustain a long-term relationship

American academy of pediatrics

767

PREP ® Self-Assessment

PREPSA 2020

Correct Answer: A The pregnant adolescent girl in the vignette is most likely to drop out of high school to care for her child. Data from the National Center for Health Statistics show a 7% decline in the adolescent birth rate in the United States from 2016 to 2017, to 18.8 births per 1,000 girls of age 15 to 19 years; there has been a 55% decrease since 2007. The reason for this decline is thought to be related to several factors: a decrease in the number of sexually active adolescent girls, increased use of birth control, and increased access to more effective forms of birth control (long-acting reversible contraceptive methods such as the implant and intrauterine devices). Sixty percent of adolescent mothers do not complete high school; only 2% complete college by age 30 years. Adolescent mothers are also more likely to have repeated unwanted pregnancies. Adolescents with good access to prenatal care during their pregnancy have no greater risk for medical complications than adult women with the same socioeconomic status. Adolescents who do not receive prenatal care are at increased risk of delivering preterm and low-birthweight infants. This outcome is related to social determinants of health, which are age and the conditions in which people are born, grow, work, and live that affect various outcomes. Aspects of an individual’s life affected by social determinants include access to health care, clean water, and stable housing; food security; income level; quality of education/literacy; and safety. The prevention of adolescent pregnancy starts with good communication. Parents, pediatricians, and other health care providers should begin discussions about pubertal changes and sexuality in late childhood and early adolescence. Waiting to start these conversations until adolescence is too late. A sex education curriculum can be an important tool in providing adequate information to adolescents. However, only 24 states and the District of Columbia mandate sex education in schools. Three types of curricula are taught: abstinence-only until marriage, abstinence-plus, and comprehensive sex education. "Abstinence-only" focuses on the negative consequences of having sexual relations outside a marital relationship, and omits education and discussion of abortion and contraception. "Abstinence-plus" education strongly promotes abstinence, but also includes information on contraception. Lastly, "comprehensive" sex education promotes sexuality as a normal healthy part of life. The comprehensive curriculum stresses that abstinence is the most effective way to prevent pregnancy, but also delivers positive messages about sexuality, and accurate information about contraception, condom use, and sexually transmitted infections, including HIV. Adolescents are also at decreased risk of becoming an adolescent parent if they are involved in volunteer activities that serve their communities; participate in sports or after-school clubs; or have part-time jobs. The likelihood that the pregnant adolescent in the vignette will return to high school, graduate a year off cycle, and go to college would be contingent on her support systems and financial stability. Adolescent pregnancy rarely leads to marriage to the father. American academy of pediatrics

768

PREP ® Self-Assessment

PREPSA 2020

PREP Pearls • Adolescent pregnancy is declining in the United States likely because of a decrease in the number of sexually active female adolescents, more consistent use of birth control, and better access to effective methods of birth control. • Prevention of adolescent pregnancy includes good communication, education, and engaging adolescents in activities that promote becoming productive members of society. • Social determinants of health play a significant role in adolescent pregnancy. ABP Content Specifications(s) • Plan ways to improve the health and outcome of pregnant adolescents • Understand the socioeconomic and educational problems associated with pregnancy during adolescence • Recognize the age-related risks of pregnancy complications, including associated mortality, in adolescents Suggested Readings • Berman RS, Patel MR, Belamarich PF, Gross RS. Screening for poverty and povertyrelated social determinants of health. Pediatr Rev. 2018;39(5):235-246. doi:10.1542/pir.2017-0123. • Committee on Adolescent Health Care. Committee opinion no. 699: adolescent pregnancy, contraception, and sexual activity. Obstet Gynecol. 2017;129(5):e142-e149. doi:10.1097/AOG.0000000000002045. • Cox JE. Teen pregnancy. In: Emans SJ, Laufer MR, eds. Emans, Laufer, Goldstein’s Pediatric and Adolescent Gynecology. 6th ed. Philadelphia, PA: Lippincott Williams & Wilkins; 2012:474-484. • Elfenbein DS, Felice ME. Adolescent pregnancy and parenthood. In: McInerny TK, Adam HM, Campbell DE, DeWitt TG, Foy JM, Kamat DM, eds. American Academy of Pediatrics Textbook of Pediatric Care. 2nd ed. Elk Grove Village, IL: American Academy of Pediatrics; 2017:1159-1164. Pediatric Care Online. • Martin JA, Hamilton BE, Osterman MJK. Births in the United States, 2017. NCHS data brief, no. 318. Hyattsville, MD: National Center for Health Statistics; 2018. https://www.cdc.gov/nchs/products/databriefs/db318.htm.

American academy of pediatrics

769

PREP ® Self-Assessment

PREPSA 2020

Question 215 A 3-year-old boy has been in the pediatric intensive unit since sustaining a severe hypoxicischemic brain injury after drowning in the family pool 3 weeks ago. He was pulseless when emergency medical services arrived. Return of spontaneous circulation was achieved after 30 minutes of resuscitation. Over the following 3 weeks, his hemodynamics, oxygenation, and ventilation have improved, but his neurologic prognosis has remained poor. He was extubated twice, but required re-intubation because of hypopnea and hypoxia. He is not currently on any cardiac medications or sedatives, and he is on low ventilator settings. He is tolerating feeds through a nasogastric tube. He is noncognitive. He occasionally opens his eyes spontaneously but does not appear to recognize his parents. He does not respond to verbal stimuli, but responds to painful stimuli with flexor posturing. He occasionally breathes over the ventilator. Cough and gag reflexes are absent. The medical team, including the palliative care service, has had extensive discussions with his parents since early in his admission and has described his likely neurologic prognosis as a persistent vegetative state. However, his parents are convinced that they have seen him communicate and smile in the past few days. They believe that a miracle will occur and that he will eventually wake up, but he needs more time. The medical team has informed them that in order to sustain his life, he would require a tracheostomy and a gastrostomy (G-tube), need constant care, and remain dependent on technology, without neurologic recovery. In response, the parents have expressed their wish to prolong his life and have identified numerous caretakers within the family. Of the following, the BEST next step is to A. consult the hospital ethics committee B. plan tracheostomy and gastrostomy C. transfer care to another facility for a second opinion D. withdraw life-sustaining therapies

American academy of pediatrics

770

PREP ® Self-Assessment

PREPSA 2020

Correct Answer: B The boy in this vignette has a hypoxic-ischemic injury that carries a poor neurologic prognosis. There have been extensive discussions with the family regarding the likelihood that he will remain in a persistent vegetative state, yet they wish to prolong his life, despite the need for a tracheostomy and a gastrostomy (G-tube). Therefore, the best next step would be to proceed with the tracheostomy and G-tube placement. Ethical decision-making regarding end-of-life care should center on the best interest of the patient. Determining what is in the best interest of a critically ill child can be subjective and vary due to many patient-specific factors. The 4 principles of biomedical ethics, autonomy, justice, nonmaleficence, and beneficence, can guide families and practitioners through end-of-life decisions. Autonomy is the right of self-determination. Since most children have not reached the developmental stage or chronological age to provide informed consent, the power to make medical decisions generally rests with parents or other designated caregivers. For the boy in this vignette, autonomy would be upheld by allowing his parents to make medical decisions regarding his care, which may include surgical procedures or withdrawal of life-sustaining therapies. Justice is the right of each individual in a society to receive equal distribution of resources. For the boy in this vignette, justice would be violated if the boy were denied treatments that might be granted to others based on his race, religion, or socioeconomic status. Nonmaleficence is the principle that one should do no harm, and beneficence is the concept that one should do good. Decision-makers and caregivers should balance the risks and benefits of particular therapies that are individualized to the patient and the situation. There is often disagreement between caregivers and medical professionals regarding the balance of harm and potential rewards in endof-life treatments. Indeed, there can be disagreements among groups of family members and among medical practitioners in an individual case. Some medical providers and laypeople believe the essence of the human experience consists of the ability to perceive and interact with the environment. These individuals may consider the preservation of life in a persistent vegetative state, as for the boy in this vignette, to be futile care. It would be acceptable to withdraw life-sustaining therapies in this setting if the family agrees it would be in the boy’s best interest. It would also be reasonable for the medical team to offer recommendations regarding end-of-life decisions. If the family does not agree to withdrawal of life-sustaining therapies, establishing an advance directive of care plan, also known as a “do not resuscitate” (DNR) order, may benefit the patient. Emergent lifesaving interventions, such as cardiopulmonary resuscitation, intubation, or emergent medications could be limited with a DNR order. Some families do not wish to be burdened with the decision to remove life-sustaining therapies, but would agree with not pursuing active measures to restore breathing or circulation. If sustaining life long-term is considered, harm could occur to the boy in this vignette due to consequences of the surgical procedures, as well as from complications of mechanical ventilation, permanent indwelling devices, immobility, and burdens to the caregivers, all of which could be prevented by forgoing life-sustaining therapy. However, for some families, these potential sources of harm are outweighed by the harm of death. Value judgments and subjectivity American academy of pediatrics

771

PREP ® Self-Assessment

PREPSA 2020

of caregivers and medical practitioners regarding quality of life are inevitable. Frequent, familycentered, multidisciplinary communication is helpful in making decisions. A realistic, objective prognosis should be established. Details of home life while caring for a child continuously dependent on technology should be discussed. The family of the child in this vignette has received clear communication regarding his poor neurologic prognosis and what will be required to sustain his life. Since they have determined that the benefits of further treatment outweigh the harm, it should be undertaken. Specialists from services who will be involved in his long-term care should discuss his care with the family. For the child in this vignette, these services may include otolaryngology, social work, primary care, palliative care, pulmonology, and physical therapy. More than one permanent caregiver should be trained in basic cardiopulmonary resuscitation, as well as on the use of all medical devices and equipment (eg, how to administer medications and feedings and how to emergently change the tracheostomy tube). For the boy in this vignette, withdrawing life-sustaining therapies against the parents wishes would be a violation of autonomy. The opinion of a hospital ethics committee or a transfer to another facility for a second opinion could be pursued, but these measures would merely result in the provision of more value judgments on the boy’s quality of life from people not involved in his long-term care. PREP Pearls • The biomedical principles of autonomy, nonmaleficence, beneficence, and justice can be used to guide decision-making in end-of-life care. • If caregivers of children with poor prognosis do not wish to withdraw life-sustaining therapies, establishing an advance directive of care plan, also known as a “do not resuscitate (DNR)” order, may benefit the patient. • Since most children have not reached the developmental stage or chronological age to provide informed consent, the power to make medical decisions generally rests with parents or other designated caregivers. ABP Content Specifications(s) • Recognize and apply ethical principles when involved in decisions to withdraw/withhold artificial hydration/nutrition • Recognize and apply ethical principles when involved in end-of-life care • Recognize and apply ethical principles involving cardiopulmonary resuscitation and "do not resuscitate" (DNR) orders • Recognize and apply ethical decision-making when caring for critically ill patients • Recognize and apply ethical principles with regard to limitations on medical intervention

American academy of pediatrics

772

PREP ® Self-Assessment

PREPSA 2020

Suggested Readings • Kutzsche S, Partridge JC, Leuthner SR, Lantos JD. When life-sustaining treatment is withdrawn and the patient doesn't die. Pediatrics. 2013;132(5):893-897. doi:10.1542/peds.2013-0413. • Okun AL. Palliative, end-of-life, and bereavement care. In: McInerny TK, Adam HM, Campbell DE, DeWitt TG, Foy JM, Kamat DM, eds. American Academy of Pediatrics Textbook of Pediatric Care. 2nd ed. Elk Grove Village, IL: American Academy of Pediatrics; 2017:555-568. Pediatric Care Online. • Sarnaik AA, Lieh-Lai MW, Sarnaik AP. Drowning. In: Fuhrman BP, Zimmerman JJ, Clark RSB, et al, eds. Pediatric Critical Care. 5th ed. Philadelphia, PA: Elsevier; 2016:1572-1581. • Section on Hospice and Palliative Medicine and Committee on Hospital Care. Pediatric palliative care and hospice care commitments, guidelines, and recommendations. Pediatrics. 2013;132(5):966-972. doi:10.1542/peds.2013-2731. • Weise KL, Okun AL, Carter BS, Christian CW; Committee on Bioethics; Section on Hospice and Palliative Medicine; Committee on Child Abuse and Neglect. Guidance on forgoing life-sustaining medical treatment. Pediatrics. 2017;140(3):e20171905. doi:10.1542/peds.2017-1905.

American academy of pediatrics

773

PREP ® Self-Assessment

PREPSA 2020

Question 216 A 13-year-old girl is being evaluated for recurrent abdominal pain. She reports crampy pain localized to her lower abdomen that is 10 out of 10 in severity. She has associated nausea, but has not had any vomiting, diarrhea, or fever. The pain does not improve with acetaminophen. She has experienced similar cramping approximately once a month over the past 3 months, but her current pain is much more severe. She is premenarchal and has never been sexually active. On physical examination, she has a sexual maturity rating of 4 for breast and pubic hair. There is a palpable mass in the suprapubic region. The girl is nervous and uncooperative with an external genital examination. Of the following, the MOST likely diagnosis for this girl is A. bowel obstruction B. constipation C. dermoid ovarian cyst D. imperforate hymen

American academy of pediatrics

774

PREP ® Self-Assessment

PREPSA 2020

Correct Answer: D The most likely diagnosis for the girl in the vignette is imperforate hymen, which should be considered in any adolescent with cyclical abdominal pain and primary amenorrhea. It is one of the most common obstructive anomalies of the female genital tract. An imperforate hymen can be diagnosed in the neonatal period if a bulge is noticed at the introitus, which is due to vaginal secretions occupying the uterus, stimulated by maternal estrogen levels. If unrecognized, the mucus will be reabsorbed and the child will be asymptomatic until menarche. Imperforate hymen will present during adolescence in the form of primary amenorrhea, cyclical abdominal or pelvic pain, a lower abdominal mass, or a blue discoloration of the hymen because of hematocolpos. It can also present with back pain, or difficulty with urination and defecation because of vaginal distention. The diagnosis of imperforate hymen can be made on physical examination, and can be confirmed with ultrasonography (translabial or transabdominal). The differential diagnosis includes vaginal agenesis, agenesis of the lower third of the vagina, and a transverse vaginal septum. Treatment requires surgical repair of the hymen and removal of obstructing material (blood/uterine tissue). The girl’s presentation is not consistent with the other response choices. Bowel obstruction is often associated with nausea, vomiting, and abdominal distention. Although constipation can present as acute abdominal pain, this is typically colicky in nature and a stool burden is often palpable in the left lower quadrant. Pain associated with a dermoid ovarian cyst would be due to ovarian torsion, which usually presents with sudden onset of pain, nausea, vomiting, and sometimes peritoneal signs. Ovarian torsion is a surgical emergency. PREP Pearls • An imperforate hymen is one of the most common obstructive anomalies of the female genital tract. • The diagnosis of imperforate hymen can be made with external genital physical examination. • Treatment of imperforate hymen requires surgical repair and removal of obstructing material (blood/uterine tissue). ABP Content Specifications(s) • Recognize the clinical findings associated with imperforate hymen

American academy of pediatrics

775

PREP ® Self-Assessment

PREPSA 2020

Suggested Readings • Dinerman LM. Dysmenorrhea. In: McInerny TK, Adam HM, Campbell DE, DeWitt TG, Foy JM, Kamat DM, eds. American Academy of Pediatrics Textbook of Pediatric Care. 2nd ed. Elk Grove Village, IL: American Academy of Pediatrics; 2017:12921295. Pediatric Care Online. • Friedman L, Horwitz NJ, Retterath L, Woolridge D, Adhikari S. Point-of-care ultrasound screening and diagnosis of imperforate hymen in pediatric abdominal pain. Pediatrics. 2018;142(1 meeting abstract):479. http://pediatrics.aappublications.org/content/142/1_MeetingAbstract/479. • Laufer MR. Structural abnormalities of the female reproductive tract. In: Emans SJ, Laufer MR, eds. Emans, Laufer, Goldstein’s Pediatric and Adolescent Gynecology. 6th ed. Philadelphia, PA: Lippincott Williams & Wilkins; 2012:213-219.

American academy of pediatrics

776

PREP ® Self-Assessment

PREPSA 2020

Question 217 A 10-month-old female infant is seen for follow up 2 weeks after a hospital admission for a febrile Escherichia coli urinary tract infection. Ultrasonography had shown left-sided moderate hydronephrosis without ureteral dilatation and a normal right kidney. She completed a 10-day course of cefixime. A voiding cystourethrogram performed after discharge had normal findings. Of the following, the MOST likely cause of hydronephrosis in this infant is A. extrarenal pelvis B. ureterocele C. ureteropelvic junction obstruction D. ureterovesical junction obstruction

American academy of pediatrics

777

PREP ® Self-Assessment

PREPSA 2020

Correct Answer: C The infant in this vignette has unilateral hydronephrosis but a normal voiding cystourethrogram. The absence of ureteral dilation on renal ultrasonography (RUS) favors a proximal obstruction at the ureteropelvic junction (UPJ) and rules out vesicoureteral reflux as the cause of the hydronephrosis. The differential diagnoses in a child with unilateral hydronephrosis include: UPJ obstruction, ureterovesical junction obstruction, ureterocele, ectopic ureter, extrarenal pelvis, unilateral vesicoureteral reflux, polycystic kidney disease, and transient physiologic cause. These conditions can also present as bilateral hydronephrosis. Additional differential diagnoses in a child with bilateral hydronephrosis include: posterior urethral valve, urethral atresia, prune belly syndrome, and megacystis-megaureter syndrome. The most common cause of obstructive uropathy in children is UPJ obstruction. An obstruction to the flow of urine from the renal pelvis to the ureter causes progressive dilation of the renal pelvis and, if untreated, leads to progressive renal damage. As described in the vignette, UPJ obstruction is commonly detected after a febrile urinary tract infection. Ureteropelvic junction obstruction is also suspected when a child presents with a history of antenatal hydronephrosis; it is the second most common cause of antenatal hydronephrosis, with transient physiologic hydronephrosis being the most common cause. The first imaging test to be performed in children suspected to have UPJ obstruction is RUS. As described for the infant in the vignette, RUS findings of hydronephrosis without ureteral dilation favor a UPJ obstruction. Renal ultrasonography is also helpful to differentiate a proximal from a distal cause of obstructive uropathy. Ureterovesical junction obstruction, ureterocele, and an ectopic ureter have ureteral dilation along with hydronephrosis favoring a distal obstruction; thus, these conditions are unlikely for the infant in this vignette. An extrarenal pelvis is in the differential diagnosis of hydronephrosis and refers to the presence of the renal pelvis outside the renal hilum. Extrarenal pelvis is a normal anatomic variant and is not associated with dilation of the renal calyx, dilation of the ureter, or thinning of the renal parenchyma. In children with UPJ obstruction, a renal scan such as 99mTc mercaptoacetyltriglycine (MAG-3) with furosemide is commonly performed. The diuretic renography provides an assessment of differential function of each kidney and also suggests the degree of obstruction. Children with mild to moderate hydronephrosis, preserved relative renal function, and borderline to mild degree of UPJ obstruction may be conservatively managed. However, children with severe hydronephrosis causing thinning of renal parenchyma, decreased relative renal function, or marked degree of obstruction on renal scan require a surgical correction. PREP Pearls • The common differential diagnoses of unilateral hydronephrosis are ureteropelvic junction obstruction, ureterovesical junction obstruction, ureterocele, ectopic ureter, unilateral vesicoureteral reflux, and transient physiologic cause. • Hydronephrosis without ureteral dilation favors a proximal obstruction (ureteropelvic junction) rather than distal obstruction (ureterovesical junction, ureterocele, or ectopic ureter). American academy of pediatrics

778

PREP ® Self-Assessment

PREPSA 2020

ABP Content Specifications(s) • Understand the various causes of urinary tract obstruction • Formulate a differential diagnosis of urinary tract obstruction • Recognize the clinical findings associated with hydronephrosis in patients of various ages Suggested Readings • Carmody JB, Carmody RB. Question from the clinician: management of prenatal hydronephrosis. Pediatr Rev. 2011;32(12):e110-e112. doi:10.1542/pir.32-12-e110. • Nguyen HT. Obstructive uropathy and vesicoureteral reflux. In: McInerny TK, Adam HM, Campbell DE, DeWitt TG, Foy JM, Kamat DM, eds. American Academy of Pediatrics Textbook of Pediatric Care. 2nd ed. Elk Grove Village, IL: American Academy of Pediatrics; 2017:2406-2415. Pediatric Care Online. • Schlomer BJ, Copp HL. Antenatal hydronephrosis. NeoReviews. 2013;14(11):e551-e561. doi:10.1542/neo.14-11-e551.

American academy of pediatrics

779

PREP ® Self-Assessment

PREPSA 2020

Question 218 A 9-year-old boy is being evaluated for vomiting of 2 days’ duration. He has been unable to keep anything down. He has not had loose stools. His urine output is decreased. No rashes have been noted. He feels very tired. The parents state that he was in his usual state of good health until about 2 weeks ago when he had an upper respiratory infection, which resolved without treatment. On physical examination, the boy is afebrile. His heart rate is 140 beats/min, respiratory rate is 35 breaths/min, blood pressure is 90/60 mm Hg, and oxygen saturation is 96%. He is awake, answering questions appropriately and following commands. On pulmonary examination, he is tachypneic and has mildly increased work of breathing, with good aeration throughout. He is tachycardic, with a regular rhythm and a gallop on auscultation. His abdomen is soft and nontender throughout. His liver edge is palpable 3 cm below the costal margin. He is warm peripherally with good pulses. Of the following, the test that will confirm the MOST likely diagnosis for this boy is A. abdominal radiography B. abdominal ultrasonography C. echocardiography D. hepatitis panel

American academy of pediatrics

780

PREP ® Self-Assessment

PREPSA 2020

Correct Answer: C The boy in the vignette has physical examination findings notable for tachycardia, a cardiac gallop, and hepatomegaly, and he has a recent history of a viral process. These findings are consistent with myocarditis. Of the responses, the test most likely to confirm this diagnosis is echocardiography. None of the other tests listed would be indicated at this time. There is significant variability in the way myocarditis can present. Some patients will have a history of viral symptoms. Initial presentation can include respiratory symptoms, gastrointestinal symptoms, palpitations, or chest pain. Symptoms may be nonspecific and a patient’s condition can decompensate rapidly, leading to cardiogenic shock and sudden death. Physical examination findings may include tachypnea, tachycardia, cardiac gallop, or hepatomegaly. Particular attention should be paid if the degree of tachycardia is out of proportion to fever and other clinical features. There is a subset of patients who will have fulminant myocarditis with acute decompensation of myocardial function. The vast majority of cases of myocarditis arise from a viral infection. Pathogens include adenovirus, enterovirus, parvovirus, Epstein-Barr virus, influenza, and human herpesvirus 6. Rarely, myocarditis is caused by bacteria including Mycoplasma pneumoniae, Chlamydophila pneumoniae, Borrelia burgdorferi, Listeria monocytogenes, Clostridium perfringens, Staphylococcus, Streptococcus, Meningococcus, and Corynebacterium diphtheriae. Myocarditis can also be caused by parasites, autoimmune diseases, and hypersensitivity reactions. Diagnostic evaluation often includes chest radiography, which may demonstrate pulmonary edema and/or cardiomegaly. Electrocardiography may show low voltages, rhythm abnormalities, or conduction delays. Echocardiography can show decreased systolic function, pericardial effusion, and/or valvular dysfunction. Laboratory evaluation may demonstrate elevated markers of inflammation, troponin, and/or brain natriuretic peptide. An infectious disease workup may reveal an inciting pathogen. Additional testing may ultimately include cardiac magnetic resonance imaging or an endomyocardial biopsy. Treatment is largely supportive and may include diuretics, inotropes, and respiratory support. Children who cannot be well-supported with these therapies need to be treated at a center with mechanical support capabilities. Intravenous immunoglobulin and steroids are sometimes used for treatment, but this is controversial because the literature does not currently demonstrate an evidence-based benefit. PREP Pearls • Myocarditis may present with chest pain, palpitations, respiratory symptoms, or gastrointestinal symptoms; clinical condition can decompensate rapidly, leading to cardiogenic shock and death. • Myocarditis is primarily caused by viral pathogens. Diagnostic evaluation of myocarditis includes chest radiography, electrocardiography, echocardiography, and laboratory evaluation (markers of inflammation, troponin, and/or brain natriuretic peptide).

American academy of pediatrics

781

PREP ® Self-Assessment

PREPSA 2020

ABP Content Specifications(s) • Recognize the clinical findings associated with myocarditis • Recognize pathogens commonly associated with myocarditis • Plan an appropriate diagnostic evaluation of myocarditis Suggested Readings • Hohn AR, Stanton RE. Myocarditis in children. Pediatr Rev. 1987;9(3):83-88. doi:10.1542/pir.9-3-83. • McCulloch MA, Gajarski RJ. Congenital and acquired heart disease. In: McInerny TK, Adam HM, Campbell DE, DeWitt TG, Foy JM, Kamat DM, eds. American Academy of Pediatrics Textbook of Pediatric Care. 2nd ed. Elk Grove Village, IL: American Academy of Pediatrics; 2017:1883-1917. Pediatric Care Online. • Petit MA, Koyfman A, Foran M. Myocarditis. Pediatr Emerg Care. 2014;30(11):832835. doi:10.1097/PEC.0000000000000272. • Simpson KE, Canter CE. Acute myocarditis in children. Expert Rev Cardiovasc Ther. 2011;9(6):771-783. doi:10.1586/erc.11.70.

American academy of pediatrics

782

PREP ® Self-Assessment

PREPSA 2020

Question 219 A 16-year-old sexually active girl is seen at the school-based health clinic. She is concerned that she might have genital herpes, though she has never been diagnosed with herpes simplex virus. At her recent health supervision visit 2 months ago, screening for Neisseria gonorrhoeae, Chlamydia trachomatis, syphilis, and HIV were all negative. She had coitarche at age 16 years, and is currently receiving medroxyprogesterone injections every 3 months as a birth control method. She has had 1 lifetime male partner whom she has been dating for the past year. They use condoms intermittently. Genital lesions are noted (Item Q219).

Of the following, the MOST likely diagnosis for this girl is A. condyloma acuminata B. condyloma lata C. genital herpes simplex virus D. molluscum contagiosum

American academy of pediatrics

783

PREP ® Self-Assessment

PREPSA 2020

Correct Answer: A The most likely diagnosis for the girl in the vignette is condyloma acuminata, which is caused by human papillomavirus (HPV). Human papillomavirus is the most common sexually transmitted infection in the United States. According to the Centers for Disease Control and Prevention, there are approximately 14 million new cases of HPV per year and 4,000 deaths per year in women with cervical cancer related to HPV. There are approximately 120 serotypes of HPV, and at least 40 are associated with infection in the genital tract. Almost all sexually active people have acquired HPV at some point in their lifetime, but may not be aware because they are asymptomatic. The strains of HPV are divided into categories of high risk (oncogenic) and low risk (nononcogenic). Risk factors for acquiring HPV include: 1) multiple sexual partners; 2) having a partner who has had multiple sexual partners; 3) early sexual debut; and 4) inconsistent condom use. Human papillomavirus can affect the anogenital area as well as the mouth and throat. Infection with high-risk strains of HPV can cause changes in cervical cells, which may lead to abnormal results on a Papanicolaou smear (cervical cancer screen). However, most cellular changes induced by HPV are transient. Current cervical cancer screening guidelines recommend that screening begin at age 21 years, which is older than previous recommendations. This change was made in an effort to reduce identification of low-grade lesions that will most likely self-resolve with no significant health consequences, reduce unnecessary anxiety in patients, and avoid the costs and risks of unnecessary diagnostic and therapeutic medical procedures such as colposcopy and loop electrosurgical excision procedure. Infections with the low-risk HPV strains tend to cause genital warts (condyloma acuminata) in boys and girls. Boys develop lesions on their penis, scrotum, and perianal area; less commonly, boys can have lesions in the urethral meatus. Girls develop lesions on the vulva, perineum, and perianal area, and less commonly, in the vagina or cervix. The lesions are usually flesh colored and described as cauliflower-like (Item C219A), painless, and nonpruritic. The differential diagnosis of genital warts includes molluscum contagiosum, condyloma lata associated with secondary syphilis, pearly penile papules (boys), and vestibular papillae (girls).

Item C219A: Condylomas prior to imiquimod treatment.

American academy of pediatrics

784

PREP ® Self-Assessment

PREPSA 2020

Reprinted with permission from Hatch KD. Clinical manifestations of HPV infection. Section A: benign manifestations of HPV infection. A1: anogenital condylomas. Int J Gynaecol Oncol. 2006;94(suppl 1):S34.

Prepubertal children can present with condylomata acuminata. There is no consensus regarding whether genital or anal warts are specific for sexual abuse. The presence of warts should always prompt the provider to inquire about sexual abuse. Children can develop condyloma acuminata from vertical transmission. Genital warts may spontaneously resolve. Treatment can be patient or physician applied. Of the treatment options noted in Item C219B, none are considered more effective than the others. With treatment, warts usually resolve within 3 months.Treatment does not eradicate the virus, and the genital warts may return. As a preventive measure, the 9-valent HPV vaccine is approved for use in the United States in males and females between the ages of 9 and 45 years. The vaccine has been shown to prevent genital warts and cancers associated with HPV strains 6, 11, 16, 18, 31, 33, 45, 52, and 58. For boys and girls between 9 and 14 years of age, a 2-dose regimen is recommended, with the second dose given 6 to 12 months after the first. For men and women between 15 and 45 years of age, a 3-dose regimen is recommended, with doses repeated at 1 to 2 months, and 6 months after the first.

American academy of pediatrics

785

PREP ® Self-Assessment

PREPSA 2020

The physical examination findings for the girl in the vignette are not consistent with the other response diagnoses. Condyloma lata are smooth flat warts that are associated with secondary syphilis. Genital herpes simplex virus generally presents as painful vesicles or ulcers. Molluscum contagiosum are pearly lesions with a central umbilication. American academy of pediatrics

786

PREP ® Self-Assessment

PREPSA 2020

PREP Pearls • Human papillomavirus is the most common sexually transmitted infection in the United States. • Almost all sexually active people acquire human papillomavirus at some point in their lifetime. • The 9-valent human papillomavirus vaccine is approved for use in the United States in males and females between the ages of 9 and 26 years, and was recently approved for men and women between 27 and 45 years, to prevent cancers caused by high-risk human papillomavirus strains. ABP Content Specifications(s) • Recognize the clinical findings associated with condylomata acuminata • Understand the significance of condylomata acuminata in patients of various ages, including their association with sexual abuse • Plan the appropriate management of condylomata acuminata Suggested Readings • Eliscu A. Human papillomavirus and HPV vaccines. Pediatr Rev. 2017;38(9):443-445. doi:10.1542/pir.2016-0018. • Kaskowitz A, Quint E. A practical overview of managing adolescent gynecologic conditions in the pediatric office. Pediatr Rev. 2014:35(9):371-381. doi:10.1542/pir.35-9371. • Workowski KA, Bolan, GA. Sexually transmitted diseases treatment guidelines, 2015. MMWR Recomm Rep. 2015;64(3):8493. https://www.cdc.gov/mmwr/preview/mmwrhtml/rr6403a1.htm?s_cid=rr6403a1.

American academy of pediatrics

787

PREP ® Self-Assessment

PREPSA 2020

Question 220 A 6-year-old boy with ear drainage is brought to the otolaryngology clinic. Four weeks ago he underwent bilateral tympanostomy tube placement for recurrent otitis media. For the last 2 weeks, he has had persistent foul-smelling drainage from the right ear. He has a temperature of 37.2°C, blood pressure of 100/60 mm Hg, heart rate of 90 beats/min, and respiratory rate of 22 breaths/min. There is purulent drainage in the right ear canal, and the right tympanic membrane cannot be visualized. A patent tympanostomy tube is seen in the left tympanic membrane. Of the following, the BEST management of this boy’s infection is A. oral ciprofloxacin B. oral clindamycin C. topical neomycin D. topical ofloxacin

American academy of pediatrics

788

PREP ® Self-Assessment

PREPSA 2020

Correct Answer: D The best management for the infection of the boy in this vignette is topical ofloxacin. Certain pathogens, including Pseudomonas aeruginosa and Staphylococcus aureus, must be considered as the causative agents of tympanostomy tube otorrhea in older children when water penetration could have contributed to developing otorrhea. Thus, empiric therapy should have broadspectrum activity. For tympanostomy tube otorrhea, topical therapy is more efficacious than systemic therapy. Therefore, of the agents and routes presented as therapeutic options, topical ofloxacin is preferred. The clinical manifestations of Pseudomonas infections are varied as many clinical syndromes have been associated with this pathogen. Infections associated with Pseudomonas species include chronic otitis media, hot tub folliculitis, foot osteomyelitis associated with nail puncture trauma, and pneumonia in patients with cystic fibrosis. Additionally, nosocomial infections associated with Pseudomonas species include superinfection of burn wounds, ventilatorassociated pneumonia, catheter-associated bloodstream infections, catheter-associated urinary tract infections, and surgical site infections. In immunocompromised hosts, bacteremia can lead to ecthyma gangrenosum, skin lesions that evolve from hemorrhagic pustules to black eschars (Item C220).

Item C220: Ecthyma gangrenosum. Reprinted with permission from Kimberlin DW, et al, eds. Red Book Online. Itasca, IL: American Academy of Pediatrics; 2018.

The fluoroquinolones are the only antibiotic class that offer oral options for treatment of Pseudomonas infections. Other parenteral drugs with activity include higher-generation cephalosporins, such as ceftazidime and cefepime, and β-lactamase inhibitor combinations, such as piperacillin-tazobactam, carbapenems, and aminoglycosides. However, Pseudomonas species can have multiple mechanisms of drug resistance, and therapy should be guided by results of antimicrobial susceptibilities. American academy of pediatrics

789

PREP ® Self-Assessment

PREPSA 2020

Oral ciprofloxacin has broad-spectrum activity, however, topical therapy is preferred for tympanostomy tube otorrhea. Oral clindamycin has activity against susceptible S aureus but not Pseudomonas species. Neomycin is an aminoglycoside antibiotic that has a narrower spectrum of antimicrobial activity compared to fluoroquinolone antibiotics and more potential for adverse effects including ototoxicity and contact dermatitis. Thus, neomycin is not recommended for tympanostomy tube otorrhea. PREP Pearls • Pseudomonas infections include tympanostomy tube otorrhea, chronic otitis media, hot tub folliculitis, foot osteomyelitis associated with nail puncture trauma, pneumonia in patients with cystic fibrosis, and multiple nosocomial infections. • Although select β-lactam antibiotics, fluoroquinolones, and aminoglycosides can have activity against Pseudomonas species, therapy should be guided by results of antimicrobial susceptibilities given the potential for drug resistance. • Topical ofloxacin is the treatment of choice for tympanostomy tube otorrhea. ABP Content Specifications(s) • Recognize the clinical manifestations of pseudomonal infections and manage appropriately • Recognize the risk factors for the development of pseudomonal infections Suggested Readings • American Academy of Pediatrics. Serious bacterial infections caused by Enterobacteriaceae. In: Kimberlin DW, Brady MT, Jackson MA, Long SS, eds. Red Book: 2018 Report of the Committee on Infectious Diseases. 31st ed. Itasca, IL: American Academy of Pediatrics; 2018:328331. https://redbook.solutions.aap.org/chapter.aspx?sectionId=189640085&bookId=2205 &resultClick=1. • Light M. Pneumonia. In: McInerny TK, Adam HM, Campbell DE, DeWitt TG, Foy JM, Kamat DM, eds. American Academy of Pediatrics Textbook of Pediatric Care. 2nd ed. Elk Grove Village, IL: American Academy of Pediatrics; 2017:2510-2522. Pediatric Care Online. • Steele D, Adam G, Di M, Halladay C, Balk E, Trikalinos T. Prevention and treatment of tympanostomy tube otorrhea: a meta-analysis. Pediatrics. 2017;139(6):e20170667. doi:10.1542/peds.2017-0125.

American academy of pediatrics

790

PREP ® Self-Assessment

PREPSA 2020

Question 221 A 5-year-old girl has a history of repeated otitis, sinusitis, bronchitis, and pneumonia. She was born at term via an uncomplicated vaginal delivery, and had unexplained perinatal respiratory distress. She has a daily productive cough but is otherwise generally well. The girl has maintained a normal growth pattern, and participates in usual childhood activities. On physical examination, pulse oximetry is 99% in room air. She is well-appearing. Tympanic membranes are scarred, with tympanostomy tubes present bilaterally. Chest examination reveals equal airflow bilaterally with transient localized crackles over the right midlung field; her heart has a regular rate and rhythm and point of maximal impulse in the left midclavicular line. There is no digital clubbing. The remainder of the examination findings are unremarkable. Chest radiography (Item Q221A and Item Q221B) shows a right middle lobe segmental atelectasis and mild bronchiectasis. Complete blood cell count with differential is normal. Sweat chloride determination is 23 mMol/L.

Item Q221A: Anteroposterior chest radiograph. Courtesy of M. Guill

American academy of pediatrics

791

PREP ® Self-Assessment

PREPSA 2020

Item Q221B: Lateral chest radiograph. Courtesy of M. Guill

Of the following, the test MOST likely to elucidate the cause of this child’s signs and symptoms is A. exhaled nasal nitric oxide B. genetic screen for cystic fibrosis C. qualitative immunoglobulin assay D. thoracoscopic lung biopsy

American academy of pediatrics

792

PREP ® Self-Assessment

PREPSA 2020

Correct Answer: A The girl in the vignette has recurrent sinopulmonary infections and a history of unexplained neonatal respiratory distress, together a hallmark of primary ciliary dyskinesia (PCD). Based on the 2018 Clinical Practice Guideline for diagnosis of PCD , the preferred test in a patient five years of age or older with at least two of the four key clinical features and for whom cystic fibrosis (CF) has been ruled out, is measurement of exhaled nasal nitric oxide. This is a change from the previous recommendations for use of structural or functional assay of respiratory cilia as the primary diagnostic tool. The four key clinical features of PCD are 1) year round, daily, productive cough; 2) year round, daily non-seasonal rhinitis; 3) neonatal respiratory distress in a term infant; and 4) laterality defects (eg, situs inversus totalis). Low exhaled nitric oxide is found in both PCD and CF , therefore this test is only diagnostic of PCD when CF has been ruled out. Structural and functional cilia assays have a high incidence of variability from center to center and often result in misdiagnosis. Genetic assay for a pathogenic variant in the PCD gene can be obtained, but has limitations as new variants continue to be identified, resulting in incomplete commercially available panels. Although recurrent respiratory infections are associated with quantitative or functional immunoglobulin deficiencies, perinatal respiratory distress is a marker for PCD and not for immunodeficiency. Therefore, qualitative immunoglobulin assays would not be the best test to perform for the girl in this clinical scenario. The sweat chloride value for this patient is 1 mmol/L

Urine organic acids

Increased dicarboxylic acids

Of the following, this neonate's MOST likely diagnosis is A. Barth syndrome B. α-galactosidase A deficiency C. Pompe disease D. very-long-chain acyl-coenzyme A dehydrogenase deficiency

American academy of pediatrics

803

PREP ® Self-Assessment

PREPSA 2020

Correct Answer: D The neonate in the vignette has very long-chain acyl-coenzyme A dehydrogenase (VLCAD) deficiency, an autosomal recessive inborn error of metabolism that can present in 1 of 3 scenarios. This neonate has a subtype that manifests with severe early-onset cardiac failure that can progress to multiorgan failure. Cardiac findings include hypertrophic or dilated cardiomyopathy, arrhythmias, and pericardial effusion. Affected neonates and infants will have hypotonia and hepatomegaly. Laboratory findings include hypoketotic hypoglycemia, hepatic dysfunction, elevated creatine kinase, and increased dicarboxylic acids on urine organic acid analysis. Acylcarnitine analysis, an appropriate second-tier test, would reveal elevated C14:1, C14:2, C14, and C12:1 metabolites. This disorder is typically detected via newborn screening with elevated C14:1 of more than 1 mmol/L; screening is performed because early intervention and treatment will improve morbidity and mortality in affected children. A second type of VLCAD is the hepatic or hypoketotic hypoglycemic form; this type presents in early childhood with hypotonia and hepatomegaly but typically lacks cardiomyopathy. The third type has an even later onset, manifesting with episodic myopathy and intermittent rhabdomyolysis typically provoked by muscle cramps, pain, or exercise. The diagnosis of VLCAD is confirmed with an acylcarnitine analysis and identification of biallelic pathogenic gene mutations in ACADVL. In affected children having episodes of acute illness, management requires the administration of intravenous glucose as an energy source to stimulate insulin secretion and suppress lipolysis, as well as careful monitoring for and management of arrhythmias and rhabdomyolysis. Avoidance of triggers such as fasting, dehydration, and long-chain fats is important for preventing metabolic decompensation; therefore, patients are maintained on a low-fat, high-carbohydrate diet with supplementation of medium-chain triglyceride oil and frequent regular feeding. Fats are an important source of energy and serve as the principal fuel source rather than glucose for the heart and skeletal muscle during exercise. Many tissues prefer to use fatty acids for energy, thus allowing the brain to selectively use glucose. Hepatic fatty acid oxidation is also important for ketone body synthesis. Thus, fatty acid oxidation disorders commonly lead to hypoketotic hypoglycemia, hepatomegaly, hepatic dysfunction, myopathy, rhabdomyolysis, and encephalopathy. Other fatty acid oxidation disorders include, but are not limited to, carnitine palmitoyltransferase (CPT) I deficiency, CPT II deficiency, medium-chain acyl-CoA dehydrogenase deficiency, short-chain acyl-CoA dehydrogenase deficiency, and long-chain 3hydroxyacyl-CoA dehydrogenase deficiency. Most fatty acid oxidation disorders have been associated with significant morbidity and mortality; however, because many of these disorders are now detected via newborn screening, their outcomes have improved with early treatment, dietary interventions, and avoidance of triggers. Barth syndrome is an X-linked recessive mitochondrial disorder arising from TAZ mutations, which presents in affected males with cardiomyopathy, muscular weakness, neutropenia, American academy of pediatrics

804

PREP ® Self-Assessment

PREPSA 2020

distinctive facial dysmorphology, and impaired growth. Urine organic acids would reveal significantly elevated urinary 3-methylglutaconic acid and moderately increased urinary 3methylglutaric acid and 2-ethylhydracrylic acid. α-Galactosidase A deficiency, also known as Fabry disease, is an X-linked disorder that causes increasing lysosomal deposition of globotriaosylceramide in cells. It typically presents with periodic pain crises of the distal extremities, angiokeratomas, sweating dysfunction, corneal/lenticular opacities, and proteinuria. Progressive renal deterioration ultimately leads to end-stage renal disease in untreated affected males in the 3rd to 5th decade of life. Cerebrovascular strokes and heart disease are major causes of morbidity and mortality, typically in adulthood. Pompe disease, an autosomal recessive glycogen storage and lysosomal disorder caused by a deficiency of the acid α-glucosidase enzyme, can present in infancy with cardiomyopathy, left ventricular hypertrophy, hepatomegaly, poor feeding, macroglossia, failure to thrive, muscular weakness, and respiratory difficulties. Affected children will have normal cognition. It is detectable on newborn screening; the diagnosis is then confirmed by the detection of reduced acid α-glucosidase enzyme activity or biallelic GAA pathogenic gene mutations. Other laboratory abnormalities include an elevated creatine kinase and abnormal urinary oligosaccharide levels. PREP Pearls • Very long-chain acyl-coenzyme A dehydrogenase (VLCAD) deficiency, an autosomal recessive inborn error of metabolism, can manifest in the neonatal period or infancy with severe early-onset cardiac failure that can progress to multiorgan failure if not treated promptly. Clinical findings include hypertrophic or dilated cardiomyopathy, arrhythmias, pericardial effusions, hypotonia, and hepatomegaly. • Laboratory findings in very long-chain acyl-coenzyme A dehydrogenase include hypoketotic hypoglycemia, hepatic dysfunction, elevated creatine kinase, abnormal acylcarnitine analysis, and increased dicarboxylic acids on urine organic acid analysis. • Prevention of metabolic decompensation with very long-chain acyl-coenzyme A dehydrogenase involves avoidance of triggers such as fasting, dehydration, and longchain fats; affected children are maintained on a low-fat diet with supplementation of medium-chain triglyceride oil and frequent regular feeding. ABP Content Specifications(s) • Recognize the laboratory features associated with disorders of fatty acid and carnitine metabolism • Recognize the clinical features associated with disorders of fatty acid and carnitine metabolism

American academy of pediatrics

805

PREP ® Self-Assessment

PREPSA 2020

Suggested Readings • Leslie ND, Valencia CA, Strauss AW, MD, Zhang K. Very long-chain acyl-coenzyme A dehydrogenase deficiency. GeneReviews. https://www.ncbi.nlm.nih.gov/books/NBK6816/. • Saronwala A, Kubendran S, Kahler SG. Screening for genetic-metabolic diseases. In: McInerny TK, Adam HM, Campbell DE, DeWitt TG, Foy JM, Kamat DM, eds. American Academy of Pediatrics Textbook of Pediatric Care. 2nd ed. Elk Grove Village, IL: American Academy of Pediatrics; 2017:210-225. Pediatric Care Online. • Solis JO, Singh RH. Management of fatty acid oxidation disorders: a survey of current treatment strategies. J Am Diet Assoc. 2002;102:1800-1803. doi:10.1016/S00028223(02)90386-X. • Vishwanath VA. Fatty acid beta-oxidation disorders: a brief review. Ann Neurosci. 2016;23(1):51-55.

American academy of pediatrics

806

PREP ® Self-Assessment

PREPSA 2020

Question 225 A 12-year-old, right-handed boy is seen for evaluation of right medial elbow pain. He has played baseball for the past 6 years and is considered the best pitcher on his travel team. This past year he has been noticing increasing pain over the medial aspect of his elbow. He has been icing and using ibuprofen after every practice with some improvement but continues to have pain with throwing. He reports no traumatic injury, but one week ago he had increased pain while pitching and noted medial elbow swelling. He feels better after resting the elbow and using ice and a compression wrap over the past several days. He is otherwise feeling well and has no other musculoskeletal concerns. There is a small amount of swelling and tenderness to palpation over the medial epicondyle and a minimal flexion contracture at the right elbow as compared to the left. Strength testing is normal in the right arm and elicits no pain. His physical examination findings are otherwise unremarkable. Of the following, the BEST next step in management is to A. allow a graduated return to throwing B. continue with ice and ibuprofen and follow up in 2 weeks C. obtain bilateral elbow radiographs D. start physical therapy

American academy of pediatrics

807

PREP ® Self-Assessment

PREPSA 2020

Correct Answer: C Overhead throwing places significant stress across the medial elbow, and the boy in this vignette has sustained an avulsion injury of the medial epicondyle. Radiographs are indicated to assess for possible displacement of the epicondyle, and bilateral images are often needed in skeletally immature athletes to assess for subtle changes in growth plate alignment. Overhead throwing exaggerates the intrinsic valgus alignment of the elbow, which places significant traction stress across the medial elbow and compression stress across the lateral elbow. Stability of the medial elbow is maintained by the structures shown in Item C225A. The primary stabilizer of the medial elbow is the ulnar collateral ligament (UCL), which originates on the medial epicondyle of the humerus and inserts on the proximal ulna. The common flexor tendon (CFL) overlies the UCL and provides additional dynamic stability when the elbow is in motion. The UCL and CFL pull on the medial epicondyle during an overhead throw. The medial epicondyle ossification center initially appears at 6 to 7 years of age and fuses to the remainder of the humerus at about 15 years of age in boys and 13 years of age in girls; it is still open in the boy described in this vignette. During this interval the cartilaginous growth plate is weaker than the UCL and CFL and is the first to mechanically fail during a period of overload. After this apophysis fuses at skeletal maturity, the UCL becomes the weaker link and becomes more susceptible to injury.

American academy of pediatrics

808

PREP ® Self-Assessment

PREPSA 2020

Item C225A: Stabilizers of the medial elbow. Abbreviation: UCL, ulnar collateral ligament. Courtesy of M. LaBotz

American academy of pediatrics

809

PREP ® Self-Assessment

PREPSA 2020

Youth baseball-related elbow injury, also known as “little league elbow” is an umbrella term used to describe a variety of issues in the young thrower’s elbow, including medial epicondyle apophysitis or avulsion or osteochondritis dissecans of the capitellum. The earliest symptoms of youth baseball-related elbow injury are often a decrease in throwing speed or efficiency and a mild flexion contracture of the elbow. Medial elbow pain resulting from chronic overuse without a significant acute exacerbation usually represents medial apophysitis. Young throwers who continue to play with medial elbow pain are at subsequent risk for an acute avulsion event, which is identified by a sudden increase in pain, as seen in the boy in this vignette. This usually occurs during a specific pitch or throw and is not necessarily overtly traumatic in nature. Lateral elbow pain, or mechanical symptoms such as painful catching or locking, should prompt consideration for possible osteochondral injury to the capitellum. Physical examination in a patient with medial epicondyle apophysitis or avulsion reveals tenderness and possible swelling over the medial epicondyle. There is often pain with full extension and with resisted wrist flexion and pronation. The ulnar nerve runs just behind the medial epicondyle, and patients should be checked for sensation in the fourth and fifth fingers and the ability to resist finger abduction. Instability of the elbow is assessed with valgus stress testing, which is performed with the elbow flexed 20° to 30° and is demonstrated in Item C225B. In cases of suspected fracture, valgus stress testing should be deferred until radiographs have been obtained.

Item C225B: Valgus stress testing of the elbow. Reprinted with permission from Congeni J. Elbow. In: Harris SS, Anderson SJ, eds. Care of the Young Athlete. 2nd ed. Elk Grove Village, IL: American Academy of Pediatrics; 2010:345.

There should be a low threshold for obtaining radiographs in throwing athletes with elbow pain. Radiographs should include anteroposterior, lateral, and oblique views with comparison American academy of pediatrics

810

PREP ® Self-Assessment

PREPSA 2020

views of the normal elbow to identify subtle changes in the growth plates of the elbow. Mild widening of the medial apophysis is demonstrated in Item C225C with a normal comparison shown in Item C225C-1 and Item C225C-2 . Radiographs that demonstrate more than 2 mm of displacement of the medial epicondyle should be referred for possible surgical fixation. Treatment for nondisplaced avulsion injuries should begin with cast immobilization for 2 to 3 weeks of initial healing.

Item C225C: Widened medial epicondyle apophysis of the injured elbow. Reprinted with permission from Delgado J, Jaramillo D, Chauvin NA. Imaging the injured pediatric athlete: upper extremity. Radiographics. 2016;36(6):1681.

American academy of pediatrics

811

PREP ® Self-Assessment

PREPSA 2020

Patients and families need to understand that appropriate rehabilitation is an important part of treatment for youth baseball-related elbow injury. Power for throwing should actually start in the legs and is transmitted through the kinetic chain to the upper extremity. Informing patients that “throwing should occur from the ground up” often helps them to understand this concept. Many young throwers try to generate most of their power in the throwing arm which results in overuse and injury in the elbow and shoulder. Continued throwing with any pain in the shoulder or elbow markedly increases risk of subsequent injury. Ideally, families should query coaches to identity any technical errors noted with the throwing motion, and these errors should be addressed during rehabilitation. Rehabilitation can be started while the elbow is still immobilized and should begin with strengthening of the hips and core along with neuromuscular training to assure that the young thrower is using the core to transmit lower body power to the throwing arm. After cast removal, a hinged brace is provided to allow for range of motion of the elbow, while still protecting against valgus stress. Once tenderness has resolved, strengthening of the shoulder and elbow should begin. A throwing progression may be started after the athlete has regained full strength and range of motion of the elbow and has demonstrated good mechanical fundamentals in rehabilitation. Throwing progressions are highly structured; an overview and details can be found at https://www.ncbi.nlm.nih.gov/pmc/articles/PMC3445073/. Athletes who do not American academy of pediatrics

812

PREP ® Self-Assessment

PREPSA 2020

successfully complete a comprehensive rehabilitation process and correct underlying technical errors with throwing are at high risk for recurrent injury. PREP Pearls • Early recognition of growth plate injuries in the pediatric elbow is crucial for avoidance of long-term consequences. • Growth plate injuries are more common than tendon overuse syndromes in the pediatric elbow. • Bilateral elbow radiographs are often needed for detection of physeal injuries in the elbow. ABP Content Specifications(s) • Recognize the clinical findings associated with sports-related dislocation of the elbow, including associated complications, and manage appropriately • Recognize the clinical findings associated with sports-related elbow pain, and manage appropriately Suggested Readings • Axe M, Hurd W, Snyder-Mackler L. Data-based interval throwing programs for baseball players. Sports Health. 2009;1(2):145-153. doi:10.1177/1941738108331198. • Congeni J. Elbow. In: Harris SS, Anderson SJ, eds. Care of the Young Athlete. 2nd ed. Elk Grove Village, IL: American Academy of Pediatrics; 2010:343-366. • Delgado J, Jaramillo D, Chauvin NA. Imaging the injured pediatric athlete: upper extremity. Radiographics. 2016;36(6):1672-1687. doi:10.1148/rg.2016160036. • Kannikeswaran N, Suresh S. Sports musculoskeletal injuries. In: McInerny TK, Adam HM, Campbell DE, DeWitt TG, Foy JM, Kamat DM, eds. American Academy of Pediatrics Textbook of Pediatric Care. 2nd ed. Elk Grove Village, IL: American Academy of Pediatrics; 2017:2675-2686. Pediatric Care Online. • Rice SG, Congeni JA; Council on Sports Medicine and Fitness. Baseball and softball. Pediatrics. 2012;129(3):e842-e856. doi:10.1542/peds.2011-3593.

American academy of pediatrics

813

PREP ® Self-Assessment

PREPSA 2020

Question 226 A 6-month-old infant is brought to the office for a health supervision visit. He was delivered at 28 weeks of gestation because of worsening maternal preeclampsia He remained in the neonatal intensive care unit for 3 months. His hospital course included continuous positive airway pressure for 1 month, treatment with ibuprofen for a patent ductus arteriosus, and a blood transfusion. At age 1 month, head ultrasonography demonstrated a left grade 2 intraventricular hemorrhage He was discharged from the hospital in room air. He receives physical therapy weekly for increased lower extremity tone. His mother is worried about his long-term development. Of the following, this infant’s MOST important prognostic risk factor for neurodevelopmental outcomes is A. continuous positive airway pressure B. intraventricular hemorrhage C. red blood cell transfusion D. treatment of patent ductus arteriosus with ibuprofen

American academy of pediatrics

814

PREP ® Self-Assessment

PREPSA 2020

Correct Answer: B For the premature infant, after gestational age, intraventricular hemorrhage (IVH) of any grade is the risk factor most closely associated with neurodevelopmental impairment. Prematurity is a leading cause of neonatal mortality in the United States and accounts for significant neonatal morbidity. In infants born prematurely, postnatal brain development must continue in an environment very different from that found in utero. Compared with the intrauterine environment, postnatal development in the neonatal intensive care unit often involves exposure to relatively noxious stimuli such as loud noises and painful procedures. In addition, complications of prematurity may impair developmental maturation. For example, both bacteremia and necrotizing enterocolitis result in a systemic inflammatory response that has been associated with lower scores on the Bayley Scales of Infant and Toddler Development at 36 months' corrected gestational age. Intraventricular hemorrhage is primarily seen among neonates of less than 32 weeks' gestation or with birthweights of less than 1,500 g. Although the pathogenesis is not completely understood, it likely involves impaired cerebral autoregulation in combination with insufficient vascular structural support in the periventricular region. The strongest risk factor for IVH is prematurity. Additional risk factors include male sex, respiratory distress syndrome, pneumothorax, and changes in blood pressure. Intraventricular hemorrhage is graded based on the size and location of hemorrhage: • Grade 1 IVH: Bleeding within the germinal matrix • Grade 2 IVH: Blood noted in the ventricle • Grade 3 IVH: Blood noted in the ventricle with ventricular dilation • Grade 4 IVH: Blood extends into the brain parenchyma Premature infants with grades 1 and 2 IVH generally have lower neurodevelopmental scores than those without IVH. Grades 3 and 4 IVH have a clear association with neurodevelopmental impairment. Premature infants with or without IVH are at increased risk for periventricular leukomalacia and cerebral palsy. In comparison, continuous positive airway pressure administration, red blood cell transfusion, and medical treatment of a patent ductus arteriosus with ibuprofen have not been independently associated with adverse neurodevelopmental outcomes. PREP Pearls • Premature infants with grade 1 and 2 intraventricular hemorrhage (IVH) have worse neurodevelopmental outcomes than infants without IVH; grades 3 and 4 IVH have a clear association with neurodevelopmental impairment. • In preterm infants, bacteremia and necrotizing enterocolitis are associated with neurodevelopmental impairment. • Premature infants with and without intraventricular hemorrhage are at increased risk for cerebral palsy and periventricular leukomalacia.

American academy of pediatrics

815

PREP ® Self-Assessment

PREPSA 2020

ABP Content Specifications(s) • Understand the prognostic factors for very-low-birth-weight infants • Understand the physiologic and physical abnormalities that may be present in a small-forgestational-age infant • Understand the mortality rate in small-for-gestational age infants Suggested Readings • Bolisetty S, Dhawan A, Abdel-Latif M, et al. Intraventricular hemorrhage and neurodevelopmental outcomes in extreme preterm infants. Pediatrics. 2014;133(1):55-62. doi:10.1542/peds.2013-0372. • Greenberg JM. Overview of mortality and morbidity. In: Kliegman RM, St Geme JW III, Blum NJ, Shah SS, Tasker RC, Wilson KM, eds. Nelson Textbook of Pediatrics. 21st ed. Philadelphia, PA: Elsevier; 2020:863-867.

American academy of pediatrics

816

PREP ® Self-Assessment

PREPSA 2020

Question 227 Several neonates in the newborn nursery have hand or foot polydactyly. One neonate is of African descent and has a small piece of tissue adjacent to the fifth digit on the hand. A second neonate is of European descent and has an extra fully formed first digit on the hand. A third neonate has a great toe that has 2 nails. A fourth neonate has a duplicated second digit on the foot. Each family is concerned about other potential abnormalities in their newborn. The family who can be MOST reassured is the one whose newborn has the abnormality located A. centrally on the foot B. postaxially on the hand C. preaxially on the foot D. preaxially on the hand

American academy of pediatrics

817

PREP ® Self-Assessment

PREPSA 2020

Correct Answer: B Of the families described in the vignette, the family that can be most reassured is the one whose newborn has a postaxial polydactyly on the hand, because this particular location of polydactyly is least associated with other potential abnormalities. Children with polydactyly in the preaxial and central locations are more likely to have other congenital abnormalities. Polydactyly is the condition of having more than 5 digits on the hand or foot. Postaxial polydactyly occurs on the ulnar aspect of the hand or the fibular aspect of the foot. Preaxial polydactyly occurs on the radial aspect of the hand or the tibial aspect of the foot. Central polydactyly occurs on the other digits. Postaxial polydactyly is more common in individuals of African descent with an autosomal dominant inheritance pattern, and it typically has the lowest association with other abnormalities. Postaxial polydactyly of the hand in individuals of European descent is more likely to be a recessive trait and to be associated with a syndromic condition such as Ellis-Van Creveld syndrome. Preaxial polydactyly occurs more often in individuals of European descent and is more often associated with other abnormalities including trisomy 21, Fanconi anemia, and VACTERL association. Central polydactyly is the rarest form of polydactyly and is also often associated with multiple congenital abnormalities. Management of polydactyly depends on the examination findings. A preaxial or postaxial polydactyly that is without any bony structures and attached by a thin stalk may be ligated with a suture by the general pediatrician. A polydactyly with a thicker stalk or bony structure should be referred to a specialist for removal. A genetics referral may be prudent for individuals with postaxial polydactyly without a family history and for patients with preaxial or central polydactyly. PREP Pearls • Postaxial polydactyly is less likely than other polydactylies to be associated with other abnormalities. • A genetics referral should be considered for patients who have postaxial polydactyly without a family history and for patients with preaxial or central polydactyly. ABP Content Specifications(s) • Plan the appropriate management of polydactyly and understand when referral is appropriate

American academy of pediatrics

818

PREP ® Self-Assessment

PREPSA 2020

Suggested Readings • Chung EK, Gable EK, Golden WC, et al. Current scope of practice for newborn care in non-intensive hospital settings. Hosp Pediatr. 2017;7(8):471-482. doi:10.1542/hpeds.2016-0206. • Guo B, Lee SK, Paksima N. Polydactyly: a review. Bull Hosp Jt Dis. 2013;71(1):1723. http://hjdbulletin.org/files/archive/pdfs/101.pdf. • Kaur H, Campbell DE. Physical examination of the newborn. In: McInerny TK, Adam HM, Campbell DE, DeWitt TG, Foy JM, Kamat DM, eds. American Academy of Pediatrics Textbook of Pediatric Care. 2nd ed. Elk Grove Village, IL: American Academy of Pediatrics; 2017:802-819. Pediatric Care Online. • Rosen O, Marion RW, Samanich JM. Common congenital anomalies. In: McInerny TK, Adam HM, Campbell DE, DeWitt TG, Foy JM, Kamat DM, eds. American Academy of Pediatrics Textbook of Pediatric Care. 2nd ed. Elk Grove Village, IL: American Academy of Pediatrics; 2017:828-837. Pediatric Care Online.

American academy of pediatrics

819

PREP ® Self-Assessment

PREPSA 2020

Question 228 A pediatrician in a hospital-based office arrives 45 minutes after the start of the work day. This is his second late arrival this week and his fifth late arrival this month. He appears uncharacteristically unkempt and is slurring his speech ever so slightly. His colleague is concerned that he is impaired, and she discusses her concerns with him in a nearby unoccupied office. She does not smell any alcohol but does notice an overpowering stench of cologne and breath mints. When asked if he is feeling okay, he becomes extremely defensive and yells "Mind your own business!" He then puts on his white coat and leaves the room. Of the following, the next BEST course of action for his colleague is to A. ask him to provide blood and urine specimens to prove that he is not currently intoxicated B. inform him that his medical decision-making must be confirmed by another physician because there are concerns about his judgment C. report him to the supervisor on call and prevent patient contact until the situation is resolved D. tell him to go home now, "sleep it off," and return the next day ready to work

American academy of pediatrics

820

PREP ® Self-Assessment

PREPSA 2020

Correct Answer: C The physician in this vignette is impaired, likely from alcohol or other drugs. The appropriate action is to report him to the supervisor on call and prevent patient contact until the situation is resolved. As physicians we subscribe to the bioethical principle of primum non nocere - first, do no harm - which includes not only the classical interpretation of ensuring that the potential benefits of our treatments outweigh the potential risks, but also extends to protecting patients from our colleagues or ourselves if we are not fit to provide care at that moment. The definition of an impaired physician does not necessarily mean that drugs or alcohol are the cause, although that is often the case. The American Medical Association's "Physician Responsibilities to Impaired Colleagues" (Code of Medical Ethics Opinion 9.3.2) states that physicians have an ethical obligation to intervene, report, and assist impaired colleagues. Today, all 50 states have implemented physician health programs within the guidelines set forth by their respective state legislatures with the purpose of facilitating treatment, rehabilitating, and monitoring physicians who have physical, mental health, and substance use disorders. Signs of substance use disorders include changes in behavior, missed days of work, errors in patient care or paperwork, frequent breaks from clinical activities, changes in appearance, and changes in interactions with friends and colleagues. Hospitals may have policies and procedures in place to deal with an impaired practitioner. It is not incumbent or appropriate for a colleague to attempt to collect biological specimens for drug testing nor is it the responsibility of an individual to monitor a colleague who may be impaired. While the exact words chosen when confronting an impaired colleague will depend upon the personal relationship, it is never appropriate for an individual to attempt to cover for an intoxicated colleague by simply telling them to go home. If an attempted "cover-up" is discovered, it is likely that both parties will be found to be at fault. It is a legal requirement in certain states as well as an ethical obligation for physicians to report colleagues showing signs of substance use. PREP Pearls • It is an ethical responsibility of physicians to ensure that impaired colleagues are not practicing patient care. • All 50 states have physician health programs to assist in the rehabilitation and monitoring of physicians with substance use disorders. ABP Content Specifications(s) • Recognize and apply ethical principles regarding physicians who may present a risk to patients

American academy of pediatrics

821

PREP ® Self-Assessment

PREPSA 2020

Suggested Readings • American Medical Association. Code of Medical Ethics Opinion 9.3.2. Physician Responsibilities to Impaired Colleagues. https://www.ama-assn.org/deliveringcare/ethics/physician-responsibilities-impaired-colleagues. • Federation of State Physician Health Programs. History. https://www.fsphp.org/index.php?option=com_content&view=article&id=97:his tory&catid=20:site-content&Itemid=143. • Santucci KA. Reporting an impaired colleague difficult but necessary. AAP News. December 2018:20. http://www.aappublications.org/news/2018/11/28/law112818.

American academy of pediatrics

822

PREP ® Self-Assessment

PREPSA 2020

Question 229 A 6-year-old boy is having a health supervision evaluation. He has a diagnosis of mild intellectual disability, and has been doing well with the supports and services provided through his Individualized Education Program. The boy’s mother is pregnant. Although she is excited to have another child, she is worried that this child may also have intellectual disability. She is interested in having her son tested to identify the cause of his disability. The boy's vital signs, height, weight, and head circumference are within normal limits. His physical and neurologic examination findings are unremarkable. Of the following, the test that is MOST likely to uncover the etiology is A. acylcarnitine profile B. chromosomal microarray C. electroencephalography D. magnetic resonance imaging of the brain

American academy of pediatrics

823

PREP ® Self-Assessment

PREPSA 2020

Correct Answer: B Chromosomal microarray (CMA) is the test most likely to determine the etiology of intellectual disability (ID) in this child who has a normal head circumference, normal neurologic examination, and no dysmorphic features. Brain magnetic resonance imaging should be considered when there is an abnormal head size, midline defect, significant seizures, or abnormal neurologic examination. Magnetic resonance imaging of the brain is unlikely to uncover the cause in a child without micro or macrocephaly or focal findings on neurologic examination. Electroencephalography is not indicated unless there are symptoms concerning for a seizure disorder. Because an inborn error of metabolism may be treatable, metabolic tests should be considered early on when there are signs or symptoms such as intermittent physical and mental decompensation with illness, developmental regression, severe hypotonia, organomegaly, and growth abnormalities. An acylcarnitine profile is a biochemical test for fatty oxidation disorders and some organic acid disorders. This child is not exhibiting signs or symptoms suggestive of a metabolic disorder. There are many possible causes for intellectual disability, including genetic conditions (eg, Down syndrome, fragile X, Rett), infections (eg, rubella, encephalitis), inborn errors of metabolism (eg, phenylketonuria), central nervous system disorders/malformations, teratogens (eg, alcohol, radiation), or trauma. The most common inherited cause of ID is fragile X. Identifying a cause whenever possible is important, because it can provide valuable information for genetic counseling; monitoring for associated medical, learning, and mental health problems; connecting with condition-specific support groups and research; understanding prognosis; and, in some cases, targeted treatment (eg, for an inborn error of metabolism). Knowing the answer to this question can be particularly important for parents who are struggling with guilt or other concerns about presumed reasons for their child’s disability. A comprehensive history, physical, and neurologic examination should be conducted to evaluate for possible etiologies of ID. If a specific genetic syndrome or condition is suspected, the appropriate laboratory studies should be ordered (eg, karyotype for suspected Down syndrome). In the nonsyndromic patient with ID, CMA and DNA analysis for fragile X are the first-line recommended tests. Chromosomal microarray identifies duplications and deletions via copy number analyses but will not identify gene mutations. Irrespective of etiology, ID should be addressed via early intervention services before 3 years of age and special education services starting at 3 years of age. An Individualized Family Service Plan (IFSP) under early intervention and an Individualized Education Program (IEP) under special education should be developed to provide the services, support, and guidance required to maximize the child's skills in learning, communication, socialization, and activities of daily living. Services may include specialized academic instruction, speech and language therapy, occupational therapy, adaptive physical education, and counseling. Goals include achieving maximum independence in the community; children should be in a general educational setting when possible. As the child approaches adulthood, it is important to assess and make plans for the individual’s capacity for independent living (eg, employment, living arrangements, finances) and to consider the extent of their decision-making abilities (eg, conservatorship). By age 16 years, with the child's input, an Individualized Transition Plan is incorporated into the IEP to American academy of pediatrics

824

PREP ® Self-Assessment

PREPSA 2020

include transition services (eg, community experiences, college/career counseling, independent living skill development). Intellectual disability may be accompanied by additional medical and mental health needs, and these should be identified and treated. Individuals with ID are more likely to have vision and hearing impairments and neurologic conditions such as epilepsy and cerebral palsy. They can have problems with sleep, nutrition, weight, and physical fitness. They are more likely to have coexisting developmental or mental health conditions such as autism, attentiondeficit/hyperactivity disorder, anxiety, and depression, as well as problematic aggressive and self-injurious behaviors. Children with ID are also at higher risk for being physically or sexually abused. The primary care provider serves an important role as the medical home for children with ID, assisting in their identification, connection to services, and etiologic workup and in advocating for and addressing their health care needs.

PREP Pearls • Identifying the cause of intellectual disability can provide valuable information for genetic counseling; monitoring for associated medical, learning, and mental health problems; connecting with condition-specific support groups and research; understanding prognosis; and, in some cases, for targeting treatment (eg, for an inborn error of metabolism). • In the nonsyndromic patient with intellectual disability, chromosomal microarray and DNA analysis for fragile X are the recommended first-line tests for identifying a cause. • By age 16 years, with the child's input, an Individualized Transition Plan is incorporated into the Individualized Education Program to include transition services (eg, community experiences, college/career counseling, independent living skill development). ABP Content Specifications(s) • Plan appropriate laboratory evaluation of various intellectual disabilities • Plan appropriate management for children with various intellectual disabilities Suggested Readings • Medford HC, Bashaw ML, Hoffman EP. Genomics, Intellectual disability, and autism. N Engl J Med. 2012;366(8):733-743. doi:10.1056/NEJMra1114194. • Morscher JB, Shovel M, Committee on Genetics. Comprehensive evaluation of the child with intellectual disability or global developmental delays. Pediatrics. 2014;124(3):e903e918. doi:10.1542/peds.2014-1839. • Phelps RA, Cohen WI. Intellectual disability. In: McInerny TK, Adam HM, Campbell DE, DeWitt TG, Foy JM, Kamat DM, eds. American Academy of Pediatrics Textbook of Pediatric Care. 2nd ed. Elk Grove Village, IL: American Academy of Pediatrics; 2017:2208-2217. Pediatric Care Online. • Purugganan O. Intellectual disabilities. Pediatr Rev. 2018;39(6):299-309. doi:10.1542/pir.2016-0116. American academy of pediatrics

825

PREP ® Self-Assessment

PREPSA 2020

Question 230 A 6-day-old female neonate is brought to the emergency department for evaluation of abnormal movements. She was born at 39 weeks' gestation to a gravida 2 para 2 mother with an uncomplicated pregnancy via normal spontaneous vaginal delivery. Apgar scores were 9 and 9 at 1 and 5 minutes, respectively. Her neonatal course was unremarkable, and she was discharged home on the second day after birth. On the day of presentation, her parents noted abnormal movements that were described as asymmetric twitching of her legs with accompanying lip smacking and perioral cyanosis with periods where she appeared to hold her breath. These periods would last up to 2 minutes before self-resolving with return to her typical feeding and activity. Family history is significant for her mother and a maternal uncle with seizures around the time they were born, which subsequently resolved. The neonate's vital signs are within normal limits. She appears alert and has normal neurological examination findings. During examination, she has a typical event where her eyes open and she stares straight ahead with clonic movements of her legs and lip smacking with apnea. Intravenous phenobarbital (20 mg/kg) is administered, and the movements stop. Results of noncontrast computed tomography of the head are normal. Results of a complete blood cell count, comprehensive metabolic panel, urinalysis, ammonia level, C-reactive protein, lactate, serum and urine toxicology screen, and capillary blood glucose are within normal limits. Lumbar puncture yields clear cerebrospinal fluid. Laboratory data from cerebrospinal fluid analysis are shown: Laboratory test

Result

White blood cell count 4/μL Lymphocytes

75%

Red blood cell count

0/μL

Protein

120 mg/dL

Glucose

100 mg/dL (5.6 mmol/L)

Cultures are pending. Of the following, the MOST likely diagnosis is A. bacterial meningitis B. benign familial neonatal convulsions C. hypoxic-ischemic encephalopathy D. pyridoxine-dependent epilepsy

American academy of pediatrics

826

PREP ® Self-Assessment

PREPSA 2020

Correct Answer: B The clinical presentation of the neonate in this vignette is consistent with benign familial neonatal convulsions, 1 of 2 recognized benign neonatal seizure syndromes. An affected neonate will present with focal clonic seizures responsive to antiepileptic medication. The seizures typically resolve spontaneously in the first few days to week after birth. A positive family history of neonatal seizures distinguishes benign familial neonatal convulsions from benign neonatal seizures, or "fifth-day fits." Key features supportive of the diagnosis of benign familial neonatal convulsions include a positive family history and lack of clinical, radiologic, or laboratory risk factors for seizure in a neonate with normal neurological examination findings and normal interictal electroencephalography (EEG) background. In both syndromes, prognosis is excellent with a benign clinical course. Antiepileptic medication can be withdrawn after a few weeks to months of therapy. Neonatal seizures are a relatively common condition, typically occurring secondary to a variety of etiologies in the first week after birth. Causes include symptomatic seizures in the setting of hypoxic-ischemic encephalopathy, infection, a vascular event, congenital malformations, or an underlying genetic or metabolic disorder. The neonatal brain is immature with incomplete myelination and reduced capacity for sufficient inhibition of the hypersynchronous excitatory neuronal firing that causes seizures. As a result, neonatal seizures clinically can be subtle, arising from either a single focus or multiple foci that may not generalize. Neonatal seizures are classified as focal/multifocal clonic (Item C230A), focal tonic (Item C230B), generalized tonic, myoclonic (Item C230C), and subtle. Close clinical observation and a high suspicion for seizure is crucial for prompt diagnosis and treatment. Once the neonate is stabilized, a diagnostic evaluation to identify an etiology for the seizures must occur to allow for targeted treatment. In a stepwise approach, initial testing is aimed at treatable conditions such as correctable metabolic derangements (ie, hypoglycemia, hyponatremia). Hypoxic-ischemic encephalopathy is the most common cause of neonatal seizures. Affected neonates typically show some degree of clinical encephalopathy with abnormalities noted on interictal EEG and magnetic resonance imaging demonstrating brain injury. This is in contrast to the infant in the vignette who had a normal interictal clinical examination. Central nervous system infections, such as meningitis and encephalitis, can cause seizures and are evaluated by cerebrospinal fluid analysis. Congenital brain malformations and genetic/metabolic disorders can present with seizures at any age including the neonatal period. Pyridoxine-dependent epilepsy is a rare treatable cause of neonatal seizures that are refractory to typical antiepileptic medications but responsive to pyridoxine and/or folinic acid, administration of which can be both diagnostic and therapeutic. In contrast to the 2 benign neonatal seizure syndromes described above, infantile epileptic encephalopathies represent neonatal seizure syndromes that result in medically refractory epilepsies with severe clinical course and profound developmental delay. These encephalopathies include early infantile epileptic encephalopathy or Ohtahara syndrome and early myoclonic encephalopathy. Typically these infants will present with tonic spasms, a burstsuppression pattern on EEG, encephalopathy, and treatment-resistant epilepsy. Presence of either syndrome warrants further investigations to identify a specific etiology, which can be structural (cortical malformations such as polymicrogyria) or genetic/metabolic. American academy of pediatrics

827

PREP ® Self-Assessment

PREPSA 2020

Once identified, neonatal seizures are treated symptomatically with antiepileptic medication, commonly phenobarbital. The management of neonatal seizures is outlined in Item C230D. Phenytoin and fosphenytoin are equally effective for initial seizure control in neonates; however, it becomes challenging to maintain a therapeutic level once converted to enteral dosing because of neonatal metabolism of the drug. Once the neonate is stabilized, management is tailored to the underlying etiology of the seizures with the goal of seizure control.

American academy of pediatrics

828

PREP ® Self-Assessment

PREPSA 2020

PREP Pearls • Seizures in the neonatal period are common and are often acute symptomatic seizures related to hypoxic-ischemic encephalopathy. Other symptomatic etiologies include metabolic derangements, infection, hemorrhage, and stroke. • Neonatal seizures are classified as focal/multifocal clonic, focal tonic, generalized tonic, myoclonic, and subtle. Generalized tonic-clonic and secondarily generalized seizure do not occur in this age group due to immature myelination of the brain. • Several neonatal seizure syndromes exist, including 2 benign conditions: benign familial neonatal convulsions and benign neonatal convulsion. Both conditions respond to antiepileptic medications and have an excellent prognosis. MOCA-Peds Objective • Evaluate an infant with suspected neonatal seizures. ABP Content Specifications(s) • Understand the various etiologies of neonatal seizures and plan appropriate diagnostic evaluation • Recognize the clinical findings associated with neonatal seizures and manage appropriately Suggested Readings • Abend NS, Jensen FE, Inder TE, Volpe JJ. Neonatal seizures. In: Inder TE, Darras BT, deVries LS, du Plessis AJ, Neil JJ, Perlman JM, eds. Volpe's Neurology of the Newborn E-Book. 6th ed. Philadelphia, PA: Elsevier; 2017:275. • Glass HC. Neonatal seizures: advances in mechanisms and management. Clin Perinatol. 2013;41(1):177-190. doi:10.1016/j.clp.2013.10.004. • Olson D. Neonatal seizures. NeoReviews. 2012;13(4):e213-e223. doi:10.1542/neo.13-4e213. • Roddy SM, McBride MC. Seizure disorders. In: McInerny TK, Adam HM, Campbell DE, DeWitt TG, Foy JM, Kamat DM, eds. American Academy of Pediatrics Textbook of Pediatric Care. 2nd ed. Elk Grove Village, IL: American Academy of Pediatrics; 2017:2599-2617. Pediatric Care Online.

American academy of pediatrics

829

PREP ® Self-Assessment

PREPSA 2020

Question 231 The pediatric clinic supervising attending physician and pediatric intern discuss the management of otitis media in a 12-month-old female patient. They agree on treatment with amoxicillin 80 to 90 mg/kg per day divided twice daily for 10 days. The intern writes the prescription using the electronic medical record. Later that day, the patient's mother calls the clinic asking for clarification of the amoxicillin dose, because the dose seems too high. The girl's 3-year-old sister recently took amoxicillin for otitis media at a lower dose. Upon review of the prescription order, it was found that a weight of 20 kg was used to calculate the medication dose. The girl's weight was recorded in the electronic medical record as 20 lbs. Of the following, the BEST way to prevent similar errors from occurring is to A. counsel the intern on ensuring that the correct weight is used to calculate medication doses B. have the clinic supervising attending physician review all prescriptions before the patient leaves the clinic C. review medication dose calculations with the parent(s) D. standardize the use of kilograms when recording weight for all clinic patients

American academy of pediatrics

830

PREP ® Self-Assessment

PREPSA 2020

Correct Answer: D The vignette describes a situation in which a medication error was made because the dose was calculated using kilograms, but the weight was recorded in the medical record in pounds. Presumably, the units for which weight is recorded in the medical record was not standardized, thus causing the confusion. The best way to prevent similar errors from occurring is to standardize the use of kilograms when recording the weight of all clinic patients. This solution is systems-based versus focusing on individual improvements. Systems-based solutions help prevent similar errors from occurring again and improve the overall system. They help to reduce error caused by human factors. In the vignette, if all weights were recorded in kilograms, the human factors of deciding to record weight in kilograms versus pounds, realizing the weight was recorded in pounds, and the need to convert pounds to kilograms when calculating the medication dose would be eliminated. A culture of safety that encourages health care team members to report errors for the purpose of improving systems facilitates systems-based solutions. Counseling the intern on ensuring that the correct weight is used to calculate medication doses is an individual-based improvement. It does nothing to improve the system or prevent a similar error from being made by another individual. Having the clinic supervising attending physician review all prescriptions before the patient leaves the clinic would add an additional check on the medication dose but would not be the most efficient way to improve the system. Similarly, reviewing the medication dose calculations with the parent(s) would not be the most efficient way to improve the system. PREP Pearls • Systems-based rather than individual improvements should be sought for health care systems problems. • A culture of safety that encourages health care team members to report errors for the purpose of improving systems facilitates systems-based solutions. ABP Content Specifications(s) • Apply the psychology of change (eg, motivating people to improve) to improve healthcare systems • Understand that quality improvement is based on applying a scientific method to improving human systems • Understand what a system is (eg, people, procedures, equipment) and how each component of that system affects outcome Suggested Readings • Bartman T, McClead RE. Core principles of quality improvement and patient safety. Pediatr Rev. 2016;37(10):407-417. doi:10.1542/pir.2015-0091. • Leonard MS. Patient Safety and quality improvement: reducing risk of harm. Pediatr Rev. 2015;36(10):448-456. doi:10.1542/pir.36-10-448. • Neuspiel DR. Medical errors, adverse events, and patient safety. In: McInerny TK, Adam HM, Campbell DE, DeWitt TG, Foy JM, Kamat DM, eds. American Academy of Pediatrics Textbook of Pediatric Care. 2nd ed. Elk Grove Village, IL: American Academy of Pediatrics; 2017:2287-2295. Pediatric Care Online. American academy of pediatrics

831

PREP ® Self-Assessment

PREPSA 2020

Question 232 A 2-week old neonate is having a health supervision evaluation. He was born at term to first-time parents. Prenatal ultrasonography revealed shortened femur length and macrocephaly; postnatal evaluation confirmed the diagnosis of achondroplasia. The boy’s parents wish to know what the typical pattern of development is in children with this diagnosis. Of the following, the MOST accurate statement is that children with this diagnosis typically have A. delayed cognitive development B. delayed gross motor development C. global developmental delay D. normal development

American academy of pediatrics

832

PREP ® Self-Assessment

PREPSA 2020

Correct Answer: B The neonate in the vignette has achondroplasia; children with this condition typically have delayed gross motor milestones but development in other realms, including cognition, is normal. Achondroplasia is a skeletal dysplasia caused by mutations in the fibroblast growth factor receptor 3 gene (FGFR3) and is inherited in an autosomal dominant fashion. It is the most common type of skeletal dysplasia, a group of conditions that affect bone development. Children with achondroplasia have disproportionate short stature, long bone shortening that is most prominent in the humerus and femur, brachydactyly (short fingers and toes), kyphoscoliosis, lumbar lordosis, and macrocephaly. Early gross motor delays typically exist because of atypical physical proportions; delays resolve by age 2 to 3 years. Children with achondroplasia are at increased risk of developing otitis media, obstructive sleep apnea, obesity, chronic joint pain, and cervical medullary compression. Osteogenesis imperfecta is a heterogeneous bone matrix disorder characterized by increased bone fragility. Although there are variable phenotypic presentations, all children with osteogenesis imperfecta have bone fragility, osteoporosis, bone deformities, and multiple or atypical fractures. Some also have blue sclera, hearing loss, dental abnormalities, increased ligamentous and skin laxity, and/or easy bruising. Several gene mutations have been identified, but the most common are autosomal dominant mutations in COL1A1 or COL1A2 which encode chains of type 1 collagen. Arthrogryposis (also known as "arthrogryposis multiplex congenita") is a condition in which multiple joint contractures develop before birth. It can be associated with multiple diagnoses that decrease fetal movement in utero, including central nervous system disorders, neuromuscular diseases, connective tissue or muscular disorders, and trisomies. PREP Pearls • Achondroplasia is an autosomal dominant disorder that is characterized by disproportionate short stature, long bone shortening that is most prominent in the humerus and femur, brachydactyly (short fingers and toes), kyphoscoliosis, lumbar lordosis, and macrocephaly. Infants and toddlers with achondroplasia have gross motor delays but otherwise normal development. • Osteogenesis imperfecta is a heterogeneous disorder characterized by bone fragility, osteoporosis, bone deformities, and multiple or atypical fractures. Affected children may have blue sclera, hearing loss, dental abnormalities, increased ligamentous and skin laxity, and/or easy bruising. • Arthrogryposis is a condition in which multiple joint contractures develop before birth. It can be associated with multiple diagnoses that decrease fetal movement in utero, including central nervous system disorders, neuromuscular diseases, connective tissue or muscular disorders, and trisomies. ABP Content Specifications(s) • Recognize the clinical findings associated with achondroplasia, including complications • Recognize the clinical findings associated with osteogenesis imperfecta • Recognize the clinical features associated with arthrogryposis American academy of pediatrics

833

PREP ® Self-Assessment

PREPSA 2020

Suggested Readings • Clunie G. Prenatal diagnosis. In: McInerny TK, Adam HM, Campbell DE, DeWitt TG, Foy JM, Kamat DM, eds. American Academy of Pediatrics Textbook of Pediatric Care. 2nd ed. Elk Grove Village, IL: American Academy of Pediatrics; 2017:699-710. Pediatric Care Online. • Hall JG. Arthrogryposis (multiple congenital contractures): diagnostic approach to etiology, classification, genetics, and general principles. Eur J Med Genet. 2014;57(8):464-472. doi:10.1016/j.ejmg.2014.03.008. • Pauli RM, Legare JM. Achondroplasia. GeneReviews. https://www.ncbi.nlm.nih.gov/books/NBK1152/. • Steiner RD, Adsit J, Basel D. COL1A1/2-related osteogenesis imperfecta. GeneReviews. https://www.ncbi.nlm.nih.gov/books/NBK1295/.

American academy of pediatrics

834

PREP ® Self-Assessment

PREPSA 2020

Question 233 A 7-month-old female infant born at 28 weeks’ gestation is being evaluated in the office for her weight. She has a history of bronchopulmonary dysplasia; she was discharged from the neonatal intensive care unit with oxygen supplementation, which was discontinued 1 month ago. Her diet consists of maternal breast milk, 25 oz per day fortified to 24 kcal/oz, and pureed solids once per day. She has constipation, and produces hard, yellow stools every 2 to 3 days. Her weight is 7.6 kg (~50th percentile for chronologic age and ~90th percentile for adjusted gestational age) (Item Q233). Physical examination findings are unremarkable.

Item Q233: Growth chart. Courtesy of K. Montez. Of the following, the BEST recommendation for this infant is A. B. C. D.

continued breast milk fortification to 24 kcal/oz implementation of feeding strategies that provide 120 kcal/kg per day reduced caloric intake to achieve 15 g/day of weight gain soy milk–based formula (20 kcal/oz) to help with constipation

American academy of pediatrics

835

PREP ® Self-Assessment

PREPSA 2020

Correct Answer: C The premature infant in this vignette is growing well and has achieved catch-up growth, as evidenced by her growth chart. At this point, her caloric intake may be decreased such that she achieves a weight gain of 15 g/day (Item C233A ). Depending on their clinical condition, premature infants typically require 105 to 130 kcal/kg per day, with a weight gain goal of 15 to 20 g/day. Once she has achieved catch-up growth, continuing to give her 24 kcal/oz formula or 120 kcal/kg per day would increase her risk of overweight or obesity. Soy milk–based formula is nutritionally inappropriate for preterm infants because of the risk of osteopenia, aluminum toxicity, and inadequate absorption of zinc, phosphorus, and iron.

The preferred milk for most premature infants is human milk, though human milk by itself may not provide an adequate amount of calories, protein, minerals, and vitamins without supplementation. Fortification should be individualized to optimize growth trajectory over the first year after birth. For formula-fed infants, preterm infant formulas mixed to 22 to 24 kcal/oz provide the higher vitamin content these infants require. Daily dietary recommendations for stable premature infants are as follows: Carbohydrate 7-20 g/kg Fat 5-7 g/kg Protein 3-4 g/kg Calcium 100-220 mg/kg Chromium 0.1-2.25 μg/kg Copper 120-150 μg/kg Iron 2-4 mg/kg Magnesium 7.9-15 mg/kg Manganese 0.7-7.75 μg/kg Phosphorus 60-140 mg/kg Zinc 1,000-3,000 μg/kg Throughout infancy, the primary objective of feeding is the acquisition of nutrients for optimal growth. The secondary goal is the development of oral-motor skills and appropriate eating behaviors. Target weight gain for term infants follows the trajectory listed in Item C233A. American academy of pediatrics

836

PREP ® Self-Assessment

PREPSA 2020

During early childhood, important nutritional issues include the acquisition of self-feeding skills, fluctuations in appetite, achieving adequate iron and zinc intake, avoiding overconsumption of added sugar, and developing healthy eating and activity routines. School-aged children and adolescent nutritional issues include adequate intake of fruits, vegetables, calcium, vitamin D, and fiber; avoidance of energy-rich/nutrient-poor foods and added sugar; and the development of a healthy body image. Item C233B highlights the recommended daily dietary fat, protein, and mineral intake for various ages.

Recommended dietary allowances (RDAs) are in bold type and adequate intakes (AIs) are followed by an asterisk (*). • Either RDAS or AIs may be used as goals for individual intakes. • RDAs are set to meet the needs of almost all individuals in a group. • For healthy breastfed infants, the AI is the mean intake. The AI for other life stage and gender groups is believed to cover the needs of all individuals in the group but lack of data or uncertainty in the data prevent being able to specify with confidence the percentages of individuals covered by this intake. Adapted from Institute of Medicine, Standing Committee on the Scientific Evaluation of Dietary Reference Intakes, Food and Nutrition Board. Nutrition recommendations: dietary reference intakes. Washington, DC: National Academies Press.

American academy of pediatrics

837

PREP ® Self-Assessment

PREPSA 2020

PREP Pearls • Premature infants typically require 105 to 130 kcal/kg per day, with a weight gain goal of 15 to 20 g/kg per day, until the infant reaches approximately 2 kg. • The preferred milk for premature infants is human milk, but human milk by itself may not provide an adequate amount of calories, protein, minerals, and vitamins without supplementation or fortification. • Soy milk–based formula is nutritionally inappropriate for preterm infants because of the risk of osteopenia, aluminum toxicity, and inadequate absorption of zinc, phosphorus, and iron. ABP Content Specifications(s) • Understand the dietary protein requirements for patients of various ages, including those born prematurely, and the circumstances in which those requirements may change • Understand the dietary mineral requirements of patients of various ages, including those born prematurely, and the circumstances in which those requirements may change • Understand the dietary fat requirements of patients of various ages, including those born prematurely, and the circumstances in which those requirements may change Suggested Readings • American Academy of Pediatrics Committee on Nutrition. Feeding the infant. In Kleinman RE, Greer FR, eds. Pediatric Nutrition. 7th ed. Elk Grove Village, IL: American Academy of Pediatrics; 2013:13-110. • DiMaggio DM, Cox A, Port AF. Updates in infant nutrition. Pediatr Rev. 2017;38(10):449-462. doi:10.1542/pir.2016-0239. • Institute of Medicine, Standing Committee on the Scientific Evaluation of Dietary Reference Intakes, Food and Nutrition Board. Dietary Reference Intakes. Washington, DC: The National Academies Press. https://ods.od.nih.gov/Health_Information/Dietary_Reference_Intakes.aspx. • Tender JAF. Preterm infant nutrition. Pediatr Rev. 2004;25(9):328-329. doi:10.1542/pir.25-9-328. • Torrazza RM, Neu J. Evidence-based guidelines for optimization of nutrition for the very low birthweight infant. NeoReviews. 2013;14(7):e340-e349. doi:10.1542/neo.14-7-e340.

American academy of pediatrics

838

PREP ® Self-Assessment

PREPSA 2020

Question 234 A previously healthy 5-year-old girl was brought to the hospital 1 day ago for fevers, decreased appetite, and difficulty breathing. She had a temperature of 38.5°C, heart rate of 120 beats/min, respiratory rate of 24 breaths/min, blood pressure of 100/60 mm Hg, and oxygen saturation by pulse oximetry of 95% on 1 L/min oxygen by nasal cannula. She appeared tired but well. Mucous membranes were moist. Her heart had regular rhythm with no rubs, murmurs, or gallops. The girl was breathing slightly fast and shallowly, with mild intercostal retractions. She had diminished and bronchial breath sounds throughout the right hemithorax, with dullness to percussion at the right base. The left hemithorax had normal breath sounds. The abdomen was soft, nontender, and nondistended, with no organomegaly. She was warm and well-perfused, with 2+ pulses throughout. Her chest radiograph is shown in Item Q234. She was continued on oxygen, given maintenance intravenous fluids, allowed to eat and drink, and encouraged to use her incentive spirometer. Intravenous vancomycin and ceftriaxone were administered. Over the next 24 hours, she has become progressively tachypneic and less active. She now requires 50% oxygen by simple facemask to maintain an oxygen saturation of 95%. She has been persistently febrile between 38.5°C and 39.5°C, and her respiratory rate is above 30 breaths/min.

Item Q234: Chest radiograph of the girl in the vignette. Courtesy of A. Sarnaik.

American academy of pediatrics

839

PREP ® Self-Assessment

PREPSA 2020

Of the following, the BEST next step is to A. change antibiotics to linezolid and cefepime B. insert a chest tube C. obtain magnetic resonance imaging of the chest D. start furosemide intravenously

American academy of pediatrics

840

PREP ® Self-Assessment

PREPSA 2020

Correct Answer: B The girl in this vignette has pneumonia with radiographic evidence of a pleural effusion. Despite receiving 24 hours of broad-spectrum antibiotics, oxygen, and incentive spirometry, her overall status has worsened. Of the response choices, the best next step is to insert a chest tube. The pleural space is encompassed by the visceral pleura overlying the lung and the parietal pleura attached deep to the chest wall. Under normal conditions, a small amount of fluid (0.3 mL/kg) occupies the pleural space to lubricate the lungs during breathing. The pleural membrane maintains the volume and electrolyte composition of the fluid by homeostatic mechanisms. Filtration of fluid into the pleural space is dependent on capillary permeability, hydrostatic pressure, and oncotic pressure, as described by the Starling principle. Increased venous hydrostatic pressure, most commonly from congestive heart failure, or decreased plasma oncotic pressure can lead to filtration of fluid into the pleural space. Inflammatory conditions or lung infections, most commonly bacterial pneumonia, can also cause pleural effusions by increasing capillary permeability and impairing the function of the pleural membrane to resorb fluid. In addition to Starling forces, reabsorption of pleural fluid occurs via the downstream lymphatic system. Other causes of pleural effusion in children (Item C234A) include lymphatic disorders, trauma, intra-abdominal processes, and malignancy.

American academy of pediatrics

841

PREP ® Self-Assessment

American academy of pediatrics

PREPSA 2020

842

PREP ® Self-Assessment

PREPSA 2020

Children with pleural effusions initially may not exhibit symptoms beyond the underlying cause. In such scenarios, management can be limited to the treatment of the underlying condition. However, if the pleural effusion is large or if the child has limited reserve caused by congestive heart failure or respiratory insufficiency, the presence of fluid in the pleural space may impair lung expansion and lead to hypoxia and hypercapnia. If breathing is significantly impaired, insertion of a chest tube may be warranted regardless of the etiology. Another indication for drainage of a pleural effusion is if the diagnosis is uncertain. Pleural effusions can be transudative or exudative. Transudative effusions are often thin and clear and are usually caused by increased hydrostatic pressure or decreased oncotic pressure, for example in congestive heart failure or hypoalbuminemia, respectively. Exudative effusions are cloudy, thick, or frankly purulent and are usually caused by fluid or protein leakage across an altered capillary membrane, as in pneumonia, malignancy, or chylothorax. In addition to visual appearance, analysis of the pleural fluid can differentiate between a transudative and an exudative effusion and often suggests an underlying etiology (Item C234B).

Item C234B: Characteristics of Exudative and Transudative Effusions. Abbreviation: LDH, lactate dehydrogenase. Reprinted with permission from Cashen K, Petersen TL. Pleural effusions and pneumothoraces. Pediatr Rev. 2017;38(4):173.

Since the widespread use of the pneumococcal vaccine, the incidence of bacterial pneumonia in children has decreased. However, hospitalizations for empyema increased from 3.5 cases per 100,000 hospitalizations from 1996 to 1998 to 7 cases per 100,000 hospitalizations from 2005 to 2007. Parapneumonic effusions can occur in up to 28% of children who are hospitalized for community-acquired pneumonia. The 3 phases of parapneumonic effusion/empyema are exudative, fibrinopurulent, and organizational. During the exudative phase, which usually lasts several days from the onset of pneumonia, impaired resorption due to pleural inflammation leads to the fluid collection. During the fibrinopurulent phase, which may last up to 2 weeks, fibrin and inflammatory debris collect in the pleural space, which can lead to loculations. Stasis of the fluid American academy of pediatrics 843

PREP ® Self-Assessment

PREPSA 2020

can become secondarily infected, leading to empyema. In the organizational phase of parapneumonic effusion, fibroblasts migrate into the pleural space, causing healing, scarring, and pleural thickening. Mature loculations and abscess cavities can become secondarily infected. Some pathogens, particularly methicillin-resistant Staphylococcus aureus, can cause necrotizing pneumonia, lung abscess, and bronchopleural fistulas. Management of most parapneumonic effusions centers around treatment of the underlying pneumonia, including providing supportive care, antibiotics, and oxygen if needed. Indications for insertion of a chest tube for a parapneumonic effusion include significant breathing impairment, impairments in oxygenation or ventilation, and overall declining clinical course, as described for the girl in the vignette. Cases with a severe clinical course, invasive pathogens such as methicillin-resistant Staphylococcus aureus, or loculated empyemas may warrant a consultation with a pediatric surgeon. Minimally invasive video-assisted thoracoscopic surgery is an effective technique to debride infected tissue, lyse walled-off loculations, and perform decortication if necessary. Fibrinolytic therapy with tissue plasminogen activator, urokinase, or streptokinase administered through the chest tube is also an effective treatment for loculated empyema. Placement of a chest tube is indicated for the girl in this vignette because of her worsening condition despite conservative treatment consisting of antibiotics, oxygen, and incentive spirometry. Since she has community-acquired pneumonia, further broadening her antimicrobial coverage for nosocomial infections would not be effective. Since the etiology of the effusion is infectious as opposed to cardiac, furosemide would not be beneficial. Further imaging is sometimes helpful in detecting loculated empyemas if suspected clinically or based on radiographic findings, but ultrasonography, rather than magnetic resonance imaging, is the best diagnostic test. PREP Pearls • Although the incidence of bacterial pneumonia in children has decreased in recent years, complicated parapneumonic effusions are more common. • Most cases of parapneumonic effusion do not require placement of a chest tube. Indications for chest tube placement include significant impairment in breathing or an overall decline in clinical course. • Video-assisted thoracoscopic surgery and fibrinolytic therapy instilled through the chest tube are management options for loculated parapneumonic effusions. • Pleural effusions can be characterized as exudative or transudative based on analysis of the fluid. ABP Content Specifications(s) • Plan the appropriate management of an empyema • Recognize the normal and abnormal characteristics of pleural fluid • Plan the appropriate diagnostic evaluation of pleural disease

American academy of pediatrics

844

PREP ® Self-Assessment

PREPSA 2020

Suggested Readings • Bradley JS, Byington CL, Shah SS, et al. The management of community-acquired pneumonia in infants and children older than 3 months of age: clinical practice guidelines by the Pediatric Infectious Diseases Society and the Infectious Diseases Society of America. Clin Infect Dis. 2011;53(7):e25-e76. doi:10.1093/cid/cir531. • Breuer O, Picard E, Benabu N, et al. Predictors of prolonged hospitalizations in pediatric complicated pneumonia. Chest. 2018;153(1):172-180. doi:10.1016/j.chest.2017.09.021. • Cashen K, Petersen TL. Pleural effusions and pneumothoraces. Pediatr Rev. 2017;38(4):170-181. doi:10.1542/pir.2016-0088. • McGraw MD, Robison K, Kupfer O, Brinton JT, Stillwell PC. The use of Light's criteria in hospitalized children with a pleural effusion of unknown etiology. Pediatr Pulmonol. 2018;53(8):1101-1106. doi:10.1002/ppul.24065.

American academy of pediatrics

845

PREP ® Self-Assessment

PREPSA 2020

Question 235 A 13-year-old adolescent girl is brought to the emergency department for increasing fatigue. She had been a competitive soccer player until about 6 months ago when she began to experience weakness and fatigue. At first, she was unable to play for as long as she used to, and eventually she had to quit the team because of her inability to perform. Until the fatigue began, she had been healthy without any hospitalizations or surgeries. She takes no medications and has had normal growth and development. She reports that she has been bruising more easily, and her parents comment that she appears pale. She has a temperature of 37°C, heart rate of 118 beats/min, blood pressure of 110/78 mm Hg, respiratory rate of 24 breaths/min, and oxygen saturation of 98% on room air. She appears pale and fatigued. She has bruises on her forearms and shins of various ages and sizes. Findings from her heart and lung examinations are normal. Her abdominal examination has normal findings, without hepatomegaly or splenomegaly. Her neurologic examination has normal findings. A urine pregnancy test and toxicology screen have negative results. Laboratory data are shown: Laboratory test

Result

White blood cell count

900/μL (0.9 x 109/L)

Neutrophils

2%

Lymphocytes

98%

Hemoglobin

6.4g/dL (64 g/L)

Platelet count

18x 103/μL (18 x 109/L)

Reticulocytes

0.1%

Mean corpuscular volume 90 fL Peripheral blood smear

Normal morphology

Of the following, the test MOST likely to yield a diagnosis is A. a bone marrow biopsy for percent cellularity B. peripheral blood flow cytometry C. serum polymerase chain reaction for the Epstein-Barr virus D. serum vitamin B12 level

American academy of pediatrics

846

PREP ® Self-Assessment

PREPSA 2020

Correct Answer: A The adolescent girl in the vignette presents with symptomatic anemia (exercise intolerance), leukopenia with very severe neutropenia (absolute neutrophil count < 200/µL), and thrombocytopenia. A decrease in more than 1 blood cell line (leukocytes, erythrocytes, or thrombocytes) is referred to as pancytopenia. As a broad conceptualization, cytopenias occur because the product of the bone marrow is being destroyed (eg, autoimmune cytopenias such as immune thrombocytopenic purpura or autoimmune hemolytic anemia) or the bone marrow is hypoproductive (eg, because it has been replaced by cancer cells as occurs in leukemia, or because the productive cellular machinery has been deactivated as occurs in aplastic anemia). Select causes of pancytopenia are shown in Item C235A. The patient in this vignette has been symptomatic for a long time (> 6 months), she does not have splenomegaly, she is mildly macrocytic (an estimate of an appropriate mean corpuscular volume is 72 fL plus age in years), her reticulocyte count is inappropriately low, and there are no leukemia blasts noted on her peripheral blood smear. This combination of symptoms, signs, and laboratory findings is most consistent with a diagnosis of aplastic anemia. The diagnostic test of choice for aplastic anemia is a bone marrow biopsy with an assessment of percent cellularity. While acute leukemia most often presents with a hypercellular marrow, aplastic anemia presents with a markedly hypocellular marrow (Item C235B).

American academy of pediatrics

847

PREP ® Self-Assessment

American academy of pediatrics

PREPSA 2020

848

PREP ® Self-Assessment

PREPSA 2020

Item C235B: Bone marrow in acute leukemia and severe aplastic anemia. (A) Normal bone marrow contains hematopoietic cells in various stages of maturation. (B) Bone marrow of a patient with acute leukemia is filled with monotonously appearing large leukemic blasts. (C) Bone marrow in severe aplastic anemia is profoundly hypocellular. Aplastic bone marrow is strikingly “empty,” with visible stromal cells but very few hematopoietic cells. Reprinted with permission from Sharma R, Nalepa G. Evaluation and management of chronic pancytopenia. Pediatr Rev. 2016;37(3):104.

Flow cytometry is the diagnostic test of choice for leukemia because it can distinguish cells with aberrant surface marker expression. It is not useful, however, in aplastic anemia where the cells in the peripheral blood are generally phenotypically normal. Epstein-Barr virus–associated mononucleosis can lead to chronic fatigue and exercise intolerance as experienced by the patient in the vignette. However, it typically presents with splenomegaly and lymphadenopathy and only rarely presents with pancytopenia. A deficiency of vitamin B12 can cause a macrocytic anemia, and although the macrocytosis is typically more severe, it is not typically associated with very severe neutropenia, and the peripheral blood smear is usually abnormal with hypersegmented neutrophils and megaloblasts. PREP Pearls • Pancytopenia refers to a state of having more than 1 blood cell line decreased on the complete blood cell count. • The diagnostic test of choice for aplastic anemia is a bone marrow biopsy with measurement of percent cellularity. • An estimate of an appropriate mean corpuscular volume is 72 fL plus age in years. ABP Content Specifications(s) • Understand the etiology of pancytopenia, and manage appropriately • Plan the appropriate diagnostic evaluation of multiple pancytopenias • Distinguish acquired aplastic anemia from childhood leukemia

American academy of pediatrics

849

PREP ® Self-Assessment

PREPSA 2020

Suggested Readings • Hartung HD, Olson TS, Bessler M. Acquired aplastic anemia in children. Pediatr Clin North Am. 2013;60(6):1311-1336. doi:10.1016/j.pcl.2013.08.011. • Sharma R, Nalepa G. Evaluation and management of chronic pancytopenia. Pediatr Rev. 2016;37(3):101-111. doi:10.1542/pir.2014-0087. • Young NS. Aplastic anemia. N Engl J Med. 2018;379(17):1643-1656. doi:10.1056/NEJMra1413485.

American academy of pediatrics

850

PREP ® Self-Assessment

PREPSA 2020

Question 236 A 3-week-old female neonate with fever is brought to the emergency department. Earlier in the day, she was noted to be fussy and was not feeding well. One hour prior to presentation, she had a rectal temperature of 38.1°C. She was full-term at birth and has no medical conditions. She has a temperature of 38°C, blood pressure of 87/55 mm Hg, heart rate of 140 beats/min, and respiratory rate of 40 breaths/min. She cries during physical examination but is consolable. A fine maculopapular rash is seen over her thorax. The remainder of the physical examination findings are normal. She undergoes an evaluation for a serious bacterial infection and is started on ampicillin and gentamicin. Her urine is negative for blood, protein, nitrite, and leukocyte esterase. Final results of a urine culture are pending. Laboratory data are shown: Laboratory Test Result Blood White blood cell 4,500/μL (4.5 x 109/L) count Hemoglobin 9.8 g/dL (98 g/L) Platelet count 175 x 103/μL (175 x 109/L) Culture Gram-positive cocci in clusters; negative for Staphylococcus aureus by molecular testing Cerebrospinal Fluid White blood cell 3/μL count Red blood cell 440/μL count Glucose 57 mg/dL (3.2 mmol/L) Protein 80 mg/dL Culture Gram stain, no organisms seen; culture, pending Of the following, the BEST next step in management is A. no further testing, add vancomycin B. no further testing, no medication change C. repeat blood culture, add vancomycin D. repeat blood culture, no medication change

American academy of pediatrics

851

PREP ® Self-Assessment

PREPSA 2020

Correct Answer: B The best management of the neonate in this vignette is no further testing and no medication change. The blood culture drawn for evaluation of serious bacterial infection is positive for gram-positive cocci in clusters, and molecular testing has excluded Staphylococcus aureus. Therefore, one can infer that the organism is coagulase-negative staphylococci (CoNS). The neonate was born at term and has no underlying medical condition or indwelling device that would predispose her to an invasive infection with CoNS. The most likely explanation for the positive blood culture is bacterial contamination of the blood culture from a common skin organism. Risk factors for CoNS infection include prematurity, especially in infants who weigh less than 1,500 g, and foreign devices or material. Indwelling devices at risk for colonization with CoNS include central venous catheters, ventriculoperitoneal shunts, and peritoneal catheters. Other foreign material at risk for colonization includes orthopedic hardware, baclofen pumps, prosthetic joints, pacemakers, and prosthetic valves. The ability of CoNS to proliferate in foreign material is mediated in part by the organism’s ability to create biofilm, which is an aggregate of bacteria encased in extracellular material. Biofilm is relatively impermeable to host defenses and antibiotics. Repeating a blood culture is not necessary for the neonate in this vignette because there is enough information provided to assume that the result represents contamination, rather than actual bacteremia. It is not necessary to add vancomycin to cover the organism identified in culture because it represents a blood culture contaminant. PREP Pearls • Distinguishing coagulase-negative staphylococci blood culture contamination from bacteremia requires consideration of the clinical context, including risk factors and symptomatology. • Risk factors for coagulase-negative staphylococci infection include prematurity (especially in infants who weigh < 1,500 g) and foreign devices or material. • Coagulase-negative staphylococci can proliferate on foreign material due to its ability to create biofilm, which is relatively impermeable to host defenses and antibiotics. ABP Content Specifications(s) • Understand that a positive culture for coagulase-negative staphylococci may represent specimen contamination or infection • Understand the association of coagulase-negative staphylococcal infections with the presence of central venous catheters or other foreign bodies

American academy of pediatrics

852

PREP ® Self-Assessment

PREPSA 2020

Suggested Readings • American Academy of Pediatrics. Coagulase-negative staphylococcal infections. In: Kimberlin DW, Brady MT, Jackson MA, Long SS, eds. Red Book: 2018 Report of the Committee on Infectious Diseases. 31st ed. Itasca, IL: American Academy of Pediatrics; 201:746748. https://redbook.solutions.aap.org/chapter.aspx?sectionId=189640186&bookId=2205 &resultClick=1. • Becker K, Heilmann C, Peters G. Coagulase-negative staphylococci. Clin Microbiol Rev. 2014;27(4):870-926. doi:10.1128/CMR.00109-13.

American academy of pediatrics

853

PREP ® Self-Assessment

PREPSA 2020

Question 237 A lethargic 2-year-old child is brought to the emergency department after his grandmother called emergency medical services to her home. The grandmother found the child lying on the floor of the bathroom in a puddle of vomit with multiple medications strewn about the room. She did not directly witness him ingest anything, but she believes he did because he was in his usual state of health when she put him down for a nap on her bed 2 hours earlier. She has been unable to wake him. The child has a temperature of 38.3°C, heart rate of 138 beats/min, respiratory rate of 34 breaths/min, blood pressure of 82/54 mm Hg, and oxygen saturation of 100% on room air. He is in no distress, and there are no signs of trauma. He is only minimally responsive to pain. His pupils are equal, round, and reactive, and he has moist mucous membranes. Heart, lung, and abdominal examination findings are normal. His skin is warm and he smells faintly of wintergreen. A capillary blood glucose level is 50 mg/dL (2.8 mmol/L), and an electrocardiogram shows sinus tachycardia. Chest and abdominal radiographs are normal. Of the following, the blood test MOST likely to confirm the suspected diagnosis is a(n) A. acetaminophen level B. ibuprofen level C. iron level D. salicylate level

American academy of pediatrics

854

PREP ® Self-Assessment

PREPSA 2020

Correct Answer: D The child in this vignette presents with an acute intoxication, an all too common occurrence. Bathrooms, kitchens, and diaper bags are common locations of over-the-counter medications, such as acetaminophen, ibuprofen, aspirin, and iron-containing vitamins, all of which can be deadly if ingested at supratherapeutic levels. Salicylates are found in over-the-counter products that have a wintergreen odor including topical analgesics (methyl salicylate) and anti-indigestion medications (bismuth subsalicylate). Oil of wintergreen is a highly concentrated methyl salicylate solution that has a variety of uses ranging from topical analgesics to fragrances. Just a few milliliters of oil of wintergreen can be lethal to a young child. Salicylism presents primarily with gastrointestinal symptoms (nausea, vomiting, and gastrointestinal hemorrhage) and neurologic symptoms (confusion, agitation, seizures, and coma). Severe intoxication can cause hyperthermia, hypoglycemia, and hypokalemia and affect the pulmonary, renal, and hematologic systems. Salicylism typically causes a mixed acid-base disorder with a metabolic acidosis with an elevated anion gap and a respiratory alkalosis. However, younger children are often not as hyperpneic and will thus only present with a metabolic acidosis and not the respiratory component. Symptoms of salicylism will develop with ingestions of more than 150 mg/kg, while ingestions of more than 300 mg/kg typically lead to severe symptoms. Salicylates are readily detected with a serum blood test. Management of a salicylate overdose begins with stabilization of the airway, breathing, and circulation. Activated charcoal should be administered if aspiration is not a concern. Aggressive intravenous fluid resuscitation is done with the addition of sodium bicarbonate to promote urinary alkalinization to facilitate salicylate excretion. Hemodialysis may be indicated in extremely severe cases of salicylism. Both acetaminophen and iron toxicity present with similar gastrointestinal and neurological symptoms as salicylism, although the odor of wintergreen found on the boy in the vignette will be absent. Additionally, acetaminophen toxicity typically does not cause a change in glucose levels, and the initial stages of iron ingestion are more likely to cause hyperglycemia. Acetaminophen is a common co-ingestant with salicylates in both accidental ingestions in young children and intentional ingestions in adolescents, so an acetaminophen level should be determined whenever salicylism is suspected. Nonsteroidal anti-inflammatory drug toxicity presents similarly to salicylism with gastrointestinal and neurological symptoms and can cause severe symptoms at ingestions of more than 400 mg/kg; however, serum ibuprofen testing is not readily available. PREP Pearls • Gastrointestinal and neurological symptoms are the hallmarks of salicylism. • Salicylism causes a mixed acid-base disorder with a respiratory alkalosis and an elevated anion-gap metabolic acidosis. • Management of salicylism consists of intravenous fluids and urinary alkalinization to facilitate salicylate excretion.

American academy of pediatrics

855

PREP ® Self-Assessment

PREPSA 2020

ABP Content Specifications(s) • Recognize the signs and symptoms of nonsteroidal anti-inflammatory drug toxicity, and manage appropriately • Recognize the signs and symptoms of salicylate toxicity, and manage appropriately Suggested Readings • American College of Medical Toxicity. Guidance document: management priorities in salicylate toxicity. J Med Toxicol. 2015;11(1):149-152. doi:10.1007/s13181-013-0362-3. • Fine JS. Poisoning. In: McInerny TK, Adam HM, Campbell DE, DeWitt TG, Foy JM, Kamat DM, eds. American Academy of Pediatrics Textbook of Pediatric Care. 2nd ed. Elk Grove Village, IL: American Academy of Pediatrics; 2017:2924-2950. Pediatric Care Online. • Lugassy DM. Salicylates. In: Nelson LS, Howland MA, Lewin NA, Smith SW, Goldfrank LR, Hoffman RS, eds. Goldfrank’s Toxicologic Emergencies. 11th ed. New York, NY: McGraw Hill; 2019:555-567. • Rakovchik EE, Fein DM. Nonsteroidal anti-inflammatory drug and salicylate poisoning. Pediatr Rev. 2016;37(1):48-50. doi:10.1542/pir.2015-0052.

American academy of pediatrics

856

PREP ® Self-Assessment

PREPSA 2020

Question 238 A 15-year-old previously healthy adolescent girl is brought to the emergency department with a 1-day history of intermittent right flank pain and visible blood in the urine. Her pain is moderate intensity and radiates to the front of the abdomen and pelvis. She has bright red urine with small clots. She has no fever or burning with urination but has urgency to void. She has a heart rate of 104 beats/min, respiratory rate of 14 breaths/min, and blood pressure of 100/60 mm Hg. She has no guarding or rebound tenderness during abdominal examination. The rest of the physical examination findings are unremarkable. Results of a spot urinalysis with microscopy are shown: Laboratory test

Result

pH

7.0

Specific gravity

1.020

Leukocyte esterase Positive Nitrite

Negative

Blood

3+

Protein

1+

Red blood cells

> 100/HPF

White blood cells

> 5-10/HPF

Of the following, the MOST likely diagnosis in this patient is A. acute glomerulonephritis B. hemorrhagic cystitis C. nephrolithiasis D. urinary tract infection

American academy of pediatrics

857

PREP ® Self-Assessment

PREPSA 2020

Correct Answer: C The patient in this vignette has right flank pain radiating to the suprapubic region, gross hematuria with clots, and a urinalysis showing minimal pyuria; these findings favor a diagnosis of an acute episode of nephrolithiasis or renal stone. Nephrolithiasis is increasingly seen in children. The majority of children in the adolescent age group present with the classic symptom of flank pain or renal colic. Older children, like adults, have ureteral stones which can cause obstruction and present with renal colic as the stone tries to pass through the ureterovesical junction. However, children younger than 5 years may be asymptomatic; in this case, nephrolithiasis is detected when abdominal imaging is done for other clinical indications. Younger children may have stones within the kidney and can present with recurrent abdominal pain. The other presenting features of nephrolithiasis include gross hematuria, dysuria, urinary urgency, and sometimes urinary tract infection. The evaluation of a child with nephrolithiasis includes a detailed history of urinary tract anomalies (obstruction, renal cyst), recurrent urinary tract infection (Proteus, Klebsiella), metabolic conditions (malabsorption syndrome, ketogenic diet), medications (sulfadiazine, indinavir), and family history. Physical examination includes measurement of blood pressure and growth parameters and abdominal examination to look for a mass suggestive of urinary obstruction or for other causes of abdominal pain. The initial laboratory studies are urinalysis with microscopic examination for the presence of crystals, urine culture to rule out urinary tract infection, and renal function tests. The adolescent girl in this vignette has renal colic and favors a diagnosis of nephrolithiasis. Acute glomerulonephritis usually presents with gross hematuria without clots, proteinuria, hypertension, and azotemia. In acute glomerulonephritis, hematuria originates from the glomerulus and usually presents with cola- or tea-colored urine. Urine microscopy may show red blood cell casts that are diagnostic of acute glomerulonephritis. However, in nephrolithiasis, hematuria is from the collecting system, and the urine is bright red and has clots. Acute hemorrhagic cystitis presents with suprapubic pain or may be painless and is associated with bright red urine. The patient in this vignette has a history of flank pain, however acute urinary tract infection (pyelonephritis) is unlikely because there is no history of fever or significant pyuria on urinalysis. The diagnosis of nephrolithiasis is confirmed by abdominal imaging. Renal ultrasonography is the initial test, but it may not detect small stones or ureteral stones. A plain abdominal radiograph helps in detecting radiopaque stones. Noncontrast helical computed tomography is the most sensitive test for detection of nephrolithiasis in children. The acute management of nephrolithiasis includes pain control, aggressive fluid administration, and facilitating passage or removal of stone. The long-term management includes evaluation of the type of nephrolithiasis and the risk factors for stone recurrence. Accordingly, dietary interventions and medications are recommended to prevent recurrent nephrolithiasis.

American academy of pediatrics

858

PREP ® Self-Assessment

PREPSA 2020

PREP Pearls • Young children with nephrolithiasis may be asymptomatic or have recurrent abdominal pain. • Evaluation of nephrolithiasis includes history of urinary tract abnormalities, recurrent urinary tract infections, metabolic conditions, medications, and family history • Noncontrast computed tomography is the most sensitive imaging modality for confirming nephrolithiasis. ABP Content Specifications(s) • Plan the evaluation of urinary tract stones in patients of various ages • Recognize the signs and symptoms of urinary tract stones in patients of various ages Suggested Readings • Gellin CE. Urinary tract stones. Pediatr Rev. 2019;40(3):154-156. doi:10.1542/pir.20170235. • McKay CP. Renal stone disease. Pediatr Rev. 2010;31(5):179-188. doi:10.1542/pir.31-5179. • Tayaba M, Kamat D. Pediatric nephrolithiasis: a review. Pediatr Ann. 2017;46(6):e242e244. doi:10.3928/19382359-20170517-02.

American academy of pediatrics

859

PREP ® Self-Assessment

PREPSA 2020

Question 239 A neonate with ambiguous genitalia is being evaluated in the newborn nursery. The neonate was born today via an uncomplicated spontaneous vaginal delivery at 39 weeks of gestation, with a birthweight of 3.2 kg. The pregnancy was uncomplicated and maternal medication consisted only of prenatal vitamins. The neonate is breastfeeding well and has normal vital signs for age. Physical examination findings are normal except for the genitalia. The neonate has a 1.5-cm long phallic structure, complete fusion of the labioscrotal folds, and no palpable gonads (Item Q239). Pelvic ultrasonography shows a uterus. A karyotype is ordered.

Item Q239: Physical examination findings for the neonate described in the vignette. Reprinted with permission from Antal Z, Zhou P. Pediatr Rev. 2009;30(7):e50.

Of the following, the BEST next test to determine this neonate's diagnosis is A. adrenocorticotropic hormone B. cortisol C. 17-hydroxyprogesterone D. testosterone

American academy of pediatrics

860

PREP ® Self-Assessment

PREPSA 2020

Correct Answer: C The neonate described in the vignette has congenital adrenal hyperplasia (CAH) arising from 21hydroxylase deficiency. The findings of no palpable gonads and a uterus noted on ultrasonography are consistent with a virilized female or 46,XX disorder of sex development (DSD). Congenital adrenal hyperplasia arising from 21-hydroxylase deficiency is the most common 46,XX DSD. The best next test to determine the diagnosis for the neonate in the vignette is a 17-hydroxyprogesterone level. 17-hydroxyprogesterone is an intermediate steroid in the adrenal glucocorticoid pathway, which is a substrate for 21-hydroxylase and builds up in 21hydroxylase deficiency. This defect in adrenal glucocorticoid production results in an elevated adrenocorticotropic hormone (ACTH) level, which stimulates the adrenal gland to make excess androgen. Adrenal mineralocorticoid production is also negatively affected by this enzyme defect. The excess androgen exposure in utero causes virilization of female external genitalia. Congenital adrenal hyperplasia should be suspected in a neonate with virilized, atypical genitalia and no palpable gonads. Male (46,XY) neonates with 21-hydroxylase deficiency have normal male genitalia. Unlike affected females, males have no physical findings to identify them at birth. Therefore, they are at risk of presenting with an adrenal salt-wasting crisis, which occurs after age 1 week. Signs and symptoms may include failure-to-thrive, vomiting, hypotension, hyponatremia, hyperkalemia, hypoglycemia, and metabolic acidosis.The newborn screen is critically important for identification of male infants with CAH before an adrenal salt-wasting crisis occurs. If measured, an ACTH level would be high, cortisol level would be low, and testosterone level would be high in CAH, but none of these is the best next diagnostic test for this neonate, because the results would not be specific for the diagnosis. PREP Pearls • Congenital adrenal hyperplasia should be suspected in a neonate with virilized, atypical genitalia and no palpable gonads. • A 17-hydroxyprogesterone level is the first-line diagnostic test for congenital adrenal hyperplasia arising from 21-hydroxylase deficiency. • The newborn screen is critically important for identifying male infants with congenital adrenal hyperplasia before an adrenal salt-wasting crisis occurs. Males have no physical findings to identify them at birth. MOCA-Peds Objective • Recognize and manage ambiguous genitalia in a neonate.

American academy of pediatrics

861

PREP ® Self-Assessment

PREPSA 2020

ABP Content Specifications(s) • Plan the appropriate management of congenital adrenal hyperplasia, including that associated with an adrenal crisis • Plan the appropriate diagnostic evaluation of congenital adrenal hyperplasia, including during the perinatal period • Recognize the clinical features associated with congenital adrenal hyperplasia Suggested Readings • Antal Z, Zhou P. Congenital adrenal hyperplasia: diagnosis, evaluation, and management. Pediatr Rev. 2009;30(7):e49-e57. doi:10.1542/pir.30-7-e49. • Auron M, Raissouni N. Adrenal insufficiency. Pediatr Rev. 2015;36(3):92-102. doi:10.1542/pir.36-3-92. • Loomba-Albrecht LA, Styne DM. Disorders of sex development. In: McInerny TK, Adam HM, Campbell DE, DeWitt TG, Foy JM, Kamat DM, eds. American Academy of Pediatrics Textbook of Pediatric Care. 2nd ed. Elk Grove Village, IL: American Academy of Pediatrics; 2017:1968-1976. Pediatric Care Online. • Speiser PW, Arlt W, Auchus RJ, et al. Congenital adrenal hyperplasia due to steroid 21hydroxylase deficiency: an Endocrine Society clinical practice guideline. J Clin Endocrinol Metab. 2018;103(11):4043-4088. doi:10.1210/jc.2018-01865.

American academy of pediatrics

862

PREP ® Self-Assessment

PREPSA 2020

Question 240 A 4-month-old male infant is seen for a health supervision visit. His mother has no concerns and feels he is eating well and developing normally. His vital signs are within normal limits. He has a normal sexual maturity rating stage 1 penis. The right testicle is palpated in the scrotum, but the left testicle cannot be palpated. The remainder of his physical examination findings are normal. The patient is referred to a urologist, and the diagnosis is explained to his mother To reduce the risk of complications, treatment for this condition should be completed by the age of A. 6 months B. 12 months C. 18 months D. 24 months

American academy of pediatrics

863

PREP ® Self-Assessment

PREPSA 2020

Correct Answer: B The most recent international guidelines state that orchiopexy to treat cryptorchidism or undescended testicle should be completed by 12 months of age. During normal development, the testes form in the abdomen and then descend into the scrotum. Having one or both testes undescended at birth is not uncommon (2%-9% of male neonates), but only 1% remain undescended at 3 months of age. Those infants with testes that remain undescended require surgery to bring the testicle into the scrotum (orchiopexy) or remove the testicle (orchiectomy) because they will not spontaneously descend from the abdomen into the scrotum after this age. If a testicle is palpated in the scrotum at times or easily brought into the scrotum on examination, it is considered a retractile testicle and not an undescended testicle. This does not require surgical correction but does increase the risk of developing an ascending or acquired cryptorchidism later in life. A retractile testicle is likely a result of a lack of formation of attachment within the scrotum after the testes descend. An undescended testicle that remains in the abdomen is exposed to body temperature instead of the preferred 2°F to 3°F cooler temperature of the scrotum. The increased temperature can inhibit differentiation of the germ cells as early as 4 months of age and can lead to decreased fertility and increased risk of malignancy. Due to the location in the abdominal cavity, if a malignancy develops in an undescended testis, the diagnosis is often delayed until there are systemic symptoms or the mass is large. There is also an increased risk of testicular torsion, which can be difficult to diagnose due to the intra-abdominal location. Due to the risk of complications, especially decreased fertility and malignancy, waiting until 18 or 24 months for surgical treatments is not recommended. Surgery can be completed as early as 6 months of age. PREP Pearls • Individuals with undescended testes are at increased risk of testicular malignancy. • Individuals with untreated cryptorchidism are at risk of decreased fertility. • Surgical treatment of undescended testes is recommended between 6 and 12 months of age. ABP Content Specifications(s) • Differentiate the findings associated with undescended testes from those of retractile testes • Recognize complications associated with undescended testes • Plan the appropriate management of undescended testes

American academy of pediatrics

864

PREP ® Self-Assessment

PREPSA 2020

Suggested Readings • Inouye B, Tourchi A, Gearhart JP. Hypospadias, epispadias, and cryptorchidism. In: McInerny TK, Adam HM, Campbell DE, DeWitt TG, Foy JM, Kamat DM, eds. American Academy of Pediatrics Textbook of Pediatric Care. 2nd ed. Elk Grove Village, IL: American Academy of Pediatrics; 2017:2180-2184. Pediatric Care Online. • Schneuer F, Holland A, Pereira G, Jamieson S, Bower C, Nassar N. Age at surgery and outcomes of an undescended testis. Pediatrics. 2016:137(2):e220152768. doi:10.1542/peds.2015-2768. • Urology Care Foundation. What are undescended testicles (cryptorchidism)? https://www.urologyhealth.org/urologic-conditions/cryptorchidism.

American academy of pediatrics

865

PREP ® Self-Assessment

PREPSA 2020

Question 241 A 4-month-old infant is seen for a routine health supervision visit. She recently moved from another state. Her parents have no concerns about her physical or mental development. Her medical record indicates that she was born at full term after an unremarkable pregnancy and delivery and has had no medical problems. She appears healthy with no apparent developmental delays and average growth for all parameters. Specific findings on active and passive range of motion of the neck are shown in Item Q241.

Item Q241: Image of the examination findings described in the vignette. Abbreviations: Arom, active range of motion; Prom, passive range of motion; Lrot, left rotation; Lsb, left side bend; Rrot, right rotation; Rsb, right side bend. Reprinted with permission from Kuo AA, Tritasavit S, Graham Jr JM. Pediatr Rev. 2014;35(2):83.

American academy of pediatrics

866

PREP ® Self-Assessment

PREPSA 2020

Of the following, the physical examination findings in this patient are MOST suggestive of A. bilateral torticollis B. normal range of motion C. unilateral torticollis on the left D. unilateral torticollis on the right

American academy of pediatrics

867

PREP ® Self-Assessment

PREPSA 2020

Correct Answer: A The examination findings for the infant in the vignette are most suggestive of bilateral torticollis. Normal passive rotational range of motion of the neck for children ages 0 to 3 years is about 110° on each side. Normal lateral flexion of the neck in this age group is about 70° on each side. This infant has limited range of motion on both sides. She does not have normal range of motion of her neck. Likewise, unilateral torticollis does not accurately describe her examination findings. Torticollis refers to the symptom of abnormal tilting and rotation of the head. The tilt and rotation in torticollis are most commonly directed towards opposite sides, but may be towards the same side. The latter is much less common and may be caused by congenital vertebral malformations, tumors, or eye abnormalities. The most common cause of torticollis is congenital muscular torticollis (CMT) in which the sternocleidomastoid (SCM) muscle is abnormal, usually resulting in the head tilting toward the affected side and rotated away from the affected side. The definitive cause of CMT is unknown but presumed to be from delivery trauma and/or intrauterine positioning. Congenital muscular torticollis may not be clinically evident until 2 weeks of age or later. At that time, a palpable mass in the affected SCM muscle may be found. This mass represents fibrosis of the muscle and increases in size until about 1 month of age, after which it slowly disappears, resulting in muscle contracture. Other examination findings of CMT include facial asymmetry, as shown in Item C241A. Mild CMT, however, may not exhibit these classic findings and may present only as positional plagiocephaly. Congenital muscular torticollis is a major risk factor for positional plagiocephaly; likewise, positional plagiocephaly is a major risk factor for CMT.

American academy of pediatrics

868

PREP ® Self-Assessment

PREPSA 2020

Item C241A: Facial asymmetry characteristic of torticollis. Reprinted with permission V. Wall

Treatment of CMT is usually conservative, involving exercises with passive and active stretching of the SCM and tummy time. Botulinum toxin injections and orthotics may also be considered for more severe CMT. If, after 6 months of therapy, CMT persists, physicians should consider obtaining radiography to assess for other causes of torticollis. If CMT persists after 1 year of age, surgery may be necessary. Other causes of torticollis are in Item C241B. Treatment of these causes are aimed at the underlying etiologies.

American academy of pediatrics

869

PREP ® Self-Assessment

PREPSA 2020

Item C241B: Differential Diagnosis of Torticollis. Adapted and reprinted from Ozuah PO, Skae CC. Torticollis. In: McInerny TK, Adam HM, Campbell DE, DeWitt TG, Foy JM, Kamat DM, eds. American Academy of Pediatrics Textbook of Pediatric Care. 2nd ed. Elk Grove Village, IL: American Academy of Pediatrics; 2017:1651.

PREP Pearls • Children younger than 3 years have a normal passive neck rotation of about 100° and a lateral flexion of about 70° on each side. • Infants with congenital muscular torticollis that persists despite therapy should be evaluated for other causes of torticollis. ABP Content Specifications(s) • Plan the appropriate management of torticollis • Formulate a differential diagnosis of torticollis American academy of pediatrics

870

PREP ® Self-Assessment

PREPSA 2020

Suggested Readings • Kuo AA, Tritasavit S, Graham Jr JM. Congenital muscular torticollis and positional plagiocephaly. Pediatr Rev. 2014;35(2):79-87. doi:10.1542/pir.35-2-79. • Mistovich RJ, Spiegel DA. Torticollis. In: Kliegman RM, St Geme JW III, Blum NJ, Shah SS, Tasker RC, Wilson KM, eds. Nelson Textbook of Pediatrics. 21st ed. Philadelphia, PA: Elsevier; 2020:3646-3647. • Ozuah PO, Skae CC. Torticollis. In: McInerny TK, Adam HM, Campbell DE, DeWitt TG, Foy JM, Kamat DM, eds. American Academy of Pediatrics Textbook of Pediatric Care. 2nd ed. Elk Grove Village, IL: American Academy of Pediatrics; 2017:16501653. Pediatric Care Online.

American academy of pediatrics

871

PREP ® Self-Assessment

PREPSA 2020

Question 242 A 14-year-old adolescent girl is brought to the emergency department with a 1-day history of fever and left eye swelling, with an inability to open the eye. Two weeks ago, she had a mild upper respiratory tract infection. On physical examination, she is alert and interactive with a temperature of 39.2°C. Examination of the left eye reveals periorbital redness and swelling, proptosis, conjunctival edema, and limited extraocular movements. The remainder of the physical examination findings are normal. Laboratory findings are notable for a white blood cell count of 18,000/μL (18 × 109/L) and elevated C-reactive protein level. Of the following, the MOST likely source of this patient's infection is the A. bloodstream B. ethmoid sinus C. maxillary sinus D. skin

American academy of pediatrics

872

PREP ® Self-Assessment

PREPSA 2020

Correct Answer: B The adolescent girl in the vignette has a fever, unilateral periorbital swelling, and redness, with proptosis, restricted extraocular movements, and conjunctival chemosis after an upper respiratory tract infection. These physical examination findings and patient history are consistent with a diagnosis of orbital (postseptal) cellulitis. Orbital cellulitis is a serious infection of the orbital tissue posterior to the orbital septum, usually complicating sinusitis (especially the ethmoid sinus). The orbital septum is a protective layer of periosteal fibrous connective tissue extending to the upper and lower eyelids. The ethmoid sinus is separated from the orbit by a thin medial orbital wall (lamina papyracea). The microbiology of orbital cellulitis includes organisms associated with acute or chronic sinusitis including Streptococcus pneumoniae, Staphylococcus aureus, Streptococcus anginosus, Moraxella catarrhalis, nontypeable Haemophilus influenzae, and upper respiratory tract anaerobes. The clinical manifestations of orbital cellulitis include proptosis, conjunctival chemosis, ophthalmoplegia, and pain with eye movement (Item C242A). In patients with marked eye swelling, it may be difficult to distinguish periorbital cellulitis from orbital cellulitis by physical examination alone because of difficulties in assessing visual acuity or extraocular movements. All patients with a suspected diagnosis of orbital cellulitis should undergo orbital computed tomography imaging with contrast to confirm the diagnosis and exclude orbital complications such as subperiosteal abscess and orbital abscess (Item C242B).

Item C242A: Clinical manifestations of orbital cellulitis.

American academy of pediatrics

873

PREP ® Self-Assessment

PREPSA 2020

Reprinted with permission from Kimberlin DW, et al, eds. Red Book Online. Itasca, IL: American Academy of Pediatrics; 2018.

Item C242B: Contrast-enhanced orbital computed tomography scan showing orbital cellulitis. Left: proptosis, phlegmon formation, left paranasal sinus opacification, intraorbital free air (solid arrow), and stranding of the intraconal fat along the orbital floor (dotted arrow) Reprinted with permission from Hauser A, Fogarasi S. Periorbital and orbital cellulitis. Pediatr Rev. 2010;31(6):247. Management of orbital cellulitis warrants an urgent and multidisciplinary approach, including specialist consultations from pediatric infectious disease, otolaryngology, and ophthalmology. Empiric antimicrobial therapy of orbital cellulitis should be rapidly initiated; intravenous ampicillin-sulbactam and vancomycin are recommended to treat methicillin-resistant S aureus and other microorganisms associated with sinusitis. A 5- to 7-day course of parenteral antibiotic therapy (until the eye examination results are greatly improved), followed by 2 weeks of oral antibiotic therapy, is recommended. Surgical drainage of the affected sinus and abscess is indicated in patients with large, well-defined subperiosteal and/or orbital abscess, visual impairment, or complete ophthalmoplegia. Medical management alone is often adequate in patients with orbital cellulitis associated with phlegmon or small subperiosteal abscess, if the patient clinically improves within 48 to 72 hours. Distinguishing periorbital cellulitis from orbital cellulitis is critical to avoid potentially serious complications of the latter, including vision loss and intracranial infection. The microbiology, pathogenesis, diagnostic evaluation, and management of periorbital cellulitis and orbital cellulitis are very different. Periorbital cellulitis is an infection of the eyelid and surrounding skin and soft tissue anterior to the orbital septum. It occurs most commonly in preschool-aged children and American academy of pediatrics

874

PREP ® Self-Assessment

PREPSA 2020

typically begins at the site of local skin trauma (eg, an insect bite) which becomes infected. The most common etiologic agents causing periorbital cellulitis include S aureus (including methicillin-resistant S aureus) and group A Streptococcus. In the era of H influenzae type b vaccination and pneumococcal conjugate vaccination, bacteremic periorbital cellulitis is very unusual. Periorbital cellulitis may be treated with appropriate oral antibiotics for the likely causative organisms. PREP Pearls • Orbital cellulitis is a serious infection of the orbital tissue posterior to the orbital septum usually complicating sinusitis (especially ethmoid sinus). • The microbiology of orbital cellulitis includes organisms commonly associated with acute or chronic sinusitis. • Management of orbital cellulitis warrants an urgent and multidisciplinary approach with consultations with specialists in pediatric infectious disease, otolaryngology, and ophthalmology. ABP Content Specifications(s) • Recognize pathogens commonly associated with orbital cellulitis • Recognize and differentiate the clinical and radiologic findings associated with orbital cellulitis and periorbital (preseptal) cellulitis • Plan the appropriate management of orbital cellulitis, including associated complications • Plan the appropriate diagnostic evaluation of orbital cellulitis Suggested Readings • Adamson J, Waterfield T. Fifteen-minute consultation: preseptal and orbital cellulitis [published online ahead of print June 22]. Arch Dis Child Educ Pract Ed. 2019;104(2):79-83. doi:10.1136/archdischild-2017-314297. • Hauser A, Fogarasi S. Periorbital and orbital cellulitis. Pediatr Rev. 2010;31(6):242-249. doi:10.1542/pir.31-6-242. • Wald ER. Preseptal and orbital cellulitis. In: McInerny TK, Adam HM, Campbell DE, DeWitt TG, Foy JM, Kamat DM, eds. American Academy of Pediatrics Textbook of Pediatric Care. 2nd ed. Elk Grove Village, IL: American Academy of Pediatrics; 2017:2537-2543. Pediatric Care Online. • Wong SJ, Levi J. Management of pediatric orbital cellulitis: a systematic review. Int J Pediatr Otorhinolaryngol. 2018;110:123-129. doi:10.1016/j.ijporl.2018.05.006.

American academy of pediatrics

875

PREP ® Self-Assessment

PREPSA 2020

Question 243 A 14-year-old girl is being evaluated for concern about evolving seizures. She has a history of partial complex seizure disorder, which has been well-controlled with medication. She recently had a prolonged episode of abnormal motor movements that included backward arching of her head, neck, and back, and pelvic thrusting. Her eyes were closed tightly during the episode, but she became alert quickly when the movements subsided. The adolescent also has an anxiety disorder. She has been diligent about taking her antiepileptic and antianxiety medications as prescribed. She has otherwise been well. She has a family history of epilepsy, hypertension, anxiety, and diabetes, and a cousin who recently died of meningitis. On physical examination, her vital signs are within normal limits. The remainder of her physical and neurologic examination findings are normal. Of the following, the test or study MOST likely to reveal this girl's diagnosis is A. antiepileptic medication level B. brain magnetic resonance imaging C. cerebrospinal fluid studies D. video electroencephalography

American academy of pediatrics

876

PREP ® Self-Assessment

PREPSA 2020

Correct Answer: D The girl in the vignette is demonstrating behaviors consistent with psychogenic nonepileptic seizure (PNES). Video electroencephalography (EEG) is the gold standard test for this diagnosis and will have normal findings at the time of the apparent seizure behavior. The girl is unlikely to have low blood levels of antiepileptic medication if she has been taking her medication as prescribed. Brain magnetic resonance imaging can be used to evaluate for a structural brain abnormality causing seizures, but is not diagnostic for PNES. She is unlikely to have meningitis given her normal vital signs and physical examination findings. In conversion disorder (functional neurologic symptom disorder), neurologic symptoms (motor or sensory) are present, which are not consistent with recognized pathophysiology of medical or neurologic conditions. These symptoms cause distress or impairment in functioning and may include weakness, paralysis, posturing, abnormal movements or gait, tremors, seizures, altered speech, visual disturbance, or altered sensation. Conversion disorder can be seen in early childhood, but is unusual before 10 years of age. Girls are more likely to be affected than boys. Associated features include symptom onset at the time of a stressor, conflict, or trauma and dissociative symptoms (eg, depersonalization, derealization). While "la belle indifference" (lack of concern about a serious symptom) and concerns about secondary gain have been associated with conversion disorder, they are also seen in other conditions. The differential diagnosis of conversion symptoms includes a neurologic or medical disorder, somatic symptom disorder, factitious disorder, malingering, dissociative disorder, body dysmorphic disorder, depressive disorder, and panic disorder. Some of these conditions, such as neurologic disorder (eg, epilepsy), panic disorder, and dissociative disorder may co-occur with conversion disorder. Patients with seizures are more likely to have PNES. A comprehensive history is essential to evaluate the concerning symptoms. Neurologic symptoms should be explored in detail, paying close attention to the circumstances at the onset of the symptoms, characteristics of the symptoms, and the presence of dissociative symptoms. In conversion disorder, symptoms often start suddenly and a feeling of being disconnected from oneself or the environment may be present. Forward pelvic thrusting, side-to-side head or body movements, closed eyes resistant to opening, and a lack of postictal confusion are more often seen in PNES. The child with anesthesia due to conversion disorder may describe a pattern of sensory loss that does not correspond to known sensory nerve pathways. Family history is likely to be positive for medical and psychiatric disorders in close relatives of children with conversion disorder. Any psychiatric conditions (eg, anxiety, depression) and neurologic disorders (eg, epilepsy) in the child should be identified. Psychosocial stressors such as school problems, family disruption, interpersonal conflicts, and traumatic events should be ascertained. A psychiatric evaluation can identify mental health conditions that coexist or underlie the patient’s presentation. A complete physical and neurologic examination is required to determine if the findings are consistent and the symptoms are anatomically and physiologically feasible. The child with weakness caused by conversion disorder may have varying symptoms during the visit. Laboratory and radiologic studies may be indicated, based on the presenting symptoms. Video EEG can distinguish between PNES and epileptic seizures. American academy of pediatrics

877

PREP ® Self-Assessment

PREPSA 2020

PREP Pearls • In conversion disorder (functional neurologic symptom disorder), neurologic symptoms (motor or sensory) are present, which are not consistent with recognized pathophysiology of medical or neurologic conditions. • Forward pelvic thrusting, side-to-side head or body movements, closed eyes resistant to opening, and a lack of postictal confusion are findings suggestive of psychogenic nonepileptic seizures rather than epileptic seizures. • Neurologic disorder (eg, epilepsy), panic disorder, and dissociative disorder may cooccur with conversion disorder. Patients with epileptic seizures are more likely to have psychogenic nonepileptic seizures. ABP Content Specifications(s) • Plan an appropriate evaluation of psychosomatic disorders • Recognize the various features associated with conversion disorders • Identify the various features associated with psychosomatic disorders • Formulate an appropriate differential diagnosis of conversion symptoms Suggested Readings • American Psychiatric Association. Diagnostic and Statistical Manual of Mental Disorders, Fifth Edition. Arlington, VA: American Psychiatric Association; 2013. • Patel H, Dunn DW et al. Psychogenic nonepileptic seizures (pseudoseizures). Pediatr Rev. 2011;32(6):e66-e72. doi:10.1542/pir.32-6-e66. • Prazar GE. Conversion reactions and hysteria. In: McInerny TK, Adam HM, Campbell DE, DeWitt TG, Foy JM, Kamat DM, eds. American Academy of Pediatrics Textbook of Pediatric Care. 2nd ed. Elk Grove Village, IL: American Academy of Pediatrics; 2017:1927-1933. Pediatric Care Online.

American academy of pediatrics

878

PREP ® Self-Assessment

PREPSA 2020

Question 244 A 3-year-old boy was missing for approximately 15 minutes before he was found face down in his family's above-ground pool. When emergency medical services arrived, he was unresponsive, pulseless, and apneic. Of the following, the goal with the MOST positive impact on outcome for this boy is to A. prevent seizures with antiepileptic medications B. prevent water aspiration with cricoid pressure C. restore spontaneous circulation with cardiopulmonary resuscitation D. reverse hypothermia with rapid rewarming

American academy of pediatrics

879

PREP ® Self-Assessment

PREPSA 2020

Correct Answer: C The boy in this vignette has cardiac arrest from drowning. Although there are many factors impacting outcome following drowning, the most important is restoration of spontaneous circulation. The moment the airway is submerged below the surface of the water, reflex laryngospasm is initiated by the contact of liquid with the larynx. As the larynx relaxes after a period of hypoxia and hypercarbia, water is aspirated into the lungs. When the water reaches the alveoli, hypoxia occurs due to ventilation-perfusion mismatch. The most important consequence of drowning is decreased oxygen delivery to the tissues. Decreased myocardial oxygen delivery can result in asphyxial cardiac arrest, as has occurred in the boy in this vignette. Lifesaving return of spontaneous circulation depends on interrupting the drowning process by rescue from the water and on the success of cardiopulmonary resuscitation (CPR). Management of airway, breathing, and circulation is the immediate goal after the initial rescue from the water, which should occur as soon as possible. Protocols for basic life support should be followed as in any other circumstance requiring CPR. However, since cardiac arrest in drowning is precipitated by a respiratory etiology, it is important that CPR be performed in the traditional airway-breathing-circulation (ABC) sequence rather than the circulation-airway-breathing (CAB) sequence. Emergency medical services should be called immediately if the individual is not breathing. The airway should be opened using chin-lift and jaw-thrust maneuvers, and mouth-to-mouth breathing should be performed. If the lungs do not adequately expand despite appropriate ventilation technique, a foreign body should be suspected, and a subdiaphragmatic thrust could be applied. If the individual is not breathing, it is unlikely that there is a perfusing rhythm, thus chest compressions should be initiated. Recent protocols of basic life support have de-emphasized checking for a central pulse, since it can be unreliable and lead to less time providing vital chest compressions. When the equipment becomes available, bag-valve mask ventilation with 100% oxygen should be initiated if the individual is not breathing adequately. Higher than anticipated pressures during artificial ventilation may be required due to the reduced compliance of the edematous lungs. An automatic external defibrillator should be applied as soon as it becomes available if return of spontaneous circulation is not accomplished. A large quantity of water may be swallowed during drowning. Aspiration of water can vary because initially laryngospasm can protect the lungs from aspiration, but water can enter the lungs during prolonged periods of submersion. The most effective method of preventing further aspiration is timely removal from the water. The application of subdiaphragmatic pressure (ie, Heimlich maneuver) can dislodge a foreign body, but it is not helpful in clearing the lungs or airway of water. In fact, increased abdominal pressure may result in the regurgitation of swallowed water to the oropharynx and subsequent aspiration. Similarly, although it is sometimes used in rapid sequence intubation protocols, cricoid pressure has not been shown to prevent water aspiration in drowning victims during artificial ventilation. As in any cause of cardiac arrest, hypothermia in drowning can affect survival due to its adverse effects on cardiac rhythm. Although a pulseless, hypothermic drowning victim should be rewarmed, rapid rewarming has less of a positive effect on outcome than CPR. In fact, animal models and human studies of trauma and asphyxia have shown that hypothermia mitigates secondary brain injury. American academy of pediatrics

880

PREP ® Self-Assessment

PREPSA 2020

Good outcomes have been reported in children submerged in ice water (< 5°C) for prolonged periods. Although recent studies of therapeutic hypothermia in pediatric cardiac arrest victims have shown no benefit, once a core temperature of 30°C is reached, warming should not exceed 0.5°C/h to prevent rises in cerebral blood flow and intracranial pressure. Lastly, there is evidence that seizures in the post–cardiac arrest patient are deleterious due to increasing cerebral metabolic demand. Empiric antiepileptic medications may be warranted, however, this is of much lower priority compared to successful CPR and return of spontaneous circulation at the scene. PREP Pearls • In drowning, lifesaving return of spontaneous circulation depends on interrupting the drowning process by rescue from the water and on the success of cardiopulmonary resuscitation. • Since cardiac arrest in drowning is precipitated by a respiratory etiology, it is important that cardiopulmonary resuscitation be performed in the traditional airway-breathingcirculation (ABC) sequence rather than the circulation-airway-breathing (CAB) sequence. • Good outcomes have been reported in children submerged in ice water (< 5°C) for prolonged periods. In cases of hypothermic drowning, the patient should be quickly rewarmed to 30°C and then warming should not exceed 0.5°C/h to prevent rises in cerebral blood flow and intracranial pressure. ABP Content Specifications(s) • Plan the appropriate management of near-drowning • Understand the prognostic factors associated with near-drowning Suggested Readings • Dyson K, Morgans A, Bray J, Matthews B, Smith K. Drowning related out-of-hospital cardiac arrests: characteristics and outcomes. Resuscitation. 2013;84(8):1114-1118. doi:10.1016/j.resuscitation.2013.01.020. • Frankel LR. Drowning and near drowning (submersion injuries). In: McInerny TK, Adam HM, Campbell DE, DeWitt TG, Foy JM, Kamat DM, eds. American Academy of Pediatrics Textbook of Pediatric Care. 2nd ed. Elk Grove Village, IL: American Academy of Pediatrics; 2017:2826-2831. Pediatric Care Online. • Mtaweh H, Kochanek PM, Carcillo JA, Bell MJ, Fink EL. Patterns of multiorgan dysfunction after pediatric drowning. Resuscitation. 2015;90:91-96. doi:10.1016/j.resuscitation.2015.02.005. • Sarnaik AA, Lieh-Lai MW, Sarnaik AP. Drowning. In: Fuhrman BP, Zimmerman JJ, Clark RSB, et al, eds. Pediatric Critical Care. 5th ed. Philadelphia, PA: Elsevier; 2016:1572-1581. • Topjian AA, Berg RA, Bierens JJ, et al. Brain resuscitation in the drowning victim. Neurocrit Care. 2012;17(3):441-467. doi:10.1007/s12028-012-9747-4.

American academy of pediatrics

881

PREP ® Self-Assessment

PREPSA 2020

Question 245 A newborn is having a follow-up evaluation after a neonatal intensive care unit (NICU) stay. The child had tachypnea at birth, required support with oxygen and continuous positive airway pressure for several hours, and ultimately was diagnosed with transient tachypnea of the newborn. He was quickly weaned to room air over 2 days. He was breathing comfortably, latching on, and breastfeeding well before being discharged from the hospital. In the NICU, he was noted to have a cardiac murmur and echocardiography was performed, which showed a moderate ventricular septal defect (VSD) with left-to-right flow and a bicuspid aortic valve without stenosis or regurgitation. The mother reports that she was told that the VSD will likely require surgical closure, but wishes to know what needs to be done about the bicuspid aortic valve. Of the following, the MOST appropriate response to this mother is A. full anticoagulation at this time B. long-term pediatric cardiology follow-up C. no further treatment D. valve repair or replacement at the time of VSD surgery

American academy of pediatrics

882

PREP ® Self-Assessment

PREPSA 2020

Correct Answer: B The neonate in the vignette is known to have a moderate ventricular septal defect (VSD), as well as a bicuspid aortic valve (BAV) without stenosis or regurgitation. The VSD is likely to need surgical repair. There is no indication to intervene for the aortic valve at this time or to start anticoagulation. However, patients with BAV can develop regurgitation, stenosis, endocarditis, and aortopathy, and therefore need follow-up throughout life. Bicuspid aortic valve is one of the most common congenital heart defects. The normal aortic valve is trileaflet; in the setting of a BAV, there are 2 leaflets. This is thought to occur because of an abnormal fusion during embryonic development. Patients with BAV are likely to have some complication during their life. Adult studies have found that a bimodal distribution of surgical intervention is required at age 30 to 40 years or after age 50 years. Some children require intervention as well. Bicuspid aortic valve can be familial or sporadic. Because of the familial inheritance risk, first-degree relatives should be screened. Bicuspid aortic valve is also associated with genetic syndromes such as Turner syndrome. Ventricular septal defect (VSD) is also one of the most common congenital heart defects. It consists of a communication between the right and left ventricles via an opening in the interventricular septum. Ventricular septal defects are named based on their location along the septum (membranous, muscular, atrioventricular canal, conal septal). They are further described as small, medium, or large based on their size relative to the size of the aortic valve. A small VSD is unlikely to cause hemodynamic effects and if located in the muscular septum, may close spontaneously. Moderate and large defects will need to be surgically closed. Before closure, some children will have tachypnea and increased work of breathing because of the increased pulmonary blood flow. This can be medically treated with diuretics and higher caloric feeds. PREP Pearls • Patients with bicuspid aortic valve can develop regurgitation, stenosis, endocarditis, and aortopathy, and therefore need cardiology follow-up throughout life. • Ventricular septal defects are named based on their location (membranous, muscular, atrioventricular canal, conal septal) and size of the defect. • Moderate and large ventricular septal defects require surgical closure because of the impact of increased pulmonary blood flow. ABP Content Specifications(s) • Understand the natural history of a bicuspid aortic valve • Understand the natural history of ventricular septal defect

American academy of pediatrics

883

PREP ® Self-Assessment

PREPSA 2020

Suggested Readings • Freeze SL, Landis BJ, Ware SM, Helm BM. Bicuspid aortic valve: a review with recommendations for genetic counseling. J Genet Counsel. 2016;25(6):1171-1178. doi:10.1007/s10897-016-0002-6. • Masri A, Svensson LG, Griffin BP, Desai MY. Contemporary natural history of bicuspid aortic valve disease: a systematic review. Heart. 2017;103(17):1323-1330. doi:10.1136/heartjnl-2016-309916. • McCulloch MA, Gajarski RJ. Congenital and acquired heart disease. In: McInerny TK, Adam HM, Campbell DE, DeWitt TG, Foy JM, Kamat DM, eds. American Academy of Pediatrics Textbook of Pediatric Care. 2nd ed. Elk Grove Village, IL: American Academy of Pediatrics; 2017:1883-1917. Pediatric Care Online. • McDaniel NL. Ventricular and atrial septal defects. Pediatr Rev. 2001;22(8):265-270. doi:10.1542/pir.22-8-265. • Puri K, Allen HD, Qureshi AM. Congenital heart disease. Pediatr Rev. 2017;38(10):471486. doi:10.1542/pir.2017-0032.

American academy of pediatrics

884